You are on page 1of 421

PHARMACOLOGY

USMLE WORLD STEP 1 PHARMACOLOGY

Question List

Pharmacology Q No: 1 Hepatobiliary system Pharmacology Q No: 42 Cardiology

Pharmacology Q No: 2 Blood vessels Pharmacology Q No: 43 Endocrinology

Pharmacology Q No: 3 Hematology Pharmacology Q No: 44 Hematology

Pharmacology Q No: 4 Head and neck Pharmacology Q No: 45 Dermatology


Pharmacology Q No: 5 Gastrointestinal system Pharmacology Q No: 46 Musculoskeletal

Pharmacology Q No: 6 Neurology Pharmacology Q No: 47 Neurology

Pharmacology Q No: 7 Head and neck Pharmacology Q No: 48 Gastrointestinal system

Pharmacology Q No: 8 Musculoskeletal Pharmacology Q No: 49 Neurology

Pharmacology Q No: 9 Neurology Pharmacology Q No: 50 Neurology

Pharmacology Q No: 10 N euro logy Pharmacology Q No: 51 Gastrointestinal system


Pharmacology Q No: 11 Blood vessels Pharmacology Q No: 52 Neurology

Pharmacology Q No: 12 Musculoskeletal Pharmacology Q No: 53 Musculoskeletal

Pharmacology Q No: 13 Head and neck Pharmacology Q No: 54 Oncology

Pharmacology Q No: 14 Neurology Pharmacology Q No: 55 Neurology

Pharmacology Q No: 15 Endocrinology Pharmacology Q No: 56 Blood vessels


Pharmacology Q No: 16 Pharmacology Q No: 57

Neurology Neurology
Pharmacology Q No: 17 Neurology Pharmacology Q No: 58 Hematology

Pharmacology Q No: 18 Blood vessels Pharmacology Q No: 59 Cardiology

Pharmacology Q No: 19 Pulmonology Pharmacology Q No: 60 Blood vessels

Pharmacology Q No: 20 Endocrinology Pharmacology Q No: 61 Neurology

Pharmacology Q No: 21 Neurology Pharmacology Q No: 62 Dermatology


Pharmacology Q No: 22 Genitourinary Pharmacology Q No: 63 Neurology

Pharmacology Q No: 23 Hematology Pharmacology Q No: 64 Endocrinology

Pharmacology Q No: 24 Neurology Pharmacology Q No: 65 Neurology

Pharmacology Q No: 25 Neurology Pharmacology Q No: 66 Cardiology

Pharmacology Q No: 26 Genitourinary Pharmacology Q No: 67 Cardiology

Pharmacology Q No: 27 Neurology Pharmacology Q No: 68 Neurology


Pharmacology Q No: 28 Cardiology Pharmacology Q No: 69 Pulmonology

Pharmacology Q No: 29 Neurology Pharmacology Q No: 70 Hematology

Pharmacology Q No: 30 Cardiology Pharmacology Q No: 71 Cardiology

Pharmacology Q No: 31 Cardiology Pharmacology Q No: 72 Renal

Pharmacology Q No: 32 Genitourinary Pharmacology Q No: 73 Neurology


Pharmacology Q No: 33 Neurology Pharmacology Q No: 74 Blood vessels

Pharmacology Q No: 34 Renal Pharmacology Q No: 75 Hematology

Pharmacology Q No: 35 Oncology Pharmacology Q No: 76 Musculoskeletal

Pharmacology Q No: 36 Endocrinology Pharmacology Q No: 77 Neurology

Pharmacology Q No: 37 Hepatobiliary system Pharmacology Q No: 78 Neurology

Pharmacology Q No: 38 Renal Pharmacology Q No: 79 Endocrinology


Pharmacology Q No: 39 Hepatobiliary system Pharmacology Q No: 80 Neurology

Pharmacology Q No: 40 Endocrinology Pharmacology Q No: 81 Pulmonology

Pharmacology Q No: 41 Cardiology Pharmacology Q No: 82 Head and neck

USMLE WORLD STEP 1 PHARMACOLOGY

Pharmacology Q No: 83 Neurology Pharmacology Q No: 125 Pulmonology

Pharmacology Q No: 84 Hematology Pharmacology Q No: 126 Blood vessels

Pharmacology Q No: 85 Pulmonology Pharmacology Q No: 127 Cardiology

Pharmacology Q No: 86 Oncology Pharmacology Q No: 128 Blood vessels

Pharmacology Q No: 87 Pulmonology Pharmacology Q No: 129 Renal


Pharmacology Q No: 88 Blood vessels Pharmacology Q No: 130 Neurology

Pharmacology Q No: 89 Neurology Pharmacology Q No: 131 Blood vessels

Pharmacology Q No: 90 Endocrinology Pharmacology Q No: 132 Cardiology

Pharmacology Q No: 91 Cardiology Pharmacology Q No: 133 Cardiology

Pharmacology Q No: 92 Neurology Pharmacology Q No: 134 Cardiology


Pharmacology Q No: 93 Genitourinary Pharmacology Q No: 135 Genitourinary

Pharmacology Q No: 94 Oncology Pharmacology Q No: 136 Cardiology

Pharmacology Q No: 95 Hepatobiliary system Pharmacology Q No: 137 Neurology

Pharmacology Q No: 96 Musculoskeletal Pharmacology Q No: 138 Head and neck

Pharmacology Q No: 97 Blood vessels Pharmacology Q No: 139 Neurology

Pharmacology Q No: 98 Neurology Pharmacology Q No: 140 Musculoskeletal


Pharmacology Q No: 99 Neurology Pharmacology Q No: 141 Musculoskeletal

Pharmacology Q No: 100 Genito urinary Pharmacology Q No: 142 Cardiology

Pharmacology Q No: 101 Renal Pharmacology Q No: 143 Neurology

Pharmacology Q No: 102 Musculoskeletal Pharmacology Q No: 144 Hepatobiliary system

Pharmacology Q No: 103 Endocrinology Pharmacology Q No: 145 Pulmonology

Pharmacology Q No: 104 Neurology Pharmacology Q No: 146 Neurology


Pharmacology Q No: 105 Hepatobiliary system Pharmacology Q No: 147 Hematology

Pharmacology Q No: 106 Pulmonology Pharmacology Q No: 148 Pulmonology

Pharmacology Q No: 107 Neurology Pharmacology Q No: 149 Blood vessels

Pharmacology Q No: 108 Musculoskeletal Pharmacology Q No: 150 Endocrinology

Pharmacology Q No: 109 Blood vessels Pharmacology Q No: 151 Hematology


Pharmacology Q No: 110 Pulmonology Pharmacology Q No: 152 Cardiology

Pharmacology Q No: 111 Cardiology Pharmacology Q No: 153 Neurology

Pharmacology Q No: 112 Blood vessels Pharmacology Q No: 154 Neurology

Pharmacology Q No: 113 Neurology Pharmacology Q No: 155 Cardiology

Pharmacology Q No: 114 Gastrointestinal system Pharmacology Q No: 156 Endocrinology

Pharmacology Q No: 115 Cardiology Pharmacology Q No: 157 Endocrinology


Pharmacology Q No: 116 Blood vessels Pharmacology Q No: 158 Musculoskeletal

Pharmacology Q No: 117 Pulmonology Pharmacology Q No: 159 Blood vessels

Pharmacology Q No: 118 Neurology Pharmacology Q No: 160 Hematology

Pharmacology Q No: 119 Pulmonology Pharmacology Q No: 161 Head and neck

Pharmacology Q No: 120 Musculoskeletal Pharmacology Q No: 162 Hepatobiliary system


Pharmacology Q No: 121 Gastrointestinal system Pharmacology Q No: 163 Blood vessels

Pharmacology Q No: 122 Cardiology Pharmacology Q No: 164 Pulmonology

Pharmacology Q No: 123 Musculoskeletal Pharmacology Q No: 165 Cardiology

Pharmacology Q No: 124 Endocrinology Pharmacology Q No: 166 Oncology

USMLE WORLD STEP 1 PHARMACOLOGY

Pharmacology Q No: 167 Cardiology Pharmacology Q No: 209 Blood vessels

Pharmacology Q No: 168 Pulmonology Pharmacology Q No: 210 Musculoskeletal

Pharmacology Q No: 169 Renal Pharmacology Q No: 211 Hematology

Pharmacology Q No: 170 Neurology Pharmacology Q No: 212 Cardiology

Pharmacology Q No: 171 Cardiology Pharmacology Q No: 213 Neurology


Pharmacology Q No: 172 Neurology Pharmacology Q No: 214 Neurology

Pharmacology Q No: 173 Neurology Pharmacology Q No: 215 Blood vessels

Pharmacology Q No: 174 Blood vessels Pharmacology Q No: 216 Endocrinology

Pharmacology Q No: 175 Genitourinary Pharmacology Q No: 217 Neurology

Pharmacology Q No: 176 Cardiology Pharmacology Q No: 218 Neurology


Pharmacology Q No: 177 Endocrinology Pharmacology Q No: 219 Cardiology

Pharmacology Q No: 178 Neurology Pharmacology Q No: 220 Hematology

Pharmacology Q No: 179 Neurology Pharmacology Q No: 221 Blood vessels

Pharmacology Q No: 180 Pulmonology Pharmacology Q No: 222 Neurology

Pharmacology Q No: 181 Neurology Pharmacology Q No: 223 Pulmonology

Pharmacology Q No: 182 Neurology Pharmacology Q No: 224 Blood vessels


Pharmacology Q No: 183 Neurology Pharmacology Q No: 225 Hematology

Pharmacology Q No: 184 Gastrointestinal system Pharmacology Q No: 226 Cardiology

Pharmacology Q No: 185 Blood vessels Pharmacology Q No: 227 Neurology

Pharmacology Q No: 186 Renal Pharmacology Q No: 228 Endocrinology

Pharmacology Q No: 187 Hematology Pharmacology Q No: 229 Cardiology

Pharmacology Q No: 188 Blood vessels Pharmacology Q No: 230 Renal


Pharmacology Q No: 189 Hepatobiliary system Pharmacology Q No: 231 Cardiology

Pharmacology Q No: 190 Pulmonology Pharmacology Q No: 232 Neurology

Pharmacology Q No: 191 Musculoskeletal Pharmacology Q No: 233 Blood vessels

Pharmacology Q No: 192 Neurology Pharmacology Q No: 234 Cardiology

Pharmacology Q No: 193 Neurology Pharmacology Q No: 235 Renal


Pharmacology Q No: 194 Neurology Pharmacology Q No: 236 Hematology

Pharmacology Q No: 195 Blood vessels Pharmacology Q No: 237 Neurology

Pharmacology Q No: 196 Endocrinology Pharmacology Q No: 238 Endocrinology

Pharmacology Q No: 197 Hepatobiliary system Pharmacology Q No: 239 Reproductive system

Pharmacology Q No: 198 Pulmonology Pharmacology Q No: 240 Blood vessels

Pharmacology Q No: 199 Hematology Pharmacology Q No: 241 Hematology


Pharmacology Q No: 200 Neurology Pharmacology Q No: 242 Pulmonology

Pharmacology Q No: 201 Pulmonology Pharmacology Q No: 243 Blood vessels

Pharmacology Q No: 202 Dermatology Pharmacology Q No: 244 Neurology

Pharmacology Q No: 203 Blood vessels Pharmacology Q No: 245 Pulmonology

Pharmacology Q No: 204 Pulmonology Pharmacology Q No: 246 Genitourinary


Pharmacology Q No: 205 Neurology Pharmacology Q No: 247 Neurology

Pharmacology Q No: 206 Endocrinology Pharmacology Q No: 248 Neurology

Pharmacology Q No: 207 Hematology Pharmacology Q No: 249 Musculoskeletal

Pharmacology Q No: 208 Endocrinology Pharmacology Q No: 250 Cardiology

Pharmacology Q No: 251 Neurology Pharmacology Q No: 290 Cardiology

Pharmacology Q No: 252 Pulmonology Pharmacology Q No: 291 Renal

USMLE WORLD STEP 1 PHARMACOLOGY

Pharmacology Q No: 253 Hematology Pharmacology Q No: 292 Renal

Pharmacology Q No: 254 Pulmonology Pharmacology Q No: 293 Cardiology

Pharmacology Q No: 255 Neurology Pharmacology Q No: 294 Dermatology

Pharmacology Q No: 256 Neurology Pharmacology Q No: 295 Blood vessels

Pharmacology Q No: 257 Head and neck Pharmacology Q No: 296 Blood vessels
Pharmacology Q No: 258 Cardiology Pharmacology Q No: 297 Hepatobiliary system

Pharmacology Q No: 259 Blood vessels Pharmacology Q No: 298 Neurology

Pharmacology Q No: 260 Hematology Pharmacology Q No: 299 Renal

Pharmacology Q No: 261 Musculoskeletal Pharmacology Q No: 300 Cardiology

Pharmacology Q No: 262 Neurology Pharmacology Q No: 301 Hepatobiliary system

Pharmacology Q No: 263 Pulmonology Pharmacology Q No: 302 Blood vessels


Pharmacology Q No: 264 Endocrinology Pharmacology Q No: 303 Endocrinology

Pharmacology Q No: 265 Blood vessels Pharmacology Q No: 304 Hematology

Pharmacology Q No: 266 Endocrinology Pharmacology Q No: 305 Gastrointestinal system

Pharmacology Q No: 267 Neurology Pharmacology Q No: 306 Blood vessels

Pharmacology Q No: 268 Blood vessels Pharmacology Q No: 307 Neurology

Pharmacology Q No: 269 Hematology Pharmacology Q No: 308 Renal


Pharmacology Q No: 270 Endocrinology Pharmacology Q No: 309 Pulmonology

Pharmacology Q No: 271 Genitourinary Pharmacology Q No: 310 Endocrinology

Pharmacology Q No: 272 Cardiology Pharmacology Q No: 311 Endocrinology

Pharmacology Q No: 273 Pulmonology Pharmacology Q No: 312 Neurology

Pharmacology Q No: 274 Oncology Pharmacology Q No: 313 Neurology


Pharmacology Q No: 275 Gastrointestinal system Pharmacology Q No: 314 Hematology

Pharmacology Q No: 276 Renal Pharmacology Q No: 315 Blood vessels

Pharmacology Q No: 277 Blood vessels Pharmacology Q No: 316 Neurology

Pharmacology Q No: 278 Neurology Pharmacology Q No: 317 Neurology

Pharmacology Q No: 279 Endocrinology Pharmacology Q No: 318 Blood vessels

Pharmacology Q No: 280 Musculoskeletal Pharmacology Q No: 319 Neurology


Pharmacology Q No: 281 Hematology Pharmacology Q No: 320 Pulmonology

Pharmacology Q No: 282 Musculoskeletal Pharmacology Q No: 321 Head and neck

Pharmacology Q No: 283 Blood vessels Pharmacology Q No: 322 Blood vessels

Pharmacology Q No: 284 Genitourinary Pharmacology Q No: 323 Genitourinary

Pharmacology Q No: 285 Renal Pharmacology Q No: 324 Blood vessels


Pharmacology Q No: 286 Endocrinology Pharmacology Q No: 325 Musculoskeletal

Pharmacology Q No: 287 Neurology Pharmacology Q No: 326 Neurology

Pharmacology Q No: 288 Neurology Pharmacology Q No: 327 Gastrointestinal system

Pharmacology Q No: 289 Endocrinology Pharmacology Q No: 328 Neurology

USMLE WORLD STEP 1 PHARMACOLOGY

Q NO 1: After a positive PPD test, a 64-year-old male begins isoniazid therapy. One

month later he presents complaining of fever, anorexia and nausea. What is the

most likely cause of his current symptoms?

A. Peripheral nerve damage

B. Hepatocyte damage
C. Gastric mucosal damage

D. Serum sickness

E. Factitious disorder

Explanation:

Isoniazid (INH) is directly hepatotoxic in 10-20% of patients causing


acute mild hepatic dysfunction with transient increases in serum AST

(SGPT) ALT (SGPT) and bilirubin and symptoms like fever, anorexia, and

nausea. This adverse effect usually occurs during the first 4-6 months
of treatment and in most cases liver function tests return to baseline

with continued INH therapy. In rare instances however severe hepatitis

and progressive liver dysfunction bilirubinuria, and jaundice occur.


(Choice A) Peripheral neuropathy is also a potential side effect of INH

therapy if simultaneous pyridoxine is not administered. However

peripheral neuropathy would not cause the above symptoms.


(Choice D) Serum sickness drug hypersensitivity generally causes fever

urticaria, arthralgias, proteinuria and lymphadenopathy 5-10 days after

exposure to the drug (antigen). This patient does not have arthralgias
or skin findings.

Educational Objective:
Isoniazid (INH) can be directly hepatotoxic causing acute mild hepatic

dysfunction in 10-20% of patients. In a smaller percentage of cases

frank hepatitis may develop causing fever anorexia and nausea.

USMLE WORLD STEP 1 PHARMACOLOGY

Q NO 2: A 47-year-old Caucasian female was diagnosed with hyperlipidemia that

responded poorly to dietary changes. Her past medical history is significant for

acute cholecystitis that required a five-day hospitalization. Her father died of a

myocardial infarction at the age of 54 and her mother had diabetes mellitus.
Which of the following drug combinations is most likely to precipitate gallstone

formation in this patient?

A.
B.
Atorvastatin and ezetimibe
Atorvastatin and cholestyramine

C. Atorvastatin and gemfibrozil

D.
E.
Gemfibrozil and cholestyramine
Niacin and ezetimibe

F. Niacin and gemfibrozil

Explanation:

Bile acid-binding resins (cholestyramine, colestipol, colesevelam) work


by binding to bile acid in the gastrointestinal tract, interfering with

its enterohepatic circulation. LDL is reduced as a consequence, because

hepatic cholesterol is consumed in the re-synthesis of bile acids,


which in turn increases the uptake of LDL from the circulation. Bile

acid production and secretion is increased 10-fold because of the

interruption in the enterohepatic circulation of bile acids. Bile acid-


binding agents increase the cholesterol content of bile, increasing the

risk of gallstone formation. Fibrates also increase the cholesterol

content of bile, and thus also increase the risk for gallstones. Both
fibrates and bile acid-binding resins should be used with caution in

patients with preexisting gallbladder disease.

Educational Objective:

Both gemfibrozil and cholestyramine increase cholesterol excretion by

the liver. Along with the reduction in serum LDL there is an increased
risk for gallstone formation.

USMLE WORLD STEP 1 PHARMACOLOGY

Q NO 3: A 6-year-old female is brought to the FR with fever, headache and

confusion. The family notes that three years ago she suffered a severe

allergic reaction to penicillin that required hospitalization. She is admitted,


and on day4 of her hospitalization, her CBC shows:

Complete blood count

Hemoglobin 9.0 g/L

Erythrocyte count 2.2 mln/mm3


Platelets 90000/mm3

Leukocyte count 2500/mm3

The patient most likely received treatment with:

A. Clindamycin

B. Gentamycin
C. Chloramphenicol

D. Metronidazole

E. Vancomycin

Explanation:

The patient’s CBC shows pancytopenia (decreased erythrocytes,


leukocytes, and platelets). This condition is also called aplastic

anemia. It occurs due to suppression of stem cell function in the bone

marrow.
Common causes of acquired pancytopenia include radiation exposure,

environmental toxins (e.g. benzene, arsenicals), certain infections,

and drugs. Chloramphenicol can cause both dose-dependent and dose-


independent aplastic anemia. Chloramphenicol elicits its antibacterial

effect by binding to the ribosomal 50S subunit and inhibiting the

peptidyl transferase enzyme. Thus, it suppresses bacterial protein


synthesis. Dose-dependent aplastic anemia associated with

chloramphenicol is reversible after the medication is withdrawn. Dose-

independent anemia is usually severe and may be fatal.


(Choice A) Clindamycin is active against Bacteroides and Gram positive

cocci. The most significant side effect of clindamycin is


pseudomembranous colitis.

(Choice B) Side effects of gentamycin include vestibular and cochlear

ototoxicity, nephrotoxicity and neuromuscular paralysis (with large


dose or intrapleural administration).

(Choice D) The common side effects of metronidazole are

gastrointestinal (nausea, vomiting, crampy abdominal pain) and


neurologic (paresthesias, dizziness). When taken with alcohol,

metronidazole induces a disulfiram-like reaction.

(Choice E) Rapid administration of vancomycin can cause histamine


release which results in flushing (“red man syndrome”). This antibiotic

may also cause dose-related ototoxicity.

Educational Objective:

Aplastic anemia (pancytopenia) is caused by many drugs and

environmental toxins. Chloramphenicol can lead to both dose-dependent


(reversible) and dose-independent (often irreversible) pancytopenia.

USMLE WORLD STEP 1 PHARMACOLOGY

Q NO 4: A chemotherapeutic agent is shown to bind in vitro with viral encoded

enzymes such as DNA polymerase, RNA polymerase, and reverse

transcriptase. The agent does not require intracellular activation to

demonstrate its antiviral effect. Which of the following agents is studied in the
experiment?

A. Acyclovir

B. Foscarnet
C. Lamivudine

D. Saquinavir

E. Ganciclovir
F. Amantadine

G. Oseltamivir

Explanation:

Foscarnet is a pyrophosphate analog that does not require intracellular

activation. It directly inhibits both DNA polymerase in herpesvirus and


reverse transcriptase in human immunodeficiency virus (HIV). Foscarnet

must be administered intravenously and is most commonly used in

treating advanced AIDS patients who have acyclovir-resistant


herpesvirus infections or ganciclovir-resistant cytomegalovirus

infections.

(Choice A) Acyclovir is a nucleoside analog that must be


intracellularly converted to its monophosphate form by a virally

encoded thymidine kinase. Cellular kinases convert the monophosphate

form into a triphosphate form, which inhibits herpesviral DNA


polymerase-mediated replication.

(Choice C) Lamivudine is a cytosine analog classified as a nucleoside

reverse transcriptase inhibitor (NRTI). It must be phosphorylated to


its active form, lamivudine triphosphate, by intracellular kinases.

Lamivudine inhibits HIV reverse transcription through viral DNA chain

termination.
(Choice D) Saquinavir does not require intracellular activation. It is

a protease inhibitor that binds to the catalytic site of an HIV

aspartic protease, preventing the cleavage of polyprotein precursors


necessary for the generation of functional viral proteins.

(Choice E) Ganciclovir is a guanine nucleoside analogue that is

structurally similar to acyclovir. It requires intracellular conversion


to its monophosphate form by a virally encoded kinase, and has greater

activity than acyclovir against cytomegalovirus DNA polymerase.

(Choice F) Amantadine is an antiviral agent that does not require


intracellular activation. It binds to and inhibits the M2 ion channel

protein of influenza A viruses, blocking viral un coating after host


cell endocytosis.

(Choice G) Oseltamivir is an antiviral agent that does not require

intracellular activation. It is a sialic acid analogue inhibitor of


influenza A and B virus neuraminidase.

Educational Objective:
Of all the antiviral agents that bind and inhibit DNA polymerase in

herpesvirus and reverse transcriptase in HIVI the pyrophosphate analog

foscarnet is one of few that do not require intracellular activation by


viral or cellular kinases.

USMLE WORLD STEP 1 PHARMACOLOGY

Q NO 5: Enterococci strains isolated from a patient with bacteremia are known to

substitute D-alanine for D-lactate in the synthesis of pentapeptide proteoglycan

precursors. This substitution decreases pentapeptide binding for which of the

following antibiotics?

A. Vancomycin
B. Penicillin

C. Cycloserine

D. Polymixin
E. Tetracycline

F. Amikacin

Explanation:

Vancomycin binds to D-alanyl-D-alanine termini in cell wall peptide

precursors and prevents the formation of peptidoglycan. The mechanism


of vancomycin resistance in Vancomycin Resistant Enterococcus (VRE) is

a change in the peptide precursor structure that alters the vancomycin-

binding site from a D-alanyl-D-alanine terminus to a D-alanine-D-


lactate terminus.

(Choice B) Penicillin resistance occurs by two mechanisms. First,

bacteria that produce beta-lactamase (penicillinase) are able to


destroy the beta-lactam ring of the penicillins and render them

ineffective. The cephalosporins, carbapenems and penicillinase-

resistant penicillins such as nafcillin and methicillin are not


susceptible to penicillinase and, therefore, retain their antimicrobial

effect. Second, some bacteria have modified the penicillin binding

proteins in their peptidoglycan cell walls such that beta-lactam


antibiotics are unable to bind and interfere with cell wall synthesis.

This is the mechanism by which methicillin-resistant Staphylococcus

aureus (MRSA) is resistant to even the penicillinase resistant beta-


lactam antibiotics. Remember that vancomycin is nota beta lactam drug.

(Choice C) Cycloserine inhibits the incorporation of D-alanine into the

UDP-MUrNAc-oligopeptide during bacterial cell wall synthesis. This


bacterial cell wall synthesis inhibitor has a high level of ototoxicity

and is rarely used.

(Choice D) Polymyxin binds to disrupts, and interferes with the


permeability of the cytoplasmic membrane.

(Choice E) Tetracycline is an inhibitor of protein synthesis that binds


to the 303 subunit and inhibits binding of aminoacyl-tRNAs. The

mechanisms of tetracycline resistance include an increased efflux of

drug from within the bacterial cell or production of a protein that


allows translation to take place even when tetracycline is present

within the bacterium.

(Choice F) Amikacin is an aminoglycoside antibiotic that acts as an


inhibitor of the 30s ribosomal unit, preventing bacterial protein

synthesis. The mechanism of resistance to the aminoglycosides is an

increased efflux of drug from within the bacterial cell.

Educational Objective:

The mechanism of vancomycin resistance in organisms such as VRE is a


substitution of D-lactate in the place of D-alanine during the process

of peptidoglycan cell wall synthesis. This prevents the binding of

vancomycin to its usual D-alanyl-D-alanine binding site in the cell


wall.

USMLE WORLD STEP 1 PHARMACOLOGY

Q NO 6: A 21-year-old Caucasian male is brought to the ER with prolonged tonic-

clonic seizures. His wife says that he had a similar episode three months ago for

which he was prescribed some medications that he never took. IV lorazepam is


administered in the ER, and the seizures stop. An intravenous infusion of another

drug is simultaneously instituted to prevent seizure recurrence. Which of the

following is the most likely mechanism of action of the second drug infused in this
patient?

A. Decreases calcium current in thalamic neurons

B. Decreases sodium current in cortical neurons


C. Increases chloride current on multiple levels

D. Blocks NMDA receptors in hippocampal neurons

E. Blocks outgoing potassium current on multiple levels

Explanation:
Recurrent or continuous generalized tonic-clonic seizures that last for

more than 30 minutes without a return to consciousness are called

status epilepticus. It is a life-threatening condition that has a


number of systemic effects, including hypertension tachycardia, cardiac

arrhythmias, and lactic acidosis. Treatment of status epilepticus

should be started immediately. It consists of the following:


1. Benzodiazepines are the first-line drugs for management of status

epilepticus. Lorazepam is the drug of choice.

2. Phenytoin (or fosphenytoin) is administered simultaneously to


prevent the recurrence of seizures. Benzodiazepines are preferred to

phenytoin for initial seizure management because benzodiazepines have a


more rapid onset of action. The onset of action of phenytoin is about

15 minutes after IV infusion—benzodiazepines begin working within a few

minutes.
If seizure does not stop after benzodiazepines and phenytoin is

administered phenobarbital is indicated. Alternatively midazolam,

propofol, or inhaled anesthetics may be used to induce a state of


general anesthesia.

10

USMLE WORLD STEP 1 PHARMACOLOGY

Phenytoin inhibits neuronal high-frequency firing by reducing the

ability of sodium channels to recover from inactivation. Thus at high-


frequency firing rates the neuron becomes refractory to reactivation

and reduces seizure activity.

(Choice A) Ethosuximide acts by blocking T-type calcium channels in the


thalamic neurons. It is first-line treatment for absence seizures.

Ethosuximide is not effective in status epilepticus.

(Choice C) Benzodiazepines, barbiturates and alcohol bind to the


component of GABAA receptor that is a ligand gated chloride channel.

These substances enhance the inhibitory action of GABA on the receptor

and increase chloride current.


(Choices D and E) Valproic acid reduces abnormal electric activity in

the brain by blocking NMDA receptors and affecting potassium current.

This drug also affects Na+ channels and GABA receptors.

Educational Objective:

Benzodiazepines (lorazepam) are first-line agents in the management of


status epilepticus. Phenytoin (or fosphenytoin) is administered

simultaneously to prevent the recurrence of seizures. Phenytoin

inhibits neuronal high- frequency firing by reducing the ability of


sodium (Na+ channels to recover from inactivation.

11

USMLE WORLD STEP 1 PHARMACOLOGY

Q NO 7: A 46-year-old nurse is brought to the emergency room with confusion and

fever. On physical exam, her skin is flushed. Her oral mucosa is dry, and her pupils

are dilated and poorly responsive to light. A bottle of atropine is found in her

pocket. Which of the following drugs can cause a similar clinical presentation?

A. Diazepam
B. Amitriptyline

C. Propranolol

D. Carbamazepine
E. Prazosin

Explanation:
Classic signs and symptoms of anticholinergic toxicity include fever,

mucosal and axillary dizziness, cutaneous flushing, mydriasis,

cycloplegia, and delirium (use the mnemonic “hot as a hare dry as a


bone, red as a beet, blind as a bat, and mad as a hatter”). Of the

drugs listed, amitriptyline has the most potent anticholinergic side

effects. Amitriptyline and the other tricyclic antidepressants have


these effects because they block muscarinic receptors. Overdose can

cause anticholinergic symptoms mimicking atropine toxicity.

(Choice A) Typical symptoms of benzodiazepine overdose include sedation


anterograde amnesia and respiratory depression. Benzodiazepines are

rarely fatal in overdose.

(Choice C) Adverse effects associated with nonselective 3-adrenergic


blockers include bronchial constriction, masked symptoms of

hypoglycemia in diabetics, bradyarrhythmias, worsening of Raynaud

phenomenon, and CNS depression.


(Choice D) Carbamazepine is an anticonvulsant used in seizure disorders

trigeminal neuralgia and bipolar disorder. In overdose, its toxic

effects include stupor, coma and increased seizure risk.


(Choice E) Prazosin is an al-adrenergic blocker used to treat

hypertension and urinary retention due to benign prostatic hypertrophy.

Its major adverse effect is hypotension (especially postural


hypotension).

Educational Objective:
Fever, cutaneous flushing, dry oral mucosa, dilated poorly reactive

pupils and confusion are all signs of anticholinergic toxicity.


Tricyclic antidepressants, particularly amitriptyline, have

antimuscarinic side effects that may mimic atropine toxi city.

12

USMLE WORLD STEP 1 PHARMACOLOGY

Q NO 8: A 65-year-old female is hospitalized with a hip fracture. Her bone density,

measured by quantitative X-ray densitometry, corresponds to the lowest 10%. Her

comorbidities include hypertension; diabetes mellitus, type 2; and congestive

heart failure (CHE). The attending physician suggests that if this patient had
received long-term treatment with a drug affecting calcium metabolism her

fracture could have been avoided. Which drug is most likely mentioned by the

physician?

A. Furosemide

B.
C.
Hydrochlorothiazide
Spironolactone

D. Digoxin

E.
F.
Enalapril
Glyburide

G. Acarbose

Explanation:

This patient has osteoporosis, which is reflected by her very low bone
density and her fractured hip. Hydrochlorothiazide is a diuretic agent

used to treat hypertension and to treat congestive heart disease. An

opportune side effect of hydrochlorothiazide is that it increases the


absorption of calcium from the distal convoluted tubules, making it an

ideal agent for treating hypertension/CHE in a woman who is also at

risk for osteoporosis. Studies have shown that patients who are on
thiazide diuretics have relatively higher bone mineral densities.

Hydrochlorothiazide has another fortuitous side effect—the increased

renal calcium absorption causes hypocalciuria, which helps prevent


painful renal stones in some patients.

(Choice A) Furosemide is a loop diuretic used to treat hypertension and

CHF. It causes an increase in urinary calcium loss that could only


worsen osteoporosis in this woman.

(Choice C) Spironolactone inhibits action of other steroid hormones,

such as testosterone. The antiandrogenic action of spironolactone is


utilized for the treatment of hirsutism and androgen-dependent

malignancies. It does not have an affect on calcium homeostasis.

(Choice D) Therapeutic digoxin levels do not lead to a significant


alteration in calcium levels. However, digitalis toxicity is associated

with hypercalcemia. This is not the correct answer because the risk of

digitalis toxicity far outweighs any possible benefit of an increased


calcium level.

(Choice E) Enalapril has not been associated with significant change in

bone mineral density.


(Choice F and G) Glyburide and acarbose are oral hypoglycemic agents

used to treat type 2 diabetes mellitus. Both of these medications have

an insignificant effect on bone mineral density.

Educational Objective:

Thiazide diuretics decrease urinary calcium excretion and may improve


bone density. Furosemide will increase urinary calcium loss, making it

a possible treatment for hypercalcemia, but not for women with porus

bones.
Extremely important concept

13

USMLE WORLD STEP 1 PHARMACOLOGY

Q NO 9: A 23-year-old female presents to the ER with chest pain dyspnea,

palpitations perioral numbness and sweating. Soon after treatment with

lorazepam the patient feels better and wants to go home. Cardiac

enzymes are normal and an ECG shows normal sinus rhythm with no ST
segment or T-wave abnormalities. Which of the following indicates the site

of action of the drug on the diagram below?

A. A

B. B
C. C

D. D

E. E
F. F

Explanation:
GABA (y-aminobutyric acid) is the main inhibitory neurotransmitter in

the CNS. Its synthesis from glutamate is mediated by glutamate

decarboxylase (GAD) within neurons. Metabolism of GABA occurs by means


of transamination performed by enzyme GABA transaminase (GABA-T). Both

GAD and GABA-T use Vitamin B6 (pyridoxal phosphate) as a cofactor.

There are thee types of GABA receptors known at this time (see the
table below for the summary). Activation of each type of receptor

ultimately leads to neuron hyperpolarization (inhibition).

This case scenario describes use of the benzodiazepine lorazepam for

treatment of a panic attack. Benzodiazepines act by binding to the


GABAA receptor and stimulating the influx of chloride ions into the

neurons. Binding to the

Receptor facilitates the inhibitory action of GABA in the CNS.


Barbiturates, alcohol and zolpidem display a similar mechanism of

action. These substances, however, bind to a different component of the

GABAA receptor than do benzodiazepines.

14

USMLE WORLD STEP 1 PHARMACOLOGY

(Choice A) Benzodiazepines, barbiturates, zolpidem and alcohol are all

GABA-agonists. They act by binding to GABAA receptor, not to GABA


itself.

(Choice C) Stimulation of the GABAA receptor leads to activation of ion

channels and increased chloride influx into the cell. GABA agonists do
not bind to the ion channels directly; they bind to the receptor

binding sites.

(Choices D, E and F) Stimulation of the GABA6 receptor leads to


activation of an inhibitory G-protein with a subsequent change in

activity of an ion channel. It results in inhibition of adenylyl

cyclase, a decrease in calcium influx and an increase in potassium


efflux.

Educational Objective:
The GABAA and GABAC receptors are ion channels, while the GABAB

receptor is linked to a G- protein. Benzodiazepines, zolpidem,

barbiturates and alcohol bind to a component of GABAA receptors and


facilitate the inhibitory action of GABA in the CNS.

15

USMLE WORLD STEP 1 PHARMACOLOGY

Q NO 10: A 60-year-oldfemale presents to your office complaining of involuntary

face and tongue movements. You note that although adequately responding to

your questions, the patient appears to grimace and writhe. She says that the

symptoms get worse after emotional stress and fatigue. Her past medical history
is significant for long-standing schizophrenia effectively controlled with

medications. Which of the following is the most likely diagnosis in this patient?

A.
B.
Acute dystonia
Neuroleptic-induced Parkinsonism

C. Akathisia

O
D.
E.
Tardive dyskinesia
Manic episode

F. Neuroleptic malignant syndrome


G. Drug-induced delirium

Explanation:
Extrapyramidal symptoms (EPS) occur frequently as side effects of

typical antipsychotics but may also occasionally occur with the use of

atypical antipsychotics. This patient has developed tardive dyskinesia


as a side effect of antipsychotic usage. Tardive dyskinesia is

characterized by involuntary pen oral movements such as biting,

chewing, grimacing, and tongue protrusions. Involuntary choreoathetoid


movements of the head, limbs, and trunk may also be observed. The

condition usually arises between four months and four years of

treatment and may be irreversible.


Risperidone is the atypical antipsychotic most likely to cause EPSI

while clozapine is the atypical antipsychotic least likely to cause

EPS. Therefore tardive dyskinesia is best managed by decreasing the


dose or discontinuing the offending antipsychotic and replacing it with

clozapine. Because clozapine is associated with agranulocytosis, it is

typically considered to be a medication of last resort.


(Choice A) Acute dystonia can develop abruptly at any point between

four hours and four days after receiving an antipsychotic medication.

The condition is characterized by muscle spasms or stiffness, tongue


protrusions or twisting opisthotonus, and oculogyric crisis (a forced

sustained elevation of the eyes in an upward position). Treatment of

acute dystonia is with antihistamines (eg, diphenhydramine) or


anticholinergics (eg, benztropine or trihexyphenidyl).

(Choice B) Parkinsonism can also occur, developing between four days

and four months after receiving an antipsychotic medication. It


presents with cogwheel rigidity, masked facies, bradykinesis, pill-

rolling finger tremors and shuffling gait. Treatment is with

anticholinergics such as benztropine.


(Choice C) Akathisia is a subjective feeling of restlessness that

compels patients to constantly move around. It can occur at anytime

during treatment with antipsychotics.


(Choice E) Acute mania is marked by grandiose delusions (millions of

dollars are held in trust for them; the President calls them; they are
chosen by God for a magnificent purpose etc.). Flight of ideas and

pressured speech become very intense and hyperactivity and impulsivity

becomes more pronounced.


(Choice F) Neuroleptic malignant syndrome is a rare and potentially

fatal syndrome characterized primarily by delirium fever muscle

rigidity, and autonomic instability.

16

USMLE WORLD STEP 1 PHARMACOLOGY

(Choice G)The hallmark of delirium is confusion ora clouding of the

sensorium. Patients may appear somewhat dazed and unclear about their
surroundings. Disorientation to time and place are common accompanying

features.

Educational Objective:

Tardive dyskinesia is characterized by involuntary perioral movements

such as biting, chewing, grimacing, and tongue protrusions. Involuntary


choreoathetoid movements of the head, limbs, and trunk may also be

observed. The condition usually arises between four months and four

years of treatment and maybe irreversible.

17

USMLE WORLD STEP 1 PHARMACOLOGY

Q NO 11: A medical student is conducting a pharmacology experiment. He infuses

Drug X intravenously over different dose ranges and measures several important

hemodynamic parameters. Graphs plotting the recorded measurements of renal

blood flow and cardiac output change with increasing doses of Drug X are shown
below. Which of the following is most likely to be the drug used in the

experiment?

A. Epinephrine

B.
C.
Phenylephrine
Dopamine

D. Edrophonium
E. Esmolol

Explanation:
Dopamine is unique among adrenergic stimulators due to its ability to

simultaneously increase both myocardial contractility and renal blood

flow. Because of this unique effect dopamine is used to increase renal


perfusion in low- output cardiac states, such as hypovolemic and

cardiogenic shock. Dopamine executes its renal effects via stimulation

of D1 adrenergic receptors in the walls of renal blood vessels. These


D1 adrenergic receptors are also located in the splanchnic and

mesenteric arteries, which are also dilated by dopamine’s effects.

Dopaminergic increases in renal blood flow results in increased


glomerular filtration, increased urinary output, and increased sodium

excretion.

The dose of dopamine that affects Dl receptors of renal arteries also


acts as an agonist on β1 receptors of myocardium, increasing both heart

rate and myocardial contractility, leading to overall increases in

cardiac output. Systolic blood pressure subsequently increases due to


the rise in cardiac output. Higher doses of dopamine stimulate al-

adrenoreceptors in the arteries of skin and viscera (including

kidneys), resulting in vasoconstriction.


[See the table below for comparison of the effects of dopamine with

epinephrine (a and agonist) and phenylephrine (a agonist)].

(Choice A) Epinephrine stimulates both α and β adrenoreceptors and

increases myocardial contractility and cardiac output (β stimulation),


while decreasing renal blood flow (al stimulation).

18

USMLE WORLD STEP 1 PHARMACOLOGY

(Choice B) Phenylephrine is a selective agonist of a-adrenoreceptors

that does not affect [31 receptors of myocardium. Stimulation of al


receptors causes renal splanchnic and mesenteric vasoconstriction.

(Choice D) Edrophonium is a short-acting cholinesterase inhibitor used

in the diagnosis of myasthenia gravis. Through its cholinergic effects


edrophonium reduces heart rate, cardiac conduction, and cardiac

contractility.

(Choice E) Esmolol is a cardioselective β blocker with a short duration


of action. Esmolol decreases heart rate, myocardial contractility, and

cardiac conduction without affecting renal blood flow.

Educational Objective:

Low doses of dopamine stimulate both Dl receptors in renal and

splanchnic blood as well as β1 receptors of the myocardium, leading to


increases in both renal blood flow and myocardial contractility. On the

other hand, high doses of dopamine stimulate α1 receptors, resulting in

peripheral vasoconstriction.

19

USMLE WORLD STEP 1 PHARMACOLOGY

Q NO 12: A 58-year-old female with a long history of rheumatoid arthritis and

peptic ulcer disease presents to the emergency room with acute low back pain

after too quickly sitting down onto a hardwood chair. She reports that she

takes many medications, and has been for several years. X-ray shows a
fracture of the fourth lumbar vertebra. Which of the following may have

contributed to this injury?

A. Rheumatoid arthritis progression

B.
C.
Methotrexate side effect
Prednisone side effect

D. Indomethacin side effect

E. Omeprazole side effect

Explanation:
This patient presents with a pathologic fracture of a lumbar vertebra.

A pathologic fracture is one that results from a force significantly

less than that required to fracture a normal bone. Underlying bone


pathology is generally present. Patients taking as little as 7.5mg of

daily prednisone for longer than six months can develop osteoporotic

bone changes, and 30-50% of patients on prolonged systemic


glucocorticoids may develop pathologic vertebral body fractures. Long

term use of systemic steroids is thought to promote osteoporosis by

causing decreased synthesis of bone matrix, inhibiting the intestinal


action of vitamin D to promote calcium absorption, and causing

increased parathyroid hormone levels.

(Choice A) Rheumatoid arthritis (RA) is an autoimmune inflammatory


disorder that predominantly affects synovial joints in the hands and

feet, but also affects larger joints like the wrists and shoulders.

Intervertebral body joints lack a synovial space. RA would be unlikely


to directly cause a pathologic thoracolumbar vertebral fracture.

(Choices B, D and E) Methotrexate, NSAIDs and proton pump inhibitors

are not known to cause osteoporosis.

Educational Objective:

Osteoporosis is a common cause of pathological vertebral fractures.


Chronic systemic use of corticosteroids like prednisone promotes

osteoporosis and therefore may cause such fractures.

20

USMLE WORLD STEP 1 PHARMACOLOGY

Q NO 13: A chemotherapeutic agent classified as a guanosine derivative

demonstrates significant antiviral activity against herpes simplexvirus


A.
Viral

type 1 herpes simplexvirus type 2, and varicella zoster virus. The same

agent demonstrates weak antiviral activity against Epstein-Barr virus


and cytomegaly virus. Which of the following best explains this

difference in viral susceptibility to the drug?

protease activity

B. DNA polymerase structure


C. Structural protein composition

D. Drug phosphorylation rate

E. Drug degradation rate

Explanation:
The chemotherapeutic agent described here is most likely acyclovir, a

guanosine analog. Once acyclovir enters the herpesvirus-infected host

cell, it is converted to acyclovir monophosphate principally via a


virally-encoded thymidine kinase (TK). This is the rate-limiting step

in the activation of acyclovir. The acyclovir monophosphate is then

phosphorated by cellular enzymes into the active triphosphate form,


which impairs viral DNA polymerase-mediated replication of the virus.

Epstein-Barr virus (EBV) and cytomegalovirus (CMV) do not produce the

same thymidine kinase that herpes simplex viruses (HSV) and varicella
zoster virus (VZV) do. As a result EBV- or CMV-infected cells cannot

easily convert acyclovir into its pharmacologically active triphosphate

form. At very high intracellular concentrations some acyclovir appears


to be activated by unidentified cellular phosphorylating enzymes, which

may explain the partial susceptibility of EBV and CMV to higher

acyclovir doses. In addition, the partial activity of acyclovir against


CMV may be mediated by inhibited synthesis of a virus-specific

polypeptide.

(Choice A) Viral protease activity is not responsible for differences


in viral susceptibility to acyclovir.

(Choice B) Cytomegalovirus has increased sensitivity to ganciclovir

(when compared to other herpesviruses) because of differences in viral


DNA polymerase structure.

(Choice C) Anti-herpesviral drugs currently in use include acyclovir,

famciclovir, valaciclovir, ganciclovir, cidofovir, and foscarnet. The


active forms of these drugs inhibit viral DNA polymerase. The first

21

USMLE WORLD STEP 1 PHARMACOLOGY

four drugs listed require mono phosphorylation by a virally-encoded

kinase. Because these drugs are activated enzymatically, the structural


protein composition of the virus does not affect drug sensitivity.

(Choice E)The drug degradation rate is not responsible for differences

in viral susceptibility to acyclovir.

Educational Objective:

Mono phosphorylation of acyclovir by a viral thymidine kinase is the


first (and rate-limiting) step in the conversion of acyclovir to its

active triphosphate form. Acyclovir and related drugs (eg, famciclovir,

valaciclovir) are more effective against herpes simplex virus and


varicella zoster virus than cytomegalovirus and Epstein-Barr virus.

22

USMLE WORLD STEP 1 PHARMACOLOGY

Q NO 14: A 38-year-old traveler presents to your office for a routine check-up. He

will be leaving for a cruise next week, and asks for a drug that would prevent the

severe nausea and vomiting he experiences on ships. After recommending the

appropriate drug, you warn about which of the following side effects?

A. Diarrhea

B. Nasal congestion
C. Cough

0
D.
E.
Dry mouth
Frequent urination

Explanation:

Antihistaminics, or Hi histamine receptor blockers, are very useful


drugs in the treatment of allergy. Hi receptor blockers decrease the

activity of this receptor by increasing the proportion of inactive Hi

receptors, via a process known as reverse blockade. In addition to


blocking Hi receptors, first-generation antihistaminics have

antimuscarinic, antiadrenergic and anti serotonergic properties.

In motion sickness, the muscarinic Mi and histaminic Hi pathways are


stimulated, resulting in the nausea and vomiting of motion sickness.

Because of their anti muscarinic and antihistaminic properties, first-

generation antihistaminic drugs like meclizine and dimenhydrinate are


effective at preventing these symptoms. Other first generation drugs

are also effective, but are generally avoided because of their sedating

properties. Scopolamine has only antimuscarinic effects and is also


effective at preventing the symptoms of motion sickness.

Side effects of antimuscarinics include blurry vision, dry mouth,

palpitations, urinary retention and constipation. The options in


choices A, B, C and E occur with stimulation rather than blockade of

the muscarinic pathway.

Educational Objective:

Antimuscarinic agents and antihistamines with antimuscarinic action are

most effective for motion sickness prevention.

23

USMLE WORLD STEP 1 PHARMACOLOGY

Q NO 15: A new drug is being developed for the treatment of diabetes. This drug

activates the peroxisome proliferator-activated receptor gamma (PPAR-gamma),

a nuclear receptor and transcription factor. Activation of this receptor would most
likely result in which of the following?

A. Down regulation of leptin activity

B. Increased insulin release


C. Decreased insulin resistance

D. Increased free fatty acid levels

E. Downregulation of adiponectin

Explanation:

Thiazolidinediones (TZD5) exert their glucose-lowering effect by


decreasing insulin resistance. TZDs bind to peroxisome proliferator

activated receptor gamma (PPAR-gamma), which is a transcriptional

regulator of the genes involved in glucose and lipid metabolism. One of


the most crucial genes regulated by PPAR-gamma is adiponectin, which is

a cytokine secreted by fat tissue (adipocytokine). Adiponectin levels

are low in type 2 diabetes which TZDs alter. As the glucose lowering
effect of TZDs requires alteration in gene transcription and protein

synthesis, it takes days to weeks after initiation of therapy to

observe a significant reduction in glucose levels. TZDs do not cause


hypoglycemia. The main side effects of TZDs are fluid retention, weight

gain, and the precipitation of congestive heart failure from fluid


retention.

(Choice A) Leptin is the hormone secreted by fat cells. It is

responsible for appetite suppression and decreased insulin resistance,


which is mediated via the central nervous system. Although PPAR-gamma

activation increases fat cell mass, circulating leptin levels are

essentially unchanged. Some studies have shown that PPAR-gamma


activation leads to suppression of the transcription of the leptin

24

USMLE WORLD STEP 1 PHARMACOLOGY

gene. The activity of leptin is not significantly altered with PPAR

gamma activation, however.


(Choice B) PPAR-gamma is present in small amounts in pancreatic beta

cells. TZDs do not directly alter insulin secretion: however, as

insulin resistance diminishes, circulating levels of insulin tend to


decrease overtime.

(Choice D) PPAR-gamma activation leads to an increase in fat mass

secondary to the increased differentiation of preadipocytes into mature


adipocytes. The movement of free fatty acids into fat cells is

increased, and circulating free fatty acids levels decrease.

(Choice E) Adiponectin is a newly-discovered cytokine secreted by fat


cells. Adiponectin levels are decreased in patients with type 2

diabetes mellitus. One of the mechanisms by which PPAR-gamma activation

decreases insulin resistance is by increased expression of the


adiponectin gene.

Educational Objective:
TZDs activate PPAR-gamma, which is the nuclear receptor that alters the

transcription of genes responsible for fat and lipid metabolism.

Thiazolidinediones (TZD5) exert their glucose-lowering effect by


decreasing insulin resistance.

25

USMLE WORLD STEP 1 PHARMACOLOGY

Q NO 16: A 12-year-old male is hospitalized with headache, nausea and fever. A

physical examination is significant for neck stiffness. Gram-negative cocci in

pairs are revealed during CSF microscopy. Which of the following is the best way
to prevent infection in his close contacts?

A. Capsular polysaccharide vaccine

B. Pilus protein vaccine


C. Lipopolysaccharide vaccine

D. Penicillin

E. Sulfamethoxazole
F. Rifampin

Explanation:
The gram stain description of gram-negative cocci in pairs makes the

etiologic agent of this infection most likely Neisseria meningitidis.


Because the agent is transmitted by direct contact with contaminated

respiratory secretions or airborne droplets, family members and other

close contacts of persons with meningococcal disease are at high risk


of acquiring the disease or becoming carriers and must receive

immediate prophylaxis.

Rifampin has been used successfully since the 1960’s for


chemoprophylaxis for house hold members and close contacts of patients

with invasive meningococcal disease and is the most likely chemo

prophylactic agent to be administered in this subject. Rifampin is used


for chemoprophylaxis because it penetrates well into the respiratory

tract and will eliminate nasopharyngeal colonization. Rifampin’s most

notable side effect is an orange discoloration of secretions (urine,


breast milk and tears), and patients should be alerted to the fact that

contact lenses will be permanently stained orange. Also, remember that

rif AMP in “AMPlifies” CYP45O and will increase the metabolism of drugs
such as warfarin that are processed by this system.

(Choice A) Vaccination is an important prophylactic strategy for a

large population. The United States military currently vaccinates new


recruits entering basic training to avoid meningococcal outbreaks as

the carrier rate has been shown to increase from approximately 5% to up

to 4O% in army barracks. However, routine post exposure prophylaxis


with vaccine is not recommended for the following reasons:

1. Group B meningococcus is one of the most common serogroups

responsible for infections, but there is no effective vaccine available


for group B meningococci because this strain is considered to be poorly

immunogenic.

2. Children younger than two years of age respond poorly to the


capsular polysaccharide vaccine as their immune system are not well

adapted to mounting a response triggered by polysaccharide alone

without protein conjugation.


(Choices B and C) There is not a pilus protein vaccine or a

lipopolysaccharide vaccine currently available for Neisseria

meningitidis.
(Choice D) While penicillin is the preferred agent for treatment of N.

meningitidis infections, it is not useful for prophylaxis where

rifampin, followed by ceftriaxone, is the drug of choice. The reason


for this is that only rifampin and the third generation cephalosporins

have been shown to eliminate nasal carriage.


(Choice E) Sulfamethoxazole was historically used as chemoprophylaxis

for contacts of an index case of meningococcal disease, but presently

this would not be a correct choice due to the fact that N. meningitidis

26

USMLE WORLD STEP 1 PHARMACOLOGY

has developed widespread resistance to sulfonamides, so these drugs are

no longer used.

Educational Objective:

Rifampin is most typically used as chemoprophylaxis of meningococcal


meningitis. It must be prescribed to all close contacts of any patient

who has active disease within 2 weeks of diagnosis in order to be

effective. Vaccine is typically not used for post exposure prophylaxis:


it is used to develop population immunity in at-risk groups such as

military recruits, college freshmen and healthcare workers.

27

USMLE WORLD STEP 1 PHARMACOLOGY

Q NO 17: A 50-year-old man is brought to emergency room after developing

blurred vision while cutting several trees in his garden. His past medical
A.

history is insignificant and he is not taking any medications. He has no

history of illicit drug use. His temperature is 38.9°C (102°F), blood pressure
is 100/70mm Hg, pulse is 120/mm, and respirations are 22/mm. Physical

examination shows pushed skin and dry oral mucosa. Both pupils are dilated

and non-reactive to light. Which of the following drugs can potentially


reverse this patient’s condition?

Diazepam

B. Haloperidol

C. Benztropine
D. Physostigmine

E. Metoprolol

F. Atropine
G. Thiamine

H. Morphine

I. Naloxone

Explanation:

This patient most likely has Jimson Weed (Datura stramonium) poisoning.
This condition is also called as “Gardener’s mydriasis.” Jimson Weed

produces toxins (belladonna alkaloids) that possess strong


anticholinergic properties. Jimson weed and atropine poisoning are

strikingly similar. Blockade of visceral muscarinic receptors produces

the following effects:


1. Heart: diminished vagal tone at the SA node causes relative

tachycardia.

2. Blood vessels: vasoconstriction via muscarinic receptor blockade in


endothelial cells results in decreased nitric oxide synthesis. In spite

of this effect atropine poisoning is associated with cutaneous

flushing; the pathogenesis of this effect is unknown.


3. GI: delayed gastric emptying, decreased intestinal motility, and

secretion.

4. Respirators’: bronchodilatation.
5. GU: urinary retention via detrusor relaxation and contraction of the

external urethral sphincter.

6. Secretions: decreased lacrimation (dry eyes), salivation (dry mouth)


and sweating (dry and hot skin). Atropine decreases one’s ability to

sweat, contributing to hyperthermia.

7. Eye: mydriasis (dilated pupils) and cycloplegia (inability to focus


on the near objects blurry vision).

8. CNS: hallucinations, agitation and delirium.

Atropine’s antimuscarinic effects can be counteracted by increasing the


concentration of acetylcholine in the synaptic cleft. Increased

acetylcholine concentrations are produced by cholinesterase inhibitors

that suppress acetylcholine degradation. Physostigmine, a


cholinesterase inhibitor can be used for treatment of atropine

overdose.

(Choice A) Diazepam is a long-acting benzodiazepine that facilitates


GABA action by increasing the frequency of chloride channel opening.

Diazepam is used to treat seizures associated with atropine poisoning

but does not affect muscarinic cholinergic receptors.


(Choice B) Haloperidol is a neuroleptic drug that blocks dopamine

receptors in the CNS. It also has anticholinergic and antihistamine

properties.

28

USMLE WORLD STEP 1 PHARMACOLOGY

(Choice C) Benztropine is a centrally acting anti-cholinergic

medication used for treatment of idiopathic and drug induced


Parkinson’s disease. Its administration would increase the patient’s

symptoms.

(Choice E) Metoprolol is a selective β-adrenergic receptor antagonist.


It is used to treat angina acute coronary syndromes heart failure,

hypertension and arrhythmias. Metoprolol does not have any effect on

muscarinic cholinergic receptors.

Educational Objective:

The mnemonic for the clinical manifestations of atropine poisoning is:


“blind as a bat, mad as a hatter, red as a beet. Hot as a hare, dry as

a bone, the bowel and bladder lose their tone, and the heart runs

alone.” Atropine is a reversible cholinergic antagonist that acts


selectively on muscarinic receptors. Its effects can be reversed by

cholinesterase inhibitors (physostigmine).

29

USMLE WORLD STEP 1 PHARMACOLOGY

Q NO 18: A new drug that is used to treat hypertensive emergencies causes

arteriolar dilation. It also increases renal perfusion and promotes


A.

natriuresis. The drug described above is most similar with which of the

following agents?
Diazoxide

B. Nitroprusside

C. Hydralazine
D. Esmolol

E. Nicardipine

F. Fenoldopam

Explanation:

The management of hypertensive emergency requires immediate but gradual


blood pressure reduction over minutes to hours to minimize target organ

damage. Intravenous therapy is necessary to allow immediate effects and

the preferred and optimal parenteral agent will vary according to the
patient’s clinical situation and presence of coexisting conditions and

disease states.

Fenoldopam is a newer, novel intravenous agent that is a benzazepines


derivative of dopamine. In contrast to dopamine, itis a selective

dopamine-i receptor agonist with no effect on alpha or beta receptors.

Dopamine-i receptor stimulation activates adenylyl cyclase and raises


intracellular cyclic AMP, resulting in vasodilation of most arterial

beds, especially renal, mesenteric, and coronary beds. The main effect

is a significant reduction in systemic vascular resistance. Stimulation


of dopamine-receptors in the kidneys not only improves renal blood

flow, but also leads to increased sodium and water excretion. Thus

fenoldopam is the only available intravenous agent that improves renal


perfusion while it lowers blood pressure.

Fenoldopam is indicated for short term management of severe

hypertension and can be safely used in all hypertensive emergencies. It


may be exceptionally beneficial in patients with concomitant renal

insufficiency.

(Choices A and C) Although diazoxide and hydralazine are both arterial


vasodilators, neither agent therapeutically improves renal perfusion.

Both agents cause significant reflex sympathetic activation, resulting

in increased heart rate and contractility and extensive sodium and


fluid retention. They are usually considered third line agents and are

not recommended in hypertensive emergencies with aortic dissection.


Hydralazine is considered safe and useful in pregnancy related

hypertensive emergency.

(Choice B) Nitroprusside is a very potent direct acting arterial and


venous vasodilator. Due to its favorable pharmacokinetics profile

(quick onset and short duration of action), it is considered the most

effective agent for most cases of hypertensive emergency. Nitroprusside


causes slight reflex sympathetic activation, and thus can cause modest

tachycardia and sodium and fluid retention. Since nitroprusside is

metabolized to cyanide and thiocyanate, the primary limiting factor to


its use is the risk for cyanide toxicity with prolonged use, high

doses, and renal insufficiency.

(Choice D) Esmolol is a short acting selective beta-i receptor


antagonist that decreases heart rate contractility, and cardiac output.

It has an immediate onset of action and upon discontinuation of

esmolol; the effects are reversed within 20 minutes. Esmolol can cause
bradycardia, left ventricular dysfunction, and bronchospasm. It is

30

USMLE WORLD STEP 1 PHARMACOLOGY

mainly used in the setting of postoperative hypertension and may be

preferred in critical patients where rapid withdrawal of drug effects


is needed.

(Choice E) Nicardipine is a dihydropyridine calcium channel blocker

that works by blocking calcium channels in the vascular smooth muscle


and the myocardium, resulting in relaxation of smooth muscles and

coronary arteries. Because nicardipine causes significant arterial

vasodilation, it can cause mild to moderate tachycardia. Other side


effects include flushing, headache, and venous irritation. Since

nicardipine has a longer half life compared to other agents, the

hypotensive effect may be prolonged and rapid titration of the agent is


difficult.

Educational Objective:
Fenoldopam is a newer parenteral agent that is classified as a

selective dopamine-1 receptor agonist. It causes arteriolar dilation

and natriuresis leading to decreased systemic vascular resistance and


blood pressure reduction. Since fenoldopam is the only intravenous

agent that improves renal perfusion, it may be exceptionally beneficial

in hypertensive patients with concomitant renal insufficiency.

31

USMLE WORLD STEP 1 PHARMACOLOGY

Q NO 19: Isolates of M. tuberculosis obtained from a 43-year-old HI V-positive male

demonstrate resistance to a number of antibiotics. Which of the following best

explains resistance to streptomycin in these bacteria?

A. Decreased activity of bacterial catalase-peroxidase

B. Increased activity of enzymes involved in cell wall polysaccharide

synthesis
C. Altered structure of bacterial ribosomal proteins

D. Altered structure of enzymes involved in DNA winding-unwinding

E. Altered structure of enzymes involved in RNA synthesis

Explanation:

The treatment of active Mycobacterium tuberculosis infection is best


accomplished through the use of isoniazid and rifampin in addition to

any combination of streptomycin, pyrazinamide, ethambutol or

fluoroquinolone. Because streptomycin is one of the older drugs in the


aminoglycoside family, bacterial resistance to this antibiotic is

widespread. As a result streptomycin usage is currently limited to the

treatment of tuberculosis plague, and tularemia. Like all other


aminoglycosides, streptomycin can only be administered parenterally,

and it works by inhibiting the bacterial 30S ribosomal subunit

(ultimately preventing bacterial protein synthesis). Mutations of the


genes that encode ribosomal proteins are responsible for aminoglycoside

resistance because they modify the ribosomal binding sites for these

drugs.
(Choice A) Decreased activity of bacterial catalase-peroxidase is one

mechanism of mycobacterial resistance to isoniazid. Mycobacterial

catalase-peroxidase is required for the initial enzymatic conversion of


isoniazid to its active metabolite within the mycobacterial cells;

without this enzyme isoniazid is unable to inhibit mycobacterial

mycolic acid synthesis.


(Choice B) Increased activity of enzymes involved in cell wall

polysaccharide synthesis is the means by which mycobacteria develop


resistance to ethambutol. Ethambutol interferes specifically with

mycobacterial peptidoglycan cell wall synthesis through an unclear

mechanism that appears to differ from that of isoniazid. This drug is


ineffective against organisms other than mycobacteria.

(Choice D) Structural alteration of enzymes involved in DNA winding-

unwinding is the means by which many microorganisms become resistant to


fluoroquinolone antibiotics. Fluoroquinolones inhibit the bacterial

enzyme DNA gyrase (topoisomerase II).

(Choice E) Structural alteration of enzymes involved in RNA synthesis


is the mechanism through which organisms become resistant to rifampin.

Rifampin inhibits the bacterial DNA-dependent RNA polymerase, thereby

preventing the transcription of DNA into mRNA.

Educational Objective:

* The aminoglycoside streptomycin inhibits protein synthesis by


inactivating the 30S (small) ribosomal subunit.

• Decreased activity of bacterial catalase-peroxidase is one mechanism

of mycobacterial resistance to isoniazid.


* Structural alteration of enzymes involved in RNA synthesis (DNA-

dependent RNA polymerase) is the mechanism through which organisms

become resistant to rifampin.

32

USMLE WORLD STEP 1 PHARMACOLOGY

Q NO 20: A 60-year-old Caucasian male comes to your office with a 2-month

history of back pain that is not responsive to over-the-counter pain relievers.

The pain is worse at night and interferes with his sleep. Lumbar vertebrae

are tender to percussion, and the prostate is enlarged and firm. After a
thorough evaluation, you decide to proceed with leuprolide therapy. Which of

the following changes in testosterone (T) and dihydrotestosterone (DHT)

levels are most likely in this patient after initiation of the therapy?

A. Steady decrease in both T and DHT levels


B. First concordant increase, then concordant decrease in T and DHT

levels

C. Discordant decrease in DHT level


D. First discordant increase, then discordant decrease in DHT level

E. No change in T and DHT levels

Explanation:

Pulsatile release of gonadotropin-releasing hormone (GnRH) from the


hypothalamus is the natural state of human functioning causing release

of gonadotrophins from the pituitary that in turn stimulates release of

testosterone in a man. Testosterone is converted to DHT in target


tissues by the enzyme 5-alpha reductase. Alternatively if GnRH levels

are constantly elevated. Rather than pulsed the secretion of

luteinizing hormone (LH) and follicle-stimulating hormone (FSH) from


the pituitary will be suppressed.

A number of GnRH analogs such as leuprolide, have been generated by

amino acid substitutions—alterations that allow longer half lives and


increased activity. Leuprolide is a long-acting GnRH analog that causes

continuous GnRH activity. The result is ultimately a suppression of the

pituita—gonadal axis for the duration of treatment although there is a


brief period of initial stimulation sometimes called a “start up

flare.” An increase in gonadotrophin levels during the initial flare-up

period causes an increase in both testosterone and DHT levels (a


concordant increase). Soon the start up flare burns out and both

testosterone and DHT are typically suppressed to castrate levels. These

yew low levels of androgens are useful for the treatment of androgen-
dependent cancers such as prostate cancer.

(Choice A) With the use of a GnRH analog androgen levels are suppressed

after a transient increase so the “decrease only” option here cannot be


correct. Suppression of testosterone and DHT without an initial flare

is seen with the use of GnRH antagonists.

(Choices C and D) Leuprolide has no effect on 5-alpha reductase


activity. Therefore the changes in testosterone and DHT levels are

always concordant following leuprolide administration. Finasteride is a

5-alpha-reductase inhibitor used for the treatment of benign prostatic


hypertrophy. It prevents the conversion of testosterone to DHT, causing

a discordant decrease in DHT levels.

(Choice E) The purpose of administering a GnRH is to change androgen


levels (testosterone and DHT levels) so “no change” cannot possibly be

correct.

Educational Objective:

Leuprolide is a GnRH agonist that causes first a transient increase,

then a decrease in both testosterone and DHT levels. Finasteride causes


a discordant decrease in DHT level.

33

USMLE WORLD STEP 1 PHARMACOLOGY

Q NO 21: A 19-year-old Caucasian female is brought to the ER after a tonic-clonic

seizure. She was diagnosed with epilepsy several years ago and has been

successfully treated with phenytoin. Blood tests reveal her plasma phenytoin level

to be low. She states that she has been compliant with her medication. Which of
the following drugs may be responsible for this patient’s condition if co-

administered with phenytoin?

A.
B.
Amoxicillin
Rifampin

C. Metronidazole

D.
E.
Fluconazole
Cimetidine

F. Omeprazole
G. Oral contraceptives

Explanation:
As with many other medications phenytoin is metabolized by hepatic P450

oxidase. Plasma drug level, efficacy, and the severity of side effects

are directly influenced by the rate of metabolism of phenytoin. The


following features of phenytoin metabolism are important:

1. Hepatic hydroxylation of phenytoin is dose-dependent. When low doses

of phenytoin are administered, a sufficient number of enzyme molecules


are available, and the drug is eliminated rapidly. Higher doses of

phenytoin may fully saturate the enzyme; hence, increasing the dose

above the saturation limit causes an inability to metabolize the drug


and a rapid rise in plasma levels. Even a small increase in phenytoin

intake above the prescribed dose may lead to severe toxicity.

2. Phenytoin is an inducer of P450 oxidase. Induction of this enzyme


increases the consumption of many medications that are metabolized by

the liver, such as oral contraceptives. When the P450 oxidase system is

induced, the serum concentration of such medications will decrease,


thereby reducing efficacy.

3. Because phenytoin is metabolized by P450 enzymes its level if

affected when it is co-administered with other medications that induce


or inhibit the P450 system. The co-administration of phenytoin with

P450 inducers decreases concentration of phenytoin in plasma and

diminishes its effectiveness. Some P450 inducers are barbiturates


rifampin, carbamazepine, griseofulvin, and chronic alcohol consumption.

P450 inhibitors slow hepatic metabolism, which increases the toxicity

of phenytoin. Some of P450 inhibitors are isoniazid, cimetidine,


macrolides, azole antifungals, and grape juice.

(Choices A and F) Amoxicillin and omeprazole do not affect the hepatic

metabolism of phenytoin.
(Choices C and G) Both metronidazole and oral contraceptives are

metabolized by hepatic P450 oxidase, but these two drugs neither induce

nor inhibit the activity of P450 oxidase.


(Choices D and E) Fluconazole and cimetidine inhibit P450 enzymes and

increase the concentration of phenytoin in the serum.


Educational Objective: Phenytoin metabolism depends on the function of

hepatic P450 oxidases and is dose-dependent. Drugs that induce hepatic

microsomal enzymes (phenobarbital, carbamazepine, and rifampin) enhance


phenytoin metabolism and decrease its serum concentration. You should

know all of the commonly-prescribed medications that are metabolized by

the P450 system and you should know which drugs induce or inhibit the
function of these enzymes.

34

USMLE WORLD STEP 1 PHARMACOLOGY

Q NO 22: A 65-year-old male treated with sildenafil experiences significant

improvement in his sexual performance. The net intracellular effect of this


A.

drug is most similar to that of which of the following substances?

Insulin
B. Platelet-derived growth factor (PDGE)

C. lnterleukin-2 (IL-2)

D. Atrial natriuretic peptide


E. 1, 25-dihydroxjcholecalciferol

F. Gamma-aminobutyric acid (GABA)

Explanation:

Atrial natriuretic peptide (ANP) and nitric oxide (NO, endothelium

derived relaxing factor) are hormones that exert their intracellular


effects by binding a receptor protein that also has a guanylate cyclase

enzymatic ability. This is known as the cGMP second messenger system.

The cGMP second messenger system is similar to the cAMP second


messenger system in that hormone binding causes formation of a cyclic

nucleotide monophosphate from a nucleotide triphosphate. These systems

are also similar in that the cyclic nucleotide monophosphate is able to


activate a protein kinase to carry out the intracellular effects of the

hormone and also in that the effect of the hormone is terminated by

degradation of the cyclic nucleotide monophosphate by


phosphodiesterase. The cGMP system is different because the receptor

and guanylate cyclase are part of the same protein (as opposed to the

cAMP system, which consists of four distinct proteins). This receptor


guanylate cyclase protein exists in both transmembrane form (for ANP)

and in a free cytosolic form (for NO which can freely diffuse through

cell membranes.
Sildenafil causes a net increase in the intracellular cGMP

concentration by inhibiting cGMP phosphodiesterase in target cells.

This effect is similar to that of nitric oxide (NO) and atrial


natriuretic peptide (ANP) in that these hormones activate a cGMP second

messenger system upon binding to their specific receptors, and


activation of the cGMP second messenger system will also lead to

increased intracellular cGMP. It is important to note though that NO

and ANP increase intracellular cGMP by a different mechanism than that


of the drug sildenafil which inhibits breakdown of cGMP by inhibiting

phosphodiesterase. These hormones increase production of cGMP; they do

not decrease degradation of cGMP as sildenafil does.


(Choice A) Insulin acts by the tyrosine kinase second messenger system.

(Choice B) Platelet-derived growth factor (PDGF) is a factor that

stimulates cell growth and division including angiogenesis. It also


acts via a tyrosine kinase receptor.

(Choice C) lnterleukin-2 (IL-2) is a cytokine that primarily functions

in an autocrine manner whereby activated T lymphocytes provide


costimulus to themselves by secreting IL-2 and producing IL-2

receptors.

(Choice E) 1, 25-dihydrox’’cholecalciferol is the active form of


Vitamin D. Vitamin D freely crosses cell membranes and binds to

cytosolic receptors. The vitamin-receptor complex then crosses the

nuclear membrane to modify DNA transcription.


(Choice F) Gamma-aminobutyric acid (GABA) is an inhibitory

neurotransmitter that acts to hyperpolarize nerve cells by binding a

voltage gated ion channel.

35

USMLE WORLD STEP 1 PHARMACOLOGY

Educational Objective:

Sildenafil is a selective inhibitor of the cGMP phosphodiesterase, and


use of this drug will prevent degradation of cGMP leading to higher

intracellular levels. Nitric oxide and atrial natriuretic peptide act

via a cGMP second messenger system (NO being primarily responsible for
causing erection), and binding of these hormones to their receptors

will also increase intracellular cGMP concentrations.

36

USMLE WORLD STEP 1 PHARMACOLOGY

Q NO 23: A 67-year-old male comes to the physician’s office complaining of mouth

ulcers and fever. His past medical history is significant for long-term hypertension

and an episode of transient ischemic attack which occurred one month ago and

lasted 20 minutes. His complete blood count shows:

Hb 13.2 g/dL

WBC 1100/mm3
Platelet 180000/mm3

Which of the following drugs is most likely to be associated with this

patient’s condition?

A. Heparin

B. Argatroban
C. Warfarin

D. tPA

E. Aspirin
F. Ticlopidine

G. Abciximab

H. Cilostazol

37

USMLE WORLD STEP 1 PHARMACOLOGY

Explanation:

Antiplatelet drugs work by one of three basic mechanisms: 1) formation


of ligands (aspirin decreases thromboxane A2 formation), 2) blocking

interaction of ligands with receptors on platelets (clopidogrel and

ticlopidine work as ADP antagonists), or 3) interfering with


intracellular signaling (cilostazol and dipyridamole increase cAMP by

decreasing phosphodiesterase activity).

Ticlopidine and clopidogrel are useful in the treatment and prevention


of ischemic strokes, acute coronary syndrome and peripheral vascular

disease. These agents can be combined with aspirin to get an additive

antiplatelet effect, as the mechanism of action of ticlopidine and


clopidogrel is different from aspirin. Sometimes patients are allergic

to or cannot tolerate aspirin or clopidogrel (the two first line drugs)

and in those patients ticlopidine is the other option. Otherwise


ticlopidine is rarely used due to the occurrence of serious side

effects. Neutropenia is seen in about percent of patients on

ticlopidine and typically presents with fever and mouth ulcers. Though
this is rare, it is a serious complication and complete blood count

should be monitored biweekly for the first three months.

(Choice A) Heparin usually causes thrombocytopenia but not neutropenia.


(Choice G) Abciximab and other IIb/IIIa inhibitors are also associated

with significant thrombocytopenia typically within 24 hours. They are

typically used in acute coronary syndrome and after stent placement.


(Choices B, C, D, E and H) These drugs do not cause neutropenia.

Educational Objective:
Neutropenia is seen in about percent of patients on ticlopidine and

typically presents with fever and mouth ulcers. Though this is rare,

itis a serious complication and complete blood count should be


monitored biweekly for the first three months.

38

USMLE WORLD STEP 1 PHARMACOLOGY

Q NO 24: A 78-year-old male inpatient about to undergo bronchoscopy is

premedicated with intramuscular atropine and becomes acutely restless,


A.

disoriented, and combative. On physical examination, his pupils are


widely dilated and non-reactive to light. An EKO monitor shows sinus

tachycardia. Which of the following agents will reverse a/i of this patient’s

symptoms and signs:


Haloperidol

B. Physostigmine
C. Diazepam

D. Neostigmine

E. Edrophonium

Explanation:

Atropine can be administered prior to bronchoscopy to decrease


respiratory mucous secretions and promote bronchodilation. However,

this patient is now manifesting symptoms of ONS and peripheral anti-

muscarinic toxicity. In the elderly (patients 70+ years of age)


atropine’s half-life may be prolonged from its usual 3 hours to up to

10-30 hours due to reduced clearance, causing an increased

susceptibility to toxicity. An organic delirium or psychosis can result


from blockade of muscarinic receptors in the CNS. In severe atropine

overdose, CNS effects may progress to coma and respiratory failure.

Peripherally, atropine can cause: dry and flushed skin, hyperthermia


(atropine fever), mydriasis and cycloplegia, bronchodilation,

tachycardia constipation and urinary retention.

Physostigmine inhibits acetylcholinesterase both peripherally and


centrally (a tertiary amine, physostigmine is capable of crossing the

blood-brain barrier). The increased acetylcholine availability

counteracts atropine’s blockade of muscarinic cholinergic receptors.


(Choice A) The antipsychotic haloperidol may sedate this patient and

suppress his CNS symptoms, but it would not reverse the other

anticholinergic manifestations (e.g. mydriasis, tachycardia) of his


atropine toxicity.

(Choice C) Diazepam is sedating and is sometimes used to treat

atropine-induced agitation. However, it would not reverse the


peripheral antimuscarinic effects. (Choices D and E) These are

anticholinesterase drugs with actions similar to physostigmine.

However, they both have a quaternary ammonium structure that prevents


penetration of the blood-brain barrier at moderate doses. These drugs

would thus fail to alleviate the patient’s ONS symptoms.

Educational Objective:

The tertiary amine physostigmine can reverse both the CNS and
peripheral symptoms of severe atropine toxicity. The anticholinesterase

agents neostigmine and edrophonium have a quaternary ammonium structure

that limits CNS penetration.

39

USMLE WORLD STEP 1 PHARMACOLOGY

Q NO 25: A 36-year-old male is undergoing a major surgical procedure under

general anesthesia. A fluorinated inhaled anesthetic (isoflurane) is used to

achieve the desirable depth of central nervous system depression. An

increase in which of the following parameters is most likely to happen during


the anesthesia in this patient?

A. Glomerular filtration rate (GFR)

B. Effective renal plasma flow


C. Left ventricular ejection fraction (EP)

D. Hepatic blood flow

E. Cerebral blood flow

Explanation:

General anesthesia encompasses loss of consciousness, analgesia,


amnesia, skeletal muscle relaxation and inhibition of reflexes. This

complex of symptoms occurs due to the inhibition of electrical activity

of neurons. Most of inhalation anesthetics, barbiturates, and


benzodiazepines achieve CNS depression by influencing GABA receptors

and increasing the inhibitory action of GABA. Inhaled anesthetics have

also been shown to affect potassium channels in the neuronal membranes


and lock them in the state of hyperpolarization. They may also affect

nicotinic and glycine receptors.

Apart from their desirable action on CNSI inhalation anesthetics affect


almost all organ systems of the body.

1. Cardiovascular effects include myocardial depression that leads to a

decrease in cardiac output and an increase in atrial and ventricular


pressures. Hypotension associated with fluorinated anesthetics is a

result of decrease in cardiac output.

2. Respiratory: all inhalation anesthetics, except nitrous oxide, are


respiratory depressants. They decrease tidal volume and minute

ventilation and cause hypercapnia. Another undesirable effect is the

suppression of mucociliary clearance, which may predispose to


postoperative atelectasis. Halothane and sevoflurane have

bronchodilation properties and are preferred in patients with asthma.

3. In the brain, fluorinated anesthetics decrease vascular resistance


and lead to an increase in cerebral blood flow. It is an undesirable

effect as it results in increased intracranial pressure.

4. In kidneys, inhaled anesthetics decrease glomerular filtration rate,


increase renal vascular resistance and decrease renal plasma flow.

5. Fluorinated anesthetics decrease hepatic blood flow.

(Choices A and B) Renal function in general anesthesia is characterized


by decrease in glomerular filtration rate and decrease in renal plasma

flow.
(Choice C) Fluorinated anesthetics are myocardial depressants. They

decrease cardiac output which leads to hypotension.

(Choice D) Fluorinated anesthetics decrease hepatic blood flow.

Educational Objective:

Almost all volatile anesthetics increase cerebral blood flow. It is an


undesirable effect as it results in increased ICP. Other important

effects of inhalation anesthetics are myocardial depression,

hypotension, respiratory, depression and decreased renal function.

40

USMLE WORLD STEP 1 PHARMACOLOGY

Q NO 26: A 33-year-old Caucasian female begins treatment with the abortifacient

mifepristone six weeks after her last menstrual period. She experiences

abdominal cramps, nausea and vaginal bleeding soon after initiating therapy.

Which of the following effects is most likely responsible for this patient’s
symptoms?

A. Anti-progestin
B. Inhibition of progesterone synthesis

C. Prostaglandin agonist

D. Inhibition of cell division


E. Anti-glucocorticoid

F. Anti-mineralocorticoid

Explanation:

Mifepristone (previously called RU-486) is an abortifacient approved by

the EDA for clinical use. It can be used for therapeutic abortions up
to 49 days after conception. Mifepristone is a progesterone antagonist

with an affinity for the progesterone receptor five times that of

natural progesterone.
Progesterone is necessary for implantation and maintenance of

pregnancy. When its effects are antagonized by mifepristone, there is

decidual necrosis and expulsion of the products of conception. The


success rate of mifepristone at inducing medical termination of

pregnancy is about 80%: however, the success rate is higher when

mifepristone is combined with the prostaglandin analog misoprostol.


(Choice B) Epostane and trilostane inhibit the formation of

progesterone from pregnenolone by inhibiting the enzyme 3-β-

hydroxysteroid dehydrogenase. These agents have been studied for use in


the medical termination of pregnancy, but their success rates have been

quite limited.

(Choice C) Endogenous prostaglandin causes uterine contraction and


cervical dilatation, important steps in the process of labor.

Prostaglandin analogues have been used for years in the medical

termination of pregnancy. The prostaglandin-E1 analogue, misoprostol,


is available for clinical use in combination with the abortifacient,

mifepristone. (Misoprostol has limited success in pregnancy termination

when used alone, but enhances the efficacy of mifepristone.)


Additionally, misoprostol is sometimes used for the prevention of

NSAID-induced gastrointestinal ulceration.

(Choice D) Folic acid antagonists like methotrexate inhibit trophoblast


division thus decreasing trophoblast survival, hindering implantation

and encouraging expulsion. In studies methotrexate has a higher success


rate when combined with vaginal misoprostol. Methotrexate is also used

for the treatment of ectopic pregnancy.

(Choices E and F) Mifepristone has mild anti-glucocorticoid and anti-


mineralocorticoid effects. However, these effects do not contribute to

its utility as an agent for the medical termination of pregnancy.

Educational Objective:

Mifepristone is an anti-progestin agent that can be used to terminate

early pregnancy. The prostaglandin-E1 analogue, misoprostol, is


available for clinical use in combination with the abortifacient,

mifepristone.

41

USMLE WORLD STEP 1 PHARMACOLOGY

Q NO 27: A 65-year-old Caucasian male is undergoing rehabilitation after a stroke.

You prescribe diazepam to this patient in order to decrease muscle spasticity of

involved extremities. The patient should be cautioned to avoid which of the

following drugs?

A. Chlorpheniramine

B. Loratadine
C. Acetaminophen

D. Ranitidine

E. Omeprazole
F. Folic acid

Explanation:
Diazepam is a long-acting benzodiazepine indicated as follows:

1. As an anxiolytic, it is effective for the management of generalized

anxiety disorder and panic attacks.


2. The sedative-hypnotic effect of diazepam is useful for the short-

term treatment of insomnia.

3. As an anticonvulsant, diazepam is indicated for tonic-clonic


seizures. Itis a first-line agent (along with chlordiazepoxide) for the

treatment of seizures associated with alcohol withdrawal.

4. As a muscle relaxant, diazepam can stop the spasticity caused by


upper motor neuron disorders (multiple sclerosis, strokes, and spinal

cord trauma) and tetanus.

5. Due to its sedative and amnestic properties, diazepam is used as an


adjunct to anesthesia.

The most common side effect of diazepam is sedation. Diazepam may also

impair coordination and balance (sometimes avoided in the elderly for


fear of falls) decrease memory and concentration, and produce

confusion. These effects are amplified by the co-administration of

other CNS-depressants. As a class, all benzodiazepines should be


excluded from use in conjunction with alcohol, barbiturates,

neuroleptics, or 1st generation antihistamines.

Among drugs listed in the answer choices, only chlorpheniramine causes


sedation and should not be used with diazepam. Chlorpheniramine is a

15t generation antihistamine that acts by blocking both central and

peripheral Hi receptors. Chlorpheniramine and other 15t generation


antihistamines can be very sedating because they easily penetrate the

blood-brain barrier and accumulate in the CNS. The most sedating of


this generation of antihistamines are diphenhydramine (Benadryl),

promethazine, and hydroxyzine. Despite the associated sedation these

antihistamines are often still used to prevent/treat allergic reactions


and motion sickness and as antiemetics.

(Choice B) Loratadine is a second-generation antihistamine that blocks

peripheral H1 histamine receptors. It does not enter the CNS and does
not cause drowsiness.

Educational Objective:
First generation H1-histamine receptor antagonists including

diphenhydramine and chlorpheniramine can cause significant sedation

especially when used with other medications that cause CNS depression
(such as benzodiazepines).

42

USMLE WORLD STEP 1 PHARMACOLOGY

Q NO 28: In a patient with severely impaired left ventricular function, IV digoxin

administration increases cardiac output and decreases right atrial pressure and

pulmonary capillary wedge pressure. Which of the following is the initial cellular

event triggering this response to digoxin?

A. Decreased sodium efflux

B. Increased calcium influx


C. Increased potassium influx

D. Increased cAMP concentration

E. Troponin sensitization to calcium

Explanation:

Digoxin inhibits Na-K-ATPase, causing an increase in intracellular

sodium, which in turn decreases the transmembrane sodium gradient that


acts as the driving force of the Na-Ca transporter. This leads to

calcium accumulation in the cell. In excitation-contraction coupling,

when a cardiac myocyte is depolarized sodium and calcium enter the cell
and potassium exits. This depolarization causes the release of a large

amount of calcium from the sarcoplasmic reticulum within the cell, and

this high calcium concentration allows the binding of calcium to


troponin C and subsequent actin-myosin cross bridge formation and

muscle contraction. The greater the intracellular calcium

concentration, the greater the contraction will be. Cardiac relaxation


then occurs via sequestration of calcium back into the sarcoplasmic

reticulum.

43

USMLE WORLD STEP 1 PHARMACOLOGY

Thus calcium gluconate should not be given to patients for hyperkalemia

in the setting of digoxin toxicity. Digoxin and potassium compete with


each other for Na-K- ATPase; thus digoxin toxicity results in

hyperkalemia. Hypokalemia worsens digoxin toxicity.

(Choice A) Digoxin exerts its effect on the Na-K-ATPase thus causing


decreased sodium efflux from the cardiac myocyte.

(Choice B) Digoxin causes decreased calcium efflux, not increased

calcium influx. High levels of intracellular calcium are necessary for


excitation-contraction coupling leading to contraction of the myocyte.

(Choice C) The Na-K-ATPase causes K influx and Na efflux. Digoxin acts

to inhibit the Na-K-ATPase.


(Choice D) Cyclic AMP (cAMP) is an important second messenger in

excitation-contraction coupling though digoxin has no effect on the

production of cAMP. cAMP is formed from AlP by adenylyl cyclase, the


activity of which is stimulated by beta agonists. cAMP increases the

conductance of the calcium channels in the sarcoplasmic reticulum. As a

result, more calcium can enter the cell and strengthen the force of
contraction.

(Choice E) Digoxin does not cause increased troponin sensitivity to

calcium. New drugs are being developed to increase the sensitivity of


calcium.

Educational Objective:
The mechanism of action of digitalis is the inhibition of the Na-K-

ATPase in cardiac pacemaker cells leading to AV nodal blockade

(increased diastolic filling time for greater contraction by the Frank-


Starling mechanism) and increased contractility from increased

intracellular calcium.

44

USMLE WORLD STEP 1 PHARMACOLOGY

Q NO 29: A 65-year-old male presents to your office with impaired vision. He says

that the problems with his vision started a long time ago and gradually progressed

overtime. His past medical history is significant for long-standing schizophrenia

effectively controlled by medications. Ophthalmoscopy suggests the diagnosis of


retinitis pigmentosa in this patient. Which of the following antipsychotic agents has

most likely been used in this patient?

A.
B.
Chlorpromazine
Haloperidol
Cornealdeposits

C. Ziprasidone

D.
E.
Thioridazine
Olanzapine

F. Clozapine

Explanation:

Thioridazine is an anti-psychotic medication that may cause retinal


deposits that resemble retinitis pigmentosa.

(Choice A) Chlorpromazine is associated with corneal deposits.

(Choice B) Haloperidol is associated with extrapyramidal symptoms.


(Choice C) Ziprasidone is associated with prolonged QT.

(Choice E) Olanzapine is associated with weight gain.

(Choice F) Clozapine is associated with agranulocytosis and seizures.

Educational Objective:

Thioridazine causes retinal deposits that resemble retinitis


pigmentosa. Chlorpromazine is associated with corneal deposits.

45

USMLE WORLD STEP 1 PHARMACOLOGY

Q NO 30: A new antiarrhythmic drug (Drug A) is tested in a series of experiments.

These experiments measure ion flow changes in cardiac muscle cells as the result

of the action of the drug with inward ion flow represented as a positive value and

outward ion flow represented as a negative value. The following chart has been
obtained. Drug A has an antiarrhythmic effect most similar to which of the

following drugs?

A. Quinidine

B. Lidocaine
C. Flecainide

D. Dofetilide

E. Verapamil
F. Esmolol

G. Digoxin

H. Adenosine

46

USMLE WORLD STEP 1 PHARMACOLOGY

Explanation:

An effect typical of the class Ill antiarrhythmic agents is


demonstrated on this graph where potassium efflux from the cell

(negative black line) as well as the length of the action potential is

prolonged while the movement of sodium (positive blue line) and calcium
(positive black line) are relatively unaffected. Dofetilide (Choice D),

ibutilide, amiodarone and sotalol are representative of class III

agents.
(Choice A) Quinidine is a class IA antiarrhythmic agent. It blocks

sodium channels and has the effect on phase 0 depolarization and phase

3 repolarization.
(Choice B) Lidocaine is a class IB antiarrhythmic agent and acts by

blocking sodium channels only in veri rapidly depolarizing cells. It

does not have much effect on potassium efflux.


(Choice C) Flecainide is a class IC antiarrhythmic agent. It blocks

sodium channels and primarily acts by slowing phase 0 depolarization.

(Choice E) Verapamil is a class IV antiarrhythmic that acts by blocking


calcium channels primarily in cells of the AV node and pacemaker cells.

They would have no effect on the graph shown above because the above

graph represents ion flow in ventricular myocytes.


(Choice F) Esmolol is a class II antiarrhythmic that acts by blocking

beta-adrenergic receptors. The beta-blockers primarily act to slow

phase 4 depolarization in cells displaying automaticity and slow


conduction through the AV node. They would have no effect on ion flow

in the ventricular myocardium illustrated in the above graph.

(Choices G and H) Digoxin and adenosine exert their antiarrhythmic


effects by slowing conduction through the AV node though by different

mechanisms. Because of this they would have no effect on the ion flow

represented in the above graph.

Educational Objective:
Class Ill antiarrhythmics such as amiodarone, sotalol, ibutilide and

dofetilide will slow potassium efflux from the ventricular myocyte,

prolong repolarization and prolong the refractory period.

47

USMLE WORLD STEP 1 PHARMACOLOGY

Q NO 31: A 60-year-old male presents with worsening fatigue and dyspnea on

exertion that have recently limited his daily activities. He also notes that

recently he has required four pillows to sleep at night. His past medical history

is significant for a myocardial infarction 2 years ago. Physical examination


reveals bilateral lung crackles and lower extremity edema. Chest x-ray shows

cardiomegaly. He is admitted to the hospital, where his blood pressure is

stable at 150-160/90-100. His hypertension should be treated with which of


the following agents:

A. Hydrochlorothiazide

B. Terazosin

C. Lisinopril
D. Amlodipine

E. Minoxidil

F. Methyldopa

Explanation:

This patient is persistently hypertensive in the setting of a previous


myocardial infraction and current congestive heart failure. Of the

antihypertensives listed, only ACE inhibitors (lisinopril) have been

shown to inhibit the chronic angiotensin Il-mediated left ventricular


hypertrophy and remodeling that occurs in association with myocardial

failure. Overtime, such remodeling diminishes contractile function. A


beta blocker would also be beneficial for this patient.

(Choice A) A thiazide diuretic is the best initial treatment for

essential hypertension in a patient without congestive heart failure or


diabetes. In patients with CH or diabetes. ACE inhibitors should be

used instead.

(Choice B) Terazosin is sometimes chosen for patients with both


hypertension and benign prostatic hypertrophy. (Choices D — F) These

agents are considered second- or third-line for the treatment of

hypertension.

Educational Objective:

In patients with hypertension and chronic ischemic myocardial failure,


ACE inhibitors are considered first-line treatment, as they inhibit

myocardial remodeling and the associated deterioration of ventricular

contractile function in addition to reducing blood pressure. A beta


blocker would also be beneficial for these patients.

48

USMLE WORLD STEP 1 PHARMACOLOGY

Q NO 32: A 23-year-old mildly obese woman is treated for infertility. Her menstrual

cycles are irregular occurring once every two to three months. Examination shows

hirsutism. Once treatment is started, her serum progesterone level increases

sharply and secretory changes are noted on endometrial sampling. Which of the
following agents has been most likely used in this patient?

A. Progesterone antagonist
B. Estrogen antagonist

C. Androgen antagonist

D. Aromatase inhibitor
E. GnRH antagonist

Explanation:
Obesity, oligomenorrhea, hyperandrogenism (hirsutism, acne) and

infertility in a young woman are suggestive of polycystic ovarian

syndrome (PCOS, Stein-Leventhal syndrome). This disease is


characterized by increased circulating androgens and LH, and a high

LH/FSH ratio. Hypersecretion of LH by the anterior pituitary is

considered a primary abnormality in PCOS. Increased LH stimulates


secretion of androgens by ovarian theca cells. Elevated LH and androgen

levels coupled with decreased ESH levels lead to dysregulation of the

menstrual cycle and anovulation due to failure of follicular


maturation. Decreased frequency of ovulation in PCOS causes menstrual

irregularities (oligomenorrhea) and infertility. Unopposed estrogen

influence on the endometrium leads to endometrial proliferation and an


increased risk of endometrial carcinoma. Other complications of PCOS

include diabetes mellitus due to peripheral insulin resistance and an

increased risk of cardiovascular disease due to elevated triglycerides


and cholesterol.

Infertility in PODS is treated with clomiphene and weight loss.

Clomiphene structurally resembles estrogen and reacts with estrogen


receptors in the ONS (Choice B). When estrogens are secreted in

sufficient quantities, they elicit negative feedback on the

hypothalamus and inhibit production of gonadotropin-releasing hormone


(GnRH). Clomiphene blocks these receptor sites thereby suppressing

negative feedback on the hypothalamohypophysial axis by estrogen. The

result of clomiphene therapy is continued secretion of gonadotropins


despite normal or elevated estrogen levels. Increased amounts of ESH

and LH ultimately stimulate ovulation.

(Choice A) Mifepristone (RU486) is a progesterone antagonist. If


administered to pregnant women, this drug interferes with the action of

progesterone causing failure of maintenance of the endometrium and


sensitization of the uterus to procontractile hormones such as PGE-2

thereby inducing abortion.

(Choice C)The androgen antagonists cyproterone and flutamide


competitively inhibit the action of androgens by interacting with

receptors on the target cells. Flutamide is used occasionally for the

treatment of prostate cancer though GnRH agonists are preferred, while


cyproterone is indicated to treat hirsute females.

(Choice D) Aromatase inhibitors include anastrozole and letrozole. They

inhibit peripheral conversion of androgens into estrogens and are used


in women with estrogen-receptor-positive breast cancer.

(Choice E) GnRH agonists include leuprolide and goserelin. Continuous

administration of these drugs inhibits production of gonadotropin-


releasing hormone by the hypothalamus by causing downregulation of the

49

USMLE WORLD STEP 1 PHARMACOLOGY

GnRH receptor. They are indicated in prostate cancer, precocious

puberty and endometriosis.

Educational Objective:

Obesity, oligomenorrhea and infertility are seen in patients with


polycystic ovarian syndrome (PODS). Infertility in PODS occurs due to

anovulation resulting from abnormal FSH and LH levels. Clomiphene is an

estrogen antagonist that increases secretion of gonadotropins and


stimulates ovulation.

50

USMLE WORLD STEP 1 PHARMACOLOGY

Q NO 33: A 66-year-old Caucasian male was recently diagnosed with Parkinson

disease. His current medication is levodopa/carbidopa. He presents to your

office complaining of increased motion restriction although he reports that he is

highly compliant with his medications. His other medical problems include
peptic ulcer disease hypertension, and osteoarthritis of his right knee. Which of

the following medications may be responsible for this patient’s restriction of

motion?

A. Beta-blockers for hypertension

B. Cimetidine for peptic ulcer disease

C.
D.
Ibuprofen for osteoarthritis
A “baby” aspirin for stroke prevention metabolism

E. An over-the-counter multivitamin
delevodopa B 6 increases theperipheral
Explanation:

Most over-the-counter vitamins contain B6. Vitamin B6 supplementation


should not be taken by those on levodopa therapy, however, because B6

increases the peripheral metabolism of levodopa and decreases its

effectiveness. The more peripheral conversion of levodopa there is, the


less levodopa enters the central nervous system.

(Choices A, C, and D) Beta-blockers ibuprofen and aspirin do not

directly interact with levodopa/carbidopa.


(Choice B) Cimetidine is an H2-receptor antagonist that decreases

hydrochloric gastric acid production. Cimetidine may cause

gynecomastia, a very frequently-asked USMLE Step 1 question. Cimetidine


also inhibits the hepatic cytochrome enzymatic system CYT P450 and may

increase levels of warfarin, Perryton, propanolol, metoprolol,

quinidine, and theophylline.

Educational Objective:

Vitamin B6 increases the peripheral metabolism of levodopa, which


decreases its effectiveness.

51

USMLE WORLD STEP 1 PHARMACOLOGY

Q NO 34: A 47-year-old Caucasian male undergoing treatment with amphotericin B

for disseminated histoplasmosis complains of weakness and palpitations. An ECG

reveals frequent premature ventricular beats. If caused by drug toxicity, these new

signs and symptoms are most likely related to:

A. Cardiac muscle cell damage

B. Bone marrow suppression


C. Renal tubular dysfunction

D. Hepatocyte necrosis

E. Pulmonary hypertension

Amphotericin B is the drug of choice for many systemic mycoses. It is

also the most toxic antifungal medication. The most dangerous adverse
effect of amphotericin B is its nephrotoxicity. Amphotericin B causes

both a decrease in the glomerular filtration rate and direct toxic

effects on the tubular epithelium. Anemia and electrolyte abnormalities


may stem from amphotericin B-associated nephrotoxicity. Hypokalemia and

hypomagnesemia are common, due to an increase in the membrane

permeability of the distal tubule. Hypokalemia can cause weakness and


arrhythmias. ECG findings in hypokalemia include T-wave flattening, ST-

segment depression, prominent U waves, and premature atrial and

ventricular contractions. Profound hypokalemia can cause ventricular


tachycardia or fibrillation.

(Choice A) Doxorubicin and daunorubicin are associated with

irreversible dose-dependent cardiotoxicity. This is not a side effect


of amphotericin B.

(Choice B) Bone marrow suppression is associated with chloramphenicol,

zidovudine, phenylbutazone, and gold- containing medications. Bone


marrow suppression would not cause ECG changes.

(Choice D) Acetaminophen and halothane are examples of medications that

can cause liver necrosis. Amphotericin B is not hepatotoxic.


(Choice E) Busulfan and bleomycin are examples of the drugs that cause

pulmonary fibrosis and can lead to pulmonary hypertension.

Educational Objective:

Hypokalemia and hypomagnesemia are common electrolyte disturbances in

patients undergoing treatment with amphotericin B. Hypokalemia and


hypomagnesemia reflect an increase in distal tubular membrane

permeability.

52

USMLE WORLD STEP 1 PHARMACOLOGY

Q NO 35: A 65-year-old female with ovarian cancer is being treated with cisplatin-

based chemotherapy. Which of the following prophylactic measures would most

likely limit the toxicity associated with this chemotherapy regimen?

A. N-acetylcysteine

B. Folinic acid

C. Filgrastim
D. Fomepizole

E. Dexrazoxane

F. Amifostine

Explanation:

Cisplatin is a platinum-containing compound that exerts its


chemotherapeutic effect by forming a reactive oxygen species that can

form DNA cross links. The most prominent adverse effect associated with

use of cisplatin is nephrotoxicity. This drug causes acute tubular


injury, and a significant percentage of patients will develop mild

renal insufficiency after the first course of therapy if preventative

measures are not taken.


Amifostine is a thiol-based cytoprotective free-radical scavenging

agent used to decrease the cumulative nephrotoxicity associated with

platinum-containing agents, thereby disallowing reaction with the renal


tubules. Another preventative measure is establishing a chloride

diuresis (via intravenous normal saline) because cisplatin stays in a

nonreactive state when in a higher chloride concentration.


(Choice A) N-acetylcysteine (NAC) works in acetaminophen overdose by

enhancing glutathione production and conjugation of the toxic NAPQ

metabolite. NAC is also used as a mucolytic agent in patients with


influenza, bronchitis, and cystic fibrosis. Additionally, in patients

with renal insufficiency who require an IV-contrast CT scan. NAG

prevents radiocontrast-induced nephropathy.


(Choice B) Leucovorin, or folinic acid, is a drug used in the treatment

of methotrexate overdose.
(Choice C) Filgrastim is granulocyte colony-stimulating factor (G-CSF)

analog, produced by recombinant DNA technology, used to stimulate the

proliferation and differentiation of granulocytes.


(Choice D) Fomepizole is used as an antidote in suspected methanol

(rubbing alcohol) or ethylene glycol (antifreeze) poisoning. It is a

competitive antagonist of alcohol dehydrogenase, and thereby prevents


the conversion of methanol and ethylene glycol into their toxic

metabolites.

(Choice E) Dexrazoxane is an iron-chelating agent that can help prevent


anthracycline-induced (i.e. doxorubicin) cardiotoxicity (CHF).

Educational Objective:
Aggressive hydration and amifostine should be utilized to prevent

nephrotoxicity in patients receiving a platinum- based chemotherapeutic

regimen.

53

USMLE WORLD STEP 1 PHARMACOLOGY

Q NO 36: A 48-year-old male is diagnosed with diabetes mellitus. After evaluating

various treatment modalities, you recommend monotherapy with pioglitazone.

Which of the following should be checked periodically if the patient agrees to

proceed with pioglitazone therapy?

A. Thyroid function tests

B. BUN and creatinine


C. Liver function tests

D. CBC with differential

E. Ejection fraction

Explanation:

Thiazolidinediones (TZD5) are a new group of antidiabetic medication


that reduces insulin resistance. Troglitazone was the first TZD

released for clinical use; however it was withdrawn after two years

because of a high incidence in severe hepatotoxicity. The newer TZDs,


pioglitazone and rosiglitazone, have a lower incidence of

hepatotoxicity. However, liver function tests should be monitored in

all patients started on TZDs. Liver function tests— chiefly alanine


aminotransferase (ALT)—are measured at baseline, 12 weeks after

starting treatment, and periodically thereafter. In addition to

treating type 2 diabetes TZDs are also very useful medications in the
treatment of metabolic syndrome, nonalcoholic fatly liver disease (also

called “NASH”), and polycystic ovarian disease. Insulin resistance is

crucial in the development of these disorders.


(Choice A) Thyroid function tests are not required for any antidiabetic

medication. Thyroid function tests are routinely performed on patients

taking lithium and amiodarone.


(Choice B) Renal failure is an absolute contraindication for metformin

use. Creatinine is typically monitored on a yearly basis in patients on

metformin therapy because its use in the face of renal failure


increases the risk of lactic acidosis. A creatinine of 1.5 mg/dL in

males and 1.4 mg/dL in females is contraindication for the use of

metformin.
(Choice D) Hematological side effects are very rarely seen with

treatment with oral antidiabetic medications, and a CBC is not

routinely required.
(Choice E)The side effects of TZDs include fluid retention and the

aggravation of congestive heart failure. Patients on concurrent insulin


and TZD therapy have the highest risk for congestive heart failure.

NYHA class Ill and class IV heart failure is an absolute

contraindication for TZD use. For class land II heart failure, the
decision whether to start TZD is based on clinical evaluation.

Determining ejection fraction is not mandatory before or during

treatment with TZDs.

Educational Objective:

1. Because troglitazone was with drawn from the market due to


hepatotoxicity, periodic liver function tests are now recommended in

all patients treated with thiazolidinediones.

2. The other important side effect of TZDs includes fluid retention


which can exacerbate congestive heart failure in patients with

underlying cardiac dysfunction.

54

USMLE WORLD STEP 1 PHARMACOLOGY

Q NO 37: A group of investigators is performing an experiment designed to assess

genetic variability in drug biotransformation. A fixed dose of isoniazid is given to a

group of volunteers and the plasma drug concentration is measured four hours

following administration of the drug. The following distribution of plasma drug


concentration in these subjects is obtained. Variation of which of the following

processes provides the best explanation for the shaded area of the curve?

A.
B.
Methylation
Glucuronidation

C. Hydrolysis
D. Acetylation

E. Amine oxidation

Explanation:

The rate and extent of

drug metabolism normally varies from person to person. These slight


interpersonal variations in the ability to metabolize drugs are

typically reflected graphically by a unimodal distribution, usually in

the shape of a bell curve, when plasma levels of drug are measured at a
fixed time following a fixed dose of drug. This is the method that was

used to generate the graph given in the question stem. With most drugs,

the majority of people fall within one standard deviation and 95% of
people fall within two standard deviations of the population mean of

plasma concentration. A single peak in this type of graph indicates

that the population being tested possesses a similar genetic drug


metabolizing capacity.

A bimodal (discontinuous polymorphic) curve, as shown in the question

stem, results from the presence of two apparently distinct groups


within the study population and suggests a pharmacogenetics

polymorphism in drug metabolizing capacity. In other words, the two

peaks indicate two sets of responders to the drug within the


population: one that rapidly converts the drug into its metabolite

(considered normal) and another that converts the drug slowly, leading

to accumulation of the original drug in the plasma.


Isoniazid is metabolized by acetylation to N-acetyl-isoniazid in the

hepatic microsomal system by the enzyme N-acetyl transferase and is

subsequently excreted in the urine. The first and second peaks in the
above graph represent fast and slow acetylators, respectively. Slow

acetylators of isoniazid also metabolize (acetylate) dapsone,

hydralazine, and procainamide slowly, causing accumulation of these


drugs as well. Slow acetylators of these drugs are at increased risk of

toxic effects, while fast acetylators may require much higher


therapeutic doses to achieve a therapeutic effect.

(Choice A) Methylation is an important drug biotransformation method to

consider when prescribing drugs such as azathioprine and 6-


mercaptopurine, drugs used in the treatment of some inflammatory

disorders of the bowel and skin.

(Choice B) Glucuronidation is a biotransformation pathway utilized for


the metabolism of numerous drugs as well as endogenous substances such

55

USMLE WORLD STEP 1 PHARMACOLOGY

as bilirubin. No bimodality has been described with this pathway, but

conditions such as Gilbert syndrome involve dysfunction of the


glucuronyl transferase system that can lead to toxic accumulation of

some drugs.

(Choice C) Hydrolysis occurs with enzymes such as esterases and


amidases. Isoniazid is not metabolized by this pathway, and hydrolysis

does not exhibit polymorphic metabolism.

(Choice E) Amine oxidation is usually undertaken by monoamine oxidases


or by cytochrome oxidative deamination. Neither process metabolizes

isoniazid nor exhibits bimodality.

Educational Objective:

Isoniazid is metabolized by acetylation. The speed with which a patient

is able to acetylate drugs depends on whether they are genetically


“fast” or “slow” acetylators. The presence of fast and slow acetylators

within the same population results in a bimodal distribution of the

speed of isoniazid metabolism. Slow acetylators are at increased risk


of adverse side effects.

56

USMLE WORLD STEP 1 PHARMACOLOGY

Q NO 38: A 43-year-old male with advanced HIV infection is admitted for impaired

vision. On the third day of hospitalization, he develops generalized seizures and is

found to be hypocalcemic and hypomagnesemic. Which of the following drugs is

most likely responsible for the observed effects?

A. Acyclovir

B. Ganciclovir
C. Foscarnet

D. Indinavir

E. Lamivudine

Explanation:

Visual impairment in an HIV-infected patient is most commonly secondary


to cytomegalovirus-induced retinitis. Treatment options for

cytomegalovirus-induced retinitis include ganciclovir, foscarnet, and

cidofovir.
Foscarnet is a pyrophosphate analog that can chelate calcium. Moreover

foscarnet-induced renal wasting of magnesium may lead to hypomagnesemia

and a reduction in the release of parathyroid hormone, which


contributes to the hypocalcemic state. Both hypocalcemia and

hypomagnesemia can promote seizures. Thus, this patient was most likely

started on foscarnet during his current hospitalization.


(Choice A) Major acyclovir toxicities include crystal nephropathy and

neurotoxicity that manifests as delirium and/or tremor.

(Choice B) Ganciclovir is the first-line drug for cytomegalovirus-


induced retinitis. However ganciclovir is associated with severe

neutropenia. Patients with advanced HIV who are on zidovudine may

already have bone marrow suppression: consequently they cannot be


started on ganciclovir because of the potential for worsening

neutropenia.

(Choice D) Indinavir is a protease inhibitor. As a class effect


protease inhibitors can cause fat redistribution, insulin resistance

(hyperglycemia), and hypertriglyceridemia. Adequate hydration is

recommended because indinavir is also associated with kidney stones.


(Choice E) Lamivudine is a nucleoside reverse transcriptase inhibitor

(NRTI). Side effects are unusual with lamivudine, in contrast with the

other NRTIs in the class. Lamivudine is occasionally associated with


peripheral neuropathy and lactic acidosis.

Educational Objective:

Foscarnet is an analog of pyrophosphate that can chelate calcium and

promote nephrotoxic renal magnesium wasting. These toxicities can


result in symptomatic hypocalcemia and hypomagnesemia.

57

USMLE WORLD STEP 1 PHARMACOLOGY

Q NO 39: After a series of pharmacokinetic studies, it is determined that a newly

developed non-opioid cough suppressant has a volume of distribution of 10 L and

clearance of 7 L/h for a healthy 70kg man. How long would it take for 75% of

this drug to be excreted after a single dose?

A. 1 hour

B. 1.5 hours
C. 2 hours

D. 2.5 hours

E. 3 hours

Explanation:

The half-life (t1/2) of a drug is the time period required for the
plasma concentration of a drug to decrease by 50%. This definition

assumes a one-compartment model with linear pharmacokinetics (as

opposed to multi-compartment models, which undergo separate


distribution and elimination phases). The half-life of a drug depends

on its physicochemical properties such as lipid solubility, protein

binding, volume of distribution, and metabolic conversion or


elimination pathways. The half-life of a drug after administration of a

single dose can be calculated from the volume of distribution and the

clearance rate as follows:


t1/2=(Vd * ln 2) / CL

Where Vd is the volume of distribution, CL is the clearance and “In 2”

refers to the natural “Log of 2”, which is equal to 0.693 and is


typically rounded to 0.7.

The half-life of a drug can be used to estimate how much of the drug

remains to be eliminated at a specified time interval or how fast the


steady state can be achieved following repeated dosing. With the

passage of each subsequent half-life half of the remaining drug is

eliminated. For example at time zero 100% of the drug is present


following intravenous dosing. After half-life 50% is eliminated and 50%

remains. After 2 half-life intervals, half of the remaining 50% (50% +

(half of 50%)) or 75% of the original total dose is eliminated. After 3


half-life intervals, 75% + (half of 25%) 87.5% is eliminated. After4

half-life intervals, 87.5% + (half of 12.5%) = 93.75% is eliminated.

After 5 half-life intervals, 93.75% + (half of 6.25%) = 96.875% is


eliminated. Consequently, a drug is virtually totally eliminated after

five half-life intervals.


With respect to the question, the equation above can be used with the

values provided in the question stem to calculate the half-life as

follows:
t1/2=(10L x 0.7) / 7L/hr = 1 hour

It takes two half-life intervals to eliminate 75% of the drug:

therefore. It will take 2 hours to excrete 75% of a drug with a t1/2 of


1 hour.

Educational Objective:
The half-life of a drug (t1/2) is a measure of how quickly a drug is

eliminated from the body and how soon steady-state concentrations are

achieved after repeated dosing. Generally the half-life of a drug in


plasma is a determinant of the duration of its pharmacologic effects in

the body. A drug is virtually totally eliminated after 5 half-life

intervals. To calculate half-life, use the formula:


t1/2 (Vd * ln 2) / CL

58

USMLE WORLD STEP 1 PHARMACOLOGY

Q NO 40: A 22-year-old female is diagnosed with hyperthyroidism after presenting

with palpitations, weight loss, and insomnia. Methimazole monotherapies initiated;

two months later, her condition improves significantly. Inhibition of which of the

following processes is most likely responsible for improvement in this patient?

A. Iodine uptake by the thyroid

B. Iodine organification and coupling of iodotyrosine


C. Colloid deposition

D. Peripheral T4 to T3 conversion

E. T3/T4 effect on target tissues

Explanation:

The normal thyroid takes up iodine (iodine trapping) against a


concentration gradient by an energy-dependent process, using a sodium

iodine symporter. After dietary, inorganic iodine enters the thyroid

follicular cells, it is oxidized to organic iodide by the enzyme


thyroid peroxidase, in a process known as “organification.” The iodine

then binds to tyrosine residues in the thyroglobulin to form

monoiodotyrosine. Several combinations are then possible. Two


monoiodotyrosine can combine to form diiodotyrosine, which can then

unite with another diiodotyrosine to form thyroxine (T4).

Alternatively, a monoiodotyrosine can link with a diiodotyrosine to


make triiodothyronine (T3). The thyroid follicular cells then engulf

thyroglobulin, which contains any and all of the iodinated tyrosine

compounds (mono- and diiodotyrosine, T3, and T4) by pinocytosis. In the


thyroid cytoplasm, the iodinated tyrosine residues is removed from the

rest of the thyroglobulin and secreted from the basolateral border. The

nonfunctional mono- and diiodotyrosine are de-iodinated so that iodine


in the thyroglobulin mix can be reused (deiodination and iodine

recycling).

Thyroid peroxidase is responsible for oxidation of inorganic iodine,


formation of mono and diiodotyrosine, and coupling which forms T3 and

T4. Methimazole acts as an anti-thyroid drug by inhibiting thyroid

peroxidase, thereby treating hyperthyroidism.


(Choice A) The thyroid gland uptakes iodine against the concentration

gradient using a sodium iodine symporter. The activity of this pump is

energy dependent. There are other factors that determine the degree of
iodine uptake. For instance, several other ions like perchlorate and

thiocyanate can compete against the iodine for entry into the thyroid
gland, henceforth reducing iodine uptake by the thyroid follicular

cells. Alternatively, iodine uptake is positively up-regulated by TSH.

(Choice C) Colloid is the gelatinous product of the thyroid gland,


consisting mainly of thyroglobulin, a high-molecular- weight

glycosylated protein. TSH secreted from the pituitary increases the

synthesis of thyroglobulin by increasing its gene transcription.


Thyroglobulin synthesis is not increased by thyroid peroxidase enzyme

and is not affected by methimazole.

(Choice D) Methimazole does not inhibit the peripheral conversion of


the inactive T4 to the activeT3. This might be a tempting choice,

however, because another antithyroid medication, propylthiouracil (PTU)

both inhibits thyroid peroxidase and decreases conversion of T4 to T3.


(Choice E)The effect of thyroid hormones on the peripheral tissues is

accomplished by interaction of T3 with an intracellular thyroid hormone

receptor. There are no antithyroid drugs that effect thyroid hormone


receptor interaction.

59

USMLE WORLD STEP 1 PHARMACOLOGY

Educational Objective:
Thionamide (methimazole and propylthiouracil) act as antithyroid

medications, by decreasing the formation of thyroid hormones via

inhibition of the enzyme thyroid peroxidase. Propylthiouracil also


decreases the peripheral conversion of T4 to 13

60

USMLE WORLD STEP 1 PHARMACOLOGY

Q NO 41: A new antiarrhythmic drug is found to be effective against both atrial and

ventricular arrhythmias. On EKG it causes ORS prolongation but has little effect on

the QT interval duration. Which of the following parts of the action potential curve

pictured below is most affected by this drug?

A. A
B. B

C. C

D. D
E. E

Explanation:

The ORS complex corresponds to ventricular depolarization. which occurs


during phase 0 of the ventricular myocyte action potential and is

represented above by Choice B. Depolarization occurs when there is

rapid sodium movement into the cardiac myocyte, thus this new
medication most likely blocks sodium influx (phase 0), as do class I

antiarrhythmics. Additionally, because this drug does not prolong the

QT interval itis most similar to the class IC agents (e.g.


propafenone).

(Choice A) This is phase 4 of the myocyte action potential, the resting

portion of the curve where ingoing and outgoing currents are balanced.
Antiarrhythmic drugs do not affect phase 4 of the myocyte action

potential, but they can affect phase 4 of the pacemaker cell action

potential which slowly depolarizes and is mediated by sodium influx.


(Choice C) This is phase of the action potential, a brief period of

repolarization corresponding to a sharp decrease in sodium conductance

and an increase in potassium permeability. It is not modulated to an


appreciable extent by anti arrhythmic medications.

(Choice D) This is phase 2 of the cardiac myocyte action potential the

“plateau” of the action potential where movement of calcium ions into


the cell and potassium ions out of the cell is relatively equal.

(Choice E) Phase 3 of the action potential is the main repolarization

phase of the ventricular myocyte. Repolarization is mediated by an


outward movement of potassium ions and can be modulated by class 3

antiarrhythmic agents. These medications prolong phase 3 repolarization


and the OT interval on EKG.

Educational Objective:
The ORS complex corresponds to ventricular depolarization and phase 0

on the action potential graph. Ventricular myocyte depolarization is

mediated by inward sodium movement and can be modulated by class I


antiarrhythmics.

61

USMLE WORLD STEP 1 PHARMACOLOGY

Q NO 42: A 65-year-old Caucasian male presents to your office after a long period

without medical follow-up complaining of disturbed color perception anorexia,

nausea vomiting and diarrhea. His past medical history is significant for congestive

heart failure that is effectively controlled with multiple medications. Which of the
following is the most likely complication of this patient’s current condition?

A. Arrythmia
B. Angioedema

C. Bronchoconstriction

D. Pulmonary edema
E. Renal failure

F. Hypotension

Explanation:

The key to getting this question correct is the knowledge that digoxin

toxicity can lead to changes in color vision as a characteristic side


effect and nausea and decreased appetite as common side effects. Once

you have determined that digoxin is responsible for this patient’s

constellation of symptoms, you need to recall that ventricular


tachyarrhythmias (Choice A) are the most serious and potentially fatal

effects of digoxin toxicity and need to be treated with management of

hyperkalemia and possibly with anti-digoxin antibody fragments.


(Choice B) Angioedema is a severe side effect clinically manifest as

edema of the lips and larynx. Of the drugs used to treat heart failure,

it is most commonly associated with the angiotensin converting enzyme


inhibitors (ACE-I). ACE- Is do not cause the side effects mentioned,

and angioedema is not a known side effect of digoxin.

(Choice C) Bronchoconstriction is a feared side effect of nonspecific


beta-adrenergic blockers in patients with chronic obstructive pulmonary

disease and asthma. Beta-blockers such as metoprolol have been shown to

be useful in patients after myocardial infarction and in treating


patients with heart failure. Bronchoconstriction is not a known side

effect of digoxin.

(Choice D) Pulmonary edema may occur with uncontrolled heart failure.


In systolic dysfunction, the ejection fraction is low and there is poor

forward flow causing blood to pool in the lungs and pulmonary edema

results. In diastolic dysfunction, the ejection fraction can be normal


or even high, but high heart rate or poor ventricular compliance causes

insufficient diastolic filling time. This result in an inadequate

volume of blood being pumped forward leading to accumulation in the


lungs and pulmonary edema.

(Choice E) Renal failure can be precipitated by a variety of drugs from


antimicrobials such as amphotericin B and aminoglycosides to drugs used

commonly for heart failure such as furosemide. Over diuresis with the

loop diuretic furosemide can cause a decrease in intravascular volume


leading to prerenal acute renal insufficiency from poor renal blood

flow.

(Choice F) Digoxin is an inotropic agent and causes increased cardiac


output. It would lead to hypotension only if ventricular fibrillation

set in from digoxin toxicity leading to poor forward flow.

Educational Objective:
In this case, digoxin toxicity manifested as changes in color vision

and anorexia. Digoxin toxicity can also cause ventricular dysrhythmias,

headache, fatigue and confusion.

62

USMLE WORLD STEP 1 PHARMACOLOGY

Q NO 43: A 25-year-old Caucasian female presents to your office complaining of

excessive hair growth on her face. She is very concerned because she thinks

that it makes her look ‘ugly’. Her past medical history is insignificant. Her

menstrual periods are regular, occurring every 24 days. She is not sexually
active and her last menstrual period was 2 weeks ago. Which of the following is

the best treatment option for this patient?

A.
B.
Clomiphene
Mitotane

C. Medroxyprogesterone

D.
E.
Mifepristone
Danazol

F. Spironolactone

Explanation:

Terminal hair growth in a male-like pattern in female is called


hirsutism. It should be distinguished from virilization that consists

of hirsutism in association with clitoromegaly, increased muscle mass,

acne, increased libido and voice deepening.


The growth of terminal hair (coarse, curly and pigmented) is dependent

on the action of a testosterone metabolite dihydrotestosterone (DHT)

that is synthesized by 5-a-reductase. Hirsutism may occur due to


increased testosterone production (such as in polycystic ovary

syndrome, Cushing syndrome, ovarian and adrenal tumors). Idiopathic

hirsutism develops due to increased conversion of testosterone into DHT


by 5-a-reductase or due to higher sensitivity of hair follicles to DHT.

In women with regular menstrual cycles and no symptoms of virilization,

increased hair growth is likely to be caused by idiopathic hirsutism.


It is treated by inhibition of testosterone synthesis or its effect on

skin.

Among the drugs listed in the answer choices, only spironolactone has
anti-androgenic properties. It blocks androgen receptors at the hair

follicles, and decreases testosterone production. Apart from its anti-

androgenic properties, spironolactone is widely used as a potassium-


sparing diuretic. Spironolactone causes gynecomastia and testicular

atrophy in men.

The other drugs that can be used for hirsutism include antiandrogens
flutamide (testosterone receptor antagonist) and finasteride (5-alpha-

reductase inhibitor).

(Choice A) Clomiphene is an antiestrogen. It interferes with the


negative feedback of estrogens on hypothalamus and pituitary. It

increases the synthesis of Gn-RH and, consequently gonadotropins. It is

used for treating infertility especially when associated with


anovulation.

(Choice B) Mitotane is an adrenocorticolytic drug used primarily for

the treatment of adrenocortical carcinoma.


(Choice C) Medroxyprogesterone (Depo Provera) is a progesterone-only

contraceptive administered as an intramuscular injection once every 12


weeks.

(Choice D) Mifepristone (RU-486) is a synthetic steroid with anti-

progestin and anti-glucocorticoid effects. It is used primarily as an


abortifacient.

(Choice E) Danazol is a synthetic androgen used for the treatment of

endometriosis and hereditary angioedema. Its side effects include


hirsutism, masculinization, fluid retention and weight gain.

63

USMLE WORLD STEP 1 PHARMACOLOGY

Educational Objective:

Hirsutism is an excessive growth of terminal hair in a male-like


pattern. It occurs due to increased testosterone secretion or increased

conversion of testosterone to DHT. Spironolactone has anti-androgenic

properties and is used for treatment of hirsutism. The other drugs that
can be used for hirsutism include antiandrogens flutamide (inhibits

binding to testosterone receptors) and finasteride (5-alpha-reductase

inhibitor).

64

USMLE WORLD STEP 1 PHARMACOLOGY

Q NO 44: A 24-year-old pregnant female comes to the physician’s office because of

right leg swelling and pain. She is 8 weeks pregnant. Doppler examination of the

leg shows deep venous thrombosis. Which of the following is the most effective

anticoagulation for this patient during her pregnancy?

A. Aspirin

B. Warfarin
C. Heparin

D. Clopidogrel

E. t-PA

Explanation:

Pregnancy itself is an independent risk factor for deep venous


thrombosis (DVT) due to several reasons. During pregnancy, there are

increased levels of clotting factors, decreased fibrinolysis, and

reduced levels of the natural anticoagulant protein S. Additionally,


decreased venous tone and the pressure of the gravid uterus on the

inferior vena cava predispose pregnant women to venous stasis. DVT

occurs with equal frequency during all trimesters of pregnancy. The


classic presentation is unilateral leg pain, swelling, and warmth.

Tenderness of the calf muscle on dorsiflexion of the foot (Homan’s

sign) is characteristic. A Doppler ultrasound of the affected extremity


confirms diagnosis.

The mainstay of DVT treatment in pregnancy is heparin. (Coumadin is not

used because it is teratogenic.) Heparin increases antithrombin Ill


activity. Activated antithrombin III inhibits the activity of thrombin

(factor IIA), factor Xa, and other coagulation factors, by forming

complexes with them. Low-molecular weight heparin (LMWH) has more


activity against factor Xa than unfractionated heparin.

(Choice A) Aspirin inhibits platelet aggregation by suppressing the

synthesis of thromboxane A2 in platelets. Aspirin alone is not an


effective anticoagulant for deep venous thrombosis. Aspirin is adequate

for the treatment and prevention of myocardial infarction and ischemic

strokes, however.
(Choice B) Warfarin is the best long-term anticoagulant used for the

treatment of DVT. However, warfarin is contraindicated in pregnancy

because it is teratogenic, capable of causing a constellation of


embryopathy termed “fetal warfarin syndrome.”

(Choice D) Clopidogrel inhibits ADP mediated platelet aggregation. It


is used in the treatment of coronary artery disease, peripheral

vascular disease, and cerebrovascular disease, but not for DVT.

(Choice E) t-PA (tissue plasminogen activator) is a thrombolytic drug


that converts plasminogen to plasmin and facilitates the dissolution of

thrombi. This and other thrombolytics are used in the management of ST

elevation acute myocardial infarction and ischemic stroke.

Educational Objective:

In pregnant women heparin is used to treat DYT. Heparin increases


antithrombin III activity. Although warfarin is normally the drug of

choice for DVT it is contraindicated in pregnancy because it is

teratogenic. Neither aspirin nor clopidogrel are independently


sufficient to treat DVT.

65

USMLE WORLD STEP 1 PHARMACOLOGY

Q NO 45: A 26-year-old male presents to your office complaining of hair loss. He

says he fears going bald like his father. Physical examination reveals moderate

hair loss from the anterior scalp and vertex. Which of the following drugs can be

used to treat this patient’s condition?

A. Local glucocorticoids

B. Isotretinoin
C. Progesterone

D. Finasteride

E. Terbinafine

Explanation:

In adults age 18-451 the three most common causes of hair loss, in
order of frequency, are: male pattern baldness (androgenetic alopecia),

tinea capitis (ringworm), and telogen effluvium. Of these only

androgenetic alopecia is inherited (polygenic or autosomal dominant


inheritance). The pattern of hair loss described above is classic for

androgenetic alopecia. This type of hair loss generally follows the

progression pictured below.

The pathogenesis of male pattern baldness is thought to involve scalp

5-a-reductase activity and the androgenic effects of the


dihydrotestosterone produced. Thus, this condition may be treated by

androgen antagonists, in particular 5-a-reductase inhibitors like

finasteride.
In contrast tinea capitis causes isolated pruritic lesions on the

scalp. The lesions typically have a central clearing producing a ring-


like configuration (hence, “ringworm”). Terbinafine (Choice E) is an

oral anti-fungal agent (squalene epoxides inhibitor). Systemic

medications are used to treat tinea capitis because topical agents do


not adequately penetrate the hair follicle.

66

USMLE WORLD STEP 1 PHARMACOLOGY

(Choice A) Topical corticosteroids are used to treat alopecia areata,

an autoimmune disorder that causes patchy or diffuse hair loss.


(Choice B) Oral isotretinoin is used to treat severe acne. Like other

retinoids, it inhibits follicular epidermal keratinization.

Educational Objective:

Male pattern baldness is an inherited trait that produces a distinctive

pattern of progressive hair loss. The anterior (frontal) scalp is


affected first, then the vertex. Pathogenesis involves scalp 5-a-

reductase activity and the androgenic effects of the resulting

dihydrotestosterone. 5-a-reductase inhibitors like finasteride can be


used to treat this condition.

67

USMLE WORLD STEP 1 PHARMACOLOGY

Q NO 46: A 45-year-old Caucasian female is being treated for severe joint pain and

swelling. Soon after beginning treatment she presents with painful mouth ulcers

and nausea. Liver function tests reveal AST and ALT elevations. Which of the

following medications is most likely responsible?

A. Naproxen

B. Methotrexate
C. Hydroxychloroquine

D. Prednisone

E. Minocycline

Explanation:

Methotrexate is the preferred first-line disease-modifying treatment


for most patients with moderate to severe rheumatoid arthritis. It

produces substantial improvements in disease activity in 60-70% of

patients. Leflunomide and TNF-a inhibitors are other disease-modifying


agents that may be used in combination with methotrexate if the patient

does not respond to methotrexate alone.

Methotrexate works by inhibiting the enzyme dihydrofolate reductase to


block folinic acid synthesis. Although it is generally well tolerated,

serious adverse effects can occur, including stomatitis (painful mouth

ulcers) and hepatotoxicity (hepatitis, fibrosis, and cirrhosis). Other


potential adverse effects include myelosuppression, increased risk for

opportunistic infections, B-cell lymphomas, and pulmonary fibrosis.

Methotrexate is contraindicated during pregnancy. Supplementation of


folic acid reduces the occurrence of stomatitis.

(Choice A) Naproxen is a nonsteroidal anti-inflammatory drug (NSAID)

that works by blocking prostaglandin synthesis. Gastrointestinal


bleeding, fluid retention, and worsening of hypertension are important

potential side effects of NSAID therapy. There may be mild elevations

of liver function enzymes in a small percentage of patients. Liver


changes are generally asymptomatic and nonprogressive. Mouth

ulcerations do not typically occur with NSAID therapy.

(Choice C) Hydroxychloroquine can be used to treat early mild


seronegative rheumatoid arthritis, though it is more commonly used in

the treatment of systemic lupus erythematosus. A serious potential

adverse effect of hydroxychloroquine is permanent retinal damage.


Baseline and follow-up retinal examinations are necessary for all

patients using this medication.


(Choice D) Corticosteroids are typically used for a short period of

time in patients who have acute joint pain and swelling, as a bridge to

relief from other therapies. Side effects of prolonged corticosteroid


use include weight gain (especially centrally), hyperglycemia,

decreased bone density, skin striae, and proximal muscle weakness. When

used with NSAIDs, corticosteroids potentiate the risk for


gastrointestinal bleeding.

(Choice E) Like sulfasalazine and hydroxychloroquine, minocycline can

be used in some patients with early mild rheumatoid arthritis. Side


effects are minimal, and include photosensitivity.

Educational Objective:
Methotrexate is the preferred disease-modifying treatment for patients

with moderate to severe rheumatoid arthritis. Methotrexate treatment

may cause stomatitis and liver function abnormalities.

68

USMLE WORLD STEP 1 PHARMACOLOGY

Q NO 47: A 44-year-old Caucasian male who is hospitalized for open

cholecystectomy becomes severely agitated and tremulous two days


A.
Baclofen

after admission. He denies any illicit drug use, but admits to 3-4 B.

beers every day for the last several years. He has no known drug
allergies, and his family history is insignificant. His BP is 150/90mm

Hg, and pulse is 110 beats per minute. Which of the following drugs

is the best treatment option for this patient’s condition?

Haloperidol
C. Diazepam

D. Carbamazepine

E. Propranolol
F. Phenobarbital

G. Phenytoin

Explanation:

Alcohol is a CNS depressant that affects GABAA receptors and enhances


the inhibitory effect of GABA in the brain. Long-term alcohol

consumption leads to down-regulation of GABA receptors; therefore,

discontinuation of alcohol can cause CNS excitation (withdrawal).


Alcohol-withdrawal symptoms typically start within 8-12 hours after the

last drink and include insomnia, tremulousness, mild anxiety, headache,

and autonomic hyperactivity (variable BP, diaphoresis, and


tachycardia). If untreated, delirium tremens (generalized tonic-clonic

seizures and hallucinosis) may develop, usually on the third day

without alcohol.
Alcohol, barbiturates, and benzodiazepines have similar effects on GABA

receptors and act by enhancing GABA action. Benzodiazepines have a

better safety profile than barbiturates, and are the drugs of choice in
the treatment of alcohol withdrawal. Long-acting benzodiazepines, such

as diazepam or chlordiazepoxide, are preferred in the majority of

patients. They have a prolonged half-life (up to a few days) and are
metabolized by the liver into active metabolites. Short-acting

benzodiazepines, such as lorazepam, are indicated for patients with

advanced liver disease.


(Choice A) Baclofen is an agonist of GABA6 receptors and is used to

treat spastic conditions. It is not the drug of first choice for

alcohol withdrawal.
(Choice B) Phenothiazines and butyrophenones (including haloperidol)

lower the seizure threshold. They should not be used in patients

suffering from alcohol withdrawal because these patients are already at


significant risk for seizure.

(Choices D and G) The role of anticonvulsants such as phenytoin in

alcohol-withdrawal associated seizures is controversial. However,


carbamazepine should never be used to treat alcohol withdrawal.

(Choice E) Propranolol can treat both tremor and hypertension. However,

administering benzodiazepines is the most crucial step in the


management of these patients. If a patient is still symptomatic after

benzodiazepine administration, propranolol can be tried.

(Choice F) Phenobarbital is a barbiturate anticonvulsant that enhances


GABA activity. It is not first-line treatment for alcohol withdrawal as

it has a worse safety profile than benzodiazepines.

69

USMLE WORLD STEP 1 PHARMACOLOGY

Educational Objective:

Benzodiazepines substitute for the action of alcohol on GABA receptors


and are indicated for the treatment of alcohol withdrawal. Long-acting

benzodiazepines (chlordiazepoxide, diazepam) are first-line

medications. Short-acting benzodiazepines (lorazepam, oxazepam) are


preferred in patients with advanced liver dysfunction.

70

USMLE WORLD STEP 1 PHARMACOLOGY

Q NO 48: A 78-year-old nursing home resident has not had a bowel movement for

the last five days. He has suffered from constipation for many years; he is

demented and spends most of his time in bed. Magnesium hydroxide was

administered and produced a bowel movement in a few hours. The mechanism of


the clinical effect observed in this patient mimics the pathophysiology of which of

the following disorders?

A.
B.
Irritable bowel syndrome
Crohn’s disease

C. Lactase deficiency

D.
E.
Carcinoid syndrome
Rectal prolapse

Explanation:

Constipation is common in elderly, debilitated patients as well as

those on chronic opiate therapy. Factors that contribute to development


of constipation are inactivity, low-Fiber diet, medications (such as

opioids or TCAs) and hypercalcemia. As lifestyle changes are not an

option in patients such as the one mentioned in the question stem,


constipation in this population is commonly treated with osmotic

laxatives such as magnesium hydroxide, stool softeners and enemas.

Magnesium-containing oral drugs such as magnesium hydroxide and


magnesium citrate, as well as preparations such as lactulose and

GoLYTELY (polyethylene glycol) are osmotic laxatives. They are non-

absorbable or poorly absorbable substances that attract water into the


intestinal lumen, thus distending the intestinal wall and increasing

peristalsis. The laxative effect is usually rapid.

Lactase deficiency is a disease state characterized by osmotic diarrhea


(Choice C). Inherited or acquired deficiency of the intestinal brush

border enzyme lactase (disaccharidase) causes inability to break down

lactose into glucose and fructose. Undigested lactose is a non-


absorbable osmotic substance, and its accumulation in the small

intestine leads to increase in secretion of water and electrolytes into

the intestinal lumen. Lactase deficiency (lactose intolerance) presents


with abdominal pain, distension and watery diarrhea. Abdominal pain and

distention result from metabolism of lactose by normal gut flora by

fermentation causing production of gas. The symptoms resolve when milk-


containing products are eliminated from the diet.

(Choice A) Irritable bowel syndrome is a functional intestinal disorder

that presents with diarrhea alternating with constipation, abdominal


pain and distention without organic cause. Fecal water and electrolyte

content in IBS is normal.

(Choice B) Diarrhea in Crohn’s disease is of the secretory type which


is characterized by high electrolyte content due to poor absorption and

increased losses from the inflamed intestinal mucosa.

(Choice D) Diarrhea in carcinoid syndrome is secretory and high in


electrolytes.

(Choice E) Rectal prolapse is a protrusion of rectal mucosa through the


anus associated with pregnancy and constipation; it is not associated

with diarrhea.

Educational Objective:
Magnesium hydroxide causes osmotic diarrhea because magnesium ions are

poorly absorbed in the gut. Diarrhea associated with lactase deficiency

is also osmotic and occurs due to accumulation of non-absorbable


lactose in the intestinal lumen.

71

USMLE WORLD STEP 1 PHARMACOLOGY

Q NO 49: A 43-year-old male begins treatment for depression. Several weeks after

starting the medication, he notes that his depressive symptoms have improved

markedly, but that he is now experiencing impaired sexual performance. He

requests an alternative treatment, without sexual side effects. Which of the


following drugs is most appropriate for this patient?

A. Fluoxetine

B. Phenelzine

C. Bupropion
D. Imipramine

E. Trazodone

Explanation:

Bupropion is an antidepressant medication that does not cause sexual

dysfunction. Side effects of bupropion can include agitation, insomnia,


and seizures. It is thought that bupropion exerts its effects by acting

on norepinephrine transmission, without significantly influencing

serotonin, acetylcholine, or histamine metabolism. Bupropion is useful


for the treatment of depression with associated psychomotor retardation

or hypersomnia, because it has stimulatory effects. Bupropion is also

effective in the treatment of nicotine dependence.


(Choice A) Fluoxetine is a selective serotonin reuptake inhibitor

(SSRI). Sexual dysfunction is a relatively common side effect of the

SSRIs and limits their use for many patients.


(Choice B) Monoamine oxidase inhibitors (MAOI5), such as phenelzine and

tranylcypromine, are not considered first- line antidepressant

medications because of necessary dietary restrictions (cheese, wine),


potential drug-drug interactions (TOA5), and numerous other side

effects.

(Choice D) Imipramine is a tricyclic anti-depressant with numerous side


effects. It is not a first-line treatment for depression.

(Choice E) Trazodone is a highly sedating anti-depressant that can be

helpful for patients with insomnia. Trazodone can cause priapism and
other sexual side effects.

Educational Objective:
Sexual dysfunction is seen in up to 50% of patients treated with SSRIs.

Symptoms include decreased libido, anorgasmia, and increased latency to

ejaculation. Bupropion is an excellent alternative.

72

USMLE WORLD STEP 1 PHARMACOLOGY

Q NO 50: A 55-year-old patient being treated for depression is admitted to the

emergency department with severe confusion and hallucinations. His mouth is

dry and his face is flushed. His blood pressure is 80/60 mmHg and his heart

rate is 100 beats per minute. ECG reveals ORS prolongation (0.12 msec) and
frequent premature ventricular beats. You start intravenous hydration. Which

of the following agents would best correct this patient’s cardiac abnormalities?

A. Acetylcysteine

B.
C.
Potassium solution
Propranolol

D. Flumazenil

E.
F.
Naloxone
Atropine

G. Sodium bicarbonate

Explanation:

Tricyclic anti-depressants (TCA5) can have quinidine-like effect on the


cardiac conduction system, potentially causing QRS and QT prolongation

and cardiac dysrhythmias. These effects are due to inhibition of fast

sodium channels. In instances of cardiovascular toxicity due to TCA


overdose, the most effective intervention is hypertonic sodium

bicarbonate (NaHCO3). NaHCO3 can correct QRS prolongation, reverse

hypotension, and treat ventricular dysrhythmias.


(Choice A) Acetylcysteine is a thiol used to treat acetaminophen

overdose. In acetaminophen (Tylenol) overdoses, toxic acetaminophen

metabolites exhaust glutathione stores. N-acetylcysteine reduces


hepatic damage in these cases.

(Choice B) Potassium replacement is one of the first steps in treating

digitalis toxicity when there is accompanying hypokalemia.


(Choice C) Propranolol causes hypotension and should be not be used in

this patient.

(Choice D) Flumazenil is a specific benzodiazepine antagonist used for


benzodiazepine toxicity.

(Choice E) Naloxone is a rapidly-acting opioid antagonist that

competitively binds opioid receptors, including K, 8, and especially p.


(Choice F) Atropine is used for organophosphate poisoning.

Educational Objective:
Sodium bicarbonate is the single most effective agent in treating TCA-

associated cardiac abnormalities.

73

USMLE WORLD STEP 1 PHARMACOLOGY

Q NO 51: A 28-year-old male complains of nighttime abdominal pain that is relieved

by food. He has no significant past medical history and does not take any

medications. Upper gastrointestinal endoscopy reveals a deep mucosal defect in the

first portion of the duodenum. Which of the following is most likely to provide long-
term relief in this patient?

A. Ranitidine
B. Sucralfate

C. Metoclopramide

D. Misoprostol
F. Antibiotics

E. NSAIDs

Explanation:

Based on his symptoms and endoscopic findings, this patient appears to

have a duodenal ulcer (peptic ulcer disease, PUD). Eighty to 95% of


patients with duodenal ulcers harbor H. pylon bacteria. (The remaining

cases are associated with NSAID use. We are told that this patient does

not take any medications and thus can assume that his condition is the
result of H. pylon infection.) To minimize the likelihood of PUD

recurrence after his ulcer heals, the H. pylon infection must be

eradicated. The most effective regimens to cure H. pylon infection


combine two antibiotics (metronidazole, tetracycline, amoxicillin, or

clarithromycin) with an adjunctive agent (proton pump inhibitor and/or

bismuth) for 14 days.


(Choice A) Proton pump inhibitors are superior to H2 blockers for

treating peptic ulcer disease.

(Choice B) Sucralfate binds to the base of mucosal ulcers protecting


against gastric acid. This can help duodenal ulcers to heal but would

be less effective at preventing ulcer recurrence than treatment to

eradicate mucosal H. pylon infection.


(Choice C) Metoclopramide is a dopamine antagonist with both central

and peripheral effects. It has prokinetic and antiemetic properties and

is used to treat 01 motility disorders like gastroparesis and to


prevent nausea and vomiting. It does not have a significant effect on

peptic ulcer disease.

(Choice D) Misoprostol is a prostaglandin El analog used to prevent


NSAID-induced ulcer disease.

(Choice F) NSAIDs promote ulcer formation and should be avoided in all

patients with gastritis and PUD.

Educational Objective:
For patients with peptic ulcer disease. the most effective way to

provide long-term relief is to eradicate Helicobacter pylon infection

of the gastric mucosa with antibiotic therapy.

74

USMLE WORLD STEP 1 PHARMACOLOGY

Q NO 52: A 2-month-old boy is brought to the ER with fever irritability and

vomiting. Lumbar puncture reveals CSF pleocytosis and you proceed with
A.

empirical ceftriaxone therapy. You must also administer ampicillin to cover

for which of the following bacteria?


Neisseria meningitidis

B. Streptococcus pneumoniae

C. Group B Streptococci
D. Listens monocytogenes

E. Haemophilus influenzae hysteria

Explanation:

Ceftriaxone adequately covers all of the organisms listed except

Listeria monocytogenes. Ampicillin should always be added to the


empiric treatment of meningitis in infants and in immunocompromised

adults to cover for Listeria.

L. monocytogenes is a facultative intracellular organism that can


survive and multiply within the cells of patients with deficiencies of

cell-mediated immunity, including neonates up to three months of age

HIV patients, organ transplant recipients, and individuals with certain


malignancies.

(Choice A) Neisseria meningitidis is most commonly treated with

Penicillin G but can also be effectively treated with a third


generation cephalosporin such as ceftriaxone.

(Choice B) Streptococcus pneumoniae is the most common cause of

meningitis in adults. Historically Penicillin G has been universally


effective, but increasing penicillin resistance has made the third-

generation cephalosporins first-line agents.

(Choice C) Group B Streptococci (Streptococcus agalactiae) is the most


common cause of neonatal meningitis, sepsis and pneumonia. Penicillin 0

is the antibiotic of choice for this organism in both the colonized

mother and the infected infant. Ceftriaxone is also effective.


(Choice E) Haemophilus influenzae is most effectively treated with a

third-generation cephalosporin such as ceftriaxone.

Educational Objective:

Ampicillin is the treatment of choice for Listeria. It is not sensitive

to cephalosporins. Listeria causes disease in neonates and


immunocompromised adults.

75

USMLE WORLD STEP 1 PHARMACOLOGY

Q NO 53: A 65-year-old diabetic male has a chronic, non-healing ulcer of the left

foot. He comes to the ER because of fever chills, and confusion. His BP is 90/70

mm Hg. He is started on broad spectrum antibiotics. Blood cultures grow

Bacterioid species, beta lactamase producing. The results of the culture allow
you to narrow the antibiotic treatment to which of the following?

A. Ciprofloxacin
B. Ceftriaxone

C. Azithromycin

D. Vancomycin
E. Piperacillin-Tazobactam

Explanation:
Bacteroid species are Gram (-) anaerobic rods. Areas of necrosis and

diminished blood supply (such as chronic non-healing ulcers) facilitate

the growth of anaerobes. Bacteroides produce 3-lactamase, an enzyme


that breaks down penicilli ns. β-lactamase inhibitors prevent these

enzymes from functioning. Tazobactam is a β-lactamase inhibitor as are

clavulanic acid and sulbactam.


Piperacillin is an extended spectrum penicillin. The combination of

piperacillin and tazobactam is effective against a large number of Gram

(+) Gram (-) and anaerobic bacteria. Among the medications listed, only
piperacillin tazobactam would be an effective treatment for this

infection.

(Choice A) Ciprofloxacin is a first-generation fluoroquinolone that


acts by interfering with bacterial topoisomerase II (DNA gyrase).

Ciprofloxacin is active against most Gram (-) rods and bacilli,

including Legionella and P. aeruginosa. It is not effective for


anaerobic infections.

(Choice B) Ceftriaxone is a third-generation parenteral cephalosporin.

It is active against Gram (-) bacilli, including most of the enteric


organisms: H. influenza, Klebsiella pneumoniae, Neisseria, and Serratia

species. It has poor activity against anaerobes.

(Choice C) Azithromycin is a macrolide antibiotic that binds to the 50S


subunit of bacteria ribosomes, thereby inhibiting protein synthesis. It

is used for Chlamydia, Mycoplasma species, H. influenza, and Moraxella

catarrhalis infections.
(Choice D) Vancomycin is a tricyclic glycopeptide that inhibits the

synthesis of peptidoglycan, which is a main component of the cell wall

of Gram-positive bacteria. It is used against methicillin-resistant


Staphylococcus aureus (MRSA) and S. epidermidis (MRSE), as well as C.

difficile.

Educational Objective:

Piperacillin-tazobactam is a combination of extended-spectrum


penicillin with β-lactamase inhibitor. It is effective against most

Gram (-) enteric rods, including Pseudomonas aeruginosa and Bacteroides

fragilis.

76

USMLE WORLD STEP 1 PHARMACOLOGY

Q NO 54: A 57-year-old male treated with high-dose methotrexate experiences

fever, painful mouth ulcers, and pancytopenia. Which of the following agents

could have prevented this patient’s condition?

A. Allopurinol

B. Folinic acid

C. Ondansetron
D. Filigrastim

E. Mesna

F. Dexrazoxane
G. Amifostine

Explanation:
Methotrexate (MTX) is a folio acid analog that inhibits DNA synthesis

by preventing the synthesis of purines and thymidylic acid. MTX

inhibits dihydrofolate reductase, the enzyme that converts dietary


folic acid to tetrahydrofolate, through direct competition.

Tetrahydrofolate is a single carbon group (typically a methyl or a

formyl group) donor in the synthesis of purines. The ultimate function


of MTX as a chemotherapeutic agent is through inhibition of DNA

synthesis.

Unfortunately, methotrexate is not specific to tumor cells; it causes


death of all rapidly-dividing cells particularly those of the 01 tract

mucosa and the bone marrow. The patient’s symptoms are therefore

classic—aphthous ulcers and pancytopenia.

Folinic acid (leucovorin) can reverse methotrexate toxicity if given

early. Leucovorin, a 5-formyl-derivative of tetrahydrofolic acid, does

not require the action of dihydrofolate reductase to be converted to


tetrahydrofolate and is therefore unaffected by methotrexate.

Leucovorin is administered following high-dose methotrexate as part of

a chemotherapeutic plan, where it serves to rescue bone marrow,


gastrointestinal, and other mucosal cells from methotrexate toxi city.

(Choice A) Allopurinol and its metabolite, oxipurinol, are

predominantly noncompetitive inhibitors of xanthine oxidase.


Allopurinol is used in the treatment of gout and the prevention of

tumor lysis syndrome.

77

USMLE WORLD STEP 1 PHARMACOLOGY

(Choice C) Ondansetron inhibits serotonin 5-HT3 receptors in the GI

tract, the area postrema, and the solitary nucleus and tract. Itis used
primarily to treat nausea and vomiting following chemotherapy.

(Choice D) Filgrastim is a granulocyte colony-stimulating factor (G-

CSF) analog, used to stimulate the proliferation and differentiation of


granulocytes in patients with neutropenia.

(Choice E) Mesna prevents hemorrhagic cystitis in patients receiving

either cyclophosphamide or ifosfamide chemotherapy. Mesna binds


acrolein, the toxic metabolite that is formed by these agents.

(Choice F) Dexrazoxane is an iron-chelating agent that can prevent

anthracycline-induced cardiotoxicity.
(Choice C) Amifostine is a cytoprotective free-radical scavenger used

to decrease the cumulative nephrotoxicity associated with platinum-

containing and alkylating chemotherapeutic agents and to decrease


xerostomia (dry mouth).

Educational Objective:
Folinic acid (leucovorin) can reverse the toxicity of methotrexate in

non-cancerous cells in the GI mucosa and bone marrow if administered at

the appropriate time. Leucovorin, a 5-formyl-derivative of


tetrahydrofolic acid, does not require the action of dihydrofolate

reductase for its conversion to tetrahydrofolate.

78

USMLE WORLD STEP 1 PHARMACOLOGY

Q NO 55: A new acetylcholine receipt or inhibit or has just been approved by the

FDA. You consider the possible therapeutic implications of this agent and want to

review the autonomous nervous supply of various organs. Which of the following

nervous outputs is noradrenergic?

A. Sympathetic output to adrenals

B. Sympathetic output to sweat glands


C. Sympathetic output to the bladder

D. Parasympathetic output to the heart

E. Parasympathetic output to the bronchi

Explanation:
The autonomic nervous system (ANS) regulates visceral functions and is

not under conscious control. Both sympathetic and parasympathetic


divisions of the ANS use chemical mediators for transmission of

stimuli. Most sympathetic neurons utilize norepinephrine, whereas

cholinergic neurons use acetylcholine preferentially. Exceptions to


this rule exist but they are uncommon.

The majority of sympathetic output to the viscera is formed by two

neuron units. Preganglionic neurons start in the brain or spinal cord


nucleus and synapse on ganglia of the spinal chain. These neurons

contain acetylcholine and stimulate nicotinic cholinergic receptors on

the postganglionic neuron. Postganglionic neurons then release


norepinephrine into the synaptic cleft of target organs. This completes

the typical sympathetic output system. There are two notable exceptions

to this scheme: 1. The adrenal glands are directly innervated by


preganglionic neurons using acetylcholine: 2. Sweat glands are

innervated by a two neuron efferent system where both neurons use

acetylcholine.
Parasympathetic output to viscera is also transmitted by two neuron

units. Like in the sympathetic system, preganglionic neurons contain

acetylcholine and synapse on nicotinic receptors of the ganglia.

79

USMLE WORLD STEP 1 PHARMACOLOGY

However, in the parasympathetic system, postganglionic neurons release

acetylcholine into their synapses to interact with visceral muscarinic


receptors.

The somatic nervous system consists only of presynaptic neurons. They

transmit impulses to skeletal muscles by releasing acetylcholine that


acts on postsynaptic nicotinic receptors.

(Choice A) Adrenal glands are innervated directly by preganglionic

sympathetic neurons that use acetylcholine as a neurotransmitter.


(Choice B) Autonomic innervation of the sweat glands is sympathetic.

Unlike most of the other sympathetic efferents, postganglionic neurons

that synapse on sweat glands use acetylcholine as a neurotransmitter.


(Choices D and E) Parasympathetic output to the heart and bronchi is

mediated by muscarinic receptors.

Educational Objective:

Sympathetic output to the viscera is transmitted through two-neuron

units that synapse on noradrenergic receptors of the target organs. The


sympathetic innervation of the adrenal and sweat glands differs from

this typical setup.

80

USMLE WORLD STEP 1 PHARMACOLOGY

Q NO 56: A 57-year-old Caucasian male is hospitalized with muscle pain, fatigue

and dark urine. His past medical history is significant for stable angina. The

patient’s medications include metoprolol, atorvastatin, and aspirin. Laboratory

evaluation reveals that he is in acute renal failure. The addition of which of the
following medications is most likely to have precipitated this patient’s condition?

A. Erythromycin
B. Phenytoin

C. Rifampin

D. Griseofulvin
E. Phenobarbitone

Explanation:
Statins are very effective agents in the reduction of LDL cholesterol.

Serious side effects of statins include myopathy and hepatitis.

Myopathy is a rare complication of statin use clinically defined as


muscle pain with serum creatine kinase over 10 times the upper limit of

normal. With the exception of pravastatin, the statin drugs

(simvastatin, lovastatin, atorvastatin) are metabolized by the liver


cytochrome P-450 3A4. Erythromycin inhibits phytochrome 3A4. Thus,

concurrent use of erythromycin with these drugs causes increased serum

levels of statin, which is in turn associated with increased risk for


myopathy. Studies have shown that erythromycin increases the serum

concentration of atorvastatin by 40%. Other macrolides, such as

clarithromycin, also inhibit cytochrome P-450 3A4. Azithromycin,


however, does not have a significant effect on cytochrome P-450 3A4.

Other inhibitors of cytochrome P-450 3A4 include ketoconazole,

cyclosporine, HIV protease inhibitors, and grapefruit juice. If a


patient is on an agent that inhibits cytochrome P-450 3A4, pravastatin

is the statin of choice.

(Choices B, C, D and E) Phenytoin, rifampicin, griseofulvin and


phenobarbitone are cytochrome P-450 inducers. Thus, the addition of

these agents would reduce the plasma concentration of drugs metabolized

via the cytochrome P-450 pathway.

Educational Objective:

Most statins are metabolized by cytochrome P-450 3A4, with the


exception of pravastatin. Concomitant administration of drugs that

inhibit statin metabolism (e.g. macrolides) is associated with

increased incidence of statin induced myopathy and rhabdomyolysis.


Acute renal failure is a possible sequela of rhabdomyolysis.

81

USMLE WORLD STEP 1 PHARMACOLOGY

Q NO 57: A 28-year-old Caucasian male presents to your office complaining of

feeling tense all the time. He also complains of worrying about many things

that make no sense but he can not control it. He has been having these

symptoms for about a year. He denies feeling down or having any suicidal
ideation. His blood pressure is 120/70 mmHg and heart rate is 80/mm.

Physical examination is insignificant. You start buspirone therapy in this

patient. The patient comes again in a week saving that nothing changed
following treatment and asks for help. Which of the following is the best next

step in the management of this patient?

A. Switch to alprazolam

B. Switch to fluoxetine
C. Recommend herbal therapy

D. Add phenobarbital

E. Reassure and schedule a follow-up

Explanation:

The patient should be informed that the onset of action for buspirone
can occur several weeks following the beginning of treatment.

Buspirone, a selective agonist of the SHT1A receptor is a safe and

effective treatment for generalized anxiety disorder. Dependence does


not occur with chronic buspirone treatment but the clinical response is

often delayed for up to two weeks of regular use and is not effective

when used on an as-needed basis. Buspirone is widely used and the


recommended therapy for long-tem treatment of anxiety. It is generally

safe and well tolerated.

(Choice A) Alprazolam is a benzodiazepine with an onset of action of 1-


2 hours and a relatively short half-life (10-15 hours). It is used on

an as-needed basis.
(Choice B) Fluoxetine is a serotonin-specific reuptake inhibitor

(SSRI5) that is used in treating panic disorder social phobia

generalized anxiety disorder and obsessive-compulsive disorder. For the


most part, SSRIs have a favorable side-effect profile (although sexual

dysfunction may limit their use). Interestingly, SSRIs may cause an

initial worsening of anxiety and therefore treatment often is started


at very low doses.

(Choice C) Herbal therapy with Kava would not be indicated in this

case. All that is required is follow-up and adequate time to allow the
medication to exert its pharmacological effect.

(Choice D) Barbiturates are not used for generalized anxiety disorder.

Educational Objective:

Buspirone is considered a first line treatment for generalized anxiety

disorder. Dependence does not occur with chronic buspirone treatment


but the clinical response is often delayed for up to 2 weeks of regular

use and is not effective when used on an as-needed basis.

82

USMLE WORLD STEP 1 PHARMACOLOGY

Q NO 58: The infected lymphocytes of a 25-year-old male treated with Drug A are

unable to synthesize viral DNA from the RNA template. Drug A is not

phosphorylated intracellularly and is structurally unrelated to nucleosides. Drug A

is most similar to which of the following agents?

A. Zidovudine

B. Zalcitabine
C. Ritonavir

D. Nevirapine

F. Enfuvirtide

Explanation:

Retroviral reverse transcriptase (RT) inhibitors prevent the synthesis


of viral DNA from the RNA template. Nonnucleoside RI inhibitors (NNRTI)

do not require activation via intracellular phosphorylation. The more

common NNRTIs include nevirapine, efavirenz, and delavirdine. NNRTIs


are best used in conjunction with other antiretroviral agents (ie,

HAARI) in the treatment of HIV infection.

Adverse effects are common with NNRTIs. Most concerning are patients
who present with abrupt onset flulike symptoms, abdominal pain

jaundice, or fever, as these clinical manifestations maybe indicative

of potentially life- threatening hepatic failure with encephalopathy.


Severe hepatotoxic reactions are most likely to occur within the first

six weeks of NNRTI antiretroviral therapy. Life-threatening skin

reactions (eg, Stevens-Johnson syndrome toxic epidermal necrolysis)


have also been observed, typically in the first eighteen weeks of NNRII

antiretroviral therapy.

(Choices A and B) Zidovudine (ATE) is a nucleoside reverse


transcriptase inhibitor (NRII). Zidovudine, like zalcitabine and the

other NRTIs, must be converted into its monophosphate form by cellular

thymidine kinase before it can be converted into a pharmacologically


active triphosphate form.

(Choice C) Ritonavir is an inhibitor of HIV protease that prevents

assembly and maturation of the virus. Nonfunctional, noninfectious


virions are produced instead. Reverse transcription is not affected.

(Choice E) Because enfuvirtide is an HIV fusion inhibitor, reverse

transcription is not affected and intracellular phosphorylation is not


required. Enfuvirtide binds to the envelope glycoprotein 41 (gp4l) of

HIV-1 and blocks the conformational changes necessary for the fusion of
viral and cellular membranes. Through this mechanism, enfuvirtide

denies the HIV genome entry into uninfected CD4 + T-cells.

Educational Objective:

Nonnucleoside reverse transcriptase inhibitors (NNRII5) are anti

retroviral drugs that do not require activation via intracellular


phosphorylation. The more common NNRTIs include nevirapine, efavirenz,

and delavirdine.

83

USMLE WORLD STEP 1 PHARMACOLOGY

Q NO 59: A 49-year-old female is brought to the emergency room with squeezing

chest pain and profuse sweating. ECG reveals ST segment elevation in leads I,

aVL and V4 -V6. Cardiac troponin levels are high. Soon after hospitalization

ventricular tachycardia is noted and therefore anti arrhythmic therapy is


considered. Which of the following anti arrhythmic drugs has the highest

selectivity for ischemic myocardium compared to normal cardiac tissue?

A.
B.
Lidocaine
Procainamide

C. Verapamil

D.
E.
Adenosine
Digoxin

F. Labetalol
G. Amlodipine

Explanation:
The question stem describes a woman experiencing an acute lateral

myocardial infarction and subsequently experiencing ventricular

arrhythmia which is one of the most common causes of death following an


acute myocardial infarction (AMI). Lidocaine selectively depresses

conduction in rapidly depolarizing and depolarized myocardial cells as

are found in ischemic myocardium and was the agent of choice in this
setting. Currently amiodarone has replaced the lidocaine in the

management of ventricular tachycardia.

(Choice B) Procainamide is a class IA antiarrhythmic and similar to


quinidine is more specific for suppressing arrhythmias arising from

centers of normal automaticity rather than areas demonstrating abnormal

automaticity. These drugs are not useful in ischemic myocardium and can
precipitate ventricular tachycardia by prolonging the QT interval.

(Choice C) Verapamil is a class IV antiarrhythmic used primarily to

treat trial tachyarrhythmias because it slows conduction through the AV


node. It is not useful in the ischemic myocardium. Prolonged

administration of verapamil in patients with ventricular tachycardia

(VT) can lead to ventricular fibrillation and death. Thus it is


important to differentiate supraventricular tachycardia from

ventricular tachycardia.

(Choice D) Adenosine is indicated for use in the acute treatment of


paroxysmal supraventricular tachycardia; it is not indicated in

ventricular tachycardia.

(Choice E) Digoxin’s antiarrhythmic use lies in its ability to slow


conduction through the AV node, and itis used in the treatment of

atrial fibrillation.

(Choice F) Labetalol blocks both alpha and beta adrenergic receptors


leading to decreased peripheral vascular resistance, heart rate,

cardiac contractility and slowed AV nodal conduction. It is primarily

used in the treatment of refractory hypertension, hypertensive urgency


emergency and in patients with pheochromocytoma.

(Choice C) Amlodipine is a calcium channel blocker used primarily for


the treatment of hypertension and angina. Calcium channel blockers have

not shown a benefit with respect to mortality following an AMI as the

beta blockers have, so their use in this scenario would not be


indicated.

84

USMLE WORLD STEP 1 PHARMACOLOGY

Educational Objective:

Lidocaine is a class IB antiarrhythmic agent which very specifically


binds rapidly depolarizing and depolarized cells. Ischemic myocardium

is depolarized tissue; this is why lidocaine is specific for ischemic

tissue and is the agent of choice for prevention and treatment of post-
myocardial infarction arrhythmias. Currently amiodarone has replaced

the lidocaine in the management of ventricular tachycardia.

85

USMLE WORLD STEP 1 PHARMACOLOGY

Q NO 60: A 65-year-old Caucasian male was diagnosed with stable angina and

underwent coronary stenting. His past medical history is also significant for

hypertension and diabetes mellitus, type 2. If treated with niacin for

hyperlipidemia, which of the following adjustments of his hypertension and


diabetes medications, respectively, would most likely be necessary?

A. Increase, Increase

B. Increase, Decrease
C. Decrease, Increase

D. Decrease, Decrease

E. Increase, No change

Explanation:

Although niacin is a very effective antihyperlipidemic agent, its use


is associated with a number of side effects. When therapy with niacin

is initiated, most patients experience warmth, flushing, and itchiness

of the skin. This side effect is most likely mediated by


prostaglandins, because administration of aspirin 30-60 minutes before

niacin offers significant relief.

Niacin potentiates the effects of some anti-hypertensive medications,


such as vasodilators and ganglion blocking agents, because of its

vasodilatory effects. Postural hypotension can ensue. Thus, dose

adjustment is required in some patients. Niacin use is also associated


with increased insulin resistance, which sometimes manifests as

acanthosis nigricans. The need for diabetes medications is likely to

increase when niacin is used in diabetic patients. Additionally, there


is an increase in serum uric acid in some patients being treated with

niacin. Therefore, patients with gout should be cautious when niacin is

started.

Educational Objective:

Niacin can potentiate the effects of some anti-hypertensive medications


because of its vasodilatory effects. Dose adjustment may be required.

Niacin also causes insulin resistance and oftentimes necessitates an

increase in diabetes medications.

86

USMLE WORLD STEP 1 PHARMACOLOGY

Q NO 61: A 64-year-old Caucasian male presents to your office complaining of a

slowing of his movements and gait instability. He was diagnosed with Parkinson

disease one year ago. After careful history taking and physical examination you

consider pramipexole therapy in this patient. The effect of the drug can be best
described by which of the following?

A. Enhances the effect of endogenous dopamine

O
B.
C.
Stimulates dopamine receptors
Decreases central dopamine degradation

D. Increases dopamine availability to the brain


E. Inhibits central muscarinic receptors

Explanation:

Dopamine agonists have a chemical structure similar to the

neurotransmitter dopamine and directly stimulate dopamine receptors.


These drugs do not have to be metabolized to be active. Dopamine

agonists preferentially stimulate D 2 receptors. There are two classes


of dopamine agonists:

1. Ergot compounds: bromocriptine and pergolide

87

USMLE WORLD STEP 1 PHARMACOLOGY

2. Nonergot compounds: pramipexole and ropinerole

Dopamine agonists have an important role in the treatment of Parkinson


disease (PD) because these medications have a long half-life and

prolong the effects of levodopa, thus limiting motor fluctuations.

Bromocriptine also treats hyperprolactinemia.


(Choice A) Amantadine exerts several effects of interest with regard to

Parkinson disease. First, as an indirect and direct dopaminergic agent,

it alleviates some of the motor symptoms of Parkinson disease by


enhancing the effects of endogenous dopamine. It is also thought to

have some anticholinergic properties, which reduce tremors. Amantadine

is primarily used as an antiviral influenza agent.


(Choice C) “Decreased central dopamine degradation” describes the

action of selegiline, a monoamine oxidase-B (MAO-B) inhibitor, used to

delay the clinical progression of PD. The enzyme MAO-B is present in


the brain: therefore, selegiline prevents the degradation of dopamine

in the brain.

(Choice D) Both catechol-O-methyl-transferase (COMT) inhibitors and


dopa decarboxylase inhibitors (DDI) decrease the breakdown of levodopa

in peripheral tissues and increase the amount of levodopa available to

cross blood brain barrier.


(Choice E) Anticholinergics, such as trihexyphenidyl and benztropine,

inhibit central muscarinic receptors. These drugs are mainly used for

drug-induced Parkinsonism. Patients with tremor as the predominant


symptom of idiopathic Parkinson disease also respond well.

Educational Objective:
Dopamine agonists have a chemical structural similar to the

neurotransmitter dopamine and directly stimulate dopamine receptors.

There are two classes of dopamine agonists:


1. Ergot compounds—bromocriptine and pergolide

2. Nonergot compounds—pramipexole and ropinerole

88

USMLE WORLD STEP 1 PHARMACOLOGY

Q NO 62: A 20-year-old female has severe acne resistant to topical therapies. She

has no significant past medical history and her only current medication is an oral

contraceptive. Before prescribing isotretinoin, which of the following tests should

you perform?

A. Platelet count

B. Bone density test


C. Serum creatine kinase level

D. Serum β-HCG level

E. Serum retinol level


F. Serum TSH level

Explanation:
Isotretinoin (Accutane) is a synthetic 13-cis-isomer of naturally

occurring all-trans-retinoic acid (tretinoin), a derivative of vitamin

A. It can be administered systemically to treat severe acne when


topical tretinoin, benzoyl peroxide, and antibiotics fail. Like other

retinoids, it inhibits follicular epidermal keratinization, thus

loosening the keratin plugs of comedones and facilitating their


expulsion. It also reduces the size of sebaceous glands and inhibits

sebum production. Some of isotretinoin’s actions may be mediated by its

interaction with a family of nuclear retinoic acid receptors (RARs)


that affect cell proliferation and differentiation. This interaction

may account for its severe teratogenic effects. Thus, systemic use of

isotretinoin is contraindicated during pregnancy. Before prescribing


this medication to a female of child-bearing age one must ensure that a

pregnancy test is negative, even if the patient is already using

contraceptives (none of which are 100% effective at preventing


conception). Sexually active patients should be advised to use two

forms of contraception and to have monthly pregnancy tests.

Educational Objective:

Isotretinoin is used to treat severe acne refractory to topical

therapies. Its main adverse effects are hypertriglyceridemia and


teratogenicity. Pregnancy is an absolute contraindication to its use.

Sexually active females should be advised to use two forms of

contraception and to have monthly pregnancy tests.

89

USMLE WORLD STEP 1 PHARMACOLOGY

Q NO 63: A new inhaled anesthetic has been developed and is tested in a series of

experiments. Anesthetic tension in the arterial blood is shown on the graph below

as a function of time after beginning inhalation (Drug A). A similar curve for

nitrous oxide is also shown.

Which of the following best describes the properties of the new


anesthetic compared to nitrous oxide’?

A. High blood: gas partitioning coefficient


B. Low solubility in the blood

C. Rapid onset of action

D. Low potency
E. Fast equilibration with the brain

Explanation:
The depth of anesthesia depends on the partial pressure of anesthetic

in CNS. The transfer of anesthetic into the brain starts only after the

blood is fully saturated (or, in other words partial pressure of the


anesthetic in blood equals the partial pressure in the inspired air).

The speed of transfer of anesthetic to the brain determines its onset

of action (rapid vs slow induction of anesthesia) and is dependent on


the solubility of anesthetic in the blood. Solubility of an anesthetic

is directly related to its blood/gas partition coefficient: highly

soluble anesthetics have high blood/gas partition coefficient.


If the agent is poorly soluble the amount of gas needed to saturate the

blood is small and saturation occurs fairly quickly. Nitrous oxide is


an example of poorly soluble gas with a blood/gas partition coefficient

of 47. On the graph above the curve of partial pressure of NO in blood

rises rapidly. In the highest point on the curve the partial pressure
on NO in blood equals that in the inspired air, and the transfer to

brain occurs.

The second curve (drug A) portrays the process of blood saturation for
a highly soluble gas. The higher the solubility the more gas can be

taken up by blood before it is saturated. Note that the curve of the

partial pressure of drug A in blood rises slower than that for NO. When
the blood is fully saturated with NO the partial pressure of drug A in

blood is approximately 25% of that in inspired air. For drug A, it

takes a longer time to fully saturate the blood and to start transfer

90

USMLE WORLD STEP 1 PHARMACOLOGY

in tissues. Drug A, therefore is characterized with high blood/gas

partition coefficient and slower onset of action.

(Choices B, C and E) Nitrous oxide is an example of a gas with low

solubility in the blood. It is characterized by rapid onset of action


and fast equilibration with the brain.

(Choice D) Potency of anesthetic is defined by minimal alveolar

concentration of the gas that produces the desired effect in 50% of the
patients (MAC). Anesthetic tension in the blood is nota measure of

potency.

Educational objective:

The onset of action of inhalation anesthetic depends on its solubility

in the blood (blood/gas partition coefficient). Drug A is an example of


higher soluble gas with slow onset of action and slow equilibration

with the brain.

91

USMLE WORLD STEP 1 PHARMACOLOGY

Q NO 64: A 66-year-old Caucasian male is treated with flutamide for metastatic

prostatic cancer. He experiences significant relief of his bone pain soon after

initiation of the therapy. The primary tumor decreases in size. Which of the

following is the best explanation for the changes observed in this patient?

A. Decreased Leydig cell stimulation

B. Decreased Leydig cell androgen synthesis


C. Decreased peripheral androgen aromatization

D. Decreased peripheral androgen conversion

E. Impaired ligand-receptor interaction

Explanation:

Prostate cancer is the most frequently diagnosed malignancy in men.


Prostate cancer is testosterone-dependent and is treated with surgical

and pharmacological androgen ablation.

Flutamide is a non-steroid antiandrogen used for the treatment of


prostate cancer. It competes with testosterone and DHT for their

receptors in the target cells. Prevention of androgen-receptor binding

blocks the stimulatory effect of androgens on the primary tumor and


metastases and leads to a reduction in their size. If flutamide is used

as monotherapy, it causes a gradual increase in blood testosterone

levels due to negative feedback mechanism. This effect is reduced by


the simultaneous administration of GnRH agonists.

Side effects of flutamide include those found in androgen depletion

(hot flashes, gynecomastia, and impotence).


(Choice A) Gonadotropin-releasing hormone (GnRH) agonists (leuprolide,

goserelin, nafarelin and histrelin) bind to GnRH receptors in the

anterior pituitary and inhibit synthesis of LH and FSHI if administered


continuously. Decreased amount of LH leads to decreased Leydig cell

stimulation and diminished testosterone synthesis.

(Choice B) Ketoconazole is a weak anti androgen that decreases


synthesis of steroid hormones in gonads and adrenals. Flutamide does

not affect testosterone production by Leydig cells.

(Choice C) Decreased peripheral androgen aromatization refers to the


mechanism of anastrozole, a nonsteroidal aromatase inhibitor which

blocks estrogen production selectively. Anastrozole is an effective

treatment for postmenopausal women with breast cancer in whom the


greatest source of estrogen is the conversion of androstenedione,

produced in the adrenal glands, to estrone in liver, muscle, and fat,


through aromatization.

(Choice D) Finasteride decreases peripheral conversion of testosterone

into dihydrotestosterone by inhibiting the 5-α- reductase. It is used


for treatment of BPH and male baldness.

Educational Objective:
Flutamide is a non-steroid anti-androgen that competes with

testosterone and DHT for testosterone receptors. Itis used for

treatment of prostate cancer in combination with GnRH agonists.

92

USMLE WORLD STEP 1 PHARMACOLOGY

93

USMLE WORLD STEP 1 PHARMACOLOGY

Q NO 65: A 43-year-old male with multiple medical problems including seizure

disorder and mental illness comes to your office for the first time in order to

establish a primary care physician. On routine physical examination you notice

the findings shown in the photo below. Which of the following is most likely
responsible for the condition of this patient’s teeth and gums?

A. Phenytoin

B. Lithium

C. Prednisone
D. Cocaine

E. Methimazole

F. Clozapine

Explanation:
Gingival hyperplasia, like that shown in the photo above, is a common

side effect of phenytoin. It occurs in 50% of patients who have had 3-4

months of phenytoin therapy. The mechanism of this side effect is


associated with increased expression of platelet-derived growth factor

(PDGF). When gingival macrophages are exposed to increased amounts of


PDGFI they stimulate proliferation of gingival cells and alveolar bone.

Gingival hyperplasia may regress after discontinuation of phenytoin.

Phenytoin is an anticonvulsant effective in the treatment of grand mal


(tonic-clonic) seizures partial seizures, and status epilepticus.

Phenytoin inhibits abnormal electrical activity of the brain by

blocking voltage-gated sodium channels in neurons. Phenytoin has a


narrow therapeutic index with a number of potential adverse effects:

1. Phenytoin toxicity is mainly characterized by CNS manifestations.

Its effect on the cerebellum and vestibular system can cause ataxia and
nystagmus.

2. Long-term therapy with phenytoin may cause gingival hyperplasia,

coarsening of the facial features and hirsutism.


3. Phenytoin interferes with the metabolism of vitamin B12 and may

cause megaloblastic anemia.

4. Phenytoin induces the P450 cytochrome oxidase system. It increases


the metabolism—and therefore decreases the blood level—of many

medications.

5. If taken during pregnancy, phenytoin may cause fetal hydantoin


syndrome.

(Choice B) Lithium also has a narrow therapeutic index. Lithium

toxicity is associated with neuromuscular excitability (irregular


coarse tremors, fascicular twitching, rigid motor agitation, and

ataxia). It may also cause nephrogenic diabetes insipidus,

hypothyroidism, and cardiac conduction defects, but not gingival


hyperplasia.

94

USMLE WORLD STEP 1 PHARMACOLOGY

(Choice C) The adverse effects of prednisone include hyperglycemia,

weight gain, depression, fluid retention with edema, and osteoporosis.


(Choice D) Cocaine blocks the re-uptake of norepinephrine, serotonin,

and dopamine in the CNS. It can cause cardiac ischemia, hypertension,

and seizures. Cocaine withdrawal is associated with severe depression.


(Choice E) Methimazole inhibits thyroid hormone synthesis by

suppressing the iodination and coupling of tyrosine. Side effects

include edema, rash, and agranulocytosis.


(Choice F) Clozapine is an atypical antipsychotic that is most helpful

in treating the negative symptoms of schizophrenia. The most serious

side effects of clozapine include agranulocytosis and seizures.

Educational Objective:

Gingival hyperplasia is a common side effect of phenytoin and is


sometimes reversible when phenytoin is withdrawn. Phenytoin causes

increased expression of platelet-derived growth factor (PDGF). When

gingival macrophages are exposed to increased amounts of PDGFI they


stimulate proliferation of gingival cells and alveolar bone. Phenytoin

toxicity mainly affects the cerebellum and vestibular system, causing

ataxia and nystagmus.

95

USMLE WORLD STEP 1 PHARMACOLOGY

Q NO 66: A 72-year-old male is hospitalized with acute myocardial infarction

resulting in cardiogenic shock. Physical examination reveals bilateral


A.

crackles, 33 gallop distended jugular veins and 2 + edema of lower

extremities. He is treated with dobutamine, furosemide, nitroglycerin and


captopril. Compared with his other medications, which of the following

effects is most specific for dobutamine in this patient?

Increased urine output

B. Improved left ventricular function


C. Increased heart rate

D. Increased conduction velocity

E. Decreased peripheral vascular resistance

Explanation:
Dobutamine is a β-adrenergic agonist with predominant activity on β1

receptors, and some activity on β2 and al receptors. It is used for

treatment of acute heart failure associated with decreased myocardial


contractility (such as cardiogenic shock). Hemodynamic effects of

dobutamine are mediated by βl adrenoreceptors of the myocardium, which

are linked to cAMP signal transduction pathway. Stimulation of β1


adrenoreceptors results in increased production of cAMP in the target

cells and causes the following effects:

1. Increase in cardiac contractility is the most useful clinical effect


of dobutamine. It leads to increase in cardiac output and decrease in

the ventricular filling pressures.

2. Dobutamine also increases heart rate and myocardial oxygen


consumption. These are undesirable effects that can produce ischemia.

Dobutamine-associated tachycardia and increase in O2 consumption,

however, are less pronounced than those caused by dopamine.


3. Dobutamine increases cardiac conduction velocity. It is also an

undesirable effect as it may result in arrhythmias.

(Choice A) Dobutamine increases cardiac output and improves renal


perfusion, which may result in increase in urine output. Furosemide

also increases urinary output because of its diuretic effect.

(Choice B) Improved left ventricular function is associated with both


dobutamine and furosemide. By decreasing intravascular volume (through

diuretic effect) it allows the heart to pump more effectively.

(Choice C) Dobutamine and nitroglycerine both increase the heart rate.


Nitroglycerine causes venous pooling and thus leads to reflex

tachycardia.

(Choice E) Effect of dobutamine on β2 adrenoreceptors decreases


peripheral vascular resistance. Among the medications listed above this

effect is also typical for nitroglycerine and captopril.

Educational Objective:

Dobutamine is a relatively selective β1-adrenergic agonist that

increases heart rate, contractility, conduction and myocardial oxygen


consumption. Increase in conduction velocity is not associated with

other drugs mentioned in this clinical scenario.

96

USMLE WORLD STEP 1 PHARMACOLOGY

Q NO 67: A 24-year-old male presents to the emergency department with sudden

onset of palpitations. He had an episode similar to this one year ago that resolved

spontaneously. Rapid IV injection of a drug into this patient results in

instantaneous resolution of the arrhythmia but is accompanied with transitory


flushing, burning in the chest and shortness of breath. Which of the following

drugs was used to treat this patient’s condition?

A.
B.
Amiodarone
Lidocaine

C. Procainamide

D.
E.
Verapamil
Adenosine

F. Digoxin

Explanation:

This patient is exhibiting symptoms consistent with paroxysmal


supraventricular tachycardia (PS VT), and adenosine is the drug of

choice for the treatment of this arrhythmia. As the name implies PSVT

comes on suddenly and the focus of automaticity lies above the


ventricles (i.e. in the tissues of the atria or AV node). It can

frequently be dependent on a reentrant circuit. Adenosine is a very

rapid acting drug with a half-life of only less than 10 seconds. It


acts by slowing conduction through the AV node by hyperpolarizing the

nodal pacemaker and conducting cells. The side effects most frequently

associated with adenosine are flushing, chest burning (due to


bronchospasm), hypotension and high grade AV block.

(Choice A) Amiodarone is a class Ill antiarrhythmic and is useful in

both supraventricular (atrial, nodal junctional) and ventricular


tachyarrhythmia. This is one of the broadest spectrum antiarrhythmic

drug available. It is also known for its side effect profile which

includes photodermatitis, a blue/grey skin discoloration pulmonary


fibrosis, and either hyperon hypothyroidism.

(Choice B) Lidocaine is a class 1B antiarrhythmic and overdose or

toxicity with lidocaine most commonly causes neurologic symptoms. It is


usually indicated for ventricular arrhythmias.

(Choice C) Procainamide is a class IA antiarrhythmic and is commonly

associated with drug-induced lupus syndrome.


(Choice D) Verapamil is a class IV antiarrhythmic and is the most

cardioselective of the calcium channel blockers. It is a potent

negative inotrope and previously was the drug of choice for PSVT.
However this has been replaced by adenosine. Additionally the most

frequent adverse reactions noted with verapamil are constipation and

gingival hyperplasia.
(Choice F) Digoxin slows conduction through the AV node and is also a

positive inotrope. Digoxin/Digitalis causes a well-characterized

toxicity; symptoms include fatigue, blurry vision changes in color


perception nausea and vomiting, diarrhea, abdominal pain confusion and

delirium.

Educational Objective:

Adenosine is a rapidly acting antiarrhythmic used to quickly convert


people out of PSVT (drug of choice). It is also rapidly cleared and has

a half-life of only less than 10 seconds. It commonly causes chest

burning (bronchospasm), flushing and high grade block as adverse


reactions (remember, this is the drug used for chemical stress tests)

97

USMLE WORLD STEP 1 PHARMACOLOGY

Q NO 68: An 8-year-old Asian immigrant is brought to the ER after a recent episode

of brief loss of consciousness. His mother, who accompanies him to the hospital,

says that he lost consciousness suddenly, fell on the floor and ‘started shaking and

jerking’. She also claims that her son has had episodes of staring in the space that
occur several times a day for the last year. The boy seems sleepy although he is

oriented in time and place. Several hours later, he is alert and says that he wants to

go home. Which of the following agents is the best long-term treatment for this
patient?

A. Ethosuximide

B. Phenytoin
C. Phenobarbital

D. Valproate

E. Carbamazepine

Explanation:

This patient most likely has a combination of both tonic-clonic


seizures and absence seizures. Both valproic acid and ethosuximide can

be used for absence seizures. Valproate is preferred in patients with

combined absence and tonic- clonic seizures. Ethosuximide is not


effective against generalized tonic-clonic seizures.

(Choice A) Ethosuximide is approved for the treatment of absence


seizures. Ethosuximide blocks T-type Ca2 channels that trigger and

sustain rhythmic pulsed discharges in thalamic neurons.

(Choice B) Phenytoin inhibits neuronal high-frequency firing of action


potentials by blocking sodium (Na+ channels and prolonging their rate

of recovery. Phenytoin is used in the management of tonic-clonic

seizures and status epilepticus but is not the first line medication
for absence seizures.

(Choice C) Phenobarbital is effective for generalized tonic-clonic

seizures, but is not the first line medication for absence seizures.
(Choice E) Carbamazepine is the drug of choice for complex partial

seizures and is also effective in generalized tonic-clonic seizures.

The most feared, though uncommon, side effect of carbamazepine is


agranulocytosis or aplastic anemia.

Educational Objective:
Sodium valproate is the drug of choice for patients with absence and

associated tonic-clonic seizures. Ethosuximide is also effective

against absence seizures but does not suppress tonic-clonic seizures.

98

USMLE WORLD STEP 1 PHARMACOLOGY

Q NO 69: A 14-year-old Caucasian male with recurrent respiratory infections with

Pseudomonas aeruginosa suffers from chronic diarrhea and weight loss. His

younger brother died from a severe respiratory infection at the age of 9. Which of

the following agents is most likely to improve this patient’s condition?

A. Octreotide

B. Pancreatic lipase
C. Metronidazole

D. Gluten-free diet

E. Antimotility agents
F. Sulfasalazine

Explanation:
The history of recurrent respiratory infections with Pseudomonas

aeruginosa, chronic diarrhea, weight loss, and death of a 9-year-old

sibling due to respiratory infection together suggest a diagnosis of


cystic fibrosis (CE). P. aeruginosa is a common pulmonary pathogen in

CE. Diarrhea and weight loss occur as a result of malabsorption due to

obstructive fibrosis and progressive insufficiency of the exocrine


pancreas. Of the choices given, pancreatic lipase supplementation would

be most likely to improve the patient’s condition.

(Choice A) Octreotide is a somatostatin analogue that decreases gut


motility gut blood flow, and gut endocrine and exocrine function.

Octreotide decreases secretion of pancreatic enzymes into the intestine

and could therefore aggravate this patient’s malabsorptive diarrhea and


associated nutritional deficiencies.

(Choice C) Metronidazole is effective only for diarrhea due to

infectious causes.
(Choice D) A gluten-free diet is the mainstay of treatment for celiac

sprue but would be unlikely to help this patient’s malabsorption, which

is the result of pancreatic exocrine insufficiency.


(Choice E) An agent that decreases gastrointestinal motility, such as

an opiate preparation possibly combined with atropine, could help

decrease the volume of the patient’s diarrhea by decreasing intestinal


transit time and allowing greater net fluid resorption from the bowel.

However, it would be unlikely to improve the deficiencies in

macronutrient absorption due exocrine pancreatic failure.


(Choice F) 5-aminosalicylates such as sulfasalazine are used to treat

inflammatory bowel diseases like Crohn’s disease and ulcerative


colitis. They work by inhibiting prostaglandin and leukotriene

synthesis during inflammation. However, this patient’s diarrhea and

weight loss are due to a non-inflammatory cause.

Educational Objective:

In a young Caucasian, a history of recurrent respiratory infections


with P. aeruginosa, chronic diarrhea, weight loss, and death of a

sibling due to respiratory infection suggests a diagnosis of cystic

fibrosis (CE). Chronic diarrhea and weight loss in patients with CE are
typically caused by malabsorption secondary to pancreatic insufficiency

and can be corrected by pancreatic enzyme supplementation.

99

USMLE WORLD STEP 1 PHARMACOLOGY

Q NO 70: A 58-year-old female has severe osteoarthritic back pain. Which of the

following aspects of her medical history would make celecoxib the preferred

treatment?

A Renal impairment

B. Peptic ulcer disease

C. Hyperlipidemia
D. Stable angina

E. Hypertension

Explanation:

Non-steroidal anti-inflammatory drugs (NSAIDs) are commonly used to

treat pain, arthritis, and fever. NSAIDs work by inhibiting the enzyme
cyclooxygenase (CCX) in order to block prostaglandin synthesis. Most

NSAIDs inhibit both the CCX 1 and COX2 isoenzymes. Whereas CCX 1 plays

a physiologic role in a number of normally functioning tissues (e.g.


platelets, the gastrointestinal tract), CCX 2 is preferentially

expressed at sites of inflammation. Many of the potential NSAID adverse

effects, including GI ulceration and bleeding, are related to


inhibition of CCX 1. Selective CCX 2 inhibitors were developed to avoid

these complications. These agents have potent anti-inflammatory effects

with less risk of bleeding and gastrointestinal ulceration.


(Choices A and E) Both CCX 1 and CCX2 are expressed in renal tissues.

Therapy with NSAIDs or selective CCX 2 inhibitors can cause fluid

retention and aggravation of hypertension.


(Choice D) CCX 2 is expressed in vascular endothelial cells and

vascular smooth muscle cells where it plays a role in the local

production of prostacyclin (P012), a substance that promotes anti-


coagulation and vascular dilatation. Some selective CCX 2 inhibitors

have been associated with an increased risk of cardiovascular events,

which may be related to decrease PG12 production.


(Choice C) Selective CCX 2 inhibitors do not have a substantial effect

on serum lipids.

Educational Objective:

Selective CCX 2 inhibitors have no effect on platelet aggregation and


cause little 01 irritation. The reduced risks of bleeding and GI

ulceration are the only advantages over traditional NSAIDs.

100

USMLE WORLD STEP 1 PHARMACOLOGY

Q NO 71: Nitrates are useful drugs for the treatment of stable angina. They have

distinct anti-ischemic properties but sometimes can cause a paradoxical increase

in myocardial oxygen demand. Which of the following agents would be most

effective in preventing that paradoxical effect?

A. Nifedipine

B. Metoprolol
C. Hydralazine

D. Hydrochlorothiazide

E. Phenylephrine
F. Prazosin

G. N-acetylcysteine

Explanation:

Nitrates are vasodilators and can cause a decrease in blood pressure.

When this relative hypotension is sensed, the body responds by


releasing catecholamines which can cause a reflex tachycardia. The

tachycardia can increase myocardial oxygen demand. This effect can be

prevented by beta-blockers such as metoprolol which act to slow


conduction through the AV node and cardiac conduction system by

preventing catecholamines from stimulating beta-i receptors at those

sites. (Choice A) Nifedipine is a peripherally acting calcium channel


blocker and would have no effect on heart rate. More cardioselective

calcium channel blockers such as diltiazem or verapamil could be used

in this situation due to their ability to slow AV nodal conduction, but


beta-adrenergic blockers more effectively blunt the tachycardic

response occasionally induced by nitrates.

(Choice C) Hydralazine is a direct-acting arteriolar dilator which acts


by a poorly characterized mechanism. Hydralazine causes a reflex

tachycardia, so it would actually worsen the reflex tachycardia caused

by nitrates if these two agents were administered together. Hydralazine


is usually prescribed together with a betaadrenergic blocker to avoid

tachycardia.

(Choice 0) Hydrochlorothiazide is a weak diuretic and would have no


effect on preventing tachycardia. If any thing, it would decrease the

intravascular volume and cause a tachycardia.

(Choice E) Phenylephrine is an alpha-agonist agent and is classified as


a vasopressor agent. It is used in cases of shock and severe

hypotension. While this agent can cause a reflex bradycardia, its


action as a potent vasoconstrictor would be in opposition to that of a

nitrate and would therefore not be useful in this setting.

(Choice F) Prazosin is an alpha-adrenergic blocking medication useful


for the treatment of both hypertension and benign prostatic

hypertrophy. Itis a peripherally acting vasodilator and would have no

effect on the response of the heart to catecholamine stimulation.


Selective alpha-adrenergic blockers usually do not cause much reflex

tachycardia because presynaptic alpha-2 inhibition is not blocked.

(Choice G) N-acetylcysteine is a medication used as an antidote in


Tylenol (acetaminophen) poisoning as well as a renal protective agent

prior to administering intravenous iodine contrast for radiological

studies. It would have no use in this setting.


Educational Objective: Nitrates can lead to a reflex tachycardia by

causing a relative hypotension that the body responds to with

catecholamine release. This adverse effect can be prevented by


administering beta-adrenergic blockers with nitrates.

101

USMLE WORLD STEP 1 PHARMACOLOGY

Q NO 72: A 32-year-old female is being treated with antibiotics for complicated

cellulitis of the right foot. One week after starting treatment, she develops mild

fever, nausea and skin rash. Laboratory studies show elevated blood urea

nitrogen and creatinine levels as well as eosinophilia. Which of the following


medications is she receiving?

A. Nafcillin
B. Gentamicin

C. Vancomycin

D. Clindamycin
E. Erythromycin

Explanation:
A rising serum BUN and creatinine together with eosinophilia suggest

acute renal failure due to acute interstitial nephritis (AIN). The most

common cause of AIN is drug hyper sensitivity and/or toxicity. The


following drugs are very commonly associated with AIN:

1. Penicillins and cephalosporins

2. NSAIDs
3. Sulfonamides

(Choice B) Gentamicin is nephrotoxic, but causes nonoliguric acute

renal failure due to its accumulation within proximal tubule epithelial


cell lysosomes and its subsequent intracellular release and cell

damage.

(Choice C) Vancomycin is very rarely associated with nephrotoxicity,


but dose adjustments should be made in patients with renal

insufficiency. The risk of vancomycin nephrotoxicity is much increased

when itis used in combination with gentamycin.


(Choice D) Clindamycin frequently causes pseudomembranous colitis.

(Choice E) Erythromycin is associated with cholestatic jaundice. It

also inhibits hepatic cytochrome P450, resulting in significant drug


interactions (e.g. it increases plasma levels of digoxin and warfarin).

Educational Objective:
Penicillins and cephalosporins, NSAIDs, and sulfa drugs are frequently

associated with acute interstitial nephritis. Gentamicin causes acute

tubular necrosis of the proximal tubule.

102

USMLE WORLD STEP 1 PHARMACOLOGY

Q NO 73: A 44-year-old male presents to your office with impaired concentration,

loss of energy and decreased sexual drive. The symptoms started several weeks

ago and without any clear-cut precipitating event. His past medical history is

insignificant. Which of the following additional features would make phenelzine a


reasonable treatment choice for this patient?

A. Suicidal ideation MoA Ainhib

B.
C.
Auditory hallucinations
Delusions of persecution
b
resistant
treatment

0
D.
E.
Hyperphagia and hypersomnia
Weight loss depression

Explanation:
Monoamine oxidase inhibitors (MAOIs), such as phenelzine and

tranylcypromine are not considered to be first-line antidepressant

medications because of the dietary restrictions (cheese, wine), drug-


drug interactions (SSRI), and other numerous side effects. However,

they are particularly useful in patients with atypical depression and

treatment resistant depression.


Atypical depression is characterized by mood reactivity, leaden fatigue

(the patient’s arms and legs feel extremely heavy), rejection

sensitivity (overly sensitive to slight criticism), and the reversed


vegetative signs of increased sleep and appetite.

(Choice A) Suicidal ideation, while a symptom of major depression, is

not specific for atypical depression and would not be a first-line


indication for phenelzine.

(Choice B) Auditory hallucinations are seen in patients with psychosis,

especially schizophrenia. These auditory hallucinations usually are


controlled with an anti-psychotic medication such as haloperidol or

chlorpromazine.

(Choice C) Delusions of persecution may be seen in cases of


schizophrenia as well. Delusions of persecution include thoughts of a

conspiracy against the patient (i.e. the FBI has coordinated its

efforts with the local police and uses satellites to track the patient;
bugs, recorders, and listening devices have been placed in the walls;

the phone and computer are tapped). There is no escape from the

patient’s perceived delusions.


(Choice E) Weight loss has been reported with the use of SSRls; thus,

they should be used with caution in the treatment of anorectic or

underweight patients.

Educational Objective:
MAO inhibitors are particularly useful in patients with atypical

depression (eg, with hyperphagia and weight gain) and treatment

resistant depression.

103

USMLE WORLD STEP 1 PHARMACOLOGY

Q NO 74: A 70-year-old male is treated with metoprolol for long-standing

hypertension. His past medical history is significant for coronary artery


A.

disease. After initial evaluation you decide to add enalapril to his

treatment regimen. His blood pressure values normalize in response to


the therapy but laboratory tests reveal significant increase in serum

creatinine level. Which of the following is the best explanation of the

abnormality observed in this patient?

Glomerular damage
B. Renal under perfusion

C. Tubular necrosis

D. Tubular obstruction
E. Renal interstitial infiltration

Explanation:
Therapy with an ACE inhibitor can acutely reduce GER causing a rise in

serum creatinine. The pathogenesis of ACE inhibitor induced renal


failure involves a decrease in glomerular capillary hydrostatic

pressure adequate enough to reduce GER. Normally, the kidney maintains

GER by dilating the afferent arteriole and constricting the efferent


arteriole. ACE inhibitors reduce angiotensin II synthesis and activity,

causing a subsequent dilation in the efferent arteriole. This effect

reduces outflow resistance from the glomerulus and decreases


hydrostatic pressure in the glomerular capillaries ultimately

decreasing renal perfusion pressure.

It is important to distinguish between acute renal failure and a normal


predictable rise in serum creatinine secondary to ACE inhibitor

therapy. Based on their pathophysiologic effects, an increase in serum

creatinine up to 30 % within 2to 5 days can be expected in most


patients started on ACE inhibitors. This increase usually stabilizes in

2 to 3 weeks and is reversible upon drug discontinuation. However, for

patients dependent on efferent arteriole constriction to maintain renal


perfusion (those with renal artery stenosis, decompensated heart

failure, chronic kidney disease and volume depletion), ACE inhibitors

can be detrimental causing acute renal failure or complicating existing


renal disease.

(Choice A) Drug-induced glomerular disease is not common but there are

a few drugs that have been implicated. Glomerular lesions primarily


occur through immune mediated pathways rather than through direct drug

toxicity. Heroin and pamidronate are known to cause focal segmental

glomerulosclerosis. Gold therapy can cause membranous nephropathy.


(Choice C) Drug-induced tubular necrosis may be due to either direct

toxic or ischemic effects. The most common agents known to cause

tubular necrosis are aminoglycosides, radiocontrast media, cisplatin,


amphotericin B, and foscarnet.

(Choice D) Drug-induced tubular obstruction may be due to intratubular

precipitation of tissue degradation products, drugs, or drug


metabolites. Sulfonamides, methotrexate (and its metabolite),

intravenous acyclovir, and triamterene are agents that can precipitate

in renal tubules and cause renal failure.


(Choice E) Renal interstitial infiltration can be acute and reversible

or chronic and irreversible. Acute allergic interstitial nephritis is

fairly common and many drugs have been implicated. Methicillin is the
most common drug causing interstitial nephritis, but other potential

drugs include cephalosporins, acetazolamide, allopurinol, sulfonamides,

104

USMLE WORLD STEP 1 PHARMACOLOGY

and NSAIDs. Lithium and cyclosporine have been reported to cause

chronic interstitial nephritis.

Educational Objective:

Many drugs can cause nephrotoxicity and ARE via different mechanisms.
In particular, ACE inhibitors can cause an acute rise in the serum

creatinine by blocking ACE mediated efferent arteriole

vasoconstriction. This leads to decreased hydrostatic pressure and


reduced renal perfusion. For patients dependent on efferent arteriole

constriction to maintain renal perfusion (those with renal artery

stenosis), ACE inhibitors can be detrimental causing acute renal


failure.

105

USMLE WORLD STEP 1 PHARMACOLOGY

Q NO 75: A 69-year-old female presents to your office with worsening dyspnea on

exertion and easy fatigability. Laboratory evaluation reveals a hemoglobin level of

9.6 mg/dL. Which of the following supplements would most likely require

parenteral administration in this patient?

A. Iron

B. Folic acid
C. Vitamin B12

D. Vitamin C

E. Protein

Explanation:
The absorption of vitamin B12 is a complex process requiring multiple

steps. In the stomach vitamin B12 is released from food and forms a

complex with the gastric P-protein. When the complex reaches the
duodenum vitamin B12 is released and binds with intrinsic factor

produced by gastric parietal cells. Vitamin B12 binding to intrinsic


factor prevents its digestion in the small intestine. B12 is finally

absorbed in the distal ileum upon binding to a specific receptor.

In most cases deficiency of vitamin B12 is due to defective absorption


rather than insufficient dietary intake. Dietary deficiency is rare,

106

USMLE WORLD STEP 1 PHARMACOLOGY

and is usually seen only with strict vegetarians. Elderly patients may

have poor absorption due to achlorhydria. Other causes of B12


deficiency include: terminal ileal disease (e.g. inflammatory bowel

disease ileal resection) and bacterial overgrowth syndrome. Oral

administration of vitamin B12 is effective only in patients with


dietary deficiency. Thus parenteral administration of vitamin B12 is

indicated for the majority of deficient patients.

(Choice A) Dietary meat consumption provides iron in the heme form.


This type of iron is much more readily absorbed than that from

vegetable sources. Iron absorption occurs mainly in the proximal small

intestine facilitated by stomach acid. Causes of iron deficiency


include: increased demand blood loss, insufficient intake and

malabsorption. Oral iron therapy is preferred in the majority of

patients with iron deficiency anemia. Parental iron is reserved for


patients who cannot tolerate oral iron or who require rapid restoration

of the iron deficit.

(Choice B) Fruits and vegetables are the principle sources of folic


acid. Folate deficiency can occur secondary to: inadequate intake,

decreased absorption, or increased demand. Treatment of folic acid

deficiency is rarely via the parenteral route even in the setting of


folic acid malabsorption.

(Choice D) Vitamin C is nearly completely absorbed when administered in

small amounts. Vitamin C plays an important role in the intestinal


absorption of non-heme iron. Deficiency of vitamin C is usually treated

with oral supplementation.

(Choice E) Dietary protein is absorbed in the form of amino acids,


following the breakdown of polypeptides by intestinal and pancreatic

enzymes. Protein supplementation is generally via the oral route unless

the patient is unable to eat.

Educational Objective:
1. Iron deficiency is usually treated with iron supplementation along

with correcting the underlying cause (usually blood loss). Parenteral

iron therapy is reserved for patients who cannot tolerate oral iron or
who require rapid restoration of the deficit.

2. In majority of the patients with vitamin B12 deficiency, B12 is

administered parenterally. This is because dietary B12 deficiency is


quite rare. Poor absorption (intrinsic factor deficiency and terminal

ileal disease) is usually the culprit.

107

USMLE WORLD STEP 1 PHARMACOLOGY

Q NO 76: A 62-year-old Caucasian female is treated with alendronate after a

vertebral compression fracture she experienced six months ago. The follow-

up quantitative X-ray densitometry demonstrates an increase in bone mass

when compared to the baseline. The agent used to treat this patient’s bone
disease is structurally similar to which of the following substances?

A. Oxalate

B. Citrate
C. Hydroxyproline

D. Fumarate

E. Pyrophosphate
F. Urate

Explanation:
Pharmacological agents approved by the FDA for the treatment of

osteoporosis include estrogen, selective estrogen receptor modulators

(SERMs), bisphosphonates, calcitonin, and parathyroid hormone.


Bisphosphonates are the most commonly used medications for

osteoporosis. Bisphosphonates are structural analogues of

pyrophosphate, an important component of hydroxyapatite. The suffix “-


dronate” indicates a bisphosphonate, as in alendronate (Fosamax),

risedronate, and ibandronate.

In addition to making the hydroxyapatite more insoluble bisphosphonates


decrease bone resorption by interfering with osteoclasts.

The gastrointestinal absorption of most bisphosphonates is extremely

poor. Less than 1% of alendronate is absorbed after oral


administration, so these agents are administered in the fasting state

with plenty of plain water. Patients also must stay upright for at

least 30 minutes to prevent reflux because these agents are caustic to


the esophagus.

Bisphosphonates should be used carefully in patients with renal failure

as they are excreted unchanged in the urine.

Educational Objective:

Bisphosphonates are structural analogues of pyrophosphate, an important


component of hydroxyapatite. These drugs are used in the treatment of

osteoporosis Paget’s disease of the bone, and malignancy-induced

hypercalcemia. These agents are administered in the fasting state with


plenty of water. The patient must also stay upright for at least 30

minutes to prevent reflux esophagitis.

108

USMLE WORLD STEP 1 PHARMACOLOGY

Q NO 77: A 70-year-old male is brought to the ER with severe agitation and

aggressive behavior. He was diagnosed with dementia six months ago that
A.

has not been medically treated. After the initial treatment the patient is

responsive and cooperative. The following day he experiences high fever,


confusion and muscle rigidity. Which of the following is the most likely

diagnosis in this patient?

Acute dystonia

B. Neuroleptic-induced Parkinsonism
C. Serotonin syndrome

D. Akathisia

E. Tardive dyskinesia

O
F. Neuroleptic malignant syndrome

Explanation:

Haloperidol is commonly used in the treatment of agitation -- it is the

drug most commonly associated with neuroleptic malignant syndrome


(NMS). While the diagnosis requires a recent history of exposure to a

neuroleptic medication

(Usually within 7—10 days of onset of the syndrome), NMS can occur with
chronic usage. The predominant cause of

NMS appears to be the specific anti-dopaminergic activity of

antipsychotic medications, particularly antagonism of the D2 receptor.


Central dopaminergic systems are involved in thermoregulation as well

as regulation of muscle tone and movement.

The usual presentation consists of four primary features: (1)


hyperthermia, (2) extreme generalized rigidity, (3) autonomic

instability, and (4) altered mental status.

(Choice A) Acute dystonia is the least common but the most disabling of
the hyperkinetic movement disorders associated with anti-psychotic use.

It is defined by twisting involuntary movements and abnormal postures.

(Choice B) Neuroleptic-induced Parkinsonism results from excessive


dopamine blockade within the nigrostriatal tissues and may manifest

with rigidity, bradykinesia and tremor.

(Choice C) Combined use of SSRIs and MAOIs is classically associated


with the serotonin syndrome, a potentially fatal complication arising

from any pharmacologic agent that increases serotoninergic tone in the

central nervous system. As opposed to the serotonin syndrome, the


Neuroleptic Malignant Syndrome is distinguished clinically by an

absence of myoclonus and by the presence of rigidity.

(Choice D) Akathisia is a subjective feeling of restlessness or


objective motor restlessness.

(Choice E) Tardive dyskinesia is a chronic, dose-related movement

disorder, which manifests as persistent lip smacking or rhythmic tongue


and chewing movements.

Educational Objective:
As opposed to the serotonin syndrome, the Neuroleptic Malignant

Syndrome is distinguished clinically by an absence of myoclonus and by


the presence of rigidity. Dopamine agonists (bromocriptine) and/or

direct muscle relaxants (dantrolene) have been used to decrease

mortality rates associated with the neuroleptic malignant syndrome


(NMS), but there is no way to prevent NMS.

109

USMLE WORLD STEP 1 PHARMACOLOGY

Q NO 78: A 26-year-old female is brought to the physician’s office because of

constipation dry skin, hair loss and weight gain. Her past medical history is

significant or mood disorder and psychotic outbursts. Her HR is 53/mm and

BP is 110/70 mm of Hg. If her current condition is due to a side effect of


medication which of the following is most likely responsible?

A. Valproic acid

B. Fluphenazine
C. Lithium

D. Clozapine

E. Citalopram
F. Risperidone

G. Carbamazepine

Explanation:

This patient most likely has hypothyroidism. Thyroid hypofunction is a

well-known side effect of lithium therapy. Periodic measurement of


serum TSH level is recommended. Lithium is almost exclusively excreted

by the kidneys. Thus long-term lithium maintenance requires periodic

monitoring of blood lithium levels and thyroid (TSH) and renal function
(BUN and creatinine). Additionally, lithium toxicity has been

associated with nephrogenic diabetes insipidus.

(Choice A) Valproic acid is associated with hepatitis and


teratogenicity.

(Choice B) Fluphenazine is a high potency traditional antipsychotic

and, thus, is associated with extrapyramidal symptoms.


(Choice D) Clozapine (an atypical antipsychotic) causes agranulocytosis

and seizures.

(Choice E) Citalopram is a SSRII and associated side effects apart from


sexual dysfunction, are uncommon.

(Choice F) Risperidone is an atypical antipsychotic and its common side

effect is hyperprolactinemia.
(Choice G) Carbamazepine cause agranulocytosis as well and thus

requires monitoring of complete blood count.

Educational Objective:

Lithium can cause nephrogenic diabetes insipidus and hypothyroidism.

Lithium is also a teratogenic and can cause Ebstein’s anomaly.

110

USMLE WORLD STEP 1 PHARMACOLOGY

Q NO 79: A 20-year-old female is brought to the FR because of intractable vomiting

and abdominal pain. She has a several week history of anorexia and weight loss. On

physical examination, she is afebrile with a blood pressure of 80/40 mm Hg and a

heart rate of 120 beats per minute. There is generalized hyperpigmentation, most
conspicuous over the face, neck and dorsal hands. Urinalysis and chest x-ray are

within normal limits. Finger-stick blood glucose is 60 mg/dL. This patient requires

immediate treatment with which of the following?

A. Epinephrine

B. Insulin

C.
D.
Corticosteroids
Antibiotics

E. Dopamine

Explanation:

This patient’s presentation is consistent with adrenal crisis. She is


hypotensive, tachycardic, and hypoglycemic, with a history consistent

with adrenal insufficiency (vomiting, abdominal pain weight loss, and

hyperpigmentation). When there is clinical suspicion of adrenal crisis,


immediate treatment with corticosteroids (stress doses of

hydrocortisone) is indicated without waiting for biochemical

confirmation. The response to corticosteroid therapy in adrenal crisis


is generally very rapid.

In normal individuals, the adrenal glands respond to stressful

situations (e.g. infection surgery) by secreting large quantities of


glucocorticoid (3 to 9 times baseline levels depending on the degree of

stress) as cortisol is essential to cardiovascular and metabolic

adaptations during stress. Patients with adrenal insufficiency cannot


mount this response and therefore develop shock-like states.

(Choice A) Epinephrine is used in the treatment of anaphylaxis, severe

asthma and cardiac arrest. The patient in the vignette does not have
features of asthma or anaphylactic shock. Because epinephrine

stimulates both alpha- and beta-adrenergic receptors (which promote

vasoconstriction and vasodilatation, respectively), it is generally not


a potent vasoconstrictor.

(Choice B) Insulin is used to treat hyperglycemia associated with

diabetes mellitus. However this patient has blood sugars of 60 mg/dL.


Using insulin here could be catastrophic.

(Choice D) Antibiotics may necessary in this patient if any evidence of

infection is found. However, treatment with corticosteroids is


warranted immediately, whereas the decision to start her on an

antibiotic may be made subsequently when the presence or absence of

infection is more clearly established.


(Choice E) Dopamine infusion can be used in the treatment of shock and

refractory cardiac failure. However the response to vasopressors is

usually suboptimal in the setting of adrenal insufficiency. This is


because glucocorticoids have a permissive effect on vasoconstriction.

Thus, a lack of glucocorticoids means that the patient’s response to

vasopressors will be limited.

Educational Objective:

The ideal treatment for patients in adrenal crisis is administration of


stress-dose corticosteroids. Response to vasopressors in the presence

of adrenal insufficiency is generally suboptimal.

111

USMLE WORLD STEP 1 PHARMACOLOGY

Q NO 80: A 75-year-old Caucasian male is brought to your office with a pruritic

rash. He has poor vision and a history of frequent falls. Which of the following

would be the most appropriate medication considering this patient’s past

medical history?

A. Hydroxyzine

B. Promethazine
C. Fexofenadine

D. Chlorpheniramine

E. Diphenhydramine

Explanation:

The histamine released from mast cells and basophils plays an important
role in allergic inflammation by acting on the Hi histaminic receptor.

Antihistaminics (Hi histamine receptor blockers) are very useful drugs

for the treatment of allergy. The side effects of antihistaminics are


largely due to blockade of other pathways, including muscarinic (blurry

vision, dry mouth urinary retention) serotonergic (appetite

stimulation), and alpha-adrenergic (postural dizziness) pathways.


Blockade of these pathways commonly occurs with first-generation

antihistaminics. Furthermore, first- generation antihistaminics are

lipophilic, and therefore easily cross the blood-brain barrier,


interfering with neuronal transmission and causing sedation and

cognitive dysfunction.

Second-generation anti histamines do not have antimuscarinic, anti


serotonergic, or anti-alpha adrenergic properties: thus, their side

effects are minimal. Moreover, second-generation antihistamines do not

cross the blood-brain barrier, and are usually non-sedating.


The patient in the vignette has preexisting problems (visual

difficulty, frequent falls) that would likely be aggravated by first-

generation antihistaminics, such as hydroxyzine, promethazine,


chlorpheniramine and diphenhydramine. Thus, these drugs should be

avoided in this patient. Chlorpheniramine, diphenhydramine and

promethazine can cause sedation and blurry vision, increasing the risk
of falls. Second-generation antihistaminics would be a better choice

for this patient. Fexofenadine is a second-generation antihistaminic

that causes minimal sedation and antimuscarinic effects.

Educational Objective:
Second-generation antihistamines like fexofenadine have minimal

sedative and antimuscarinic effects.

112

USMLE WORLD STEP 1 PHARMACOLOGY

Q NO 81: A 56-year-old male with advanced HIV infection experiences weight loss,

fevers and severe weakness. His liver and spleen are enlarged on physical

examination. Blood cultures produce colonies of acid-fast bacteria that grow best at

41C. This patient’s condition could have been prevented with the administration of:

A. Isoniazid

B. Rifampin
C. Penicillin

D. Azithromycin

E. Trimethoprim

Explanation:

The patient described here is suffering from an infection caused by the


Mycobacterium avium complex (MAC). MAC is characterized by infection

with either M. avium or M. Intracellulare and is occasionally referred

to as the Mycobacterium avium-intracellulare (MAI) complex. MAC can


affect both immunocompetent and immunosuppressed hosts, with the

preponderance of infections occurring in HIV+ patients who have CD4

lymphocyte counts less than 50 cells/mL. Research has shown that


administering weekly azithromycin to at-risk HIV+ patients (CD4 < 50)

can prevent the majority of cases of MAC infection.

MAC typically presents with the nonspecific symptoms off ever weight
loss and diarrhea in an HIV+ patient. Entry is through the respiratory

route. Because MAC tends to be systemically disseminated in this

patient population, the diagnosis is most commonly made with a blood


culture positive for the acid-fast organisms of MAC. MAC is resistant

to many of the typical antimycobacterial drugs. When an active

infection is identified, treatment is with combination therapy. The


drugs used to treat MAC include clarithromycin or azithromycin with any

combination of rifabutin and/or ethambutol. Pharmacologic HIV treatment

to bring CD4 counts above 200 is also recommended.


(Choice A) Isoniazid is used as a prophylactic medication in HIV+

patients who have been exposed to M. tuberculosis or who have recently

had a positive PPD conversion. Remember that for HIV+ patients, a PPD
induration reaction measuring 5 mm is considered a positive result.

(Choice B) Rifampin is used as antibiotic prophylaxis for individuals

who have been exposed to an active case of Neisseria meningitidis


(meningitis meningococcemia) or Haemophilus influenzae. Unfortunately

rifampin and isoniazid have limited if any efficacy against MACI


despite the fact that these agents are first-line treatment for M.

tuberculosis.

(Choice C) Penicillin is classically used in the treatment of


infections with Treponema Neisseria, and Gram-positive organisms.

(Choice E) Trimethoprim-sulfamethoxazole is used in HIV+ patients as

prophylaxis against Pneumocystis carinii pneumonia (PCP) and Toxoplasma


gondii infections. These protozoal infections tend to strike HIV+

patients whose CD4 counts have fallen below 200 or 100 respectively.

PCP causes a pneumonia characterized by fever and productive cough.

Educational Objective:

Mycobacterium avium is a common opportunistic pathogen that causes


disseminated disease (MAC) in HIV+ patients. Weekly azithromycin is

used as prophylaxis in patients at risk for MAC (eg HIV+ patients with

CD4 counts less than 50 cells/mL).

113

USMLE WORLD STEP 1 PHARMACOLOGY

Q NO 82: A 43-year-old HI V-positive male presents to the emergency department

complaining of progressive vision impairment. He was previously placed on

antiretroviral therapy but has been noncompliant with prescribed medications. His

last CD4+ cell count was 47/mcL. Ophthalmoscopy reveals the following: Which of
the following medications is the best initial therapy for this patient?

A. Pentamidine
B. Clarithromycin

C. Acyclovir

D. Ganciclovir
E. Flucytosine

F. Interferon-a

Explanation:

The inflammatory vascular sheathing and associated hemorrhage shown in

the above image is suggestive of cytomegalovirus-induced retinitis.


Histologic examination of the affected tissue would reveal full-

thickness retinal necrosis and edema with eventual replacement by

atrophic scary tissue.


The most common cause of retinitis in HIV-positive patients is

cytomegalovirus (CMV) infection, which is thought to develop secondary

to hematogenous spread of CMV to the eye. CMV retinitis affects 30-40%


of AIDS patients who have a CD4+ cell count below 50/mm3, with 25-40%

of these patients developing retinal detachment.

CMV retinitis is treated with ganciclovir, a guanine nucleoside


analogue that is structurally similar to acyclovir and that expresses

greater activity against CMV DNA polymerase.

(Choice A) Pentamidine is used to treat Pneumocystis pneumonia.


(Choice B) Clarithromycin is used in conjunction with other antibiotics

in the treatment of Mycobacterium avium complex (MAC) infections.

(Choice C) Acyclovir is used to treat herpes simplex infections.


(Choice E) Flucytosine is a fluorinated analog of cytosine. It is

converted by a relatively fungus-specific cytosine deaminase to 5-


fluorouracil, which causes RNA miscoding and inhibition of DNA

synthesis. Flucytosine is used in the treatment of fungal infections

and can also treat cryptococcal infections when given with amphotericin
B.

(Choice F) Leukocyte lFN-α is currently approved for the treatment of

hepatitis B and C virus infection heavy cell leukemia condyloma


acuminatum, and Kaposi’s sarcoma.

Educational Objective:

The most common cause of retinitis in HI V-positive patients is


cytomegalovirus (CMV) infection. CMV retinitis affects 30-40% of AIDS

patients with a CD4+ cell count below 50/mm3 and is best treated with

ganciclovir.

114

USMLE WORLD STEP 1 PHARMACOLOGY

Q NO 83: A barbiturate-like intravenous anesthetic agent is used in a patient

undergoing a minor surgical procedure. Thirty minutes after the initial

administration of the drug the patient seems alert and responds to verbal

commands adequately. Which of the following is the most likely mechanism


that explains the recovery from anesthesia in this patient?

A. Liver elimination of the drug

B. Renal elimination of the drug


C. Spontaneous degradation in plasma

D. Lung elimination of the drug

E. Tissue redistribution of the drug

Explanation:

Accumulation of thiopental and other related lipid soluble barbiturates


in the brain is followed by its rapid redistribution into skeletal

muscles and adipose tissue (within 5-10 mm of administration). In a

classic pharmacologic study, researchers demonstrated that awakening


from thiopental occurred because the plasma level rapidly declined.

They further demonstrated that the cause of the rapid plasma decay of

thiopental was not metabolism of the drug but rather redistribution of


the drug to other tissues throughout the body (lean tissue). This rapid

brain clearance leads to recovery from anesthesia.

Educational Objective:

The cause of the rapid plasma decay of thiopental was not metabolism of

the drug but rather redistribution of the drug to other tissues


throughout the body.

115

USMLE WORLD STEP 1 PHARMACOLOGY

Q NO 84: A 44-year-old male is hospitalized with unstable angina. On the fourth

day of hospitalization he develops severe foot pain and right toe paleness. His

laboratory testing is significant for platelet count of 60000/mm3. Treatment with

argatroban is initiated immediately. The drug used to treat this patient’s current
condition has which mechanism of action?

A. Binds to the thrombin active site


B. Blocks glutamate residue carboxylation

C. Block ADP receptors

D. Block GP IIb/IIIa surface receptors


E. Inhibits phosphodiesterase

Explanation:
Heparin is the most important cause of thrombocytopenia in hospitalized

patients. Occurrence of heparin-induced thrombocytopenia (HIT) is much

more common with the use of unfractionated heparin compared to low


molecular weight heparin. HIT more commonly leads to paradoxical

thrombosis rather than bleeding. HIT is a serious disorder caused by

antibodies to heparin and platelet factor IV.


Direct thrombin inhibitors (hirudin, lepirudin and argatroban) do not

require antithrombin-III for their action and are drugs of choice in

the treatment of HIT. Patients with HIT need ongoing anticoagulation


due to the presence of. or possibility of thrombosis. Upon clinical

suspicion of HIT, the most important initial step in treatment is to

stop all forms of heparin.


(Choice B) Warfarin inhibits vitamin K dependently-carboxylation of

glutamic acid residues of clotting factors II, VII, IX and X.

(Choices C, D and E) Platelet plug formation involves three critical


steps: adhesion, aggregation and release (see diagram). Drugs that

inhibit platelet aggregation work by different mechanisms.

116

USMLE WORLD STEP 1 PHARMACOLOGY

Aspirin irreversibly acetylates platelet cyclooxygenase-l leading to

decreased formation of thromboxane A2. Aspirin is used for primary and


secondary prevention of myocardial infarction and strokes.

Ticlopidine and clopidogrel inhibit ADP mediated platelet aggregation.

They are useful following percutaneous coronary intervention (PCI), and


for treatment of unstable angina and non-Q wave myocardial infarction.

Dipyridamole and cilostazol inhibit platelet aggregation by inhibiting

phosphodiesterase activity and increasing cAMP.


Glycoprotein IIb/IIIa inhibitors inhibit binding of platelet

glycoprotein IIb/IIIa with fibrinogen and fibronectin. The glycoprotein

IIb/IIIa inhibitors abciximab, eptifibatide and tirofiban are approved


for use following percutaneous intervention (PCI) in acute coronary

syndrome.

Educational Objective:

Heparin-induced thrombocytopenia (HIT) is treated with direct thrombi n

inhibitors (DTI5) such as argatroban. Both high molecular weight


heparin and LMVVH should be avoided in these patients.

117

USMLE WORLD STEP 1 PHARMACOLOGY

Q NO 85: A 14-year-old Caucasian female is brought to the ER following a suicide

attempt. Several hours ago, she swallowed several of her grandfather’s

theophylline tablets. Which of the following describes the most likely clinical

presentation of this patient?

A. Upper gastrointestinal bleeding

B. Bradycardia, hypotension and cardiovascular collapse


C. Severe sedation and respiratory depression

D. Latent period with subsequent hepatic necrosis

E. Abdominal pain, vomiting and seizures

Explanation:

In low to moderate doses, methylxanthine cause mild cortical arousal


and insomnia, much like caffeine. Acute theophylline intoxication

results in abdominal pain, vomiting, diarrhea, cardiac arrhythmias, and

seizures. Seizures are the major cause of morbidity and mortality in


theophylline intoxication. Tachyarrhythmias are the other major

concern, but usually do not cause OT prolongation.

Treatment of theophylline intoxication includes gastric lavage followed


by administration of activated charcoal (to reduce absorption) and

cathartics (to increase elimination via the gastrointestinal tract).

Beta-blockers are the drugs of choice for theophylline-induced cardiac


tachyarrhythmias. Theophylline-induced seizures are difficult to treat.

Benzodiazepines are the most effective agents.

(Choice A) Iron poisoning can cause hematemesis and melena, as iron is


directly toxic to gastric mucosal cells.

(Choice B) Bradycardia, hypotension and cardiovascular collapse are

seen with beta-blocker toxicity. Glucagon is the specific antidote for


beta-blocker poisoning, because it increases intracellular cAMP and

cardiac contractility.

(Choice C) Severe sedation respiratory depression, and constricted


pupils are features of opioid intoxication. Treatment of opioid

intoxication includes general supportive care and administration of

opioid antagonists like naloxone, naltrexone, or nalmefene.


(Choice D) Acetaminophen causes liver damage following a latent period.

N-acetyl cysteine is a glutathione donor, used in the treatment of

acetaminophen toxicity.

Educational Objective:
Seizures are the major cause of morbidity and mortality from

theophylline intoxication. Tachyarrhythmias are the other major

concern.

118

USMLE WORLD STEP 1 PHARMACOLOGY

Q NO 86: A 56-year-old female with lymph-node-positive breast cancer is treated

with systemic chemotherapy. One month later, she develops frequent urination,

suprapubic pain, dysuria, and hematuria. Which of the following could have

prevented this patient’s condition?

A. N-acetylcysteine

B. Folinic acid
C. Ondansetron

D. Filgrastim

E. Mesna
F. Dexrazoxane

G. Amifostine

Explanation:

The patient described in the question has hemorrhagic cystitis. The

agent most likely to have caused this condition is a nitrogen mustard-


based chemotherapeutic agent, such as cyclophosphamide or one of its

analogs (i.e. ifosfamide). These agents are metabolized by the kidneys

into acrolein which is then excreted in the urine. Acrolein is toxic to


uroepithelial cells and can cause cell death and necrosis if allowed to

remain in contact with these cells for a prolonged period of time.

Fortunately, the hemorrhagic cystitis associated with nitrogen mustard-


based chemotherapy can be prevented by aggressive hydration and by the

coadministration of mesna. Mesna, or 2-mercaptoethanesulfonate, binds

and inactivates the toxic metabolites of chemotherapeutic agents that


are found in the urine.

(Choice A) N-acetylcysteine (NAC) is indicated in acetaminophen

overdose, as a mucolytic agent, and as a renoprotective agent.


(Choice B) Leucovorin, or folinic acid, is a drug used in the treatment

of methotrexate overdose.

(Choice C) Ondansetron inhibits serotonin 5-HT3 receptors and is used


primarily to treat nausea and vomiting following chemotherapy.

(Choice D) Filgrastim is granulocyte colony-stimulating factor (G-CSF)

analog used to stimulate the proliferation and differentiation of


granulocytes in patients with neutropenia, as can occur after

chemotherapy.

(Choice F) Dexrazoxane is an iron-chelating agent that can help prevent


anthracycline-induced (i.e. doxorubicin) cardiotoxicity.

(Choice G) Amifostine is a cytoprotective free-radical scavenger agent


used to decrease the cumulative nephrotoxicity associated with

platinum-containing and alkylating chemotherapeutic agents.

Educational Objective:

Hemorrhagic cystitis during therapy with cyclophosphamide or ifosfamide

is caused by the urinary excretion of the toxic metabolite acrolein.


This can be prevented by aggressive hydration, bladder irrigation, and

administration of mesna, a sulfhydryl compound that binds acrolein in

the urine.

119

USMLE WORLD STEP 1 PHARMACOLOGY

Q NO 87: A gene coding an enzyme that synthesizes a particular fungal cell wall

polysaccharide is mutated. A fungal pathogen might, in this way, develop

resistance to which of the following agents?

A. Amphotericin B

B. Itraconazole

C. Terbinafine
D. Griseofulvin

E. Caspofungin

F. Flucytosine

Explanation:

13-beta-D-glucan is a major polysaccharide component of the fungal cell

wall. Caspofungin and the other echinocandin antifungals (e.g.


micafungin) block glucan synthesis. Whereas polyene and azole

antifungals target the fungal cell membrane the echinocandins are


unique in that they suppress the synthesis of the fungal cell wall.

Caspofungin is most active against Candida species and Aspergillus. It

is not active against C1yptococcus neoformans, and has limited activity


against Mucor and Rhizopus species.

(Choice A) Amphotericin B is a polyene antifungal that binds ergosterol

in the fungal cell membrane and leads to pore formation and cell lysis.
(Choice B) Itraconazole is an azole antifungal that inhibits ergosterol

synthesis. It does not affect the polysaccharides of the fungal cell

wall.
(Choice C) Terbinafine accumulates in skin and nails and is used to

treat dermatophytosis. Terbinafine inhibits the fungal enzyme squalene-

23-epoxidase, which ultimately results in decreased synthesis of


ergosterol.

(Choice D) Griseofulvin enters fungal cells, binds microtubules and

inhibits mitosis. This antifungal drug is effective only against


dermatophyte fungi as it accumulates in keratin-containing tissues.

120

USMLE WORLD STEP 1 PHARMACOLOGY

(Choice F) Flucytosine inhibits the synthesis of both DNA (replication)

and RNA (protein synthesis) in fungal cells. It is mainly used as a


synergistic agent with amphotericin B, particularly in the treatment of

cryptococcal meningitis.

Educational Objective:

Echinocandins (e.g. caspofungin and micafungin) are a newer group of

antifungal medications that inhibit synthesis of the polysaccharide


glucan, an essential component of the fungal cell wall.

121

USMLE WORLD STEP 1 PHARMACOLOGY

Q NO 88: A 76-year-old male is brought to the ER with severe chest pain and

diaphoresis. His past medical history is signification diabetes mellitus type 2,

hypertension and asymptomatic right carotid artery stenosis. His blood

pressure is 120/70 mmHg and his heart rate is 75/mm. EKG shows ST
segment elevations greater than 1mm in leads III and aVF. The patient is

treated with streptokinase, morphine, IV fluids and a low-dose beta-blocker.

Several hours later, the patient is comatose with asymmetric pupils and an
irregular breathing pattern. This patient’s current condition is most likely

caused b

A. Dissection of the ascending aorta

B. Carotid artery thrombosis


C. Inter ventricular septum perforation

D. Pulmonary embolism

E. Intracerebral hemorrhage

Explanation:

The patient described in this clinical vignette has chest pain


diaphoresis and a history of systemic atherosclerosis and diabetes is

likely to be having a myocardial infarction. This clinical suspicion is

substantiated by EKG findings consistent with an inferior ST elevation


myocardial infarction (STEMI). Treatment with thrombolytics such as

streptokinase and tissue plasminogen activator has been shown to

increase survival of patients with acute Ml if administered within 6


hours of the onset of symptoms and in the absence of contraindications

such as recent surgeries or gastrointestinal bleeding. Emergent cardiac

catheterization is preferred to thrombolytic therapy if it is available


at the institution where the patient presents.

Streptokinase is a protein synthesized by beta-hemolytic streptococci.


It forms a complex with plasminogen that in turn, cleaves plasminogen

to form plasmin. Plasmin cleaves fibrin thereby dissolving thrombi. The

streptokinase plasminogen complex also destroys fibrinogen and clotting


factors V and VII.

The most common side effect of streptokinase is hemorrhage. The patient

described in this clinical vignette displays the signs of an


intracerebral hemorrhage (Choice E), such as decreased level of

consciousness, asymmetric pupils and irregular breathing. Hemorrhages

may also occur in the GI tract or other organ systems.


(Choice A) Dissection of the ascending aorta manifests with

excruciating tearing chest pain that radiates to the interscapular

area. When dissection leads to rupture this process is nearly


universally fatal. It occurs most commonly in patients with

hypertension, Marfan and Ehlers-Danlos syndromes. CXR will show a

widened mediastinum.
(Choice B) Carotid artery thrombosis can cause ischemic stroke and the

symptoms described in this vignette, but in this case treatment with

thrombolytics preceding onset of symptoms makes hemorrhagic stroke more


likely.

(Choice C) Interventricular septum perforation results in a left-to-

right shunt and symptoms of acute heart failure. It presents with chest
pain, dyspnea, symptoms or cardiogenic shock and a harsh holosystolic

murmur on the left sternal border.

(Choice D) Pulmonary embolism manifests with acute onset of dyspnea,


pleuritic chest pain and hemoptysis. Tachycardia, hypotension, cyanosis

and loss of consciousness can occur when severe.

122

USMLE WORLD STEP 1 PHARMACOLOGY

Educational Objective:

Streptokinase is a thrombolytic agent that acts by converting


plasminogen into plasmin, which subsequently degrades fibrin. Its most

common side effect is hemorrhage. Streptokinase is a foreign protein

derived from Streptococci and can induce hypersensitivity reactions.

123

USMLE WORLD STEP 1 PHARMACOLOGY

Q NO 89: A 56-year-old immigrant from Eastern Europe is suffering from severe

depression and has been treated with phenelzine. You consider switching to

sertraline but inform the patient that at least two weeks should pass after

discontinuation of the drug he is using in order to start the new drug. Which of
the following processes justifies the necessity for such a gap between

treatments?

A.
B.
Monoamine reuptake
Monoamine re-synthesis

C. Receptor downregulation

D.
E.
Receptor recycling
Enzyme re-synthesis

Explanation:

The etiology of depression involves several hypothesis that point to

inadequate neurotransmission and dysregulation of the monoamine


neurotransmitters serotonin, norepinephrine, and dopamine. Depression

maybe caused by insufficient serotonin, norepinephrine, or dopamine

activity.
Monoamine oxidase is an enzyme located in nerve terminals and is

responsible for the inactivation of the monoamine neurotransmitters.

Phenelzine is an antidepressant that works by irreversibly binding and


inhibiting MAO A and B, resulting in decreased inactivation of

neurotransmitters. This potentially increases serotonin,

norepinephrine, and dopamine levels. Because phenelzine irreversibly


binds to MACI it may take up to at least 2 weeks after discontinuation

of the drug to regenerate enzyme to levels adequate for normal

neurotransmitter metabolism.
Sertraline is another anti depressant that is a selective serotonin

reuptake inhibitor. It specifically increases serotonin levels. Co-

administration of an SSRI and an MAO inhibitor can result in excessive


serotonin levels and is considered contraindicated. The combined

effects of increased serotonin levels and decreased serotonin

inactivation can produce serotonin syndrome, a potentially fatal


condition of excessive serotonergic activity that is characterized by

produce serotonin syndrome, a potentially fatal condition of excessive

serotonergic activity that is characterized by restlessness, altered


mental status hyperreflexia, diaphoresis, and tremors. To avoid the

risk for serotonin syndrome it is recommended to wait at least 14 days

after MAO inhibitor discontinuation before initiating SSRI therapy.


This should allow sufficient time for the regeneration of MAO.

(Choice A) Monoamine reuptake is a mechanism by which monoamine

neurotransmitters are actively taken up by presynaptic neurons. Once


reuptake occurs, the neuro transmitter is restored and recycled for

future transmission. Monoamine reuptake inhibitors work by blocking

this reuptake, ultimately increasing neurotransmitter levels.


(Choice B) Monoamine re-synthesis allows continuous repletion of

neurotransmitters. Monoamine neurotransmitters are synthesized in the


cytosol of neuronal cells and then packaged and stored in vesicles

ready for release. Neurotransmitter release occurs only when an action

potential propagates and calcium induces release by exocytosis.


(Choice C) Consistent exposure of an agonist to a receptor can cause

receptor downregulation. Receptor downregulation involves decreased

densely of receptors and the exact mechanism remains unclear, but may
involve decreased receptor synthesis or expression. Downregulation of

124

USMLE WORLD STEP 1 PHARMACOLOGY

postsynaptic receptors can occur with antidepressants secondary to

increased synaptic neurotransmitter levels. This downregulation of


postsynaptic receptors is believed to be responsible for the delayed

clinical effects of antidepressants.

(Choice D) After binding to any ligand, most receptors go through


endocytosis allowing internalization of ligands. Receptors are then

recycled to the cell surface. Postsynaptic receptors are constantly

recycled in order to maintain adequate receptor levels at the cell


surface.

Educational Objective:
Co-administration of an SSRI and MAO inhibitor can produce excessive

serotonin levels secondary to decreased reuptake and decreased

metabolism. Excessive serotonin levels can lead to the development of a


potentially fatal condition known as serotonin syndrome. To avoid the

risk for serotonin syndrome itis recommended waiting at least 14 days

after MAO inhibitor discontinuation before initiating SSRI therapy.


This should allow sufficient time for the regeneration of MAO.

125

USMLE WORLD STEP 1 PHARMACOLOGY

Q NO 90: A 53-year-old male with a long hi stow of diabetes mellitus is admitted to

the hospital after fracturing his femur in a motor vehicle accident. In the hospital he

receives NPH insulin and regular insulin. His blood sugar readings drop below 60

g/dL at several points during his hospitalization, but he never experiences


symptoms of hypoglycemia. This is most likely because he is also taking which of

the following medications?

A.
B.
Verapamil
Losartan

C. Doxazosin

D.
E.
Hydrochlorothiazide
Propranolol

F. Atorvastatin

Explanation:

When the plasma glucose concentration is low, there is an increase in


the body secretion of epinephrine, glucagon, and, to a lesser extent

cortisol and growth hormone. The increase in sympathetic activity

stimulates lipolysis in the peripheral tissues and gluconeogenesis in


the liver and decreases peripheral glucose consumption. The brain

relies heavily on glucose as an energy source, and the level of glucose

uptake by the brain is not regulated by insulin. If hypoglycemia


persists despite this autonomic reaction, the activity of higher brain

centers diminishes in order to reduce glucose requirements. Thus, there

are two types of hypoglycemic symptoms:


1. Adrenergic symptoms such as sweating, tremor, palpitations, hunger,

and nervousness occur due to epinephrine and norepinephrine release.

(Recall that sweating is a cholinergic process under sympathetic


control.) Adrenergic symptoms are the early signs of hypoglycemia.

2. CNS symptoms develop later and at lower glucose levels. They include

behavioral changes, confusion, visual disturbances, stupor, and


seizures. Prolonged CNS hypoglycemia leads to irreversible neurological

deficits and death.

Non-selective β-blockers (propranolol timolol and nadolol) inhibit the


epinephrine and norepinephrine-mediated compensatory reactions to

hypoglycemia. Thus the adrenergic symptoms of hypoglycemia (e.g.

tremor, palpitations) are blunted. Additionally, blockade of β2


adrenergic receptors inhibits hepatic gluconeogenesis and peripheral

glycogenolysis and lipolysis. For this reason, non-selective [3-

blockers should not be used in diabetic patients. Selective blockers


are preferable in diabetics because they do not block metabolic

sympathetic effects.

(Choice A, B, C, D and F) These medications do not mask the symptoms of


hypoglycemia.

Educational Objective:
Non-selective β-blockers exacerbate hypoglycemia and mask its

adrenergic symptoms. For this reason, they should not be used in


patients with diabetes mellitus. Selective antagonists should be used

instead if a β-blocker is necessary.

126

USMLE WORLD STEP 1 PHARMACOLOGY

Q NO 91: A 55-year-old Caucasian male presents to your office with recurrent

episodes of retrosternal chest pain. The episodes occur during physical

activity, usually when he climbs stairs or walks uphill. He reports rapid relief

of the pain alters taking a single sublingual tablet. Which of the following is
the major contributor to the drug effect observed in this patient?

A. Increase in coronary blood flow

B. Increase in coronary perfusion pressure


C. Decrease in left ventricular (LV) volume

D. Decrease in arteriolar resistance

E. Decrease in heart rate

Explanation:

The clinical scenario described most closely resembles a patient using


nitroglycerin to relieve the symptoms of angina pectoris. Decreased

cardiac preload (left ventricular volume during diastole) is the main

contributor to nitrate effect in patients with stable angina. Nitrates


decrease cardiac preload by directly causing veno dilation resulting in

retention of blood in the venous system.

(Choice A) Increase in coronary artery blood flow caused by a drug such


as adenosine, would be counterproductive in an episode of angina

pectoris as this would lead to the “coronary steal syndrome.” This

syndrome causes a paradoxical decrease in blood flow to an area of


ischemic myocardium because blood will preferentially flow through the

pharmacologically dilated arteries of the heart that are not obstructed

by thrombus and will not be forced through the vessel that has been
thrombolically occluded.

(Choice B) An increase in coronary perfusion pressure would require an

increase in diastolic blood pressure as it is during this phase of the


cardiac cycle that the leaflets of the aortic valve are closed and

aortic blood is allowed to flow into the coronary arteries. An

increased systolic blood pressure would mean an increased after load


against which the left ventricle must contract. This state would be

counterproductive during an acute ischemic event unless that event was

caused during systemic hypotension as in a patient in shock where


pressor medications would be used.

(Choice D) A decrease in arteriolar resistance would lead to decreased

cardiac work due to a decreased afterload. This is accomplished by


drugs such as ACE inhibitors angiotensin receptor blockers, calcium

channel blockers and beta-adrenergic blockers to name a few. Though a

decrease in systemic vascular resistance would be useful in angina it


is not the best answer in this case.

(Choice E) A decrease in heart rate would be useful in the ischemic


heart. This is one of the mechanisms by which the cardioselective

calcium channel blockers and the beta-adrenergic blockers are able to

exert their anti-anginal effects. A decreased heart rate is not an


effect of nitrates.

Educational Objective:
Sublingual or aerosolized nitroglycerin is a rapidly acting agent taken

by patients with stable angina pectoris as needed to rapidly relieve

their symptoms. Nitrates act primarily as veno dilators causing a


decrease in cardiac work by decreasing left ventricular filling volume

or preload.

127

USMLE WORLD STEP 1 PHARMACOLOGY

Q NO 92: A 24-year-old male has a long history of heroin abuse and several

addiction treatment failures. You refer this patient to methadone


A.

detoxification program that turns out to be effective. Which of the following

methadone properties justifies its substitution for heroin in this patient?


Partial agonist activity

B. Low potency

C. No respiratory suppression
D. Long half-life

E. Short withdrawal

F. Low bioavailability

Explanation:

Heroine is an opiate that has little to no established medical use. It


is primarily a street drug and is a significant drug of abuse. It

produces similar physiologic effects as other opiates and thus treating

heroine addiction and abuse involves the use of other opiates.


Methadone is usually the drug of choice for treating heroine addiction.

Methadone is a mu receptor agonist. It is a potent, long acting opiate

with good oral bio availability. Its long half-life and sustained
effects after chronic dosing allows for continuous suppression of

withdrawal symptoms in heroine dependent patients. It causes similar

pharmacologic and physiologic effects as other opioids and its effects


on miosis and respiratory depression can be seen up to 24 hours after

one dose.

(Choice A) Methadone is a mu receptor agonist. Using agents with


partial agonist activity can precipitate withdrawal.

(Choice B) Methadone is a strong mu receptor agonist with potent

analgesic effects.
(Choice C) Methadone can cause respiratory depression like most

opioids.

(Choice E) Methadone binds to protein in many tissues. After chronic


dosing, it accumulates significantly in tissues. When methadone is

discontinued, methadone is slowly released from tissues to maintain low

levels in the blood. This allows for a very mild but extended
withdrawal syndrome.

(Choice F) One of the most notable properties of methadone is its good

oral bioavailability.

Educational Objective:
Methadone is the drug of choice for treating heroine addiction and

abuse. It is a veri potent, long acting opiate with good oral

bioavailability. Its long half-life allows for prolonged effects to


suppress withdrawal symptoms in heroine dependent patients.

128

USMLE WORLD STEP 1 PHARMACOLOGY

Q NO 93: A 28-year-old female presents to your office for a routine check-up. Her

past medical history is not significant. She does not smoke or consume alcohol.

She is in a monogamous relationship with her husband and uses combination oral

contraceptives regularly. Which of the following most likely prevents pregnancy in


this patient?

A. Low serum prolactin level


B. Low serum gonadotropin levels

C. Increased endogenous estrogen synthesis

D. Impaired sperm penetration into the uterus


E. Impaired embryo implantation

Explanation:
Combination oral contraceptives (OCPs) contain variable doses of

estrogen and progesterone. Their primary mode of action is suppression

of synthesis of the gonadotropins ESH and LH in the anterior pituitary,


which thereby leads to inhibition of ovulation. Ovulation is inhibited

because an LH spike is required to stimulate ovulation in normal

individuals: this midcycle spike is suppressed in patients taking OCPs.


OCPs also cause thickening of the cervical mucus that prevents sperm

from accessing the uterus. A third mechanism of pregnancy prevention by

OCPs is the action of progestin which prevents growth of the


endometrium making it unsuitable for implantation of an embryo.

(Choices A and B) OCPs do not directly affect the synthesis of

prolactin and estrogen.


(Choice D) OCPs increase the thickness of the cervical mucus thereby

impairing sperm penetration into the uterus, but this is not the main

mode of action of oral contraceptives.


(Choice E)The progesterone component of OCPs suppresses follicular

phase thickening of the endometrium and impairs implantation of the

embryo, but this is a minor effect of oral contraceptives.

Educational Objective:

Combination oral contraceptives inhibit ovulation by decreasing


synthesis of SH and LH in the anterior pituitary. Their effects on

cervical mucus and endometrium play a minor role.

129

USMLE WORLD STEP 1 PHARMACOLOGY

Q NO 94: A 64-year-old female develops severe nausea and vomiting soon alter

receiving high-dose chemotherapy for ovarian cancer. Which of the following

agents would be most helpful to treat her symptoms?

A. N-acetylcysteine

B. Folinic acid

C. Ondansetron
D. Filgrastim

E. Dexrazoxane

F. Amifostine
G. Allopurinol

Explanation:
5-HT3 receptor antagonists, such as ondansetron, granisetron, and

dolasetron, are highly effective in preventing chemotherapy-induced

vomiting. These agents act by two primary mechanisms: by blocking


vagus-mediated nausea and vomiting and blocking serotonin in the

chemoreceptor trigger zone.

In the GI tract, there are presynaptic vagal nerve terminals with 5-HT3
receptors that initiate vagus nerve activity in the medulla oblongata’s

“vomit center.” 5-HT3 receptors also act centrally in both the

chemoreceptor trigger zone (area postrema) and the solitary nucleus.


(Choice A) N-acetylcysteine (NAC) is indicated in acetaminophen

overdose, as a mucolytic agent, and as a renoprotective agent in

certain patient populations. It is used as a mucolytic agent in


patients with influenza, bronchitis, and cystic fibrosis. In patients

with renal insufficiency who require an IV-contrast CT scan NAC

prevents radiocontrast induced nephropathy.


(Choice B) Leucovorin, or folinic acid, is a drug used in the treatment

of methotrexate overdose.

(Choice D) Filgrastim is granulocyte colony-stimulating factor (G-CSF)


analog used to stimulate the proliferation and differentiation of

granulocytes. Filgrastim is approved for the minimization of


granulocytopenia after myelosuppressive chemotherapy.

(Choice E) Dexrazoxane is an iron-chelating agent that can help prevent

anthracycline-induced (i.e. doxorubicin) cardiotoxicity.


(Choice F) Amifostine is a cytoprotective free-radical scavenger agent

used to decrease the cumulative nephrotoxicity associated with

platinum-containing and alkylating chemotherapeutic agents.


(Choice C) When patients receive chemotherapy for a neoplasm with a

high cell turnover rate (such as lymphomas and leukemias), they are at

risk for the tumor lysis syndrome which is characterized by electrolyte


disturbances and acute renal failure. In addition to aggressive

hydration and alkalinization of the urine, allopurinol can be used to

prevent this syndrome.

Educational Objective:

Ondansetron inhibits serotonin (5-HT3) receptors and is used primarily


to treat nausea and vomiting following chemotherapy. 5-HT3 receptors

are located peripherally in the presynaptic nerve terminals of the

vagus nerve in the GI tract. These receptors are also present centrally
in the chemoreceptor trigger zone and the solitary nucleus and tract.

130

USMLE WORLD STEP 1 PHARMACOLOGY

Q NO 95: A series of animal experiments is being performed on a newly developed

drug. The drug seems to be effective against several types of seizures. Itis

metabolized by the liver, and its byproducts are excreted by the kidneys. The

curve below demonstrates the drug-glucuronide blood concentration over a range


of doses. Which of the following is the best statement about this drug’s

metabolism?

A. At point 3, a constant proportion of the drug is metabolized

B. At point 1, no dose-dependent effect is observed

C. At point 2, biotransformation of the drug stops


D. At point 2, metabolism switches to zero-order kinetics

E. At point 1, bio availability of the drug is high

Explanation:

This question and graph highlight a classic case of capacity-limited

(saturable) enzyme kinetics. The substrate drug is glucuronidated by


hepatic glucuronyl transferases to form the product metabolite (drug-

glucuronide), which is then measured in the plasma. At lower doses

(Point 1), formation of drug-glucuronide is directly proportional to


the dose administered. In other words when more drug is administered,

more drug-glucuronide is produced. Because a fixed proportion of drug

rather than a fixed amount of drug is converted to metabolite per unit


time this early portion of the graph represents first-order kinetics.

First-order kinetics is represented graphically by a straight line

showing increased conversion with increased dosing as illustrated in


the region of Point above.

When the enzyme active sites are saturated the drug-glucuronide

concentration does not change with increasing doses of the substrate


drug. The value where the graph levels out to a zero slope approximates

the maximum capacity (Vmax) of the glucuronyl transferase enzymes in

the case described assuming that the mechanism for renal elimination of
the drug-glucuronide product is not itself saturated. At this juncture

(Points 2 and 3), the kinetics change from first-order to zero-order


(saturable) kinetics. In zero-order kinetics the active sites of the

metabolizing enzyme are fully saturated drug metabolism does not depend

on substrate concentration, and a constant amount of drug is


metabolized and eliminated per unit time.

(Choice A) At point 3 a constant amount of drug is metabolized per unit

time. During this phase of zero-order kinetics further increases in

131

USMLE WORLD STEP 1 PHARMACOLOGY

drug dose are not accompanied by an increase in drug product, and an

unchanged form of the drug may accumulate.


(Choice B) Point represents a phase of first-order metabolism kinetics;

thus the drug-glucuronide concentration is directly proportional to the

substrate concentration at point 1.


(Choice C) At point 2, biotransformation continues to occur but at that

point the biotransformation of the drug switches from linear (or first-

order) to non-linear (or zero-order) kinetics.


(Choice E) Bioavailability refers to the fraction of administered drug

that is ultimately made available in the systemic circulation and can

be calculated using graphs of drug concentration vs. time. There is no


way to determine bioavailability with the information provided.

Educational Objective:
In first order kinetics a constant fraction (or proportion) of drug is

metabolized per unit time based on the serum concentration. In zero-

order kinetics a constant amount of drug is metabolized per unit time


independent of concentration.

132

USMLE WORLD STEP 1 PHARMACOLOGY

Q NO 96: A 63-year-old man presents to the ER with muscle weakness and

cramping. He has been taking hydrochlorothiazide for recently diagnosed


A.

hypertension. Which of the following is the most likely cause of his

symptoms?
Hyperuricemia

B. Hypocalcemia

C. Hyponatremia
D. Hypokalemia

E. Hypophosphatemia

F. Hypoglycemia

Explanation:

Thiazide diuretics inhibit Na/Cl co-transporters in the distal


convoluted tubules, thus decreasing reabsorption of Na and Cl and

increasing urinary excretion of these ions. Thiazide diuretics also

reduce peripheral vascular resistance and are helpful in treating


hypertension. Thiazide diuretics are associated with the following side

effects:

1. Thiazide diuretics decrease intravascular volume, stimulating


aldosterone secretion. Aldosterone increases urinary excretion of

potassium and hydrogen ions, causing hypokalemia and metabolic

alkalosis. Hypokalemia causes muscle weakness that may lead to


paralysis as well as ECS changes (prominent U waves). The patient

described in this vignette is most likely suffering from hypokalemia.

2. Increased sodium excretion may lead to hyponatremia.


3. Hypovolemia stimulates uric acid reabsorption in the proximal

tubules. Hyperuricemia is a common side effect of thiazides and may

precipitate a gout attack (Choice A).


4. Thiazides cause hyperlipidemia by increasing plasma cholesterol and

LDL.

5. Thiazides decrease insulin secretion and glucose uptake by the


tissues and may cause impaired carbohydrate tolerance and

hyperglycemia.

(Choice B)Thiazides increase calcium reabsorption in the distal


convoluted tubules and are indicated for treatment of idiopathic

hypercalciuria and recurrent calcium stone nephrolithiasis. They do not

cause hypocalcemia.
(Choice C) Hyponatremia is also a common side effect of thiazide

diuretics but the resulting symptoms are primarily neurologic (altered


mental status, seizures etc.).

(Choice E) Hypophosphatemia can also cause muscle weakness and

paralysis but hypophosphatemia is not a side effect of thiazides. It is


commonly seen in alcoholics.

Educational Objective:
A decrease in the intravascular fluid volume stimulates aldosterone

secretion and leads to increased excretion of potassium and hydrogen

ions in the urine. This results in hypokalemic metabolic alkalosis,


which is a common side effect of most diuretics other than the

potassium-sparing class. Hypokalemia manifests with muscle weakness and

cramping.

133

USMLE WORLD STEP 1 PHARMACOLOGY

Q NO 97: A 56-year-old Caucasian female presents to your office with chronic

cough. She says that the cough is dry and affects quality of her life

significantly. She denies chest pain hemoptysis and shortness of breath. Her

past medical history is significant for long-standing hypertension, diabetes


and myocardial infarction experienced two months ago. She does not smoke

or consume alcohol. Her blood pressure is 130/10 mmHg and heart rate is

70/mm. Which of the following is the best next step in the management of
this patient?

A. Careful review of family history

B. Careful review of past allergic episodes

C. Careful review of current medications


D. Careful review of diet and physical activity

F. Chest x-ray

Explanation:

With a history of hypertension, diabetes, and coronary heart disease,

the likelihood that this patient is on standard ACE inhibitor therapy


is very high. A dry, nonproductive, and persistent cough is one of the

most well-known side effects of ACE inhibitor therapy.

The mechanism of ACE inhibitor induced cough is believed to involve


increased bradykinin substance P, or prostaglandins. ACE is a very

nonspecific enzyme responsible for the breakdown of many substrates. By


blocking ACE, ACE inhibitors block degradation of bradykinin and

substance P. Bradykinin and substance P accumulation can cause

inflammation and stimulate lung irritation.


Because angiotensin receptor blockers (ARB5) do not affect ACE activity

they theoretically should not cause cough.

(Choice A) Chronic cough is one of the early signs of cancer. Screening


family history will allow risk assessment for cancer, but since patient

does not complain of any other symptoms of cancer (unexplained fatigue,

weight loss, change in bowel habits, etc.) suspicion for cancer is low.
(Choice B) Although cough can occur secondary to post nasal drip in

patients with allergies the patient describes no other allergy type

symptoms (itchy and watery eyes, runny nose sneezing etc.).


(Choice D) Diet and physical activity are not linked to causing cough.

(Choice E) Cough maybe linked to pulmonary infection or disease. Chest

x-ray would be helpful in diagnosis if suspicion for pneumonia, TB,


bronchitis, or emphysema were high. However patient has no symptoms

associated with any infection and her risk for emphysema is low since

she does not smoke.

Educational Objective:

Cough is a very well recognized side effect of ACE inhibitor therapy.


Cough secondary to ACE inhibitor therapy is characterized as dry,

nonproductive and persistent. The mechanism behind ACE inhibitor

induced cough is accumulation of bradykinin substance P, or


prostaglandins. Because angiotensin receptor blockers (ARB5) do not

affect ACE activity, they theoretically should not cause cough.

134

USMLE WORLD STEP 1 PHARMACOLOGY

Q NO 98: A businessman visits your office complaining of insomnia. You ascertain

that he has difficulty falling asleep due to anxiety over a failing investment. He

denies a depressed mood, feelings of hopelessness, or early morning awakening.

You prescribe a commonly used hypnotic agent which is also known to have
anxiolytic, muscle relaxant, and anticonvulsant actions. Which of the following

describes the mechanism of action of this drug?

A.
B.
Reversible inhibition of H1 receptors
Increased frequency of chloride channel opening

C. Inhibition of norepinephrine and serotonin reuptake

D.
E.
Increased duration of chloride channel opening
Increased synthesis of serotonin

Explanation:

Benzodiazepines are commonly prescribed sedative hypnotic drugs with

anxiolytic, muscle relaxant, and anticonvulsant actions.


Benzodiazepines bind and modulate the GABAA receptor-chloride channel

in CNS neurons, increasing its frequency of opening. The resulting

increase in chloride permeability hyperpolarizes and stabilizes the


membrane, rendering it less excitable. A rapid- and short-acting

benzodiazepine such as triazolam (half life 1 .5 to 5 hours) would be a

reasonable option for this patient.


(Choice A) This describes the action of diphenhydramine, sedating anti-

histamine that may be used at nighttime to reduce the time to sleep

onset and to increase the depth of sleep. However antihistamines are


not thought to have significant muscle relaxant or anti convulsant

actions.

(Choice C ) This choice describes tricyclic antidepressants (TCA) which


can be prescribed for insomnia particularly in depressed patients. The

patient in the vignette does not have symptoms or signs of depression

such as early morning awakening, thus he probably does not need an


antidepressant medication. Moreover TCAs do not have significant muscle

relaxant or anti convulsant actions (some actually decrease the seizure

threshold).
(Choice D) Barbiturates increase the duration of opening of the GABAA

receptor-chloride channel in CNS neurons. Although they bind to a

different site than benzodiazepines, the net neuro physiological


effects are similar. However, barbiturates tend to have longer half

lives than the relatively short-acting benzodiazepines, and residual

sedation or “hangover” effects occur frequently following hypnotic


doses. Barbiturates are not commonly prescribed for insomnia.

(Choice E)This may be the mechanism where by tryptophan (e.g. in milk

before bedtime) has a hypnotic action as serotonin can be synthesized


endogenously from tryptophan. Tryptophan does not have anticonvulsant

effects however.

Educational Objective:

Benzodiazepines increase the frequency of opening of the CNS GABAA


receptor-chloride channels and have anxiolytic, anticonvulsant, and

muscle relaxant effects as well as sedative-hypnotic effects.

135

USMLE WORLD STEP 1 PHARMACOLOGY

Q NO 99: A 38-year-old Caucasian female presents to your office because of sudden

episodes of severe, right-sided facial pain. The pain lasts several seconds and is

usually instigated by a meal or teeth brushing. There is no nausea or vomiting. She

denies any visual changes. Her vital signs are within normal limits and physical
examination shows no rash. Which of the following is the best initial treatment for

this patient?

A.
B.
Baclofen
Haloperidol

C. Diazepam

D.
E.
Carbamazepine
Propranolol

F. Phenobarbital
G. Valproic acid

Explanation:
This case presents a classic picture of trigeminal neuralgia (tic

douloureux). This condition presents with episodes of sudden and severe

pain in the distribution of CN V (particularly V2 and V3). The pain is


described as “stabbing” or like an “electrical shock”; it is usually

unilateral, and does not last more than few seconds; but occurs

repeatedly. Pain is triggered by any stimulus to CN V1 for example


chewing teeth brushing, shaving or washing an affected area of the

face, swallowing, or exposure to hot or cold temperatures. These

painful episodes may occur for several months. In the majority of


cases, trigeminal neuralgia is not caused by any structural abnormality

and its pathogenesis remains unclear.

Carbamazepine is the drug of choice for the treatment of trigeminal


neuralgia. Itis effective in pain reduction in upto 80% of patients.

Like phenytoin, it inhibits neuronal high-frequency firing by reducing

the ability of sodium channels to recover from inactivation.


Carbamazepine can cause aplastic anemia, so CBC should be monitored

regularly. Carbamazepine is a P450 inducer that increases the

metabolism of many other medications thereby decreasing their


effectiveness.

(Choices A and G) Baclofen and valproic acid can be used for trigeminal

neuralgia but are not first-line choices.


(Choice B) Haloperidol is a neuroleptic medication used for the

treatment of schizophrenia acute psychoses acute mania, and Tourette

syndrome. It is not indicated in trigeminal neuralgia.


(Choice E) Propranolol has various uses including migraine prophylaxis

and the reduction of portal venous pressure to prevent variceal bleed.

Itis not used for trigeminal neuralgia.


(Choice F) Phenobarbital is effective for the management of generalized

tonic-clonic seizures. It is not used for trigeminal neuralgia.

Educational Objective:

Trigeminal neuralgia presents with brief episodes of sudden and severe


“electric shock-like” or “stabbing” pain in the distribution of CN V

(particularly V2 and V3). Carbamazepine is the drug of choice.

136

USMLE WORLD STEP 1 PHARMACOLOGY

Q NO 100: A 21-year-old female presents to your office with dysuria and burning on

urination. Trimethoprim given to the patient leads to rapid resolution of symptoms.

Which of the following drugs has the same intracellular target as the drug used in

this patient?

A. Sulfamethoxazole

B. Metronidazole
C. Imipenem

D. Bacitracin

E. Methotrexate

Explanation:

Trimethoprim, methotrexate, and pyrimethamine all prevent the reduction


of folic acid to tetrahydrofolate by inhibiting dihydrofolate

reductase. Trimethoprim restricts microbial growth through this

process, and works particularly well in conjunction with sulfonamide


because sulfonamide inhibits a different step in the production of

folio acid. The combination of these drugs produces a “sequential

blockade” within the folic acid synthesis pathway.


Methotrexate is a folate antimetabolite that targets rapidly

proliferating cells by halting DNA synthesis through the irreversible

binding of dihydrofolate reductase. Methotrexate is cell cycle specific


for the S phase because it prevents the synthesis of purine and

thymidylic acid.

Lastly pyrimethamine is effective as an antimalarial and in the


treatment of toxoplasmosis because it inhibits parasitic dihydrofolate

reductase.

(Choice A) As a structural analog of para-aminobenzoic acid (PABA),


sulfamethoxazole competes with PABA to inhibit dihydrofolic acid

synthesis, an intermediate step in the formation of tetrahydrofolate.

(Choice B) Metronidazole binds to DNA within susceptible cells,


unraveling the DNA’s helical structure and breaking its strands.

Protein synthesis is inhibited and the cell dies soon thereafter.

(Choice C) Imipenem binds to one or more of the penicillin binding


proteins, thereby inhibiting the final step of peptidoglycan synthesis

in bacterial cell walls. Eventually the bacteria lyse because the cell

wall autolytic enzymes remain active. Cilastatin can be used in


conjunction with imipenem because it prevents the renal metabolism of

imipenem by competitively inhibiting dehydropeptidase on the renal


tubule brush border.

(Choice D) Bacitracin prevents mucopeptide transfer into the growing

cell wall, thereby inhibiting bacterial cell wall synthesis. As a


result, bacitracin is not an appropriate antibiotic for stable

bacterial populations.

Educational Objective:

Trimethoprim, methotrexate and pyrimethamine inhibit dihydrofolate

reductase.

137

USMLE WORLD STEP 1 PHARMACOLOGY

Q NO 101: A 54-year-old diabetic male is diagnosed with invasive mucormycosis.

He undergoes surgical debridement and is started on an appropriate antifungal

therapy. Considering the side effects of this medication1 which of the following

should be routinely monitored in this patient?

A. Serum potassium and magnesium

B. Serum alanine aminotransferase (ALT)


C. Thyroid stimulating hormone (TSH)

D. Electrocardiogram for QT interval

E. Serum calcium levels

Explanation:

Amphotericin B is currently the drug of choice for mucormycosis.


(Surgical debridement may also be indicated.) Amphotericin B works by

binding ergosterol in fungal cell membranes to form holes. The changes

in membrane permeability lead to cell death. Renal toxicity is the most


notorious side effect of amphotericin B. Amphotericin B causes renal

vasoconstriction and a reduction in the glomerular filtration rate. The

drug may also act as a direct toxin on renal epithelial cells, causing
acute tubular necrosis, electrolyte disturbances and renal tubular

acidosis. Hypokalemia and hypomagnesemia are the two most serious

electrolyte disturbances that may result from treatment with


amphotericin, reflecting an increase in distal tubular membrane

permeability. The majority of patients require both potassium and

magnesium supplementation while undergoing treatment. Renal injury may


lead to a decrease in erythropoietin production and produce a

normochromic normocytic anemia as well.

Educational Objective:

Amphotericin B is a polyene antifungal notorious for its renal

toxicity. Severe hypokalemia and hypomagnesemia are commonly seen


during therapy, and often require daily supplementation.

138

USMLE WORLD STEP 1 PHARMACOLOGY

Q NO 102: A 3-year-old boy has moderate growth retardation and prominent brown

stains on his teeth. His mother’s use of which of the following substances during

pregnancy may have contributed to these findings

A. Aspirin

B. Captopril

C. Tetracycline
D. Cocaine

E. Alcohol

F. Marijuana

Explanation:

In general tetracyclines should be avoided during pregnancy. Animal


studies have shown that tetracyclines cross the placenta and localize

to fetal tissues, potentially retarding skeletal development.

Tetracyclines also localize in the dentin and enamel of developing


teeth, and can cause yellow, gray, or brown staining of the deciduous

(milk) teeth and enamel hypoplasia. Tetracyclines are also

contraindicated in children less than 8 years of age because of


possible bone growth retardation and staining of permanent teeth.

(Choice A, B, D, E and F) These agents do not stain teeth when used

during pregnancy, but they are not without risk. Aspirin has the
potential to cause bleeding in neonates, and ACE inhibitors are highly

teratogenic. Cocaine use can cause fetal growth retardation, and

alcohol use during pregnancy can cause fetal alcohol syndrome (mental
retardation, mid-face abnormalities, etc.). Mothers who use marijuana

heavily during pregnancy may bear low birth weight babies.

Educational Objective:

Tetracycline use during pregnancy can cause fetal bone growth

retardation and discoloration of the deciduous teeth.

139

USMLE WORLD STEP 1 PHARMACOLOGY

Q NO 103: A 60-year-old female is diagnosed with diabetes on routine check-up.

Her past medical history is insignificant. Her BMI is 31.5 kg/mm2. She does not

smoke or consume alcohol. You and the patient agree on metformin treatment.

Which of the following tests should be considered before initiation of metformin?

A. Complete blood count

B. Serum potassium level


C. Serum TSH level

D. Three-hour oral glucose tolerance test

E. Serum creatinine
F. 24-hour urinary VMA excretion

Explanation:
Metformin is the most commonly prescribed medication for the treatment

of type 2 diabetes mellitus. The anti-diabetic mechanism of metformin

is not clearly understood. It is known that metformin decreases blood


sugar by increasing glycolysis, decreasing gastrointestinal glucose

absorption, and decreasing gluconeogenesis. There is no risk of

hypoglycemia with metformin use.


The major side effects of metformin are gastrointestinal upset and

lactic acidosis. Metformin increases the intestinal production of

lactate by anaerobic glycolysis. In normal individuals, the lactate


produced in the intestine is converted to glucose via gluconeogenesis

in the liver. Yet, metformin inhibits this very same process of

gluconeogenesis, which results in elevated circulating lactate levels,


and puts some patients at risk for lactic acidosis. In young

individuals with normal renal and hepatic functions, lactic acidosis is

a very rare complication of metformin therapy. Contraindications to


metformin use include renal failure (as denoted by a serum creatinine

higher than 1.4 mg/dL in males or 1.5 mg/dL in females), significant

hepatic dysfunction, and hypersensitivity to metformin. Serum


creatinine is monitored periodically in patients taking metformin.

Metformin should also be avoided in patients with severe congestive

heart failure and in alcoholics because of the increased risk of


development of lactic acidosis in these subpopulations.

(Choice A) Hematological side effects are very rarely seen with oral

antidiabetic medications.
(Choice B) The monitoring of serum potassium is not necessary with

metformin use. Serum potassium is routinely monitored, however, with


diuretics (especially loop diuretics), spironolactone, angiotensin

converting enzyme inhibitors, and angiotensin receptor blockers. Loop

diuretics can cause hypokalemia whereas hyperkalemia is the risk


associated with the other medications mentioned.

(Choice C) Thyroid function tests are not required for any antidiabetic

medications, including metformin. Thyroid function tests are routinely


performed in patients taking lithium and amiodarone.

(Choice D) Once diabetes is diagnosed, glucose tolerance tests are

rarely repeated. The three-hour glucose tolerance test is traditionally


used to diagnose gestational diabetes mellitus.

Educational Objective:

Metformin is absolutely contraindicated in patients with renal failure,


due to a risk of lactic acid accumulation. In fact, metformin is

contraindicated in any situation that might precipitate lactic

acidosis, such as liver dysfunction. Congestive heart failure


alcoholism, and sepsis.

140

USMLE WORLD STEP 1 PHARMACOLOGY

Q NO 104: A 65-year-old Caucasian female presents to your office with a several

month history of mild burning in her fingers. She says that the episodes are

usually precipitated by exposure to cold and are accompanied by her fingers

turning “blue and white.” She has never had such symptoms before. Her past
medical history is significant for Parkinson disease diagnosed two years ago and

adequately controlled with medication. This patient most likely treats her

Parkinson disease with which of the following agents?

A. Levodopa/carbidopa

B. Selegiline

C.
D.
Tolcapone
Pergolide

E. or bromo
Benztropine
criptine

Explanation:

Dopamine agonists preferentially stimulate D 2 receptors. There are two


classes of dopamine agonists:

1. ergot compounds: bromocriptine and pergolide

2. nonergot compounds: pramipexole and ropinerole


Ergot derivates cause typical side effects such as Raynaud phenomenon

(vasospasm), erythromelalgia, and retroperitoneal or pulmonary

fibrosis. These symptoms usually do not occur with nonergot


derivatives.

Dopamine agonists currently have an important role in the treatment of

Parkinson disease because they have a long half-life and prolong the
effects of levodopa, thus limiting motor fluctuations.

(Choice A) The most common side effects of levodopa are nausea and

vomiting. Unfortunately, chronic use of levodopa can lead to motor


fluctuations and dyskinesias. Arrhythmia is also reported because of

increased peripheral formation of catecholamines. Other major problems

with the levodopa/carbidopa combination are behavioral changes such as


confusion, hallucination, delusions, agitation, and nightmares. This is

attributed to an increased concentration of dopamine in the brain as a

result of carbidopa. Levodopa should not be administered to patients


with psychotic symptoms.

(Choice B) Selegiline is a selective monoamine-oxidase type B (MAO-B)

inhibitor. It may delay the clinical progression of Parkinson disease.


One of the major problems with this drug is that it often causes

confusion in the elderly. Serotonin syndrome can be precipitated by the

use of an SSRI with any MAOII including selegiline.


(Choice C) A new class of drugs is the catechol-O-methyl-transferase

(COMT) inhibitors, such as entacapone and tolcapone. By limiting the

metabolism of dopamine, they increase levodopa bioavailability, prolong


the “on” response to levodopa, reduce motor fluctuations, and allow a

reduction in daily levodopa dosage. Tolcapone can cause serious drug-

induced hepatitis, however, so serial monitoring of liver functions is


strongly recommended. In fact, tolcapone was removed from the European

and Canadian markets because of reported deaths from drug-induced

hepatitis. In the US, patients must give signed consent.


(Choice E) Anticholinergics, such as trihexyphenidyl and benztropine,

are especially useful against the resting tremor of Parkinson disease,

albeit less so against rigidity and akinesia. Although effective


against motor symptoms, anticholinergics are rarely used in elderly

patients because of the numerous side effects such as confusion,


hallucinations, memory impairment, dry mouth, constipation, urinary

141

USMLE WORLD STEP 1 PHARMACOLOGY

retention, blurred vision, and tachycardia. Patients with benign

prostatic hyperplasia (BPH) and closed-angle glaucoma should not be


given anticholinergics.

Educational Objective:
• Since both bromocriptine and pergolide are ergot derivatives they can

cause typical ergot side effects such as Raynaud phenomenon

(vasospasm), erythromelalgia, and retroperitoneal or pulmonary


fibrosis.

• Tolcapone is associated with hepatotoxicity, and deaths have been

reported.

142

USMLE WORLD STEP 1 PHARMACOLOGY

Q NO 105: A group of investigators is currently conducting an experiment on a

newly developed synthetic opioid analgesic. This drug has twice the analgesic

capability of morphine. After oral ingestion by several volunteers, the plasma

concentration of the drug is measured and is determined to be subtherapeutic.


After rectal administration of the same dose of the drug the measured plasma

drug concentration reached therapeutic levels and is almost double the level

measured after oral intake. Which of the following factors is the main
determinant of drug bio availability after oral intake accounting for the

difference in drug concentrations described above?

A. Intestinal blood flow

B. Kidney blood flow


C. Liver blood flow

D. Absorptive surface

E. Absorption rate

Explanation:

Oral bioavailability is dependent on the absorptive properties of the


drug as well as its first-pass metabolism. The first-pass, or pre-

systemic, metabolism of a drug occurs en route to the systemic

circulation by enzymes present in the gutwall and lumen as well as the


liver. Bioavailability refers to the fraction of administered drug that

reaches the systemic circulation in a chemically unchanged form. When a

drug is absorbed by the Cl tract after oral administration, it first


enters the portal circulation before gaining access to the systemic

circulation. If the drug is metabolized extensively by the liver (high

first-pass metabolism), the amount that reaches the systemic


circulation, and therefore the bioavailability, will be low.

In order to efficiently use drugs with a high first-pass metabolism,


other routes of administration are typically employed. For instance,

nitroglycerin can be used for preventing or treating anginal pain, but

because it has a very high first-pass metabolism, not enough drug


reaches systemic circulation to be effective. To circumvent this

shortcoming, nitroglycerin tablets are given sublingually where they

bypass the first-pass metabolism by entering the systemic circulation


directly via the sublingual capillaries and arterioles. Propranolol and

lidocaine also have a very high first-pass metabolism and thus have

better bioavailability through IV or subcutaneous methods.


Rectal drug administration, such as with suppositories, partially by

passes first-pass metabolism. Recall that the rectum is drained by the

superior, middle and inferior rectal veins. The superior rectal veins
drain to the portal circulation via the inferior mesenteric vein. The

middle and inferior rectal veins, however, drain to the systemic

circulation via the internal iliac and internal pudendal veins,


respectively. Thus, two-thirds of the venous drainage of the rectal

region goes directly into the systemic circulation, thereby increasing

the bioavailability of drugs that are otherwise highly cleared by the


liver after oral administration. The amount of drug exposed to the

liver within the portal blood flow is the major determinant of hepatic

or first-pass metabolism.
(Choice A) Increased intestinal blood flow may increase the rate of

absorption of drug from the gut by establishing a favorable

concentration gradient for absorption1 and drug bioavailability can


vary with intestinal blood flow. However, this choice is incorrect

because intestinal blood flow would affect both oral and rectal

143

USMLE WORLD STEP 1 PHARMACOLOGY

administration of a drug and therefore cannot account for the large

difference observed.
(Choice B) Changes in the renal blood flow affect the glomerular

filtration rate, which in turn affects the elimination of drugs cleared

by the kidney. Renal blood flow has no effect on drug absorption and
thus bioavailability.

(Choice D)The absorptive surface area likely plays a role in drug

absorption, but it is not the only determinant. For example, a drug


exposed for a short time to a large surface area may experience the

same amount of absorption as the same drug exposed to a small surface

area for a prolonged period of time.


(Choice E) A drug administered orally with a high absorption rate but a

high first-pass metabolism will ultimately have a low bioavailability.

A drug that is absorbed slowly but completely (given enough time) with
a low first-pass metabolism will have a high bioavailability.

Educational Objective:
Oral administration subjects a drug to a large amount of first-pass

metabolism whereas IV, sublingual, and rectal administration bypasses

some or all of this process and allows more drug to reach the systemic
circulation.

144

USMLE WORLD STEP 1 PHARMACOLOGY

Q NO 106: A 45-year-old male is diagnosed with pulmonary tuberculosis. Acid-fast

bacterial isolates obtained from this patient are resistant to some

antimycobacterial drugs. Adding ethambutol to this patient’s treatment regimen

would most likely require periodic testing of:

A. Visual acuity

B. Hepatic function
C. Hearing and vestibular function

D. Complete blood count

E. Renal function

Explanation:

Ethambutol is an effective component of the multi-agent antibiotic


regimen used in the treatment of M. tuberculosis and atypical

mycobacterial infections. The most notable side effect of ethambutol is

optic neuritis. This side effect clinically manifests as decreased


visual acuity, central scotoma, or color-blindness and may be

reversible with discontinuation of the drug.

(Choice B) Hepatic function must be monitored in patients treated with


isoniazid, rifampin, or pyrazinamide.

(Choice C) Hearing and vestibular function must be monitored with the

prolonged use of aminoglycoside antibiotics or vancomycin. The


ototoxicity results from direct damage to the eighth cranial nerve, and

can therefore also cause vertigo, tinnitus, and deafness. In addition

to ototoxicity, the aminoglycosides can cause renal toxicity. An


exceedingly rare aminoglycoside side effect is flaccid paralysis due to

neuromuscular blockade.

(Choice D) The complete blood count should be monitored in patients


treated with chloramphenicol, dapsone, or trimethoprim-

sulfamethoxazole. Chloramphenicol can cause aplastic anemia, and

dapsone can cause agranulocytosis. Trimethoprim-sulfamethoxazole can


lead to a megaloblastic anemia in some patients because it is a folate

antagonist.

(Choice E) Renal function is most commonly affected by the


aminoglycosides and amphotericin B. Specifically, aminoglycosides cause

acute tubular necrosis.

Educational Objective:

Ethambutol is noted for causing optic neuritis that results in color


blindness, central scotoma, and decreased visual acuity. This adverse

side effect may be reversible with discontinuation of the drug.

145

USMLE WORLD STEP 1 PHARMACOLOGY

Q NO 107: A 6-year-old Caucasian boy is brought to your office by his mother. She

says that the boy has become ‘inattentive recently, displaying frequent episodes of

staring into space that last for several seconds and occur several times during the

day. His past medical history is insignificant. Which of the following should be
considered as a treatment option in this patient?

A. Lorazepam
B. Ethosuximide

C. Phenobarbital

D. Carbamazepine
E. Methylphenidate

Explanation:
Absence seizures (petit mal seizures) occur mainly in children and are

characterized by sudden momentary lapses in awareness (the absence

attack), staring, rhythmic blinking, and, occasionally, small clonic


jerks of arms or hands. Following an attack, behavior and awareness

return immediately to normal. The patient does not experience a

postictal period and usually has no recollection that a seizure has


occurred. Most absence seizures last less than 10 seconds. The drug of

choice for treatment is either ethosuximide or sodium valproate. The

interictal EEG of patients with typical absence seizures reveals


generalized 3.0-Hz spike-wave complexes superimposed on normal

background activity.

(Choice A) The optimal first-line agents for anticonvulsant therapy in


patients of all ages are the benzodiazepines. Available agents include

diazepam (Valium), lorazepam (Ativan), and midazolam (Versed).

(Choice C) Phenobarbital is effective for generalized tonic-clonic


seizures, but is not the first line agent for absence seizures.

(Choice D) Carbamazepine is the drug of choice for complex partial

seizures and is also effective in generalized tonic-clonic seizures.


The most feared, though uncommon, side effect of carbamazepine is

agranulocytosis or an aplastic anemia.

(Choice E) Methylphenidate is an indirect-acting sympathomimetic,


indicated in attention-deficit-hyperactivity disorder and narcolepsy.

Educational Objective:
Absence seizures are a type of generalized nonconvulsive seizure

characterized by episodes of loss of awareness (typically 10 sec)

associated with a 3 Hz generalized spike and slow wave EEG pattern,


followed by abrupt return to full consciousness. The drug of choice for

treatment is ethosuximide or sodium valproate.

146

USMLE WORLD STEP 1 PHARMACOLOGY

Q NO 108: A patient in the operating room undergoes rapid sequence intubation to

prevent aspiration of gastric contents. During this rapid induction, he is given a

standard intravenous (IV) dose of muscle relaxant “X.” The patient is then given a

standard IV dose of vecuronium to maintain muscle relaxation during surgery. At


the end of the surgical procedure, more than an hour later, the anesthesiologist

administers a standard IV dose of neostigmine. However, the patient does not

respond and continues to display too much muscle paralysis to permit safe
extubation. Drug X was most likely which of the following?

A. Dantrolene

B. Pancuronium
C. Succinylcholine

D. Midazolam

E. Tubocurarine

Explanation:

There are two clues to the correct answer in this scenario. The first
is that the patient underwent rapid sequence induction (RSI) and

intubation. A depolarizing neuromuscular junction (NMJ) blocking drug

is commonly administered for RSI, because onset of action is generally


more rapid (within 60 seconds) than for most available non-depolarizing

NMJ blockers. The second clue is that the patient still exhibits
residual muscle paralysis even after neostigmine, an anti

cholinesterase, has been administered to reverse the action of

vecuronium, which is a non-depolarizing agent. The persistence of


paralysis indicates that Drug X is a depolarizing NMJ blocker.

Anticholinesterases do not reverse the action of depolarizing NMJ

blockers and may, in fact, enhance them.


The only depolarizing NMJ blocker listed among the answer choices is

succinylcholine. The duration of action of succinylcholine is

determined by its diffusion out of the neuromuscular junction and its


metabolism by plasma cholinesterase. This duration is normally around

10 minutes after a standard dose. About 1 in 3000 patients. However,

are heterozygous for an atypical plasma cholinesterase, which breaks


down succinylcholine more slowly, over 1 to 3 hours.

(Choice A) Dantrolene tends to relax skeletal muscle by reducing the

release of Ca++ from the sarcoplasmic reticulum. Dantrolene is not


routinely used as a neuromuscular paralytic agent. It is used to treat

malignant hyperthermia and the neuroleptic malignant syndrome.

(Choice E) Benzodiazepines have no direct action at the neuromuscular


junction and do not provide sufficient muscle paralysis to facilitate

intubation.

Educational Objective:

The paralytic action of non-depolarizing neuromuscular junction (NMJ)

blocking drugs can be reversed by anticholinesterase agents including


neostigmine. The same is not true of depolarizing neuromuscular

blockers such as succinylcholine. The duration of paralysis that is

caused by succinylcholine depends largely on its catabolism by plasma


cholinesterase.

147

USMLE WORLD STEP 1 PHARMACOLOGY

Q NO 109: A 59-year-old female presents to ER with facial swelling and difficulty

breathing. She has never had such symptoms before. You find out that’s he was

diagnosed with hypertension recently and treatment with captopril was started.

Which of the following is the most likely mechanism responsible for this patient’s
symptoms?

A. IgE-dependant mast cell degranulation


B. Non-immune mast cell degranulation

C. Decrease in angiotensin II level

D. Kinin accumulation
E. Increased renin secretion

Explanation:

Angioedema is a rare but potentially serious adverse effect of ACE


inhibitor therapy. It occurs in less than 1% of patients. Although it

can affect any tissue, angioedema most commonly involves swelling of

the tongue, lips, or eyelids. Patients can also experience laryngeal


edema and difficulty breathing.

The exact mechanism behind ACE inhibitor induced angioedema is believed

to be due to bradykinin accumulation. Normally, ACE is responsible for


bradykinin breakdown. ACE inhibitors block the effects of ACE and

decrease bradykinin degradation, leading to increased levels.

Bradykinin is a potent vasodilator that ultimately increases vascular


permeability causing significant angioedema.

Angioedema secondary to ACE inhibitors typically occurs within days of

initiation but can also occur after weeks to years of therapy. For
patients who develop angioedema, drug withdrawal is appropriate.

(Choice A) Even though IgE-dependent mast cell degranulation can cause

angioedema, it is more commonly associated with urticaria. True


hypersensitivity or allergy to ACE inhibitors is very rare and not

likely in this patient.

(Choice B) In non-immune mediated mast cell degranulation, activation


of mast cells are due to direct activation by stimuli other than IgE

(drugs, chemicals, heat, etc.). Drugs like opiates and vancomycin can

stimulate mast cells and cause pseudoallergic reactions. These


reactions are clinically similar to hypersensitivity and allergic

reactions, but do not involve drug-specific IgE. ACE inhibitors are not

known to cause non-immune mediated mast cell degranulation.


(Choice C) ACE inhibitors block the conversion of angiotensin Ito

angiotensin II, ultimately decreasing angiotensin II levels. The

decrease in angiotensin II is responsible for their pharmacologic


effect of arterial vasodilation and decreased sodium and fluid

retention. Decreased angiotensin II levels have no relation to the


etiology of angioedema.

(Choice E) In addition to blood pressure lowering effects, decrease

angiotensin II levels will also interfere with negative feedback


mechanisms ultimately activating the RAAS to promote renin release.

Increased renin levels are a natural compensatory response to ACE

inhibitor therapy and have no role in causing angioedema.


Educational Objective: Angioedema is a rare but potentially serious

adverse effect of ACE inhibitor therapy occurring in less than 1% of

patients. Although it can affect any tissue, angioedema most commonly


involves swelling of the tongue, lips, or eyelids and patients can also

experience laryngeal edema and difficulty breathing. ACE inhibitor

induced angioedema is more than likely due to increased bradykinin


levels as a result of ACE inhibition.

148

USMLE WORLD STEP 1 PHARMACOLOGY

Q NO 110: A chemotherapeutic compound that is structurally similar to pyridoxine

has been shown to inhibit the saturation of long. branched lipids within some

bacterial cells. This compound is most likely to inhibit the growth of:
A. Streptococcus

B. Legionella

C. Mycobacterium
D. Treponema

E. Mycoplasma

Explanation:

The mycobacteria are unique in their incorporation of mycolic acids


into the outer portion of their peptidoglycan cell wall. Mycolic acids

are long branched saturated fatty acids that contain approximately

ninety carbon molecules. The presence of these mycolic acids is what


makes Mycobacterium “acid fast.”

Isoniazid is chemically similar to pyridoxine, which is the B vitamin

(B6) that is depleted during isoniazid therapy. Isoniazid inhibits


mycolic acid synthesis in mycobacterial cells and is specific for

Mycobacterium species. as it has no effect on other microorganisms.

Inhibiting mycolic acid creation prevents mycobacterial cell wall


synthesis and the formation of many virulence factors. Isoniazid is not

without toxic effect on the host, however, as it causes neuropathy,

hepatotoxicity and CYP45O system induction within the liver.


(Choice A) Streptococcus consists of Gram-positive cocci that are most

frequently treated with beta-lactam antibiotics (eg, penicillin).

(Choice B) Legionella consists of Gram-negative rods that are


responsible for causing Legionnaires disease and Pontiac fever. Because

Legionella produces beta-lactamases, erythromycin is the first-line

antibiotic in the treatment of this pathogen. Erythromycin functions by


inactivating the bacterial 50S ribosomal subunit and preventing

bacterial protein synthesis.

(Choice D) All Treponema can be effectively treated with penicillin,


which is the first-line agent for this genus.

(Choice E) The genus Mycoplasma contains no peptidoglycan cell wall, so

it is not classified as Gram-positive or Gram-negative. This genus,


which includes Mycoplasma pneumoniae and Ureaplasma urealyticum, has a

single cell membrane composed of a cholesterol-containing phospholipid

bilayer. Mycoplasma therefore cannot be treated with cell wall


inhibitors. First-line treatment for infections with these organisms is

erythromycin or tetracycline. These agents inhibit bacterial protein

synthesis by blocking the 50S and the 30S ribosomal subunit,


respectively.

Educational Objective:
Isoniazid is chemically related to pyridoxine, also known as Vitamin

B6. It inhibits mycolic acid synthesis in mycobacterial cells and is


specific to the mycobacteria. Mycolic acid is a long branched chain

saturated fatty acid used in the mycobacterial cell wall and in the

formation of virulence factors.

149

USMLE WORLD STEP 1 PHARMACOLOGY

Q NO 111: A 60-year-old Caucasian male is diagnosed with exertional angina and

isosorbide dinitrate is prescribed for chronic therapy. The patient is concerned

about the possible side effects of this therapy. Which of the following is most

likely to occur in this patient?

A. AV conduction delay

B. Nocturnal wheezing
C. Constipation

D. Urinary retention

E. Throbbing headaches
F. Joint pain

G. Cold extremities

Explanation:

Headaches and cutaneous flushing are common side effects of all

nitrates (nitroglycerin and isosorbide) due to their vasodilatory


properties. These side effects are most common at higher doses where

nitrates can produce some arteriolar dilation, but the primary action

of nitrates is veno dilation.


(Choice A) AV Conduction delay is a side effect noted with drugs whose

action is to slow AV nodal conduction. These include digoxin,

quinidine, calcium channel blockers such as verapamil and diltiazem,


and betaadrenergic blockers such as metoprolol and propranolol.

(Choice B) Nocturnal wheezing can be a symptom of asthma or GERD.

Treatment of a patient who has asthma with a beta-adrenergic blocker


can cause increased bronchoconstriction leading to wheezing.

(Choice C) Constipation is a common side effect of opioid analgesics,

iron supplements, calcium channel blockers such as verapamil, and


anticholinergic medications such as atropine.

(Choice D) Urinary retention is a frequently tested side effect

associated with anticholinergic medications like atropine and drugs


exhibiting anticholinergic effects such as the tricyclic

antidepressants and antihistamines.

(Choice F) Joint pain could be caused if a patient with gout is


prescribed a thiazide diuretic as these medications can cause

hyperuricemia leading to an acute gout attack.

(Choice G) Cold extremities resulting from Raynaud’s phenomenon have


been associated with the use of beta- blockers as well as with

ergotamine used in the treatment of migraine headaches.

Educational Objective:

All nitrates, both long and short acting, are most commonly associated
with the side effects of headache and facial flushing. These reactions

occur because of the vasodilatory properties of nitrates in the

meninges and skin.

150

USMLE WORLD STEP 1 PHARMACOLOGY

Q NO 112: A 46-year-old Caucasian male is being treated for elevated cholesterol

levels. His past medical history is significant for hypertension, stable angina and

diabetes mellitus. Which of the following drugs when used as monotherapy, would

be most likely to increase triglyceride levels in this patient?

A. Atorvastatin

B. Niacin
C. Gemfibrozil

D. Cholestyramine

E. Ezetimibe

Explanation:

Bile acid-binding resins (cholestyramine, colestipol and colesevelam)


work by binding bile acid in the gastrointestinal tract, thereby

interfering with its enterohepatic circulation. LDL is reduced as a

result, because hepatic cholesterol is used up for the re-synthesis of


bile acids, which in turn increases the uptake of LDL from the

circulation.

The main side effects of bile acid-binding resins are gastrointestinal


upset, impaired absorption of nutrients and drugs, and

hypertriglyceridemia. Bile acid production is increased 10-fold in

patients on these medications because of the interruption in the


enterohepatic circulation of bile acids. Hepatic production of

triglycerides and VLDL is increased as well, which results in an

increase in serum triglyceride levels. Because of this tendency for


bile acid- binding agents to increase serum triglyceride levels, they

should not be used in hypercholesterolemia patients with who have

concomitant hypertriglyceridemia. In addition, bile acid-binding agents


increase the cholesterol content of bile thus increasing the risk for

formation of cholesterol gallstones.

Bile acid-binding resins are primarily used in combination with


statins. Concurrent administration of these two drug types results in

decreased statin absorption. For this reason, it is recommended that

these agents be administered at least four hours apart.


(Choice A) Statins cause modest reductions in serum triglyceride

concentration.

(Choice B) Niacin reduces serum triglyceride concentrations.


Additionally, niacin decreases VLDL conversion to LDL thus decreasing

serum LDL concentrations as well. Nicotinic acid also increases HDL by


25-30% making it the most effective HDL-increasing agent currently

available.

(Choice C) Gemfibrozil and fenofibrate are first-line treatments for


hypertriglyceridemia. These agents decrease serum triglycerides by

reducing hepatic triglyceride production.

(Choice E) Ezetimibe is a new drug that selectively inhibits the


intestinal absorption of cholesterol from diet and bile acid. The

result is a 20-30% reduction in serum [DL. Ezetimibe is modestly

effective at reducing serum triglycerides and increasing serum HDL


concentrations. This medication is primarily used in conjunction with

statin therapy.

Educational Objective:

Bile acid-binding resins are unique among hypolipidemic agents in that

they increase blood triglyceride levels. Fibric acid derivatives are


the first-line treatment for hypertriglyceridemia.

151

USMLE WORLD STEP 1 PHARMACOLOGY

Q NO 113: A strong, non-specific muscarinic agonist has recently been developed.

This new agent would most likely have which of the following actions?

A. Bladder wall relaxation


B. Kidney renin release

C. Gl sphincter contraction

D. Increased ventricular contractility


E. Release of endothelium-derived relaxation factor

Explanation:
Muscarinic agonists have distinct effects on the heart, peripheral

blood vessels, gastrointestinal tract genitourinary tract, eye, and

secretory glands.
Although the walls of peripheral blood vessels do not have cholinergic

innervation muscarinic receptors are present on the endothelial

surface. Binding of cholinomimetic agents to these receptors promotes


release of nitric oxide (NO), also called endothelium-derived relaxing

factor (EDRE). NO activates guanylate cyclase and increases

intracellular cGMP. cGMP activates a Ca pump and causes Ca efflux from


the cells. A decrease in calcium concentration causes vascular wall

smooth muscle relaxation.

(Choice B) Juxtaglomerular cells release renin in response to beta-i


stimulation. Cholinomimetics do not affect renal release of renin.

(Choices A and C) Cholinomimetics increase the tone of smooth muscles

of the visceral walls. They increase motility and secretion in GI


tract, manifesting clinically as nausea, vomiting, abdominal cramps and

diarrhea. Contrarily, anticholinergic drugs relax the GI wall and


increase the contraction of the sphincters, causing constipation.

Cholinomimetic-mediated increases in secretion manifest as excessive

salivation, sweating and lacrimation.


Stimulation of bladder wall smooth muscle (detrusor) promotes

urination. Cholinomimetics increase detrusor tone and stimulate its

contraction. Relaxation of the detrusor is an effect of anti


cholinergic medications.

(Choice D) Cholinomimetics cause negative chronotropic and inotropic

effects in the heart. A negative inotropic response (decreased


contractility) occurs due to decreased Ca current into cardiac

myocytes. This effect is more pronounced in the atria than the

ventricles because ventricles have more adrenergic innervation.


Cholinomimetics also decrease Ca currents in the SA and AV nodes,

decreasing conduction velocity. Decreased conduction may result in

heart block.

Educational Objective:

Cholinergic agonists bind to muscarinic receptors on endothelial cells


and promote release of NO (EDRE). NO activates guanylate cyclase and

diminishes endothelium calcium concentration. This produces

vasodilatation.

152

USMLE WORLD STEP 1 PHARMACOLOGY

Q NO 114: A 35-year-old male presents to the physician’s office with a several day

history of colicky abdominal pain, constipation irritability and headaches. He works

at a battery manufacturing factory. His past medical history is significant for iron

deficiency anemia depression and occasional illicit drug use. Which of the following
is the most likely cause of this patient’s current condition?

A. Lead poisoning
B. Iron poisoning

C. Fluoxetine overdose

D. Methanol ingestion
E. Cocaine abuse

Explanation:
Lead is a soft heavy metal used in the manufacture of a wide range of

items, including batteries, alloys, and ammunition. Individuals working

in these industries and others (eg, mining, smelting, chemical


processing, recycling, spray painting, and radiator repair) are

regularly exposed to lead.

Adult lead poisoning can have the following manifestations:


1. Colicky abdominal pain (“lead colic”), constipation, headaches,

impaired concentration and deficits in short- term memory.

2. Bluish pigmentation (“lead line”) at the gum-tooth line.


3. Wrist drop or foot drop due to peripheral neuropathy.

4. Microcytic hypochromic anemia and basophilic stippling on peripheral

smear.
Lead poisoning can be diagnosed when the blood lead level exceeds 10

µg/dL (0.48 µmol/dL) on a venous blood sample.

(Choice B) Iron poisoning is typically separated into four stages.


First, nausea, diarrhea, and abdominal pain are experienced, often

accompanied by hemorrhage, hypovolemia, and shock in severe cases. In

stage two, the gastrointestinal symptoms resolve and the patient


appears better. By stage three, metabolic acidosis, hepatic

dysfunction, and hypoglycemia may set in. Stage four is marked by

scarring of the recovering gastrointestinal tract.


(Choice C) Fluoxetine overdose appears to result in minor, nonspecific

symptoms including tachycardia, drowsiness, tremor, vomiting, and

nausea.
(Choice D) Methanol is dangerous in small doses with its most toxic

metabolites (formaldehyde and formic acid) arising after hepatic

processing. Damage to the eyes, gastrointestinal tract, and central


nervous system may be observed.

(Choice E) Cocaine abuse can affect every organ system. Common


complaints include altered mental status, new onset seizures,

hypertension, chest pain, myocardial ischemia or infarction, shortness

of breath, intracranial hemorrhage, epistaxis, or psychiatric


dysfunction.

Educational Objective:
Clinical features of adult lead toxicity include abdominal colic,

constipation, headache, lead line and peripheral neuropathy. Microcytic

hypochromic anemia and basophilic stippling is also present.

153

USMLE WORLD STEP 1 PHARMACOLOGY

Q NO 115: A group of investigators is studying cardiovascular responses to various

vasoactive drugs infused in low doses. They want to compare the relative

selectivity of different agents to various components of the cardiovascular system.

Which of the following components of the cardiovascular system is most


susceptible to the drug presented above?

A. Large veins
B. Large arteries

C. Small arteries and arterioles

D. Precapillary sphincters
E. Cardiac muscle

Explanation:
The chemical depicted above is nitroglycerin. Nitroglycerin acts

primarily as a veno dilator at the lower doses used in angina patients.

When a patient takes nitroglycerin, cardiac workload is decreased


because blood collects in the venous system (redistribution) thereby

decreasing preload. Decreased preload in the heart decreases

ventricular wall stress thereby decreasing cardiac oxygen demand. Large


veins are most susceptible to the effect of nitroglycerin.

(Choice B) Large arteries are the prima conduits for blood delivery to
the tissues and contain a large amount of smooth muscle in their walls

to regulate blood pressure and withstand high pressures.

(Choice C) Small arteries and arterioles are the prima site of hormonal
regulation of systemic blood pressure and the primary vascular site of

action of vasoactive antihypertensive drugs such as nifedipine (calcium

channel blocker), propranolol (nonspecific beta-adrenergic blocker) and


prazosin (alpha-1-adrenergic blocker). In large doses nitrates can also

have an arteriolar dilating effect which leads to cutaneous flushing

and headache.
(Choice D) Precapillary sphincters are bands of smooth muscle lying at

the junction of a capillary and an arteriole. They limit the flow of

blood into capillary beds as the capillaries themselves are formed only
of a single layer of endothelial cells and have no smooth muscle in

their walls thereby making them unable to regulate flow and pressure

themselves. These sphincters are responsive to norepinephrine and


epinephrine and depending on their location will either relax or

contract in response to these agents. Additionally the precapillary

sphincters can sense blood flow demand in target tissues and will
dilate in response to histamine low elevated 002 or decreased pH

(acidosis).

(Choice E) Cardiac muscle is not directly affected by nitrate drugs.


Cardiac work is decreased indirectly by the nitrates as a result of

their vasodilatory properties that lead to a decrease in preload and

ventricular wall stress.


Educational Objective:

Nitroglycerin is primarily a veno dilator. It decreases preload which

decreases myocardial oxygen demand and thereby treats angina pectoris.

154

USMLE WORLD STEP 1 PHARMACOLOGY

Q NO 116: A 56-year-old male is admitted to ER with severe chest pain. His initial

blood pressure is 240/130 mmHg and heart rate is 100/mm. A beta-blocker and

nitroprusside infusion is started. Several hours after admission the patient seems

confused and disoriented. You noticed that nitroprusside infusion rate is higher
than recommended. Supplying which of the following elements would help reverse

this patient condition?

A.
B.
Oxygen
Phosphorus

C. Sulfur

D.
E.
Sodium
Potassium

F. Hydrogen

Explanation:

Nitroprusside is an antihypertensive with mixed arterial and venous


vasodilatory actions. It is commonly used in the emergent setting where

quick onset of action is necessary to control blood pressure and

prevent target organ damage. Nitroprusside is given via intravenous


continuous infusion and has an onset of action within 30 seconds.

One major disadvantage to the use of nitroprusside is potential cyanide

toxicity. Nitroprusside is initially metabolized to release cyanide and


nitric oxide. Cyanide is then metabolized in the liver by liver

rhodanese to thiocyanate, which is eventually excreted in the urine.

The risk for cyanide and thiocyanate toxicity is not common but can
occur. The risk increases with higher dosages, greater infusion rates,

prolonged use, and renal insufficiency.

Clinical signs of cyanide toxicity include altered mental status and


lactic acidosis. If cyanide toxicity does occur, sodium thiosulfate can

be given. Liver rhodanese normally transfers sulfur to cyanide to form

thiocyanate. Sodium thiosulfate works by donating additional sulfur to


liver rhodanese to enhance metabolism and detoxification of cyanide to

thiocyanate.

(Choice A) Although oxygen is given in cyanide toxicity, it is more


commonly given in acute cyanide toxicity associated with poisonings to

help supplement recovery. In these situations oxygen prevents cyanide

inhibition of cytochrome oxidase which is needed for respiration.


Oxygen is also the antidote for carbon monoxide poisoning.

(Choices B, E, and F) The elements phosphorous, potassium, and hydrogen

have no significant role in the detoxification reactions of cyanide


poisoning.

(Choice D) Although sodium thiosulfate contains sodium, the sodium

component has no role in the detoxification reaction in cyanide


poisoning.

Educational Objective:
Nitroprusside is the agent of choice in treating hypertensive

emergency. It has a quick onset of action and short duration of action.


Nitroprusside is initially metabolized to cyanide, with subsequent

conversion to thiocyanate by liver rhodanese. Thus, one major

disadvantage of its use involves the risk for developing cyanide


toxicity. Sodium thiosulfate is used to treat cyanide toxicity and

works by donating sulfur to liver rhodanese to enhance conversion of

cyanide to thiocyanate.

155

USMLE WORLD STEP 1 PHARMACOLOGY

Q NO 117: A new medication inhibits fungal colony growth by altering the fungal

cell membrane composition. In human volunteers, the drug inhibits liver

cytochrome P450-dependent metabolism of several drugs. This new drug is most

similar to which of the following agents?

A. Amphotericin B

O
B.
C.
Itraconazole
Terbinafine
D. Griseofulvin

E. Caspofungin
F. Flucytosine

Explanation:
The azoles are antifungal medications that inhibit the demethylation of

lanosterol into ergosterol in fungal cells. By suppressing the

synthesis of ergosterol, an essential component of the fungal cell


membrane, these medications exert their antifungal effects. However,

the azoles also inhibit the activity of the human P450 cytochrome

oxidase system. This property leads to multiple drug interactions.


All azoles (ketoconazole, fluconazole, itraconazole and voriconazole)

increase the serum concentrations of drugs metabolized by liver P450

enzymes. This means that, when used concomitantly with azoles, drugs
metabolized by the liver P450 system may have increased toxicity. For

this reason, one should avoid using the following medications in

combination with azoles: warfarin, cyclosporine, tacrolimus, phenytoin,


isoniazid, rifampin, oral hypoglycemics and many others.

Cytochrome oxidase inducers, such as rifampin, isoniazid, phenytoin,

carbamazepine, phenobarbital and ritonavir, increase azole metabolism,


and therefore lower their concentration in serum.

(Choice A) Amphotericin B is a polyene antifungal that binds ergosterol

in the fungal cell membrane, leading to pore formation and cell lysis.
(Choice C) Terbinafine accumulates in skin and nails and is used to

treat dermatophytoses. Terbinafine inhibits the fungal enzyme squalene-

2, 3-epoxidase, which ultimately results in decreased synthesis of


ergosterol.

(Choice D) Griseofulvin enters fungal cells and binds microtubules,

inhibiting mitosis.
(Choice E) 1, 3-beta-D-glucan is a major polysaccharide component of

the fungal cell wall. Caspofungin and the other echinocandin


antifungals block glucan synthesis.

(Choice F) Flucytosine inhibits the synthesis of both DNA (replication)

and RNA (protein synthesis) in fungal cells. It is mainly used as a


synergistic agent with amphotericin B. This drug combination is

particularly important in the treatment of cryptococcal meningitis.

Educational Objective:

Azoles inhibit the synthesis of ergosterol by the fungal cytochrome

P450 enzymes. They also suppress the human P450 system resulting in
many drug-drug interactions.

156

USMLE WORLD STEP 1 PHARMACOLOGY

Q NO 118: A 24-year-old male is brought to the ER in deep coma. He is cyanotic

and unresponsive to painful stimuli. His blood pressure is 100/60 mmHg, pulse is

100/mm, respirations are 4/mm and temperature is 36.7C (98F). His pupils are

constricted and poorly responsive to light. Several minutes after admission to ER


the patient regained his consciousness and states that he wants to go home. The

drug used in this patient’s resuscitation has the greatest affinity to which of the

following receptors?

A. Delta

B. Kappa

C.
D.
Mu
NMDA

E. GABAA

F. NN

Explanation:
Several different types of opioid receptors have been identified and

include mu, delta, kappa, and N/OFQ receptors. Available narcotics like

morphine and hydromorphone produce therapeutic analgesic effects by


binding to mu receptors and modulating pain perception.

Naloxone is a pure opioid receptor antagonist devoid of any agonistic

properties. It is primarily used in the setting of opioid intoxication


or overdose. Although it antagonizes the effects of opioids at all

receptor types, it has the greatest binding affinity to mu receptors

and is thus an ideal agent for treating opioid intoxication. As an


antagonist, naloxone competes for opioid receptors and reverses their

effects. Administration of naloxone reverses analgesia, sedation,

hypotension, and respiratory depression. Reversal effects can be seen


within minutes and the duration of effects are dose dependent but

usually last anywhere from 1 to 4 hours. Naloxone must be given

parenterally secondary to complete metabolic inactivation by the liver


when given orally.

(Choices A and B) Delta and kappa are also types of opioid receptors

that modulate pain. Although naloxone does bind to these receptors, it


binds to mu opioid receptors with the greatest affinity and thus

effectively antagonizes the effects of opioid narcotics at mu

receptors.
(Choice D) Glutamate is an excitatory neuro transmitter that binds to

NMDA receptors. NMDA receptor activation is important for

neurotransmission involved with learning and memory. Ketamine, a


general anesthetic, is one example of an NMDA receptor antagonist.

Naloxone does not have effects on NMDA receptors.

(Choice E) The neurotransmitter GABA binds to GABA receptors to produce


inhibitory effects. GABAA receptors are the most prevalent GABA

receptor subtypes present in mammals. Drugs like benzodiazepines and

barbiturates work by binding to specific sites on GABA receptors.


Flumazenil is a benzodiazepine receptor antagonist used in

benzodiazepine reversal and intoxication. Naloxone does not have any

effects on GABAA receptors.


(Choice F) NN receptors are nicotinic acetylcholine receptors located

in neural tissues. They are responsible for neurotransmission.

Hexamethonium is a potent nicotinic receptor antagonist. Naloxone has


no effects on NN receptors.

157

USMLE WORLD STEP 1 PHARMACOLOGY

Educational Objective: Naloxone is a pure opioid receptor antagonist

used for treating opioid intoxication or overdose. Although it binds to


mu, kappa, and delta opioid receptors, it has the greatest affinity

form receptors making it an ideal agent for treating opioid

intoxication.

158

USMLE WORLD STEP 1 PHARMACOLOGY

Q NO 119: A 43-year-old Caucasian male undergoing treatment for invasive

aspergillosis with amphotericin B experiences headaches, hypotension and a


A.

decline in renal function. Many of amphotericin B’s toxic effects are due to

its binding
Cytochrome P450 enzymes

B. Cell membrane cholesterol

C. Microtubular proteins
D. Ribosomal subunits

E. Mitochondrial enzymes

Explanation:

Amphotericin B is a mainstay of treatment for many systemic mycoses. It

preferentially binds the ergosterol of fungal cell membranes, leading


to fungal cell lysis. This drug is relatively selective, because it has

a higher affinity for ergosterol (in fungal membranes) than for

cholesterol (in human cell membranes). This drug does, however, bind
cholesterol to a degree, which explains a large number of its adverse

effects.

Amphotericin B is administered intravenously. Its main toxicities


include:

1. Acute infusion-related reactions, such as fever, chills, rigors, and

hypotension. Acute infusion-related reactions are common, and most


frequent during initial infusions (often diminish with subsequent

infusions). Premedication with antipyretics and antihistamines can

lessen the severity of these effects.


2. Amphotericin B causes a dose-dependent nephrotoxicity because it

decreases the glomerular filtration rate. Permanent loss of renal

function is thought to be related to the cumulative total dose.


Increasing BUN and creatinine levels indicate declining renal function.

Renal function should be closely monitored in patients undergoing

treatment with Amphotericin. Concomitant administration of other


nephrotoxic drugs (e.g., aminoglycosides, cyclosporine) should be

avoided.

3. Amphotericin B can also cause significant electrolyte abnormalities


(hypomagnesemia and hypokalemia). These effects occur in the majority

of patients within the first week of therapy. Electrolytes should be

monitored daily and replaced as needed.


4. Anemia occurs due to suppression of renal erythropoietin synthesis.

This effect may be severe in HIV (+) patients taking zidovudine (a drug
that also suppresses bone marrow function).

5. Thrombophlebitis at the site of injection may occur.

(Choice A) Triazole antifungals (e.g. fluconazole, itraconazole)


inhibit cytochrome P450 enzymes, suppressing the metabolism of many

other medications. This effect explains the myriad drug interactions of

this group of antifungals.


(Choice C) Binding of microtubular proteins is the mechanism of

griseofulvin. This drug is effective against dermatophytes only.

(Choices D and E) Amphotericin B does not interact with ribosomal


subunits or mitochondrial enzymes.

Educational Objective: Amphotericin B binds the ergosterol of fungal

cell membranes to exert its antifungal effects. However, it also binds


cholesterol to some degree, causing toxicity to human tissues. The most

important adverse effects of amphotericin B are nephrotoxicity,

hypokalemia, and hypomagnesemia.

159

USMLE WORLD STEP 1 PHARMACOLOGY

Q NO 120: A new drug that is being tested in phase III clinical trial has been found

to be associated with lupus-like syndrome in 20% of patients treated for more than

one year. The syndrome is accompanied by the presence of anti-histone antibodies

in many patients. The drug tested in the clinical trial is most likely metabolized by
which of the following routes?

A. Unchanged excretion
B. Liver sulfate conjugation

C. Liver hydroxylation

D. Liver hydrolysis
E. Liver acetylation

F. Plasma hydrolysis

Explanation:

Drugs like hydralazine and procainamide have been known to trigger and

even aggravate lupus syndrome. The risk for development of drug-induced


lupus appears to increase with higher dosages and longer treatment

durations.

Both agents are metabolized via phase II N-acetylation in the liver.


Genetic predisposition determines acetylator phenotype and it has been

discovered that slow acetylators develop lupus more often, suggesting

that parent drugs are more likely responsible for the development of
lupus. A majority of patients on either medication will develop anti

histone antibodies and the exact pathogenesis of autoantibody

development remains unclear. Most patients on hydralazine or


procainamide can have positive antinuclear antibody tests without

developing clinical symptoms. Discontinuation of drug is only necessary

in patients who develop clinical symptoms, which include arthritis,


fever, rash, and more severely, pleuritis or pericarditis.

(Choice A) Very few drugs are not metabolized in the body. These drugs

are considered biochemically inert and go through unchanged excretion.


They are excreted from the body in the original, unchanged form. The

diuretic chlorothiazide is one example.

(Choice B) Liver sulfate conjugation is one of the primary phase II


metabolic pathways that bio transforms drugs into more polar drugs for

excretion. In sulfate conjugation, sulfate reacts with a substituent on

the drug’s molecule to form a new chemical compound that is more water
soluble and easily excreted. Phenol and chloramphenicol are two drugs

that go through liver sulfate conjugation.

(Choice C) Liver hydroxylation is one of several examples of oxidation


transformations that are catalyzed by the cytochrome P450 monooxygenase

system. In liver hydroxylation, a hydroxyl group is added to the


compound to produce less lipid soluble compounds. Pentobarbital and

phenobarbital go through liver hydroxylation.

(Choice D) Liver hydrolysis is a phase I type reaction of liver


metabolism where a compound is cleaved by adding water. Phase I

metabolism usually precedes phase II conjugation metabolism. Most liver

hydrolysis reactions involve esterase or amidase enzymes which


biotransform the drug. Procaine, lidocaine, and aspirin are drugs that

go through liver hydrolysis.

(Choice F) In plasmas hydrolysis, blood or plasma esterases cleave


ester linkages to biotransform or inactivate drugs rapidly. This rapid

inactivation allows for the short acting profiles of many agents. Many

drugs used in anesthesia are metabolized by plasma hydrolysis


(succinylcholine, tetracaine, and remifentanil).

160

USMLE WORLD STEP 1 PHARMACOLOGY

Educational Objective:

Drug induced lupus has been linked to drugs that are metabolized by N-
acetylation in the liver. Two classic examples include hydralazine and

procainamide. Genetic predisposition determines acetylator phenotype

and it appears that patients who are slow acetylators are at greater
risk for developing lupus-like syndrome. Discontinuation of the

offending agent is warranted only in patients who develop clinical

symptoms of lupus.

161

USMLE WORLD STEP 1 PHARMACOLOGY

Q NO 121: A 40-year-old female experiences a right knee injury during routine

sport activities. She is treated with an opioid analgesic in ER and her knee pain

decreases significantly. However, soon after the analgesic administration she

complains of severe abdominal pain that makes impossible for her to lie still.
Physical examination reveals tenderness over the right upper abdominal

quadrant. A drug effect on which of the following structures is most likely

responsible for this patient’s current condition?

A.
B.
Hepatocytes
Blood vessels

C. Enterocytes

D.
E.
Gastric epithelium
Smooth muscle cells

F. Pancreatic acini

G. Skeletal muscles

Explanation:
Administration of mu opioid analgesics like morphine can cause

contraction of smooth muscle cells in the sphincter of Oddi leading to

constriction and spasm. Constriction and spasm of the sphincter of Oddi


can increase common bile duct pressures. Pressures in the gallbladder

can also increase potentially leading to biliary colic.

Although opioid induced biliary colic can occur it is very rare.


Patients who do develop biliary colic will present with severe pain and

cramping in the right upper abdomen. Jaundice in the skin and eyes can

also occur in more severe cases. Management involves drug


discontinuation supportive treatment, and pain control. Since all mu

agonists can cause constriction of the sphincter of Oddi, itis often

recommended to avoid them in general. Meperidine is believed to cause


less sphincter of Oddi constriction and is often considered the opioid

of choice in biliary and pancreatic pain. However, scientific data is

not convincing and there is little clinical data to support this


notion. Itis recommended to use NSAIDs like diclofenac or ketorolac for

subsequent pain control.

(Choice A) All mu opioid analgesics are metabolized by the liver, but


have no direct toxic effects on hepatocytes.

(Choice B) All mu opioids cause histamine release leading to

vasodilation of blood vessels and itching. Opioids should be used


cautiously in patients with hypotension.

(Choice C) Mu opioid analgesics cause significant constipation by

binding to mu receptors in the bowel and slowing gut motility.


(Choice D) Mu opioid analgesics usually decrease parietal cell acid

secretion.

(Choice F) Although mu opioids can indirectly increase somatostatin


secretion from the Islets of Langerhans, they have no direct effects on

pancreatic acini.

(Choice C) Opioid analgesics have no direct effect on skeletal muscles.

Educational Objective:

Opioid analgesics can cause contraction of smooth muscles in the


sphincter of Oddi leading to increased pressures in the bile duct and

the gall bladder. Increased pressures can lead to a rare painful crisis

known as biliary colic. Although meperidine has been reported to cause


less constriction of the sphincter of Oddi there is little evidence to

support this notion.

162

USMLE WORLD STEP 1 PHARMACOLOGY

Q NO 122: A new drug has been developed to treat cardiac arrhythmias. The drug

has high affinity for activated and inactivated sodium channels but relatively little

affinity for resting sodium channels. A curve describing the drug’s dissociation

from non-resting sodium channels overtime after a single stimulus is given on the
picture below and labeled as ‘1’. The dissociation curve for quinidine is given for

comparison (line ‘2’). The new drug has an action most similar to which of the

following drugs?

A. Amiodarone

B. Propranolol

C.
D.
Lidocaine
Disopyramide

E. Propafenone
F. Verapamil

Explanation:
The principle of use-dependence in this case has to do with how avidly

the class I antiarrhythmic medications bind to non-resting sodium

channels in cardiac pacemaker cells and myocytes. A drug that binds


less avidly (binds with less strength) will take less time to

dissociate from the target. Additionally, a drug that binds less avidly

will be more selective for rapidly depolarizing cells. These cells


which depolarize at a high frequency are often the cause of ectopic

pacemakers and arrhythmia. In this case, the class lB antiarrh5cthmics

have the shortest dissociation time, bind less avidly to the non-
resting sodium channels, and are therefore most selective for rapidly

and frequently depolarizing cells. Examples of this class of drugs are

lidocaine (Choice C), mexiletine and tocainide. These drugs have the
effect of shortening the length of the action potential and phase 3

repolarization while they have little effect on the rapidity of phase 0

depolarization. These drugs are useful in arrhythmias that occur during


an ischemic cardiac event (ventricular arrhythmias are one of the most

common causes of death in the short-term following an acute myocardial

infarction). These drugs do not slow conduction, so they will not be


useful in atrial or AV nodal dysrhythmias. The above graph illustrates

class 13 drugs dissociating from non-resting sodium channels more

rapidly than class 1A or class 1C drugs.


(Choice A) Amiodarone is primarily classified as a class 3

antiarrhythmic medication though it has small class 1, 2 and 4 effects

as well. The class 3 anti arrhythmic act primarily by blocking


potassium channels and prolonging phase 3 repolarization.

(Choice B) Propranolol, along with the other beta-adrenergic blockers,

is categorized as a class II antiarrhythmic drug. Nonselective beta-


blockers such as propranolol block both the beta-1 and beta-2

adrenergic receptors leading principally to decreased cardiac rate-


chronotropy- and contractile force- inotropy (B-1 effect).

163

USMLE WORLD STEP 1 PHARMACOLOGY

(Choice D) Disopyramide is a class 1A antiarrhythmic drug and is

represented by “line 2” on the above graph. Other drugs in this class


include quinidine and procainamide. Class 1 A drugs have an affinity

for non-resting sodium channels that is intermediate between that of

class lB drugs (low affinity) and class 1 C drugs (high affinity).


Class 1 A drugs slow both phase 0 depolarization as well as phase 3

repolarization. They also prolong the refractory period. Quinidine is

the major drug of this class and is used for tachyarrhythmias arising
in the atria and AV junction.

(Choice E) Propafenone is a class 1 C drug and is represented by “line

3” on the above graph. Another drug in this class is flecainide. Drugs


in this class are less specific for frequently depolarizing cells due

to their high affinity for the non-resting sodium channel; therefore

they have more of an effect on normal cardiac myocytes leading to a


negative inotropic effect which limits their use. This class of drugs

slows phase 0 depolarization with little effect on repolarization or on

the length of the refractory period.


(Choice F) Verapamil as a calcium channel blocker is classified as a

class IV antiarrhythmic drug. Another drug in this class is diltiazem.

Though nifedipine is a calcium channel blocker as well, it is selective


for vascular smooth muscle and does not have a notable effect on the

heart it is used for the treatment of hypertension.

Educational Objective:

Class I antiarrhythmics:

164

USMLE WORLD STEP 1 PHARMACOLOGY

Q NO 123: A 35-year-old male presents to the physician’s office with sudden onset

pain swelling and redness of the right first metatarsophalangeal joint. A needle

aspirate of the joint shows needle-shaped, negatively birefringent crystals. You

prescribe a medication for the patient, but he calls back in one day saying that he
developed nausea, vomiting and diarrhea alter taking the medication. Which of the

following is the most likely mechanism of action of the drug prescribed to this

patient?

A. Inhibition of microtubule formation

B. Decreased renal tubular uric acid reabsorption

C.
D.
Inhibition of cyclooxygenase activity
Inhibition of xanthine oxidase activity

E. Inhibition of phospholipase A2 activity

Explanation:

This patient’s acute onset pain and swelling of the right first
metatarsophalangeal joint and joint aspirate showing needle-shaped

negatively birefringent crystals indicate that he most likely has acute

gouty arthritis. Colchicine is an effective anti-inflammatory agent in


acute gouty arthritis and acts by binding to the intracellular protein

tubulin, preventing tubulin polymerization into microtubules. This

effect of colchicine leads to impaired leukocyte migration and


phagocytosis, reducing the animation seen in gouty arthritis. Many

patients treated with colchicine develop diarrhea and less commonly,

nausea and vomiting, as the gastrointestinal mucosa is also affected by


colchicine’s inhibition of tubulin polymerization. Because of

colchicine’s toxic side effects, its use is recommended for patients

who cannot take NSAIDs or have contraindications to their use (renal


failure peptic ulcer disease etc.).

(Choice C) NSAIDs induce their anti-inflammatory effects by inhibiting

cyclooxygenase (COX) 1 and (COX) 2 isoenzymes, thus blocking


prostaglandin synthesis

(Choices B and D) Decreased proximal tubular uric acid reabsorption

describes the mechanism of action of probenecid, often used to lower


serum uric acid levels in chronic gout. Inhibition of xanthine oxidase

describes the mechanism of action of allopurinol, a medication also

prescribed to lower serum uric acid levels in chronic gout. Remember


that these agents are contraindicated in acute gouty arthritis and may

actually precipitate acute attacks.

(Choice E) Glucocorticosteroid exert their anti-inflammatory effect by


inhibiting phospholipase A2 activity. These medications can be used in

acute gout but are not associated with significant diarrhea.

Educational Objective:

Colchicine inhibits tubulin polymerization and microtubule formation in

leukocytes, reducing neutrophil chemotaxis and emigration to sites


inflamed by uric acid crystal deposition in acute gouty arthritis. Many

patients treated with colchicine also develop diarrhea as the

gastrointestinal mucosa is adversely affected by colchicine.

165
USMLE WORLD STEP 1 PHARMACOLOGY

Q NO 124: A 22-year-old pregnant patient has hyperglycemia on routine blood work


analysis. You perform a 3 hour glucose tolerance test, and based on the results,
make the diagnosis of gestational diabetes. After one week on a calorie-restricted
diet, the patient’s blood glucose levels remain elevated. You decide to initiate
pharmacotherapy. Which of the following medications is the best choice for this
patient?

A. Metformin
B. Insulin
C. Glibenclamide
D. Pioglitazone
E. Acarbose

Explanation:
Insulin is the medication of choice for the treatment of gestational
diabetes mellitus (GDM). Most patients with GDM diagnosed in the third
trimester can maintain hour postprandial blood glucose levels of less
than 130 mg/dL via diet and activity modifications alone. However, if
life stile changes fail to adequately control blood glucose levels,
insulin therapy should be initiated.
Oral hypoglycemic medications are generally avoided in GDM because of
the risk of fetal hyperinsulinemia and hypoglycemia. However, there has
been recent interest in using glyburide for GDM, because its placental
transfer is minimal and fetal hypoglycemia is less of a risk. Still,
insulin remains the drug of choice for blood glucose control in GDM.

Educational Objective:
Insulin is the medication of choice for the treatment of gestational
diabetes in patients for whom diet and light exercise have failed to
control blood glucose levels.

166
USMLE WORLD STEP 1 PHARMACOLOGY

Q NO 125: A 41-year-old male alcoholic comes to the hospital complaining of fever


weight loss and cough productive of foul smelling sputum. Physical examination
shows poor dentition. His chest CT scan is shown below.

Which of the following antibiotics would be most appropriate here?

A. Clindamycin
B. Vancomycin
C. Ciprofloxacin
D. Metronidazole
E. Cefazolin

167
USMLE WORLD STEP 1 PHARMACOLOGY
Explanation:
Streptococcus pneumoniae is the most common cause of community acquired
pneumonia in the general patient population. However, alcoholics are at
significantly increased risk for aspiration pneumonia (infection with
oral flora), because alcohol intoxication impairs the gag and cough
reflexes. Alcoholics with poor oral hygiene have increased numbers of
oral bacteria further increasing this risk. In addition, alcohol may
impair the phagocytic and/or bactericidal action of alveolar
macrophages, predisposing to infection.
Pulmonary infections in alcoholic patients thus yew commonly include
anaerobic oral flora (Bacteroides, Prevotella, Fusobacterium,
Peptostreptococcus) admixed with aerobic bacteria. Necrotizing
infections and lung abscesses may result. The CT scan above shows an
abscess. Of the antibiotic choices given, clindamycin has the most
activity against oral anaerobes and covers Gram positive organisms like
Streptococcus pneumoniae as well.
(Choices B, C, and E) Vancomycin, ciprofloxacin and cefazolin do not
cover anaerobes.
(Choice D) Metronidazole does not cover Gram positive organisms.

Educational Objective:
Alcoholics are more likely than the general public to develop pulmonary
infections and abscesses involving combinations of anaerobic oral flora
(Bacteroides, Prevotella, Fusobacterium, and Peptostreptococcus) and
aerobic bacteria. Clindamycin covers most of these organisms and is
thus the most appropriate antibiotic choice.

168
USMLE WORLD STEP 1 PHARMACOLOGY

Q NO 126: A 65-year-old male has been diagnosed with hypertension recently. You
consider an ACE inhibitor monotherapy in this patient. You study his extensive past
medical records and found that he has been taking different medications for his
multiple comorbidities. Inquiry about which of the following drugs is most important
before initiating the treatment in this patient?

A. Beta-blockers
B. Cephalosporin antibiotics
C. Vitamins
D. Benzodiazepines
E. Thiazide diuretics
F. Sulfonylurea anti diabetic agents

Explanation:
First-dose hypotension is known to be a potential limiting factor when
initiating ACE inhibitors. Although the exact etiology is unclear itis
believed that ACE inhibitors reduce venous return to the heart
secondary to a decrease in angiotensin II levels and also activate the
Bezold-Jarisch reflex causing vagally mediated hypotension and
bradycardia.
For most patients, the fall in blood pressure is relatively mild and
patients are usually asymptomatic. However, for certain patients with
predisposing risk factors, the effects can be more significant leading
to lightheadedness, dizziness, headache, general weakness, and visual
disturbances. Predisposing risk factors associated with first dose
hypotension include hyponatremia, hypovolemia secondary to diuretic
therapy, low baseline blood pressure, high renin or aldosterone levels,
renal impairment, and heart failure. Although more rare, there is also
a concern that profound reductions in blood pressure may precipitate
cardiac, renal, and cerebral hypoperfusion and ischemia.
Patients taking thiazide or loop diuretics can be hypovolemic and
hyponatremic, thus potentiating the effects of first dose hypotension.
To help prevent and minimize the risk of adverse effects, it is
generally recommended to initiate ACE inhibitor therapy at low dosages
and to slowly titrate to maximum tolerated doses. It is also
recommended to closely monitor blood pressure after ACE inhibitor
initiation and after any dosage adjustments.
(Choice A) The combination of an ACE inhibitor and a beta blocker is
often used in certain medical indications (congestive heart failure or
coronary artery disease). In these instances, the combination is
appropriate in order to optimize therapy and improve patient outcomes.
However, if the combination is used primarily for blood pressure
reduction it is believed that there may be little or no additive
effect. Part of the blood pressure lowering effects of beta blockers
involves their ability to suppress renin release and activity. By
suppressing renin, beta blockers also decrease angiotensin II. With
reduced angiotensin II levels, ACE inhibitors will not be as effective
and thus there may be little or no additive effective on blood pressure
reduction. (Choices B, C, D, and F) No significant drug interactions or
complications occur if these agents are used in combination with an ACE
inhibitor.

169
USMLE WORLD STEP 1 PHARMACOLOGY
Educational Objective:
First-dose hypotension is an adverse effect and concern when initiating
ACE inhibitors. Predisposing risk factors of first-dose hypotension
include hyponatremia, hypovolemia secondary to diuretics, low baseline
blood pressure, high renin or aldosterone levels, renal impairment, and
heart failure. To minimize risk for first-dose hypotension, identify
patients at risk and initiate therapy at low dosages and follow with
cautious dosage titration and blood pressure monitoring.

170
USMLE WORLD STEP 1 PHARMACOLOGY

Q NO 127: A 68-year-old male is admitted to the hospital for heart palpitations.


ERG shows an irregularly irregular rhythm and absent P waves. The patient is
started on warfarin for long-term anticoagulation to prevention atrial thrombus
formation. Which of the following should be monitored in this patient?

A. Bleeding time
B. Prothrombin time (PT)
C. Activated partial thromboplastin time (aPTT)
D. Fibrinogen levels
E. Fibrin split products

Explanation:
The patient described in the vignette is experiencing atrial
fibrillation as indicated by the ERG. Atrial fibrillation alters the
flow dynamics of the heart, making thrombus formation much more common.
If this thrombus somehow detaches from the wall of the atrium the clot
is likely to cause a devastating stroke. To prevent/dissolve such a
thrombus the oral anticoagulant warfarin is indicated. Warfarin is also
the long-term anticoagulant of choice for patients with deep venous
thrombosis and pulmonary thromboembolism. Warfarin, also called
coumadin inhibits the activation of vitamin K-dependent clotting
factors II, VII, IX, and X.
Decreased levels of vitamin K-dependent clotting factors especially
factor VII, prolong the prothrombin time (PT). Another useful lab value
is the ratio of the patient’s PT to a control also called the
international normalized ratio (INR). Usually the target INR for atrial
fibrillation is 2.0 - 3.0.
The most common side effect of warfarin is excessive anticoagulation
leading to hemorrhage. Patients presenting with hemorrhage who are on
warfarin therapy are treated with vitamin K. If rapid reversal of
warfarin is necessary, fresh frozen plasma can be used. In patients
with protein C deficiency warfarin may cause skin necrosis. Warfarin
should not be used during pregnancy due to its teratogenicity (possible
fetal hemorrhage jaundice, and optic neuritis). Warfarin should be used
cautiously with other medications as it interacts with numerous drugs.
(Choice A) Bleeding time is used for the assessment of platelet
function. Increased bleeding time is seen in patients with
thrombocytopenia, von Willebrand disease defects of platelet
aggregation DIC, and during treatment with ASA.
(Choice C) Activated partial thromboplastin time (aPTT) is used to
monitor unfractionated heparin.
(Choices D and E)The combination of an increased fibrinogen level and
the presence of fibrin split products (such as D-dimer) is
characteristic of DIC.

Educational Objective:
1. Warfarin inhibits the carboxylation of vitamin K-dependent
coagulation factors II, VII, IX. and X. Prothrombin time should be
monitored regularly during treatment with this medication. The most
common adverse effect is bleeding.
2. Activated partial thromboplastin time (aPTT) is used for monitoring
unfractionated heparin.

171
USMLE WORLD STEP 1 PHARMACOLOGY

Q NO 128: A new drug has been developed that inhibits one of the steps in fatty
acid oxidation. Itis considered promising for the treatment of stable angina.
Which of the following is the most likely mechanism explaining the potentially
beneficial effect of the drug in patients with this condition?

A. Increased protein synthesis


B. Less ATP use per one contraction cycle
C. Less oxygen use per one ATP synthesized
D. More oxygen extraction per minute
E. Increased oxidative/non-oxidative ATP synthesis ratio

Explanation:
Energy for myocardial cellular function is produced from three major
sources: glycolysis, glucose oxidation and fatty acid oxidation.
Although fatty acid oxidation results in greater ATP production, it
requires more oxygen use in comparison to glucose utilization and
glycolysis. Despite its oxygen requirements fatty acid oxidation is the
main source of energy production (60%), while glucose oxidation (30%)
and glycolysis (5%) produce the remainder of energy production.
In stable angina, atherosclerosis leads to reduced oxygen delivery to
cardiac mitochondria resulting in a mismatch between myocardial oxygen
demand and supply. Since glucose oxidation requires less oxygen, it is
believed that shifting energy production from fatty acid oxidation to
glucose oxidation may be more oxygen efficient and beneficial in
treating angina. Newer agents like fatty acid oxidation inhibitors are
currently under investigation as potential treatment options for
treating chronic stable angina.
(Choice A) Increased protein synthesis would have no beneficial effect
on oxygen demand and supply or in stable angina.
(Choice B) Although less ATP use per one contraction cycle would be
energy efficient, the primary danger in stable angina is related to
inadequate coronary blood flow leading to oxygen deficiency. The
cardiac muscle relies heavily on its oxygen supply. Without adequate
oxygen the cardiac muscle can not even generate sufficient ATP.
(Choice D) Increasing oxygen extraction per minute will be of little
benefit. Unlike other tissues or organs the heart already has a high
oxygen extraction rate. Under resting conditions the heart removes and
uses approximately 75% of oxygen available, which is much greater than
other tissues. Thus there is little oxygen reserve in the coronary
blood supply to further supplement the heart when demands increase.
(Choice E) Increasing the oxidative/non-oxidative ATP synthesis ratio
will only increase oxygen requirements to cause more significant oxygen
supply and oxygen demand mismatch. This will worsen angina symptoms.

Educational Objective:
Fatty oxidation inhibitors are newer agents that inhibit fatty acid
oxidation and shift energy production to glucose oxidation thus
promoting oxygen efficiency. This not only decreases the amount of
oxygen needed to support cardiac function but also decreases
potentially toxic fatty acid metabolite production.

172
USMLE WORLD STEP 1 PHARMACOLOGY

Q NO 129: A 67-year-old male is treated for congestive heart failure. A multidrug


treatment regimen results in marked improvement of the patient’s symptoms. An
agent is added to this patient’s regimen that is believed to benefit the overall
survival. Which of the following is the drug used in this patient?

A. Hydrochlorothiazide
B. Furosemide
C. Spironolactone
D. Triamterene
E. Mannitol
F. Acetazolamide

Explanation:
Spironolactone is an aldosterone antagonist with mild diuretic effects.
Based on results from the RALES trial addition of low dose
spironolactone to standard therapy (ACEIs, digoxin, a diuretic),
significantly reduced morbidity and mortality in class III and IV heart
failure patients.
In congestive heart failure patients, activation of the renin-
angiotensin-aldosterone system leads to elevated aldosterone levels.
Aldosterone is known to cause ventricular remodeling leading to cardiac
fibrosis. The benefits of spironolactone in heart failure patients are
more than likely secondary to inhibition of the neurohormonal effects
of aldosterone, particularly on the heart. Because low doses were used,
it is argued that the diuretic effect contributed little to the
benefits of aldosterone.
Recently, the EPHESUS trial reinforced the benefits of aldosterone
antagonists in the setting of left ventricular dysfunction. The EPHESUS
trial evaluated the benefits on eplerenone, a more selective
aldosterone receptor antagonist.
(Choice A) Hydrochlorothiazide is a thiazide diuretic that works by
blocking Na-Cl symporters in distal convoluted tubules resulting in
enhanced Na, Cl, and water excretion. Since only a small amount of
filtered Na reaches distal tubules, thiazides are not as efficacious
diuretics as loop diuretics and are seldom used in treating congestive
heart failure. However, in diuretic resistant patients, thiazides and
thiazide-like diuretics are often used in combination with loop
diuretics for a synergistic effect.
(Choice B) Furosemide is a loop diuretic that works by inhibiting Na-K-
2C1 symporters in the ascending limb of the loop of Henle, effectively
block Na and Cl transport resulting in increased Na, Cl, and fluid
excretion. They are the most potent class of diuretics and are used for
treating pulmonary congestion and fluid retention in heart failure
patients. Although loop diuretics have been shown to improve symptoms
significantly, they have not been shown to improve survival in heart
failure patients.
(Choice D) Triamterene is a potassium sparing diuretic that works by
blocking Na channels in the distal tubule and collecting duct. This
results in increased Na and fluid excretion. Triamterene is a very mild
diuretic and is not commonly used in treating pulmonary congestion or
fluid retention in heart failure patients.
(Choice E) Mannitol is an osmotic diuretic that woks by increasing
plasma and tubular osmolality resulting in increased Na and fluid
extraction. Mannitol is not commonly used in treating pulmonary
congestion or fluid retention in heart failure patients.

173
USMLE WORLD STEP 1 PHARMACOLOGY
(Choice F) Acetazolamide is a diuretic that works by inhibiting the
enzyme carbonic anhydrase. Acetazolamide is not commonly used in
treating pulmonary congestion or fluid retention in heart failure
patients.

Educational Objective:
Spironolactone is an aldosterone antagonist with mild diuretic effects.
Based on results from the RALES trial addition of low dose
spironolactone to standard therapy significantly reduced morbidity and
mortality in class Ill and IV heart failure patients. The benefits of
spironolactone in heart failure patients are more than likely secondary
to inhibition of the neurohormonal effects of aldosterone leading to
decreased ventricular remodeling and cardiac fibrosis.

174
USMLE WORLD STEP 1 PHARMACOLOGY

Q NO 130: A 24-year-old male is brought to the FR with altered mental status. His
past medical history is significant for epilepsy. The patient seems severely sedated
and is not oriented to person, time, and place. His blood pressure is 100/70 mmHg
and heart rate is 80/mm. Laboratory tests show elevated blood phenobarbital
levels. This patient most likely takes which of the following agents?

A. Phenytoin
B. Gabapentin
C. Topiramate
D. Lamotrigine
E. Primidone

Explanation:
Primidone is an antiepileptic that is metabolized to phenobarbital and
phenylethylmalonamide (PEMA), also active anticonvulsants. Primidone
has its own antiseizure effects but less often causes lethargy.
The other drugs do not cause elevated phenobarbital level.

Educational Objective:
Primidone is metabolized to phenobarbital and phenylethylmalonamide
(PEMA). All three compounds are active anti convulsants.

175
USMLE WORLD STEP 1 PHARMACOLOGY

Q NO 131: A 35-year-old male presents to your office with two week history of
rhinorrhea. He uses a topical decongestant that provides only partial relief. He
had episodes of rhinorrhea in the past but none of them lasted more than a
few days. He denies fever, throat pain, headaches, cough and lymph node
enlargement. Which of the following is the best next in the management of this
patient?
A. Stop the decongestant
B. Switch to ephedrine
C. Add topical corticosteroids
D. Add antihistamines
E. Start antibiotics

Explanation:
Vasoconstriction associated with a-adrenergic agonists is prominent in
the vessels of nasal mucosa. This effect makes these medications
effective decongestants. Phenylephrine, xylometazoline and
oxymetazoline are used as topical preparations for treatment of
allergic rhinitis and common cold.
These medications, however, are characterized by the rapidly declining
effect after a few days of use. This phenomenon is called
tachyphylaxis. It occurs due to the depletion of norepinephrine from
the nerve terminals and leads to exacerbation of symptoms of rhinitis.
Rebound rhinorrhea in the patient described in this clinical vignette
is associated with use of topical decongestant and is likely to be
caused by tachyphylaxis. The use of adrenergic agonist should be
stopped to allow the restoration of norepinephrine stores at the nerve
terminals.
Another medication associated with phenomenon of tachyphylaxis is
nitroglycerine. Decrease in its effect is explained by diminished
release of NO from the target cells. An 8-10-hour drug-free intervals
should be maintained during the use of nitroglycerine to prevent
tachyphylaxis.
(Choice B) Ephedrine is an indirect-acting adrenergic agonist used as a
decongestant. Like the other adrenergic medications it causes
tachyphylaxis after a few days of treatment. Switch to ephedrine would
not produce any effect in rebound rhinorrhea.
(Choices C and D)Topical corticosteroids and antihistamines are used
for treatment of allergic rhinitis. They are not effective in rebound
rhinorrhea due to tachyphylaxis.
(Choice E) Antibiotics are effective in treatment of bacterial
infections. They are not used in treatment of rebound rhinorrhea.

Educational Objective:
Topical preparations of a-adrenergic agonists cause vasoconstriction of
the vessels of nasal mucosa and are used as decongestants. Overuse of
these drugs depletes norepinephrine stores as the nerve endings and
diminishes their effect (tachyphylaxis).

176
USMLE WORLD STEP 1 PHARMACOLOGY

Q NO 132: A 64-year-old patient begins aspirin therapy for exertional chest pain.
After the first dose, however, he experiences shortness of breath and wheezing.
Which of the following medications would serve as a suitable aspirin alternative in
this patient?

A. Heparin
B. Warfarin
C. Clopidogrel
D. Streptokinase
E. Vitamin
F. Allopurinol
G. Paclitaxel

Explanation:
This patient’s chest pain is consistent with angina pectoris; thus,
initiation of aspirin therapy was appropriate. Aspirin is a nonspecific
irreversible cyclooxygenase inhibitor that helps to prevent platelet
thrombus formation by inhibiting thromboxane A2 synthesis and platelet
aggregation. In patients with an aspirin allergy, aspirin can cause
bronchoconstriction with shortness of breath and wheezing. In such
patients, the best alternative antiplatelet agent is clopidogrel.
Clopidogrel irreversibly blocks the platelet surface ADP receptors
essential for platelet activation. aggregation and fibrin binding.
Clopidogrel and aspirin are equally efficacious in the prevention of
thromboembolic disease, and have a synergistic effect when used
together because of their unique mechanisms of action.
(Choice A) Heparin functions as an anticoagulant by binding and
activating antithrombin Ill. It has been shown to reduce morbidity and
mortality in acute coronary syndrome patients, but it is not an
antiplatelet agent. Furthermore, outpatient use of heparin is
impractical because it must be parenterally administered.
(Choice B) Warfarin is not indicated for the management of angina
pectoris.
(Choice D) Streptokinase is a thrombolytic agent used in the acute
management of thromboembolic events like cerebrovascular accidents and
myocardial infarctions where emergent restoration of blood flow is
essential. (Choice E) Vitamin K can be used to reverse the effects of
warfarin, but has no anticoagulant or antiplatelet effects.
(Choice F) Allopurinol is a xanthine oxidase inhibitor used in the
treatment of tumor lysis syndrome and gout.
(Choice G) Paclitaxel is a cytotoxic chemotherapeutic agent that binds
3-tubulin and enhances microtubule polymerization, producing a
disorganized array of microtubules within the cell during metaphase and
ultimately cell death.

Educational Objective:
Clopidogrel is an antiplatelet agent that works by inhibiting the
platelet surface ADP receptor. It is as efficacious as aspirin in the
prevention of thromboembolic disease.

177
USMLE WORLD STEP 1 PHARMACOLOGY

Q NO 133: Milrinone is a strong inotropic agent. Its mechanism of action involves


inhibition of phosphodiesterase isoenzyme 3. Which of the following additional
responses is most likely an extension of its pharmacologic effect?

A. Angioedema
B. Antiarrhythmic action
C. Vasodilation
D. Increased right atrial pressure
E. Sodium and water retention
F. AV conduction block

Explanation:
Milrinone, as stated in the question stem, is a phosphodiesterase
inhibitor. In the cardiac myocyte phosphodiesterases act to metabolize
cyclic AMP (cAMP), an important second messenger in excitation-
contraction coupling, cAMP is formed from ATP by adenylyl cyclase, the
activity of which is stimulated by beta agonists. Cyclic AMP increases
the conductance of the calcium channels in the sarcoplasmic reticulum,
and as a result, more calcium can enter the cell and strengthen the
force of contraction. Thus, the inhibition of phosphodiesterases by
milrinone can increase cardiac contractility.
In vascular smooth muscle, increases in cAMP cause vasodilation, a
well-known side effect of phosphodiesterase inhibitors which can
occasionally limit their use in hypotensive patients.
(Choice A)Angioedema is most frequently manifest as rapid-onsets
welling of the lips and larynx. Itis a serious, though rare, side
effect of ACE-inhibitors.
(Choice B) Antiarrhythmic medications are broken into 6 classes based
on their mechanism of action (class IA, IB, and IC drugs modulate
sodium channels; class II drugs act on the sympathetic nervous system;
class Ill drugs modulate potassium channels; class IV drugs are calcium
channel blockers). Phosphodiesterase inhibitors do not have anti
arrhythmic properties.
(Choice D) A strong inotropic agent, which milrinone was identified as
in the question stem, would not cause increased right atrial pressure.
(Choice E) Sodium and water retention would not be a desired effect in
a patient with congestive heart failure and is not an effect of the
phosphodiesterase inhibitors.
(Choice F) AV Conduction block can be caused by digitalis toxicity, but
it is not an effect of the phosphodiesterase inhibitors.

Educational Objective:
Phosphodiesterase inhibitors lead to increased cardiac contractility
via increased intracellular cAMP concentration. cAMP promotes increased
intracellular calcium in cardiac myocytes and is normally metabolized
by phosphodiesterases. In vascular smooth muscle, increases in cAMP
cause vasodilation, a well-known side effect of phosphodiesterase
inhibitors which can occasionally limit their use in hypotensive
patients.

178
USMLE WORLD STEP 1 PHARMACOLOGY

Q NO 134: A 60-year-old Caucasian male with hypertension treated with maximum


doses of hydrochlorothiazide and ramipril presents to your office with a persistently
elevated blood pressure of approximately 160/100 mmHg and a heart rate of 56
beats/mm. You consider adding a third antihypertensive drug to his treatment
regimen. But you are concerned about his prolonged AV conduction time (PR
interval of 0.24 sec) on ERG. Which of the following drugs is most appropriate in
this patient?

A. Verapamil
B. Metoprolol
C. Diltiazem
D. Nifedipine
E. Bepridil

Explanation:
The treatment of hypertension is one of the most common reasons for
physician office visits and use of prescription drugs in the United
States. There are approximately 58 to 65 million hypertensive patients
in the adult population in the United States. The number of patients
with hypertension is likely to grow as the population ages, as both
pure systolic hypertension and combined systolic/diastolic hypertension
occur in over one-half of all people over age 65.
Therapy for hypertension can often be difficult with many patients
requiring 2 or more antihypertensive medications. The rigorous
treatment of hypertension is important as it is the most common risk
factor for heart disease, ahead of diabetes, smoking dyslipidemia, and
male gender or postmenopausal state.
Normal blood pressure (BP) is defined as systolic BP <120 mmHg and
diastolic BP <80. Prehypertension is defined as a systolic BP of 120-
139 or a diastolic BP of 80-89, and hypertension is defined as a
systolic BP of 140 or greater or a diastolic BP of 90 or greater.
Nifedipine is the correct choice in this case as it is effective as an
anti hypertensive agent and has a minimal effect on AV conduction
unlike each of the other drugs listed.
(Choices A and C) Verapamil and diltiazem are often used as therapies
for rate control in atrial fibrillation with rapid ventricular response
due to their ability to slow conduction through the atrioventricular
(AV) node. Because of this property, they would both be poor choices in
this patient with documented AV nodal delay (criteria = PR interval of
0.2 seconds or greater).
(Choice B) Metoprolol is a cardioselective beta-adrenergic blocking
agent. As such, it has a greater affinity for the beta-i receptors
which mediate AV nodal conduction and a lesser affinity for the beta-2
receptors which mediate bronchodilation and peripheral vascular
dilatation. Metoprolol would slow AV nodal conduction in this patient
leading to an increase in PR interval and decrease in heart rate.
(Choice E) Bepridil is a calcium channel blocker which is used as a
second-line anti-anginal drug. It is not indicated for use as an anti
hypertensive agent.
Educational Objective:
Knowledge of the mechanisms of action of hypertensive agents as well as
their effects on areas other than the vasculature, such as the heart in
this case, is essential for clinical practice. Nifedipine is selective
for the vasculature while the other agents all have effects on the
heart.

179
USMLE WORLD STEP 1 PHARMACOLOGY

Q NO 135: A previously healthy 45-year-old male undergoes an elective hernia


repair under spinal anesthesia. Postoperatively, he complains of difficulty
voiding. Bladder catheterization shows a post-void residual of 300cc of urine.
This patient would most likely benefit from which of the following medications?

A. Finasteride
B. Phenylephrine
C. Bethanechol
D. Oxybutynin
E. Imipramine

Explanation:
This patient most likely has postoperative urinary retention, which
occurs in up to 25% of patients after lower abdominal surgery. A normal
post-void residual urine volume in the bladder is less than 50cc.
Anesthesia and analgesia contribute to all of the following: over-
distention of the bladder, a decreased micturition reflex, decreased
contractility of the bladder detrusor muscle, and incomplete empting.
Contraction of the detrusor muscle is stimulated by muscarinic
cholinergic agonists. Bethanechol, a muscarinic agonist, often improves
bladder-emptying in patients with post-surgery urinary retention.
(Choice A) Finasteride is prescribed for patients with bladder outlet
obstruction secondary to prostatic hypertrophy. Finasteride is a 5a-
reductase inhibitor that decreases the local conversion of testosterone
to dihydrotestosterone in the prostate thereby promoting shrinkage of
the gland over6to 12 months. Since this patient’s incomplete bladder
emptying is due to weak detrusor muscle contraction rather than
prostatic hypertrophy finasteride would not be likely to improve his
acute condition.
(Choice B) Phenylephrine is an alpha-agonist with some selectivity for
α1 receptors. The occupation of al receptors in the bladder actually
encourages the trigone and sphincter to contract. Phenylephrine
promotes rather than alleviates urinary retention.
(Choice D) Oxybutynin is an antimuscarinic agent commonly used for urge
incontinence. If given. It will worsen this patient’s condition.
(Choice E) Imipramine has both alpha agonist and anticholinergic
activity, and would thus worsen this patient’s condition by two
mechanisms!

Educational Objective:
Postoperative urinary retention with incomplete bladder emptying, is a
common complication thought to involve decreased micturition reflex
activity, decreased contractility of the bladder detrusor, and/or
increased vesical sphincter tone. This condition may be treated with a
muscarinic agonist (bethanechol) or an al blocking drug.

180
USMLE WORLD STEP 1 PHARMACOLOGY

Q NO 136: A 50-year-old Caucasian male is brought to the ER with severe dizziness


and confusion. He states that he had an episode of chest pain and took several
tablets of nitroglycerin. His current medications include a daily aspirin for heart
attack prevention, an occasional acetaminophen for headaches and occasionally
tadalafil for erectile dysfunction. His blood pressure is 50/20 mmHg and his heart
rate is 120 beats/min. Which of the following cellular changes is most likely
responsible for this patient’s symptoms?

A. Receptor downregulation
B. G5 protein phosphorylation
C. Cyclic GMP accumulation
D. Tyrosine kinase overactivity
E. Enhanced phospholipid metabolism
F. Tolerance development

Explanation:
The interaction between nitrates and phosphodiesterase inhibitors used
in erectile dysfunction (tadalafil, sildenafil, vardenafil) is well
known and very important to understand. Nitrates are converted to
nitric oxide by vascular smooth muscle cells, and nitric oxide causes
increased intracellular cGMP as a second-messenger. Increased cGMP
concentration leads to vascular smooth muscle relaxation. Additionally,
cGMP is metabolized within the cells by phosphodiesterase, and
phosphodiesterase inhibitors will lead to increased intracellular cGMP.
cGMP accumulation in vascular smooth muscle cells due to both enhanced
synthesis (nitrates) and inhibited degradation (PDE inhibitors) is
responsible for profound hypotension due to extreme vasodilatation when
these drugs are used together.
(Choice A) Receptor downregulation or tachyphylaxis occurs with many
drug classes including topical glucocorticoids, opiates, L-dopa and
more. Itis one mechanism by which the body develops tolerance to drugs
on a cellular level but it is not the cause of hypotension in this
patient who took nitrates together with a PDE inhibitor.
(Choice B) G protein phosphorylation leads to activation of the S5
protein, a stimulatory protein which acts as a part of the
calcium/calmodulin, inositol triphosphate and adenylate cyclase second
messenger systems. Phosphorylation of would stimulate calcium release,
phospholipase C activity and adenylate cyclase activity in these three
systems, respectively, but it has no role in cGMP pathways that are
affected by nitrates and PDE inhibitors.
(Choice D) Tyrosine kinase overactivity would lead to an exaggeration
of the effects of insulin or insulin-like growth factor. Tyrosine
kinase has no role in the effect of nitrates or PDE inhibitors on
vascular smooth muscle. (Choice E) Enhanced phospholipid metabolism
would lead to an increase in the production of prostaglandins (via
diacylglycerol) and inositol triphosphate (leading to calcium release
from the endoplasmic reticulum). This is the inositol triphosphate
second-messenger system, and it mediates the effects of alpha-i
adrenergic receptors, angiotensin II receptors, oxytocin receptors and
vascular vasopressin receptors. It is not involved in the action of
nitrates or PDE inhibitors.
(Choice F) Tolerance refers to the need for a larger dose of drug to
achieve the same clinical effect achieved previously with a smaller
dose of drug. Development of tolerance can occur by multiple mechanisms
such as a change in cellular response to drug (decreased receptors,

181
USMLE WORLD STEP 1 PHARMACOLOGY
decreased response to receptor stimulation), as well as increased rate
of drug elimination by the body. Tolerance is not the reason for this
patient to experience hypotension with this drug combination.

Educational Objective:
Using nitrates together with phosphodiesterase (PDE) inhibitors used
for erectile dysfunction and pulmonary hypertension causes a profound
systemic hypotension because they both increase intracellular cGMP
which causes vascular smooth muscle relaxation. Their use together is
absolutely contraindicated.

182
USMLE WORLD STEP 1 PHARMACOLOGY

Q NO 137: A 15-year-old Caucasian male presents to your office with periodic,


sudden-onset jerking movements. These muscle contractions usually happen
early in the morning, soon after awakening, and are aggravated by sleep
deprivation. He has never lost consciousness. His family history reveals a
seizure disorder in his uncle. Which of the following is the best initial treatment
for this patient?
A. Baclofen
B. Haloperidol
C. Diazepam
D. Clonidine
E. Carbamazepine
F. Propranolol
G. Phenobarbital
H. Valproic acid

Explanation:
This clinical vignette describes a typical presentation of myoclonic
seizures. Myoclonic syndromes are characterized by repetitive seizures
consisting of brief, often symmetric, muscular contractions with loss
of body tone causing the patient to fall or slump forward. These
patients have a propensity for injury to the face and for mouth.
Seizures usually occur in the morning and are precipitated by stress
and sleep deprivation. Valproic acid is a first-line drug for treatment
of myoclonic seizures. The following table summarizes the appropriate
medications for different types of seizures:

183
USMLE WORLD STEP 1 PHARMACOLOGY
Educational Objective:
Sodium valproate (valproic acid) is the drug of choice for myoclonic
seizures. This drug suppresses abnormal electric activity in the cortex
by affecting GABA and NMDA receptors, as well as Na+ and K+ channels.

184
USMLE WORLD STEP 1 PHARMACOLOGY

Q NO 138: A 5-year-old boy receiving treatment for allergic rhinitis has flushed
cheeks and dilated pupils on physical exam. These findings are best explained
by:

A. Antagonism of Hi receptors
B. Antagonism of H2 receptors
C. Antagonism of muscarinic receptors
D. Antagonism of nicotinic receptors
E. Stimulation of al-adrenoreceptors

Explanation:
Facial flushing and pupillary dilation are common findings in patients
taking medications with anticholinergic effects. There are two main
forms of cholinergic receptors nicotinic and muscarinic. The toxic
effects of anticholinergic medications result primarily from blockade
of muscarinic receptors. For example inhibition of eccrine sweat gland
secretions can result in fever and compensator cutaneous vasodilation,
and inhibition of the pupillary constrictor and ciliary muscles can
cause pupillary dilation. This child is likely receiving an Hi receptor
(histamine) antagonist such as diphenhydramine for his allergic
rhinitis, a class of drugs well-known for its anticholinergic effects.
(Choice A) Though this patient is being treated with an Hi receptor
antagonist, action at this receptor cannot account for the toxic
effects observed. Hi receptors are found in the vascular endothelium
and bronchial smooth muscle where they help to mediate vascular
permeability and bronchoconstriction, respectively.
(Choice B) H2 receptor antagonists are commonly used to block gastric
acid secretion by parietal cells. They do not cause cutaneous
vasodilation or pupillary dilation.
(Choice D) Nicotinic cholinergic receptors are found on postganglionic
neurons in sympathetic and parasympathetic ganglia and on skeletal
muscle cells at the neuromuscular junction. Drugs that block skeletal
muscle nicotinic receptors, such as tubocurarine, are often used during
general anesthesia to induce paralysis. Adverse effects associated with
these agents include respiratory paralysis and autonomic ganglionic
blockade that can cause hypotension and tachycardia.
(Choice E) al-adrenergic agonists are sometimes given to patients with
allergic rhinitis because their vasoconstrictive action helps to
alleviate nasal congestion. These agents cause mydriasis if
administered intraocular or in large systemic doses, but would not be
expected to cause flushing.

Educational Objective:
Flushed skin and mydriasis result from muscarinic receptor blockade.
Itis important that students be familiar with the following medications
with antimuscarinic effects: atropine, tricyclic antidepressants (e.g.
amitriptyline), Hi receptor antagonists (e.g. diphenhydramine),
atypical antipsychotics, and anti parkinsonian drugs.

185
USMLE WORLD STEP 1 PHARMACOLOGY

Q NO 139: A new drug seems to have partial agonist/antagonist activity against


receptor X. When the drug is applied to cells expressing receptor X, there is an
immediate change in transmembrane sodium and potassium flow thought
secondary to the opening of receptor-coupled transmembrane ion channels.
Receptor X is most likely which type of receptor?

A. al adrenoreceptor
B. β1 adrenoreceptor
C. β2 adrenoreceptor
D. Muscarinic cholinergic receptor
E. Nicotinic cholinergic receptor

Explanation:
The sympathetic (adrenergic) and parasympathetic (cholinergic)
autonomic nervous systems work by releasing neuro transmitters near
corresponding receptors on target organs. Upon binding its
corresponding neurotransmitter, each type ot receptor activates a
signal transduction pathway (e.g. signal transduction cascade, ion
channel) responsible for the receptor’s actions. The autonomic nervous
system uses the following signal pathways:

Nicotinic cholinergic receptors of the neuro muscular junction are an


example of ligand-gated ion channel receptors. They are located on the
postsynaptic membrane and open in response to acetylcholine and
nicotine binding. Open ion channels allow the influx of cations (Na, K
and Ca) into cells and cause membrane depolarization. An action
potential is produced after a sufficient number of channels have
opened, resulting in muscle contraction. Nicotinic cholinergic
receptors are also found in the CNS and on postganglionic neurons of
autonomic gangli a. Activation of receptor X promotes ion influx and,
thus is most likely a nicotinic receptor.
(Choice A) al adrenoceptors are found in skin and visceral blood vessel
walls. Binding of norepinephrine to these receptors results in
activation of the inositol-lipid (1P3) pathway.
(Choices B and C) β1 and β2 adrenoceptors are associated with the cAMP
signal transduction pathway. Binding of norepinephrine to these
receptors causes protein kinase A activation.
(Choice D) Muscarinic cholinoreceptors are found in secretory glands
and smooth muscle cells. Like al agonists, they utilize the inositol-
lipid signal pathway to increase intracellular Ca concentration and
activate protein kinase C.

186
USMLE WORLD STEP 1 PHARMACOLOGY
Educational Objective:
The autonomic nervous system utilizes three types of signal pathways:
cAMP, IRS and ion channels. Nicotinic receptors are ligand-gated ion
channels that open after binding acetylcholine. Influx of Na, K and Ca
into the cell causes depolarization of cell membrane.

187
USMLE WORLD STEP 1 PHARMACOLOGY

Q NO 140: A 55-year-old Caucasian female with recalcitrant rheumatoid arthritis is


being considered for etanercept therapy. Which of the following tests should be
performed before beginning treatment?

A. Liver function tests


B. PPD skin test
C. Pulmonary function tests
D. Visual examination
E. Fecal occult blood test

Explanation:
Etanercept is a tumor necrosis factor alpha (TNF-a) inhibitor commonly
used to treat moderate to severe rheumatoid arthritis particularly in
patients who have failed methotrexate therapy. The TNF-a inhibitors can
cause serious adverse effects, including reactivation of latent
tuberculosis. All patients being considered for TNF-a inhibitor therapy
should have a baseline PPD skin test to screen for latent tuberculosis.
Patients taking TNF-a inhibitors also have increased susceptibility to
other infectious agents, including fungi and atypical mycobacteria.
TNF-a blockers should not be used in any patient with an underlying
infection.
(Choice A) Patients should have baseline liver function tests before
beginning treatment with methotrexate or leflunomide because
hepatotoxicity is an important adverse effect of these agents.
(Choice C) Pulmonary function tests maybe important for patients taking
medications known to cause pulmonary fibrosis, such as amiodarone.
(Choice D) Irreversible retinal damage can occur with long-term use of
hydroxychloroquine. Patients starting hydroxychloroquine should have
baseline and follow-up ophthalmologic examinations.
(Choice E) Patients taking NSAIDs are at increased risk for
gastrointestinal blood loss. Fecal occult blood testing is not
routinely performed unless there is a clinical suspicion of
gastrointestinal bleeding.

Educational Objective:
All patients beginning treatment with TNF-a inhibitors should be
evaluated for latent tuberculosis.

188
USMLE WORLD STEP 1 PHARMACOLOGY

Q NO 141: A 45-year-old female presents with swelling and pain of multiple joints.
The symptoms started 6-7 months ago and gradually progressed overtime. She
notes prolonged morning stiffness and significant restriction of her daily activities.
Which of the following drugs would provide the most rapid and complete relief of
her symptoms?

A. Naproxen
B. Methotrexate
C. Sulfasalazine
D. Hydroxychloroquine
E. Prednisone
F. Minocycline

Explanation:
Corticosteroids are potent immunosuppressive and anti-inflammatory
agents, capable of providing rapid and complete relief in most patients
with acute joint swelling and pain. Corticosteroids exert their anti-
inflammatory effects by inhibiting phospholipase A2 activity, which
decreases prostaglandin and leukotriene synthesis. Corticosteroids also
depress the immune response by reducing macrophage phagocytosis and
interleukin-1 secretion. In patients with acute symptoms,
glucocorticoids are typically added for a short period to achieve rapid
symptom relief until slower acting disease-modifying agents can take
effect. Nonsteroidal anti-inflammatory medications (NSAID5) such as
naproxen can also be used to treat acute symptoms. NSAIDs have only
anti-inflammatory effects: they do not dampen the general immune
response. NSAIDs offer less relief of pain and swelling compared to
corticosteroids, but cause fewer adverse effects.
(Choice B) Methotrexate is the preferred disease-modifying treatment
for most patients with moderate to severe rheumatoid arthritis.
Methotrexate treatment produces substantial improvements in disease
activity in 60-70°f of patients. However, the response typically takes
weeks to occur.
(Choices C, D and F) Sulfasalazine, hydroxychloroquine and minocycline
are sometimes used to treat mild, early, seronegative rheumatoid
arthritis. Severe disease is unlikely to respond to these medications.
These drugs do not have anti-inflammatory effects, but rather work as
immunomodulators to modify the disease course.

Educational Objective:
Although very effective and capable of producing quick symptom relief
glucocorticoids are usually used only for short term therapy of
rheumatoid arthritis due to their unfavorable side effect profile.

189
USMLE WORLD STEP 1 PHARMACOLOGY

Q NO 142: A new drug has been developed that appears to relax smooth muscle of
arterioles but does not affect veins. It is thought to be a good alternative for
patients with severe hypertension resistant to other drugs. Which of the following is
the most likely adverse effect that the drug would demonstrate in clinical trials?

A. Bradycardia
B. Cold extremities
C. Decreased cardiac output
D. Sodium and fluid retention
E. Persistent cough
F. Angioedema
G. Transient hypertension

Explanation:
Drugs that work by selective arteriolar vasodilation cause significant
vasodilation and reductions in arterial pressure, subsequently
stimulating baroreceptor mediated activation of the sympathetic system.
Reflex sympathetic activation leads to increased heart rate
contractility, and increased renin activity resulting in sodium and
fluid retention.
Thus, although direct arteriolar vasodilators like hydralazine and
minoxidil are effective in lowering blood pressure, they often cause
reflex tachycardia and edema. Due to these bothersome side effects,
these agents are rarely used first line and are usually reserved for
patients with severely uncontrolled hypertension who are resistant to
other drugs. To help mediate these side effects, these agents are often
given in combination with sympatholytics and diuretics.
(Choice A) Direct arteriolar vasodilators cause reflex sympathetic
activation resulting in tachycardia, not bradycardia.
(Choice B) Direct arteriolar vasodilator decrease vascular resistance
and enhance blood flow to organs and tissues, including the skin and
muscle. Thus, these drugs should not cause cold extremities.
(Choice C) Direct arteriolar vasodilators decrease systemic vascular
resistance and afterload, thus typically help to increase cardiac
output. This pharmacological effect is a therapeutic rationale for
hydralazine’s role in treating congestive heart failure.
(Choices E and F) Persistent cough and angioedema are well recognized
potential side effects of ACE inhibitors. They have not been reported
to occur in arteriolar vasodilators like hydralazine or minoxidil.
(Choice G) Although reflex sympathetic activation does occur with
direct arterial vasodilators, the extent of sympathetic activation is
not of magnitude to commonly cause transient hypertension. Sympathetic
activation secondary to these agents mainly results in increased heart
rate and contractility, and sodium and fluid retention.

Educational Objective:
Direct arteriolar vasodilators like hydralazine and minoxidil are
effective antihypertensives. Because they cause significant arterial
vasodilation, they also cause reflex sympathetic activation resulting
in tachycardia and edema. To counteract these compensators effects,
these agents are often given in combination with sympatholytics and
diuretics.

190
USMLE WORLD STEP 1 PHARMACOLOGY

Q NO 143: A 24-year-old female presents to your office with loss of energy,


A.
decreased appetite and insomnia. She lost four pounds over the last month.
Her family history is significant for a mother who suffered from ‘mood
swings’. Soon after treatment initiation the patient is brought to ER with
agitation and grandiose thoughts. She spent a large amount of money
buying a yacht and states that she wants to travel around the globe. Which
of the following agents was most likely used for this patient’s initial
treatment?
Lithium
B. Carbamazepine
C. Olanzapine
D. Valproate
E. Imipramine
F. Lorazepam

Explanation:
Imipramine is a tricyclic antidepressant (TCA). Antidepressants, when
used in the depressive phase of bipolar disorder, are likely to cause a
rapid switch to mania. Other drugs listed are mood stabilizers or
related drugs.
(Choices A and D) Lithium and valproate are the principal agents for
mania. Long-term lithium maintenance requires monitoring of lithium
blood levels and periodic assessment of thyroid (TSH) and renal
function (BUN and creatinine). Lithium toxicity may lead to nephrogenic
diabetes insipidus.
(Choice B) Carbamazepine is also used for bipolar disorder. The most
feared side effect of carbamazepine is agranulocytosis or aplastic
anemia.
(Choice C) Olanzapine is an atypical antipsychotic agent which, along
with clozapine, is mood stabilizing. Antipsychotics are typically used
for the acute manic phase of bipolar disorder and can be used for
chronic therapy as well.
(Choice F) Benzodiazepines are also indicated as adjunctive therapy for
acute mania.

Educational Objective:
Antidepressants, when used in the depressive phase of bipolar disorder
without an antipsychotic or mood stabilizer, can precipitate a mania.

191
USMLE WORLD STEP 1 PHARMACOLOGY

Q NO 144: A 57-year-old Caucasian male with severe pyelonephritis is admitted to


the hospital. His past medical history is significant for diabetes, hypertension and
two episodes of transient ischemic attacks. His serum creatinine level is 3.2
mg/dL; therefore, he needs to be started on an antibiotic that depends mainly on
non-renal clearance. Which of the following characteristics should the antibiotic
also have if hepatic metabolism and clearance is desired?

A. Low volume of distribution


B. Poor oral absorption
C. High lipophilicity
D. Low rate of redistribution
E. Poor penetration into the CNS

Explanation:
The kidney is the prima site of excretion of most drugs with or without
prior chemical modification in the liver. The liver is the major site
of drug biotransformation and metabolism, but some drugs are also
predominately eliminated by the liver into the bile and feces. Drugs
with high intrinsic hepatic clearance tend to have high lipophilicity
and a high volume of distribution. Highly lipophilic drugs tend to be
poorly eliminated in the kidney as these agents rapidly cross tubular
cell membranes after filtration to reenter the tissues.
High lipophilicity (lipid solubility) allows the drug to cross cellular
barriers more easily and enter hepatocytes. It can then be excreted in
the bile or through other methods of elimination. In addition, high
lipid solubility assures a wide distribution to many different tissues
including the brain, liver, and adipose tissue.
(Choice A) Drugs with a low distribution volume tend to be confined to
the bloodstream and tend not to diffuse readily through hepatocytes
into the bile. Thus, a drug with a low Vd will be highly plasma
protein-bound and hydrophilic, making it less available for hepatic
metabolism and more readily available for excretion unchanged in the
urine.
(Choice B) A drug that is not well absorbed orally may have high pre-
systemic (first-pass) elimination or be poorly lipid-soluble. In both
cases it will be unavailable to the liver for hepatic clearance.
(Choice D) A low rate of redistribution from one compartment to another
implies low lipid solubility or significant hydrophobicity. These drugs
tend to remain in the intravascular compartment and be eliminated by
the kidneys.
(Choice E) Poor central nervous system penetration indicates that a
drug is likely not lipophilic, and therefore has a low volume of
distribution. It is therefore also unlikely that the same drug will be
eliminated in large amounts by the liver.

Educational Objective:
While the kidney is the primary site of elimination of most drugs, the
liver is the main site of biotransformation of these agents in
preparation for elimination. Drugs that are more lipophilic (high Vd
good penetration into CNS) are preferentially processed by the liver
into more polar compounds for easier elimination in the bile and urine.
Liver disease (e.g., cirrhosis) or the concomitant use of other drugs
may limit or enhance the clearance of drugs metabolized in the liver.

192
USMLE WORLD STEP 1 PHARMACOLOGY

Q NO 145: A new drug has been developed that seems promising or the treatment
of asthma. It works by reversing irritant-induced bronchoconstriction. In
vagectomized animals, the compound has no bronchodilatory effects. The drug is
most similar to which of the following medications?

A. Albuterol
B. Theophylline
C. Ipratropium
D. Zileuton
E. Flunisolide
F. Nifedipine

Explanation:
When the vagus nerve is stimulated acetycholine is released. In airways
acetycholine produces bronchoconstriction by acting on muscarinic
receptors. Ipratropium blocks the action of acetylcholine at muscarinic
receptors, preventing this effect.
Ipratropium and similar asthma drugs are less effective than beta-2
adrenergic agonists. Their effect starts 60 to 90 minutes after
initiating treatment. Ipratropium enhances the bronchodilatory effects
of beta-2 adrenergic agents. Inhaled ipratropium is poorly absorbed
from the respiratory mucosa and therefore has virtually no systemic
anticholinergic side effects. The new long-acting drug tiotropium is
promising for chronic obstructive pulmonary disease (COPD).
(Choice A) Albuterol is a short-acting selective beta-2 adrenergic
agonist (sympathomimetic). Beta adrenergic receptor agonists are
considered first-line for the treatment of acute bronchial asthma. The
bronchodilatory effects of albuterol are independent of parasympathetic
innervation.
(Choice B) Methylxanthine, like theophylline and aminophylline, cause
bronchial dilatation by decreasing phosphodiesterase activity, thereby
increasing the intracellular concentration of cAMP. Methylxanthine also
block adenosine receptors, augmenting their bronchodilatory effects.
(Choice D) Leukotrienes are synthesized from arachidonic acid via the
lipoxygenase pathway. Zileuton inhibits this pathway thus reducing
leukotriene production. Zileuton is used as a prophylactic therapy for
asthma.
(Choice E) Flunisolide is one of the inhaled glucocorticoids used for
the prophylactic treatment of bronchial asthma. Inhaled glucocorticoids
are the most preferred drugs for the prophylactic treatment of
bronchial asthma. They have potent local anti-inflammatory effects.
(Choice F) Nifedipine causes bronchodilation by blocking calcium influx
into bronchial smooth muscle cells.

Educational Objective:
Antimuscarinic agents (ipratropium) only reverse vagally-mediated
bronchoconstriction. Methylxanthine like theophylline and aminophylline
cause bronchial dilatation by decreasing phosphodiesterase enzyme
activity, thereby increasing intracellular cAMP.

193
USMLE WORLD STEP 1 PHARMACOLOGY

Q NO 146: An 18-year-old male develops fever, confusion and nausea two days
after an uncomplicated laparotomy conducted under general anesthesia. His
past medical history is insignificant. Laboratory tests are significant for elevated
ASTI ALT and bilirubin level. Which of the following is most likely seen if the
liver biopsy is performed?

A. Microvesicular fatty change


B. Macrovesicular fatty change
C. Massive hepatic necrosis
D. Intrahepatic cholestasis
E. Extensive granulomatosis

Explanation:
Halogenated inhalation anesthetics (halothane enflurane, isoflurane and
sevoflurane) can cause severe liver damage. Among the gases listed
halothane is the most extensively metabolized in liver and is the most
common cause of hepatotoxicity. For this reason itis rarely used in
adults.
Clinically halothane-associated hepatitis presents in 1-2 weeks after
the exposure (surgery under general anesthesia). Liver tenderness,
hepatomegaly and increased LFts are characteristic. Light microscopy of
liver biopsy specimens reveal widespread centrilobular hepatic
necrosis. Halothane-associated hepatitis has up to an 80% mortality
rate. There is no specific therapy for this condition.
(Choice A) Microvesicular fatty change is seen in the liver of patients
with Reye syndrome. This condition occurs in children 5-10years old
after treatment with salicylates.
(Choice B) The most common cause of macrovesicular fatty change (fatty
liver) is chronic ethanol consumption.
(Choice D) Intrahepatic cholestasis is a hallmark of biliary duct
obstruction. It is characteristic for primary sclerosing cholangitis
and primary biliary cirrhosis. Drug-induced cholestasis may be
associated with oral contraceptive pills, anabolic steroids, and
chlorpromazine.
(Choice E) Hepatic granulomatosis is associated with treatment with
methyldopa, hydralazine and quinidine.

Educational Objective:
Massive hepatic necrosis is a rare but severe complication of halothane
exposure. It occurs due to direct liver injury by halothane metabolites
and formation of autoantibodies against liver proteins. Light
microscopy shows massive centrilobular hepatic necrosis.

194
USMLE WORLD STEP 1 PHARMACOLOGY

Q NO 147: A 65-year-old female is hospitalized with sudden onset of shortness of


breath, tachypnea and pleuritic-type chest pain. Pulmonary ventilation/perfusion
scanning reveals a large mismatched defect in the lower lobe of the right lung.
The agent for long-term therapy to prevent recurrences of this patient’s condition
has which mechanism of action?

A. Binds tightly to antithrombin


B. Binds to the thrombin active site
C. Blocks glutamate residue carboxylation
D. Combines with proactivator plasminogen
E. Inhibits arachidonate product formation

Explanation:
Warfarin is the most commonly used agent for long term anticoagulation
to prevent venous thrombosis and resultant pulmonary embolism (VIE).
Warfarin inhibits vitamin K dependent-carboxylation of glutamic acid
residues of clotting factors II, VII, IX and X (also known as vitamin K
dependent clotting factors). This results in production of
dysfunctional coagulation proteins. The onset of therapeutic action of
warfarin is typically delayed for a few days because of the long half-
lives of some of the vitamin K dependent clotting factors. Prothrombin
time (PT) is used for monitoring the therapeutic effect of warfarin.
International normalized ratio (INIR) standardizes P1 assays.
(Choice A) Unfractionated and low molecular weight heparin increases
the effect of the naturally occurring anticoagulant antithrombin-III.
Binding of heparin to antithrombin-lll by a pentasaccharide in the
heparin chain causes a conformational change of antithrombin Ill. This
conformational change increases antithrombin binding and neutralization
of thrombin and factor Xa. Heparin is typically used for acute
management of venous thrombosis, as warfarin takes several days to
become therapeutic.
(Choice B) Direct thrombin inhibitors (DTIs) do not require
antithrombin-III for their action. These medications directly inhibit
thrombin mediated fibrin formation. Lepirudin and argatroban are direct
thrombin inhibitors.
(Choice D) Drugs that convert inactive plasminogen to plasmin result in
fibrinolysis and clot lysis. These medications are used for clot lysis
in acute myocardial infarction, pulmonary embolism and arterial
thrombosis.
(Choice E) Aspirin irreversibly acetylates platelet cyclooxygenase-l
leading to decreased formation of thromboxane A2. Aspirin is used as a
primary and secondary prevention of myocardial infarction and strokes.
Aspirin is not as effective as warfarin in preventing recurrent
pulmonary embolism and deep venous thrombosis.

Educational Objective:
Heparin is the drug of choice for acute management of venous thrombosis
or thromboembolism. Heparin inhibits antithrombin III. Warfarin is used
for long-term prevention of recurrent venous thromboembolism (VIE).
Warfarin inhibits vitamin K dependent-carboxylation of glutamic acid
residues of clotting factors II, VII, IX and X (vitamin K dependent
clotting factors).

195
USMLE WORLD STEP 1 PHARMACOLOGY

Q NO 148: A 45-year-old male treated for pulmonary tuberculosis experiences


decreased visual acuity in both eyes. After one of the drugs is eliminated from
his treatment regimen, his vision gradually improves. The withdrawn drug
most likely affects bacterial:
A. Mycolic acid synthesis
B. Carbohydrate polymerization
C. DNA winding-unwinding
D. RNA synthesis
E. Folic acid utilization

Explanation:
Of all the antituberculous agents ethambutol is the drug most notorious
for causing adverse visual changes. The alterations in vision
associated with ethambutol stem from optic neuritis and include
decreased visual acuity, color blindness and central scotoma. With
discontinuation of this drug, most patients see an improvement in their
vision. As with all other antimycobacterial agents, ethambutol can also
cause hepatotoxicity.
Ethambutol exerts its antimycobacterial effect by inhibiting arabinosyl
transferase, an enzyme that polymerizes arabinose into arabinan and
then arabingalactan (a constituent of the mycobacterial cell wall).
This carbohydrate polymerization is necessary for mycobacterial cell
wall synthesis. Ethambutol is not effective against other organisms.
(Choice A) Mycolic acid synthesis is inhibited by isoniazid, the only
antituberculous drug to utilize this mechanism. Important adverse
reactions associated with isoniazid include liver toxicity (especially
when used in combination with the other antimycobacterial agents) and
peripheral neuropathy.
(Choice C) DNA winding-unwinding is affected by the fluoroquinolones.
These drugs inhibit DNA gyrase in bacterial organisms, leading to
torsional stress fractures of the chromosomes. In children
ciprofloxacin has been associated with inflammation of the Achilles
tendon and tendon rupture. Gatifloxacin is associated with
hypoglycemia.
(Choice D) Rifampin directly inhibits bacterial DNA-dependent RNA
polymerase, thereby disrupting the transcription of DNA into mRNA.
Adverse effects commonly associated with rifampin include
hepatotoxicity as well as the reddish- orange discoloration of bodily
secretions (eg, sweat, urine, feces, tears, and saliva).
(Choice E) Folic acid utilization can be affected by antibiotics such
as sulfamethoxazole and para aminosalicylic acid (aminosalicylic acid).

Educational Objective:
Ethambutol is an antimycobacterial agent that inhibits carbohydrate
polymerization thereby preventing peptidoglycan cell wall synthesis.
One unique adverse effect of ethambutol is optic neuritis, which
typically presents in conjunction with decreased visual acuity, central
scotoma and color blindness.

196
USMLE WORLD STEP 1 PHARMACOLOGY

Q NO 149: An experiment is conducted with IV infusion of epinephrine in dogs.


Hemodynamic parameters are recorded during the experiment. The effects of
epinephrine on diastolic blood pressure and heart rate are presented on the
graph (norma/).

After pretreatment with drug A the infusion was continued and new
curves were obtained (drug A). Which of the following medications is
most likely to be drug A?

A. Phentolamine
B. Phenylephrine
C. Propranolol
D. Isoproterenol
E. Atropine

Explanation:
Hemodynamic effects of epinephrine occur secondary to agonistic effects
at al, β1 and β2 adrenoreceptors. Stimulation of al receptors of skin
and visceral smooth muscle increases total peripheral resistance and
raises systolic blood pressure. Stimulation of 132 receptors of the
skeletal muscle vasculature produces vasodilatation, leading to
decreases in diastolic blood pressure. Epinephrine’s agonistic effect
on β1 receptors of the heart increases heart rate and contractility.
In this experiment pretreatment with drug A results in a marked rise in
the diastolic blood pressure suggesting two possibilities: (1) the
abolition of the vasodilator effect of beta-2 receptors, or (2) the
enhancement of the alpha-1 vasoconstrictor action. In this case,
however, the heart rate is also reduced, implying that the beta-1
chronotropic effect of epinephrine was reversed. Among the drugs listed

197
USMLE WORLD STEP 1 PHARMACOLOGY
in the answer choices, only propranolol is a nonselective (β1 and β2)
antagonist capable of this physiologic effect.
When administered alone, propranolol produces peripheral
vasoconstriction (β2 antagonism) without significantly increasing blood
pressure. Epinephrine, on the other hand, stimulates both α1 and β2
adrenoreceptors. Propranolol counteracts epinephrine’s β2-mediated
vasodilatation, leaving the α1 effect (vasoconstriction) intact. During
pretreatment with propranolol, the unopposed α1 effect
(vasoconstriction) of epinephrine leads to increased diastolic blood
pressure.
(Choice A) Phentolamine is a non-specific alpha-1 and alpha-2 blocker
with no beta-blocking effects that causes a profound fall in diastolic
blood pressure and reflex tachycardia.
(Choice B) Phenylephrine is a selective α-adrenergic agonist. Although
it may cause elevated systolic and diastolic blood pressures, which may
be associated with reflex bradycardia, there is a temporal inverse
relationship between the rising diastolic blood pressure and the fall
in heart rate.
(Choice D) Isoproterenol is a selective β-adrenergic agonist that
stimulates both j32 receptors on vascular smooth muscle (leading to
vasodilatation) and cardiac j3i receptors (leading to increases in
cardiac rate and contractility). A combination of isoproterenol and
epinephrine would decrease diastolic blood pressure and exacerbate
tachycardia.
(Choice E) Atropine is a competitive antagonist at post-junctional
muscarinic receptors in the heart (M2). Atropine increases the heart
rate by blocking vagal influences and after pretreatment with atropine;
epinephrine-mediated tachycardia would increase.

Educational Objective:
Epinephrine increases systolic blood pressure (α1 effect), increases
heart rate (j3i effect), and decreases diastolic blood pressure (β2
effect). Pretreatment with propranolol eliminates the β effects of
epinephrine (vasodilatation and tachycardia), leaving only a effect
(vasoconstriction).

198
USMLE WORLD STEP 1 PHARMACOLOGY

Q NO 150: A 21-year-old male presents to the ER with dizziness and extreme


fatigue. He says that he has been drinking a lot of water recently and has
been urinating more than usual. His blood pressure is 90/60 mmHg, and
heart rate is 120/mm. His urine is positive for glucose and ketones. Which of
the following insulin types depicted on the graph below is most appropriate
for the treatment of this patient’s condition?

A. A
B. B
C. C
D. D
E. E
F. F

Explanation:
This patient’s history suggests dehydration, a suspicion confirmed by
his low blood pressure and tachycardia. Worse yet, he has both ketones
and glucose in his urine, indicating that he is in a dangerous
physiological state called “diabetic ketoacidosis” (DRA), a condition
most commonly found in type 1 diabetics. This patient’s young age and
relatively acute onset of symptoms (polyuria, polydipsia, and fatigue)
also suggest type 1 diabetes although you could probably guess that
since the patient is set to receive insulin therapy. Giving intravenous
fluids administering insulin and correcting electrolyte imbalances are
crucial in the management of ketoacidosis. In fact these three simple
steps often correct DKA without further management. Short-acting
regular insulin typically given by IV infusion is preferred for initial
management of DKA. After an initial insulin bolus, an intravenous
infusion of insulin is started. The rate of infusion is adjusted to
blood glucose levels. Line B depicts short-acting regular insulin after
subcutaneous injection. The action of regular insulin starts in 30
minutes peaks at two to four hours and lasts for six to eight hours.
Typically subcutaneous insulin is not used for patients who present
with DKA. These patients are usually dehydrated which makes the
absorption of subcutaneous insulin very erratic. Intravenous insulin
has a half-life of five minutes.
(Line A) This line represents rapid-acting insulins also known as
monomeric insulins. Insulin lispro, aspart, and glulisine belong to
this category. These insulins do not polymerize and are therefore
rapidly absorbed after subcutaneous injection. Their actions start in
15 minutes and peak at around 30 minutes to one hour. Although some
studies have shown that these insulins can be used to treat DRA,

199
USMLE WORLD STEP 1 PHARMACOLOGY
regular insulin is still the preferred treatment for DKA in the United
States.
(Lines C & D) NPH and Lente insulins are intermediate acting insulins.
NPH (Neutral Protamine Hagedorn) is a crystalline suspension of insulin
with protamine and zinc. The addition of protamine and zinc serves to
retard the absorption of insulin from the subcutaneous injection site
and so prolongs duration of action. Typically NPH starts working in one
to three hours peaks in six to eight hours and lasts up to 1 2-1 6
hours (line C). Lente insulin is an amorphous and crystalline mixture
of insulin with zinc. In this case the addition of zinc alone retards
the absorption of the insulin which prolongs its action. The action of
lente insulin is similar to NPH although it has a slightly delayed peak
and a longer duration of action than NPH (line D). These insulins are
never used to treat diabetic ketoacidosis. These insulins are instead
started once the patient is stable and can accept oral feeding.
(Line E) Ultralente is the long-acting insulin containing insulin zinc
and crystals. Its action starts in two to four hours and lasts for
about 24 hours. It has a slight peak that occurs between six to eight
hours of injection. It is not commonly used to treat diabetes nowadays
due to its erratic absorption after subcutaneous injection. Ultralente
is not used for initial treatment of diabetic ketoacidosis.
(Line F) Glargine (Lantus) is a new insulin analog, which has a very
long duration of action. Glargine has a pH of 4, so—after injection—it
forms a micro precipitate in subcutaneous tissue, which results in a
yew slow release of insulin. The duration of glargine insulin lasts 18-
24 hours and does not “peak.” Glargine insulin is not used to treat
diabetic ketoacidosis.

Educational Objective:
Regular insulin is preferred in the initial treatment of diabetic
ketoacidosis (DKA). Recognize the different types insulins and their
durations of action.
Line A — rapid-acting (e.g., lispro)
Line B — regular
Line C—NPH
Line D — Lente
Line E — Ultralente
Line E — Glargine.

200
USMLE WORLD STEP 1 PHARMACOLOGY

Q NO :A 72-year-old female is brought to the ER with severe shortness of breath, tachypnea


and pleuritic-type chest pain. She had a similar episode two months ago and has been treated
with warfarin since then. She says that she has been compliant with her medications. Her
prothrombin time (PT and INR) is found to be sub-therapeutic. Which of the following
concurrent medications is she most likely taking?

A. Aspirin
B. Phenylbutazone
C. Oxycodone
D. Acetaminophen
E. Metronidazole
E. Cimetidine
G. Phenytoin
H. Amiodarone
l. Diazepam

Explanation:
USMLE WORLD STEP 1 PHARMACOLOGY

Q NO 151: A 60-year-old male non-smoker with a history of Type II diabetes mellitus,


hypertension, hypercholesterolemia and mild renal insufficiency presents to your office
complaining of a dry cough during the past month. His blood pressure is
130/80 mmHg. His medications include aspirin, glipizide, lovastatin, hydrochlorothiazide, and
captopril. In order to relieve the patient’s cough, which of the following drugs would it be best
to substitute for one of the patient’s current
medications?

A. metformin
B. propranolol
C. prazosin
D. losartan
E. tolbutamide
F. pravastatin

Explanation:
Captopril, an ACE inhibitor, was prescribed to the patient to control hypertension and prevent
diabetic nephropathy. When used chronically (greater than 3 weeks), angiotensin converting
enzyme (ACE) inhibitors may induce a non productive cough, thought to occur secondary to
increased levels of bradykinin, a substance normally degraded by angiotensin converting
enzyme (ACE). The treatment of ACE inhibitor induced cough is replacement of this drug with
an angiotensin receptor blocking drug (ARB), such as losartan. ARB’s rarely cause chronic
cough as a side effect because they actthrough directantagonism of the angiotensin receptor
to reduce peripheralvascular resistance and do not interfere with the catabolism of bradykinin
as ACE inhibitors do. Thus, the patient’s cough is best treated by replacement of captopril with
losartan.
(Choices A, B, C, E and F) Change of his other medications is notwarranted atthis time
because none of them are associated with dry cough as a frequent side effect.
Educational Objective:
The besttreatmentof chronic drycough caused byan ACE inhibitoris replacementof this
drugwith an angiotensin receptor blocking drug (ARB), such as losartan. Remember that ARBs
are also beneficial for both hypertension and diabetic nephropathy and do not interfere with
the catabolism of bradykinin as ACE inhibitors do.
USMLE WORLD STEP 1 PHARMACOLOGY

Q NO 152: A 55-year-old male is treated for depression with sertraline. His past medical
history is significant for hypertension controlled with metoprolol and diabetes mellitus
controlled with diet. Which of the following is the most likely side effect of the treatment
prescribed for this patient’s depression?

A. Urinary retention
B. Cardiac arrhythmias
C. Seizures
D. Orthostatic hypotension
E.Sedation
E. Sexual dysfunction

Explanation:
Sertraline is serotonin-specific reuptake inhibitor (SSRI5) that has a better side effect profile
compared to tricyclic antidepressants (TCAs) such as imipramine, doxepin, amitriptyline, and
clomipramine. Sexual dysfunction is a relatively common side effect of the SSRls and limits
their use for many patients.
(Choice A) Urinary retention may be seen with tricyclic antidepressants due to the
anticholinergic effects on the musculature of the urinary bladder. Anticholinergic agents
competitively inhibit acetylcholine at the postganglionic autonomic (muscarinic) receptors, and
in patients with urinary outlet obstruction (benign prostatic hypertrophy), tricyclic
antidepressants (TCA5) relax the bladder and may exacerbate urinary retention.
(Choice B) Cardiac arrhythmias are a feared sided effect of overdose with the tn cyclic
antidepressants, which create a quinidine-like effect with prolongation of the QRS and QT
interval, causing severe and often fatal cardiac arrhythmias.
(Choice D) Orthostatic hypotension is also seen with the TCAs. Rememberthattricyclic anti-
depressants (imipramine) lead to hypotension through antagonism of alpha-adrenergic
receptors. Prazosin is a selective alphal receptor blocker used to treat benign prostatic
hypertrophy (to promote urinary outflow) that may also cause orthostatic hypotension in
certain patients.
(Choice E) Sedation also commonly occurs with TCAs due to the blockade of histamine
receptors.
Educational Objective:
Sertraline is serotonin-specific reuptake inhibitor (SSRls) that has a better side effect profile
compared to tricyclic antidepressants (TCAs). Sexual dysfunction is a relatively common side
effect of the SSRls and limits their use for many patients.
USMLE WORLD STEP 1 PHARMACOLOGY

Q NO 153: A 63-year-old Caucasian male ingested several tablets of diphenhydramine for a


skin rash, and subsequently experienced blurred vision. This effect is most likely related to
antagonism of which of the following agents?

A. Histamine
B. Serotonin
C. Acetylcholine
D. Norepinephrine
E.LTB4

Explanation:
Histamine released from mast cells and basophils plays an important role in allergic
inflammation by acting on the Hi histamine receptor. Antihistaminics (Hi histamine receptor
blockers) are very useful drugs in the treatment of
allergy. Hi receptor blockers decrease the activity of this receptor by increasing the proportion
of inactive Hi receptor, a process known as reverse blockade. The first-generation
antihistaminics, chlorpheniramine and
diphenhydramine, have antimuscarinic, anti-alpha adrenergic, and anti-serotonergic properties
that are responsible for the majority of side effects. Anticholinergic side effects include
pupillary dilation, dry mouth, urinary retention, and constipation. The patient’s blurryvision
was most likely due to pupillarj dilation and failure of accommodation
secondary to diphenhydramine’s antimuscarinic effects.
(Choice A) Eirst-generation Hi receptor blockers easily cross the blood-brain barrier to
interfere with histamine
neurotransmission, producing drowsiness and cognitive dysfunction. Diphenhydramine is a
potent sedative and this properte is sometimes utilized for the treatment of insomnia.
(Choice B) Serotonin receptor blockade causes appetite stimulation and weight gain.
(Choice D) Alpha-adrenergic blockade causes erectile dysfunction, hypotension, and postural
diriness.
(Choice E) Leukotrienes (LT5) are synthesized via the liporjgenase pathway. LTs induce
allergic inflammation via their actions at leukotriene receptors. Hi receptor blockers do not
interfere with leukotriene formation or action.
Educational Objective:
The first-generation antihistaminics, chlorpheniramine and diphenhydramine, have
antimuscarinic, anti-alpha adrenergic, and anti-serotonergic properties that are responsible for
the majority of side effects.
USMLE WORLD STEP 1 PHARMACOLOGY

Q NO 154: A 45-year-old Caucasian male is brought to the Emergency Department with


palpitations and fatigue. He has never had symptoms like this before, and his past medical
history is not significant. He smokes one pack per day and consumes alcohol only occasionally.
His blood pressure is 110/70 mmHg, heart rate is 120/mm, and his rhythm is irregularly
irregular. After initial treatmentwith IV digoxin his heart rate drops to 82/mm and remains
irregular. Which of the following best explains the effect of digoxin observed in this patient?

A. Delayed afterdepolarizations
B. Increased parasympathetic tone
C. Decreased action potential duration
D. Decreased atrial refractoriness
E. Increased ventricular contractility

Explanation:
Digoxin is a second-line treatmentfor atrial fibrillation with rapid ventricular response (AE with
RVR) because of its ability to slow conduction through the AV node. Calcium channel blockers
such as diltiazem and cardioselective beta- blockers are frequently used as first line. Digoxin
causes increased parasympathetic tone through its action on the vagus nerve, which leads to
a decreased rate of AV conduction. Slowed AV conduction is useful in atrial fibrillation and
flutter with rapid ventricular response (i .e.: HR greater than 100 beats per minute) because
the focus of increased automaticity in these conditions is located in the atria and the rapidity
of the ventricular response depends on an AV node with a short refractory period. Thus, with
slowed AV conduction the atria will continue to fibrillate or flutter, but the ventricles will
contract at a more normal rate. RVR is a serious condition because there is inadequate
diastolic filling time leading to poor cardiac output and backup of blood in the lungs (heart
failure).
(Choice A) Delayed afterdepolarizations occur after complete repolarization of the cardiac
myocyte in states of hyperexcitability such as very high intracellular calcium or high
catecholamine stimulation states. Digoxin toxicity can lead to delayed afterdepolarizations via
its mechanism of increasing intracellular calcium. This is how digoxin can lead to ventricular
tachycardia and death.
(Choices C and D) Decreased action potential duration would lead to a decreased refractory
period and would allow the cardiac pacemaker cells to fire and conduct an action potential with
a greater frequency. Decreased atrial refractoriness would give the atria the ability to
depolarize and contract at an increased rate. The shorter the refractory period, the more
frequently a cell can depolarize. This state would predispose to atrial fibrillation and would not
be useful in this patient.
(Choice E) One of digoxin’s effects is increased ventricular contractility. The mechanism by
which this is accomplished is via the blockade of the Na-K-ATPase on cardiac myocytes. This
leads to an increased intracellular calcium level (stored in the sarcoplasmic reticulum). The
increased availability of calcium within the cell during excitation-contraction coupling provides
for maximal crosslinking of actin and myosin and greater contractility. Though increased
ventricular contractility is one of the mechanisms of digoxin, the question stem here asks how
digoxin slows AV nodal conduction. Increased cardiac contractility is not a mechanism for
decreased AV nodal conduction.
Educational Objective:
Digoxin will increase cardiac contractility by blocking the Na-K-ATPase in cardiac myocytes
leading to an increased intracellular calcium concentration, and it will decrease AV nodal
conduction by a mechanism of increased parasympathetic tone.
USMLE WORLD STEP 1 PHARMACOLOGY

Q NO 155: A 62-year-old Caucasian male is hospitalized with a new onset of atrial fibrillation
(AE). Eurther evaluation reveals a decreased serumTSH level. The arrhythmia is controlled,
andthe patientis scheduledforradioactive iodine (1131) therapy. Pretreatment with which of
the following will decrease the effects of therapy in this patient by competitive inhibition?

A. Propranolol
B. Potassium perchlorate
C. Methimazole
D. Propylthiouracil (PTU)
E.Prednisone

Explanation:
Most patients with hyperthyroidism in the United States are treated with ablating doses of
radioactive iodine (1131). The concentration of iodine levels within the thyroid gland is much
higher compared to anywhere else in the body because thyroid follicles transport inorganic
iodine against a concentration gradient. This iodine is needed to make the thyroid hormones,
which are iodinated amino acids. This is achieved via a sodium iodide symporter (NIS) located
at the basolateral membrane of the thyroid folli cular cell. Ions such as perchlorate and
pertechnetate and radioactive iodine are also all taken up by the thyroid gland using the same
NIS mechanism. All forms of iodine compete for receptors on the NIS. If perchlorate is
present, the uptake of radioactive iodine will decrease due to competitive inhibition.
(Choices A and E) Hyperthyroidism induces a state of increased sympathetic activity.
Therefore, treatment with beta- blockers provides symptomatic relief. Propanolol and the
other beta-blockers do not have any effect on the uptake of iodine bythe thyroid gland. In
fact1 beta-blockers are often continued during radioactive iodine treatment. Similarly,
prednisone has no effect on the uptake of iodine in the thyroid follicular cells.
(Choices C and D) Antithyroid thionamides, such as methimazole and propylthiouracil, can
decrease the uptake of radioactive iodine and alter its kinetics. For this reason, these drugs
are typically stopped 7-10 days before administration of radioactive iodine. Neither answer is
the correct choice, however, because neither acts via competitive inhibition. Instead, these
thionamides inhibitthe enzyme thyroid peroxidase, thus blocking the conversion of iodide to
iodine.
Educational Objective:
1. Anion inhibitors (perchlorate, pertechnetate) block iodide absorption by the thyroid gland
via competitive inhibition.
2. Thionamides (methimazole and propylthiouracil) decrease the formation of thyroid
hormones by inhibiting thyroid peroxidase.
3. Iodide salts inhibit synthesis as well as release of thyroid hormones.
USMLE WORLD STEP 1 PHARMACOLOGY

Q NO 156:A 22-year-old Caucasian female presents to your office with a recent onset of fever
and throat pain. Her past medical historjis significantforhyperthyroidism controlledwith
medicaltherapy. Herblood pressure is 110/70 mmHg and heart rate is 90/mm. Physical
examination is insignificant. Which of the following is the best next step in the management of
this patient?

A. Empirical oral antibiotics


B.WBC countwith differential
C. ESR and anti-neutrophil antibodies
D. TSH and T4?T3 levels
E. Ibuprofen and oral hydration

Explanation:
There are three major treatment modalities for hyperthyroidism: antithyroid drugs, radioactive
iodine, and surgical thyroidectomy. While radioactive iodine is generally the most preferred
treatment in the United States, antithyroid drugs are used in select cases. Antithyroid drugs
work by diminishing the following—oxidation of iodine in the thyroid gland, iodination of
tyrosine residues, and the coupling of iodotyrosine molecules. The two antithyroid drugs
available in the United States are propylthiouracil (PTU) and methimazole; both belong to the
thionamide class, inhibitthe same enzyme, and are associated with agranulocytosis. They
differ in that PTU also decreases the peripheral conversion of T4 to T3, has a shorter half-life,
and is the drug of choice in pregnancy, as methimazole is teratogenic.
Agranulocytosis describes an absolute neutrophil count of less than 500 /mL. Although
agranulocytosis only occurs in approximately 0.5% of cases treated with antithyroid
medications, it is a serious complication because neutrophils are vital in mounting an immune
response to many pathogens. Agranulocytosis usually occurs within the firstfewweeks of
therapy, though it can occur at anytime. Patients typically present with fever and a sore
throat. If thionamideassociated agranulocytosis is suspected, the drug is immediately
discontinued and a white blood cell count with differential is drawn.
(Choice A) Although empirical antibiotics might be reasonable in many patients, this patient is
known to be at riskfor agranulocytosis because she is taking an antithyroid drug. The
algorithm for a patient on antithyroid medications who presents with a fever is to discontinue
the antithyroid medication and measure the WBC count with a differential immediately.
Establishing this patient’s immune status and discontinuing the antithyroid drug are the best
next step.
(Choice C) It is true that antithyroid-medication-associated agranulocytosis is caused by
antibodies directed against circulating neutrophils, so this is a very tempting choice. There are
two problems with choice C, however. First, measurement of anti-neutrophil antibodies is
neither specific nor sensitive to agranulocytosis secondary to antithyroid medications. Second,
allowing the patientto continue taking her antithyroid medication will allowthe neutrophil
destruction to continue, leaving the patient vulnerable to infection.
(Choice D) Thyroid function tests are periodically measured to guide the adjustment of
antithyroid drug dosage, but this patient’s fever and sore throat pointto a far more urgent
possibility. Agranulocytosis should be ruled out before fine-tuning the antithyroid dosage.
(Choice E) Treatmentwith an anti-pyretic and intravenous hydration is a reasonable wayto
treatfever, butthe choice of anti-pyretic is problematic. Aspirin and ibuprofen are notthe
besttreatments forfever in a patientwith thyroid dysfunction because they can displace thyroid
hormones from binding proteins, thereby worsening a thyrotoxic state; acetaminophen is
preferred. Furthermore, choice E is notthe best choice because it does nothing to address the
possibility of agranulocytosis.
Educational Objective:
Agranulocytosis is a rare but very serious complication of antithyroid drugs. A WBC count with
a differential is necessary in any patient receiving either methimazole or PTU who presents
with a fever.
USMLE WORLD STEP 1 PHARMACOLOGY

Q NO 157: A 31-year-old male farm worker comes to the physician complaining of an itchy
rash on his chest. Physical examination reveals an annular and scaling plaque five centimeters
in diameter with central clearing on the chest. KOH preparation of skin scrapings shows
branching septate hyphae, and topical application of terbinafine was prescribed. Which of the
following mechanisms of action explains the antifungal activity of this drug?

A. Binding to ergosterol
B. Inhibition of squalene epoxidase
C. Blocking 3-D-glucan synthesis
D. Preventing mitosis by binding tubulin
E. Inhibition of fungal protein synthesis

Explanation:
The patient described in this vignette is presenting with symptoms of tinea corporis. Clinically
tinea corporis presents as an annular scaling plaque with well-demarcated, raised
erjthematous borders and central clearing.
Terbinafine is a commonly used drug for treatment of dermatophytosis. It belongs to the class
of antifungals called allylamines and may be used topically or orally. Terbinafine inhibits
synthesis of ergosterol of the fungal membrane by inhibiting the enzyme squalene epoxidase.
Ergosterol is a cell membrane sterol that is unique to fungi and does not occur in human cell
membranes. This drug tends to accumulate in skin and its appendages, and its side effects are
mild by the topical route.
(Choice A) Binding to ergosterol is a mechanism of action of polyene antifungals (amphotericin
B and nystatin).
(Choice C) Caspofungin belongs to the echinocandin class of antifungals. It blocks synthesis of
13 (1 ,3)-D-glucan, a main component of Candide and Aspergi//us cell walls.
(Choice D) Griseofulvin binds to polymerized microtubules and disrupts the fungal mitotic
spindle, thus preventing fungal cell mitosis. It accumulates in the skin and its appendages and
is only effective in dermatophytosis.
(Choice E) Flucytosine is an antimetabolite antifungal that is transformed into 5-fluorouracil in
the fungal cell. It inhibits fungal protein synthesis by replacing uracil with 51luorouracin in
fungal mRNA. It is used in systemic fungal infections.
Educational Objective:
Terbinafine is used fortreatment of dermatophytosis. It inhibits synthesis of fungal membrane
ergosterol by suppressing the enzyme squalene epoxidase.
USMLE WORLD STEP 1 PHARMACOLOGY

Q NO 158: A 22-year-old Caucasian male with recurrent episodes of abdominal pain and
vomiting is found to have a serum triglyceride concentration of 1500 mg/dL (normal <200
mg/dL). Which of the following combinations of drugs would be most helpful in preventing
recurrences of this patient’s condition?

A. Atorvastatin and ezetimibe


B. Atorvastatin and choleslyramine
C. Atorvastatin and gemfibrozil
D. Gemfibrozil and cholestyramine
E. Niacin and ezetimibe
F. Niacin and gemfibrozil

Explanation:
The treatment of hypertriglyceridemia consists of lifestyle changes with or without the addition
of anti-hyperlipidemic medications. Patients with mild hypertriglyceridemia are first treated
with diet, exercise, weight loss, and abstinence from smoking and alcohol. The patient
described in the vignette has severe hypertriglyceridemia. In his case, therapy with anti-
hyperlipidemic agents is necessary in addition to lifestyle changes.
Fibrates are the first-line therapy for hypertriglyceridemia, decreasing serum triglycerides by
20-50%. Fibrates work by activating paroxjsmal proliferator-activated receptor alpha (PPAR-
alpha). If further triglyceride reduction is needed, niacin can be added as a second-line agent.
Niacin improvePC:5.1ipidemia by decreasing the synthesis of hepatic triglycerides and VLDL.
Finally, fish oil containing omega-3 fatty acids is also useful in the treatment of
hypertriglyceridemia. Fish oils decrease hepatic VLDL production and decrease serum
triglycerides by 20-30%.
(Choice A, B, C, D and E) Statins decrease serum cholesterol by inhibiting the enzyme HMG-
CoA reductase, which is necessarjfor hepatic cholesterol synthesis. Statins also increase LDL
receptor activity on hepatocytes. In general, statins produce a modest reduction in serum
triglycerides. However, in patients with marked hypertriglyceridemia treated with high-dose
statin therapy, there is a significant reduction in triglycerides (up to 20%).
Bile acid-binding resins work by binding bile acids in the gastrointestinal tract, thereby
interfering with the enterohepatic circulation of bile acids. Bile acid-binding resins are now
primarily used in combination with statins for the treatment of hypercholesterolemia. Bile acid-
binding agents increase hepatic triglyceride production and can cause elevations in serum
triglyceride levels.
Ezetimibe selectively inhibits the intestinal absorption of cholesterol. Ezetimibe is primarily
used in conjunction with statin therapy. It causes modest reductions in serum triglycerides,
and modest elevations in serum HDL.
Educational Objective:
Niacin and fibrates are the most effective agents for the treatment of hypertriglyceridemia.
USMLE WORLD STEP 1 PHARMACOLOGY

Q NO 159:A 64-year-old female is treated with warfarin for chronic atrial fibrillation. She
develops acute pyelonephritis that requires ceftriaxone therapy. Soon after initiation of
therapy her laboratory testing reveals significantly increased prothrombin time and increased
INR. Warfarin action is enhanced in this patient due to the effect on which of the following?

A. Absorption
B. Hepatic metabolism
C. Plasma protein binding
D. Competitive antagonism
E. Renal excretion

Explanation:
Warfarin isthe mostcommonlyused agentforlongterm anticoagulationto
preventthromboembolism in patientswith atrial fibrillation. Warfarin inhibits vitamin K
dependent -carboxy’lation of glutamic acid residues of clotting factors Il, VII, IX and X
resulting in production of dysfunctional coagulation proteins. Warfarin has a large number of
interactions with drugs that alter P-450 activity, protein binding and gastrointestinal
absorption.
Prothrombin time (PT) is used for monitoring the therapeutic effect of warfarin. International
normalized ratio (INR) standardizes PT assays and typically an INR between 2 and 3 is used as
the therapeutic target range. Bleeding from excessive anticoagulation is the most common
complication of warfarin therapy. Vitamin K is the antidote given to reverse bleeding from
warfarin overdose. Vitamin K occurs in two natural forms, Ki or phylloquinone and K2 or
menaquinone. Ki comes from dietary sources, whereas K2 is produced in the gut by bacterial
flora. Antibiotic use typically results in a decrease in intestinal bacterial flora and decrease in
production of Vitamin K2. This can result in increased activity of warfarin without a change of
dosage because of a decrease in competitive inhibition.
Wart arm has a large number of other interactions with drugs that alter P-450 activity, protein
binding, gastrointestinal absorption and pharmacodynamic alteration. The elimination of
warfarin is almost entirely by hepatic metabolism by microsomal enzymes (cytochrome P-
450). Drugs that inhibit cytochrome P450 increase the anticoagulant activity, whereas drugs
that increase P450 activity decrease warfarin’s anticoagulant activity.
(Choice A) Cholestyramine binds warfarin and many other drugs in the intestine and thus
decreases the therapeutic effect of wart arm.
(Choice B) Rifampicin, phenobarbital and phenytoin are universal enhancers of the cytochrome
P450 pathway. Therefore, concurrent use of wart arm with these medications results in
decreased efficacy of wart arm by increasing its metabolism.
(Choice C) Aspirin and some other NSAIDs can displace warfarin from its protein-binding site.
(Choice E) Renal clearance is considered to be a minor determinant of anticoagulant response
to warfarin. No dosage adjustment is necessary for patients with renal failure.
Educational Objective:
Decreased vitamin K synthesis by intestinal flora decreases the effect of competitive
antagonism on clotting factor synthesis. This increases the activity of warfarin.
USMLE WORLD STEP 1 PHARMACOLOGY

Q NO 160: A 40-year-old asthmatic is found to have the following finding on routine


examination.
Which of the following is the most effective pharmacotherapy for the lesions on the roof of this
patient’s mouth?

A. Griseofulvin
B.Terbinafine
C. Nystati n
D. Amphotericin-B
E. Penicillin
F. Acyclovir

Explanation:
The thick white patches shown on the image of the oral mucosa above indicate oral thrush
(oropharjngeal candidiasis). Trush is a common variant of localized infection with Card/cia
fungi. Oral thrush is associated with immunosuppression and is common in patients with HIV
infection, diabetes mellitus, and cancer. Patients with asthma who take oral or inhaled steroids
are also at risk for candidiasis. Systemic antibiotic therapy and dentures are other
predisposing factors.
In immunocompetent patients, localized candidiasis is treated with local antifungal
medications. Nystatin is a polyene antifungal with a mechanism of action similar to
amphotericin B. Nystatin binds to ergosterol molecules in the fungal cell membrane causing
pores and leakage of fungal cell contents. Nystatin is not absorbed from the GI tract. It is
administered as an oral agent (“swish and swallow”) for oropharjngeal candidiasis.
(Choice A) Griseofulvin inhibits fungal cell mitosis at metaphase. It is indicated for the
treatment of dermatophytoses, not Candida.
(Choice B) Terbinafine is an allylamine antifungal that accumulates in the skin, nails, and in
adipose tissue. It is used to treat dermatophytosis (onychomycoses), but not for Candida.
(Choice D) Amphotericin B is a polyene antifungal used for systemic mycoses. Although its
mode of action is similar to nystatin, the two drugs differ in that amphotericin B has manytoxic
side effects and is, therefore, never used for simple mucocutaneous infections.
(Choice E) Penicillins are a group of IS-lactam antibiotics. They inhibit the formation of
peptidoglycan cross-links in bacterial cell walls.
(Choice F) Acyclovir is an antiviral drug effective against HSV-1, HSV-2, and VZV. It is a
nucleoside analog that is converted into acyclo-GTP in the affected cell, which inhibits viral
DNA polymerase.
Educational Objective:
Nystatin is a polyene antifungal and is the drug of choice for oropharjngeal candidiasis in an
immunocompetent patient. It acts by binding to ergosterol in the fungal cell membrane,
causing the formation of pores and leakage of fungal cell contents. Nystatin is not absorbed
from the GI tract and is administered as an oral “swish and swallow” agent.
USMLE WORLD STEP 1 PHARMACOLOGY

Q NO 161:A 20-year-old male is being evaluated for tremor and gait instabilily. Laboratory
studies reveal elevated serum transaminases. A serum ceruloplasmin concentration is
decreased. Which of the following medications is most likely to benefit this patient’s condition?

A. Dimercaprol
B. Lactulose
C. Penicillamine
D.Deferoxamine
E. Calcium disodium edetate

Explanation:
This patient’s clinical presentation is most suggestive of Wilson’s disease. Wilson’s disease is
an autosomal recessive disorder characterized bytoxic accumulation of copperwithin organ
tissues (especiallythe liver1 brain, and eye). The toxic effects of Wilson’s disease are best
treated with penicillamine chelation, which removes excess copper in tissues.
(Choices A and E) Dimercaprol and EDTA are chelating agents used to treat lead poisoning.
Clinical features of lead poisoning can include encephalopathy and abdominal colic (pain)
without hepatitis or cirrhosis. In cases of lead poisoning. however, the serum ceruloplasmin
and copper levels are unaffected.
(Choice B) Lactulose is used in the treatment of cirrhosis-related hepatic encephalopathy.
Lactulose may. in fact, benefita patientwith advanced cirrhosis dueto Wilson’s disease,
butitwould notdirectlyaffectthe pathogenesis or progression of this disease.
(Choice D) Deferoxamine is an iron-chelating agent used to treat hemochromatosis, a
condition of systemic iron overload which produces cirrhosis, pancreatic fibrosis, and skin
pigmentation (“bronze diabetes”).
Educational Objective:
The pathogenesis of Wilson’s disease (hepatolenticular degeneration) involves an excess of
non-ceruloplasmin-bound serum copper, leading to injurious accumulation of this element in
the liver, CNS lenticular nucleus, and cornea. Chelation therapywith penicillamine is indicated
to remove excess loosely bound serum copper.
USMLE WORLD STEP 1 PHARMACOLOGY

Q NO 162: A 77-year-old Caucasian male presents to your office for a routine check-up. His
blood pressure is 1 70/70 mmHg and heart rate is 70/mm. Physical examination is normal. A
drug therapy was initiated to treat this patient’s hypertension. He presents several weeks later
complaining of bilateral ankle swelling and flushing. His blood pressure is 135/65 mmHg and
his heart rate is SO/mm. Which of the following is the most likely drug used in this patient?

A. Doxazosin
B. Propranolol
C. Ramipril
D. Hydrochlorothiazide
E. Eplerenone
E. Verapamil
G. Amlodipine
H. Isosorbide dinitrate

Explanation:
Isolated systolic hypertension is a common form of hypertension, especially in the elderly,
where the systolic pressure is elevated while the diastolic is not. There is discrepancy as to
where the cutoff lies for elevated systolic hypertension; it ranges an9where from over
l4OmmHg to over l6OmmHg. Thiazide diuretics and dihydropyridine calcium antagonists are
presentlyfirst-line drugs to treat isolated systolic hypertension in nondiabetic patients (where
an ACE-I or ARB would be utilized first). The side effect profile described in this patient is most
consistentwith amlodi pine.
(Choice A) Alpha 1-blockers such as Doxazosin, Prazosin and Terazosin are useful for the
treatment of both hypertension and benign prostatic hyperplasia. The most notable issue with
these medications istheirtendencyto cause hypotension when treatment is first started. This is
known as a first-dose effect and can be ameliorated by starting with a very small first dose.
Peripheral edema is not seen with alpha-blockers.
(Choice B) Nonselective beta-blockers such as propranolol do not cause peripheral edema.
They can exacerbate bronchial asthma, peripheral vascular disease and bradycardia. They also
maskthe hypoglycemic symptoms of diabetic medications.
(Choice C) Ramipril is an angiotensin converting enzyme inhibitor (ACE-I). Side effects include
cough most commonly, and serious reactions include hyperkalemia and angioedema. ACE-Is
are an excellent choice for hypertension in patients that do not have adverse reactions to
them, and they are particularly useful in patients with diabetes where they have an anti-
proteinuric effect. ACE-Is do not cause peripheral edema.
(Choice D) Hydrochlorothiazide is presently the first-line medication for the treatment of
essential hypertension in the general public. Though hydrochlorothiazide would be a good
choice to treatthis patient’s hypertension, itwould not cause the symptoms mentioned in the
question stem. As a diuretic, it mechanisticallywould not lead to increased peripheral edema.
(Choice E) Eplerenone is a newer aldosterone antagonist and functions similarlyto
spironolactone as a potassiumsparing diuretic. The mostfrequently mentioned side effect of
these medications is their abilitito cause gynecomastia (approximately 1% with eplerenone,
9% with spironolactone).
(Choice F) Verapamil is a diphenylalkylamine calcium-channel blocker and is often used as a
therapyfor rate control in atrial fibrillation with rapid ventricular response due to its ability to
slow conduction through the atrioventricular (AV) node. The most frequent adverse reactions
noted with verapamil are constipation and gingival hyperplasia.
(Choice H) Nitrates act by relaxing smooth muscle in the venous vasculature through a
mechanism where they are metabolized into endothelium derived relaxing factor (EDRE), also
known as nitric oxide. Their most common side effect is headache, especiallywith sublingual
nitroglycerin, and they should not be used togetherwith drugs for erectile dysfunction such as
sildenafil. This drug would not address this patient’s hypertension or cause his symptoms.
Educational Objective:
Amlodipine is a calcium channel blocker used in hypertension that can lead to flushing and
peripheral edema.
USMLE WORLD STEP 1 PHARMACOLOGY

Q NO 163: A 43-year-old immigrant from Southern Asia suffers from chronic cough, weight
loss and occasional hemoptysis. Sputum cultures grow acid4ast bacilli that are susceptible to
most antimycobacterial drugs in vitro. Isoniazid monotherapy in this patient would most likely
result in:

A. Rapid mycobacterial elimination from the body


B. Selective survival of intracellular bacilli
C. j3-lactamase induction within bacterial cells
D. Adaptive gene mutation within bacterial cells
E. Decreased transport protein expression on the cell surface

Explanation:
This patient most likely has active tuberculosis, which should never be treated with drug
monotherapy due to the rapid emergence of antibiotic resistance in M. tuberculosis. Isoniazid
resistance specifically occurs byiwo adaptive gene mutations. The first is a decrease in
bacterial expression of the catalase-peroxidase enzyme that is required for isoniazid activation
once the drug enters the bacterial cell. The second mechanism of resistance occurs through
modification of the protein target binding site for isoniazid.
Treatment of active tuberculosis is accomplished by combining multiple antibiotics to avoid
selecting for resistant strains. Commonly used drug regimens include isoniazid and rifampin in
addition to streptomycin, ethambutol, and/or pyrazinamide.
Isoniazid monotherapy may be used for patients who have a positive PPD and a negative chest
x-ray (ie, no evidence of clinical disease).
(Choice A) Rapid mycobacterial elimination from the body occurs when a small inoculum is
recognized and quickly destroyed bythe cell-mediated immune system. This happens in the
majorily of exposures. Bythe time tuberculosis has progressed to active, symptomatic disease,
large caseous and liquefactive necrotic granulomas have usually formed and a lengthy
treatment period with appropriate multidrug therapy is required to eradicate the organism.
Antibiotic therapyfortuberculosis lasts a minimum of six months.
(Choice B) Porisoniazid monotherapyto resultinthe selective survival of intracellularbacilli
would suggestthat isoniazid is unable to penetrate into macrophages that house M.
tuberculosis. However, this is not the case. Isoniazid can reach therapeutic levels within
macrophages, areas of caseous necrosis, and cerebrospinal fluid.
(Choice C) j3-lactamase induction within bacterial cells would not resultfrom isoniazid
exposure because isoniazid is not a beta-lactamase antibiotic.
(Choice E) Decreased transport protein expression atthe cell surface would interfere with the
effectiveness of drugs that require protein-mediated transport into the bacterial cell to reach
the intended target (eg, fluoroquinolones, macrolides, aminoglycosides).
Educational Objective:
Active tuberculosis is never treated with drug monotherapy due to the rapid emergence of
mycobacterial antibiotic
resistance from rapid adaptive gene mutations. Isoniazid monotherapy may be used for
patients who have a positive
PPD and a negative chest x-ray (ie, no evidence of clinical disease).
USMLE WORLD STEP 1 PHARMACOLOGY

Q NO 164:A newborn is being evaluated for cyanosis. Echocardiography demonstrates apical


displacement of the tricuspid valve leaflets, increased right ventricular volume, and
atrialization of the right ventricle. If this neonate’s condition is due to a drug side effect, the
mother most likely has which of the following conditions?

A. Gestational diabetes
B. Schizophrenia
C. Narcotic abuse
D. Alcohol abuse
E. Cocaine abuse
E. Seizure disorder
G. Bipolar disorder
H. Hypothyroidism
I. Down syndrome

Explanation:
Lithium is an anti-manic medication commonly used to treat bipolar disorder. Lithium has been
linked to Ebstein’s anomaly in infants exposed in utero. Ebstein’s anomaly is characterized by
apical displacement of the tricuspid valve leaflets, increased volume of the rightventricle, and
atrialization of the rightventricle. In patients with bipolar disorder1 lithium is usedtotreatthe
manic phase of the illness,whilethe depressive phase of the illness requires concurrentuse of
an antidepressant medication.
(Choice A) Gestational diabetes is associated with fetal macrosomia, caudal regression
syndrome, hypoglycemia. hypocalcemia and hypertrophic cardiomyopathy.
(Choice B) Neither schizophrenia nor antipsychotics have been associated with Ebstein’s
anomaly.
(Choice D) Narcotic abuse may cause withdrawal symptoms in the infant upon delivery
(irritability, diarrhea, vomiting) but is not associated with Ebstein’s anomaly.
(Choice E) Alcohol abuse during pregnancy may lead to the fetal alcohol syndrome,
characterized by: (1) midfacial anomalies (2) growth retardation, and (3) mental retardation.
Educational Objective:
Bipolar disorder is commonlytreated with lithium, an anti-manic medication. Lithium has been
linked to cases of
Ebstein’s anomaly in infants exposed in ulero. Ebstein’s anomaly is characterized by apical
displacement of the
tricuspidvalve leaflets, increased rightventricularvolume, and atrialization of the rightventricle.
USMLE WORLD STEP 1 PHARMACOLOGY

Q NO 165:A 23-year-old female comes to your office stating that a home urine
pregnancytestwas positive. Further investigation reveals an ectopic pregnancy, and a single
dose of methotrexate is given to treat her condition. Which of the following substances is most
likely to accumulate in embryonic tissues as a result?

A. Dihydrofolate polyglutamate
B. Tetrahydrofolate polyglutamate
C. Para-aminobenzoic acid
D. Thym i dyl ate
E. Folinic acid

Explanation:
Methotrexate (MTX) is the treatment of choice for early (< 6 weeks gestational age) ectopic
pregnancies. MTX is toxic to the rapidly dividing cells of the trophoblast leading to a
therapeutic termination of pregnancy.
MTX competitively and irreversibly inhibits dihydrofolate (DHE) reductase. DHE reductase
catalwes the two-step process that first reduces folic acid to DHE and subsequently reduces
DHE to tetrahydrofolate (THE). THE normally participates in reactions that involve single
carbon atom transfer. MTX inhibition of THE production inhibits purine and thymidylate
synthesis.
The MTX molecule is structurally similar to folic acid. After it gains access to the target cell,
MTX is converted to a polyglutamate form. Polyglutamati on prevents the movement of MTX
out of the cell (effectively storing MTX within the cell for later use). In a similarfashion, folate
and recycled DHE are stored within the cell via polyglutamation. Because MTX inhibits DHE
reductase, the intermediate dihydrofolic acid polyglutamate accumulates within treated cells.
(Choice B) Tetrahydrofolate polyglutamate is the fully reduced intracellular form of folic acid.
This molecule is only formed when DHE reductase is fully functional.
(Choice C) Para-aminobenzoic acid (PABA) is a folic acid precursor in prokaryotes.
Sulfonamide antibiotics are chemical analogues of PABA that inhibitthe enzyme
dihydropteroate synthetase, preventing bacterial conversion of PABA to folic acid. Humans lack
the abilitito convert PABA to folic acid and so require dietarjfolate but are not adversely
affected by sulfonamide antibiotics.
(Choice D) Thymidylate is a nucleotide whose synthesis is inhibited by MTX.
(Choice E) Eolinic acid (leucovorin) is a pre-reduced form of folic acid that can be used for the
synthesis of purines and thymidylate without first being converted by DHE reductase. It is
used for rescue therapy in cancer patients receiving MTX and othertypes of chemotherapy.
Educational Objective:
Methotrexate is an anti metabolite drug that is structurally similar to folic acid. It competitively
inhibits the enzyme dihydrofolate reductase, which catalyzes the synthesis of tetrahydrofolate.
Eailure of dihydrofolate reductase causes the intermediate dihydrofolic acid polyglutamate to
accumulate within treated cells.
USMLE WORLD STEP 1 PHARMACOLOGY

Q NO 166: A 60-year-old Caucasian male is diagnosed with exertional angina. His treatment
regimen includes metoprolol, isosorbide dinitrate and aspirin. He takes isosorbide dinitrate
early in the morning and again in the afternoon, but he does not take an evening dose. Such a
pattern of drug administration is intended to decrease which of the following?

A. Pharmacokinetic drug interaction


B. Pharmacodynamic drug antagonism
C. Effect potentiation
D. Tolerance development
E. Withdrawal symptoms
F. Drug noncompliance

Explanation:
Around-the-clock nitrate administration (in anyform) rapidly results in development of
tolerance to nitrates. This is why a nitrate4ree interval must be provided every day in patients
that are using daily long acting nitrates. The mechanism bywhich this occurs has not been
fully demonstrated, but it is theorized that it is due to a decreased vascular sensitivilyto
nitrates and an increased sensitivilyto endogenousvasoconstricting agents. Usuallythe nitrate-
free period is timed to occur during the nightwhen the patient is sleeping and cardiac work is
the least.
(Choice A) A pharmacokinetic drug interaction refers to how drugs that are used together in a
patient affect the absorption and elimination of each other. An example of this is the ability of
rifampin to increase the metabolism and elimination of warfarin by inducing cytochrome P-
450. This is notthe reason for providing a nitrate-free interval.
(Choice B) Pharmacodynamic drug antagonism refers to the ability of a drug, as part of its
mechanism of action, to antagonize, or prevent, the effect of another drug orendogenous
substance. An example of this is the effectthat labetalol has on alpha-i and beta adrenergic
receptors — it prevents the binding of catecholamines to these receptors.
(Choice C) Effect potentiation refers to one drug’s ability to increase the pharmacologic effect
of another agent. The drugs mentioned in this question stem do not potentiate the effects of
each other, and this is notthe reason for providing a nitrate-free period. Beta-adrenergic
blockers are often prescribed along with nitrates to prevent reflex tachycardia as well as to
provide decreased cardiac work by a mechanism different from that of nitrates.
(Choice E) Withdrawal symptoms occur when a patient forms a physiologic dependence on a
drug or medication and that drug or medication is then discontinued. This is common in
patients using alcohol, opiates, benzodiazepines, stimulants and some antidepressants. This is
not the reason for providing a nitrate-free period in patients taking nitrates.
(Choice F) Drug noncompliance is a problem where patients do not take the medications
prescribed to them properly or at all. It is frequent among patients who do not understand
their disease, can not afford medications, or simply can notremembertotakethem. The
latterbecomes much morefrequentwhen patients aretaking multiple medications with many
times-per-day dosing schedules. This is also not the reason for providing a nitrate-free period
every day in patients taking long-term nitrates.
Educational Objective:
Patients taking daily maintenance nitrates need to have a nitrate-free period every dayto
avoid tolerance to the drug.
USMLE WORLD STEP 1 PHARMACOLOGY

Q NO 167: In the experimental setting, certain microorganisms have demonstrated treatment


resistance by decreasing ergosterol incorporation into the cell membrane. These organisms
would most likely be resistant to which of the following agents:

A. Penicillin
B. Elucytosine
C. Griseofulvin
D. Nystati n
E. Caspofungin
F. Cyclosporine

Explanation:
The main classes of antifungal medications currently in use include:
1. Polyenes (Amphotericin B and nystatin): Bind ergosterol molecules in fungal cell
membranes, creating pores and causing cell lysis.
2. Triazoles (ketoconazole, fluconazole, itraconazole and von conazole): Inhibit ergosterol
synthesis.
3. Echinocandins(caspofungin and micafungin): Inhibit glucan synthesis (a component of the
fungal cell wall).
4. Pyrimidines: Elucytosine is the only agent in this class of antifungals. It is converted to 5-
fluorouracil within the fungal cell and interferes with fungal RNA and protein synthesis.
Of these agents, onlythe polyenes (amphotericin B and nystatin) depend on the amount of
ergosterol incorporated into fungal cell membranes fortheir efficacy. These drugs bind to
ergosterol molecules, forming pores in the membrane and allowing leakage of ions (especially
K) from the cells. This disruption of cell membrane integrity leads to cell lysi s.
If an organism were to decrease the amount of ergosterol in its cell membrane, the polyenes
would not be able to form as many pores. Decreasing the cell membrane ergosterol content is
a major mechanism of polyene resistance. Amphotericin B is a systemic antifungal active
against Cendida, Aspergi/Ius, C,yptococcus, Histoplasma, Blastomyces and Coccidioides.
Nystatin is only used topically. Oral candidiasis is the main indication for Nystatin (“swish and
swallow” preparation).
(Choice A) Penicillin is a bactericidal antibiotic that inhibits cell wall synthesis. Organisms
acquire resistance to penicillins via transfer of plasmids containing genes for j3-Iactamase and
altered penicillin-binding proteins.
(Choice B) Elucytosine is used in combination with Amphotericin B mainly for the treatment of
cryptococcal meningitis in AIDS patients. It is a nucleotide analog that competitively inhibits
RNA synthesis in fungal cells.
(Choice C) Griseofulvin interacts with fungal cell microtubules, inhibiting mitosis. It
accumulates in keratin-containing tissues, and is used to treat dermatophyte infections
(Microsporum, Epidermophyton, and Trichophyton).
(Choice E) Caspofungin is an antifungal agent that blocks synthesis of the glucan component
of the fungal cell wall. It is active against Candida and Aspergillus. Caspofungin does not bind
ergosterol.
(Choice F) Cyclosporine is an immunosuppressant used in transplant recipients to prevent
rejection. It decreases the synthesis of IL-2, thus inhibiting T-ceII proliferation.
Educational Objective:
The main groups of antifungal drugs are the polyenes, azoles, echinocandins and pyrimidines.
The polyene antifungals (amphotericin B and nystatin) act by binding ergosterol in the fungal
cell membrane.
USMLE WORLD STEP 1 PHARMACOLOGY

Q NO 168: A 34-year-old Caucasian male receives a kidneytransplant. His T-lymphocytes


quickly recognize the foreign HLA molecules of the renal transplant cells. Inhibition of which of
the following substances would specifically reduce the proliferation and differentiation of these
T lymphocytes?

A. E-cadherin
B. Neurofibromin
C. Calcineurin
D.Bcl-2
E. p53

Explanation:
In normal T cells, calcineurin is a protein phosphatase that is activated upon stimulation of the
appropriate cell receptor. Once activated, the calcineurin then dephosphorjlates NPAT (nuclear
factor of activated T cells), which allows it to enter the nucleus and bind to an interleukin-2
(IL-2) promoter. lnterleukin-2 stimulates the growth and differentiation of T cells, and is
therefore an important component of the immune response. Cyclosporine and tacrolimus, two
of the more commonly used immunosuppressants in kidneytransplant patients, inhibit
calcineurin activation.
(Choice A) E-cadherin is a transmembrane glycoprotein that orchestrates epithelial cell
adhesion. If this glycoprotein is lost, cell clusters may be disrupted. The loss of e-cadherin is
associated with metastasis and is a predictor of disease progression in some cancers.
(Choice B) Neurofibromin is a tumor suppressor protein encoded for bythe NP-i gene on
chromosome
17. Neurofibromin can protect against cancer because it is a key suppressor of Ras, one of the
more powerful activators of cell growth and proliferation.
(Choice D) BcI-2 (B-cell leukemia/lymphoma 2) is an apoptosis inhibitor. When BcI-2 is
overexpressed, cell death is delayed and an accumulation of indolent malignant cells occurs.
BcI-2 involvement has been implicated in follicular cell lymphomas (85% have a characteristic
t(i4;i8)(q32;q2i)translocation).
(Choice E) p53 is a tumor suppressor that causes cell cycle arrest and apoptosis. It is rendered
ineffective in the majority of cancers.
Educational Objective:
Calcineurin is an essential protein in the activation of IL-2, which promotes the growth and
differentiation of T cells. Immunosuppressants such as cyclosporine and tacrolimus work by
inhibiting calcineurin activation.
USMLE WORLD STEP 1 PHARMACOLOGY

Q NO 169: MPTP is a toxic substance that induces a Parkinsonism-like syndrome by


destroying dopaminergic neurons in the CNS. A series of experiments demonstrated that MPTP
per se is minimally toxic. Actually, it is a metabolite of MPTP, produced by MAO tjpe B and
called MPP+, that causes the neurological damage. Pretreatment by which of the following
agents would minimize the damage induced by MPTP consumption?

A. Amantadine
B. Levodopa
C. LevoçioDa/carbiçloDa
D. Selegiline
E. Pergolide

Explanation:
Selegiline is an inhibitor of MAO, type B. Since the actual causative agent of MPTP-associated
Parkinsonism-lype symptoms is formed by MAO, type B, pretreatment with selegiline can
prevent this damage to dopaminergic neurons. Clinically, selegiline is used to delaythe
progression of Parkinson disease.
(Choice A) Amantadine was designed as an antiviral influenza agent, but it is also known to
have dopaminergic activity and possibly anticholinergic action as well; therefore, it may
provide moderate improvement in patients with Parkinson disease. Amantadine and
anticholinergic agents may produce undesirable side effects such as dry mouth, nausea,
vomiting, blurring of vision, visual hallucinations, and other mental changes; they should be
used with caution in the elderly.
(Choice B) Dopamine is absent in the nigrostriatum of patients with Parkinson disease.
Dopamine itself cannot be directly administered because it is unable to cross the blood-brain
barrier. Levodopa, or L-dopa, is the immediate precursor of dopamine. It is taken up by the
brain, engulfed in the striatum by cell processes that have cell bodies in the substantia nigra;
then, in the substantia nigra, L-dopa is converted to dopamine, and released as a
n e u rotra n s m i tte r.
(Choice C) Levodopa is a dopamine analog that remains the most effective treatmentfor
symptoms of Parkinson disease. Levodopa is absorbed in the small intestine via a large,
neutral-amino-acid transporter system and is then converted to dopamine by the enzyme
dopa-decarboxjlase. Peripheral conversion to dopamine is largely responsible forthe nausea
and vomiting associated with L-dopa use. The addition of dopa-decarboxylase inhibitors
(carbidopa) that do not cross the blood-brain barrier decrease the peripheral formation of
dopamine, relieving the intense nausea and vomiting caused by levodopa alone.
(Choice E) Pergolide is a dopamine agonist which directly stimulates dopamine D2 receptors.
It provides only modest improvement in parkinsonian symptoms when used as monotherapy;
however, this modest improvement may be sufficientto delaythe introduction of levodopa by
months oryears, thus delaying levodopa-related side effects.
Educational Objective:
1. Selegiline is an inhibitor of MAO, type B and can prevent MPTP-induced damage of
dopaminergic neurons.
2. Selegiline is used clinically to delay the progression of Parkinson disease.
3. Many neurologists favor the use of combinations of selegiline, anticholinergics, and
amantadine until they no longer provide control of symptoms. Only then is levodopa/carbidopa
introduced.
USMLE WORLD STEP 1 PHARMACOLOGY

Q NO 170: A new antiarrhythmic medication, ‘Drug A1’ is tested in several animal


experiments. The action potentials of ventricular muscle cells are recorded before and after
administration of the drug. The following change in the action potential has been recorded:
The action of Drug A most closely resembles the effect of which of the following antiarrhythmic
drugs?

A. Disopyramide
B. Lidocaine
C. Propranolol
D.Diltiazem
F. Elecainide
P. Adenosine
G.Digoxin

Explanation:
This drug has slowed phase 0 depolarization and prolonged phase 3 repolarization of the
ventricular myocyte. (Recall that cardiac pacemaker cells have different action potential
curves, where there is phase 4 depolarization and phases 1 and 2 do not exist.) Disopyramide
(Choice A) is a class IA antiarrhythmic and is the only drug listed that prolongs the action
potential in cardiac muscle cells.
(Choice B) Lidocaine is a class lB antiarrhythmic, a sodium channel blocking drug that is highly
selective for rapidly depolarizing cardiac myocytes. It has very little effect on phase 0
depolarization (rapid binding and rapid release), but it does shorten phase 3 repolarization (by
blocking sodium window current) and the action potential duration as illustrated below.
(Choice C) Propranolol is a beta-adrenergic blocking agent that slows conduction through the
AV node and prolongs phase 4 depolarization in cardiac pacemaker cells. Beta-blockers do not
affectthe ventricular myocyte action potential; their primary site of action is on the AV node
and cells with automaticity.
(Choice D) Diltiazem is a calcium channel blocker and therefore a class 4 antiarrhythmic drug.
Calcium channel blockers act primarily on tissues in the AV node and on pacemaker cells (cells
with automaticily) and are most selective for rapidly depolarizing cells (use-dependent). They
do not have an appreciable effect on the ventricular myocyte action potential.
(Choice E) A class IC antiarrhythmic, flecainide blocks sodium channels with high affinity (i.e.
little use dependence). The most prominent effect of class IC drugs is drastic slowing of the
sodium-mediated phase 0 depolarization of the ventricular myocyte action potential as
illustrated below.
(Choice F) Adenosine does not modulate the ventricular myocyte action potential. It primarily
acts on the AV node where it slows conduction and decreases automaticily by hyperpolarizing
the cells there.
(Choice C) Digoxin exerts its antiarrhythmic effects by increasing vagal outputto the AV node
and conduction system, thereby slowing conduction in these tissues. It does not alter the
ventricular myocyte action potential, but it does increase intracellular calcium in ventricular
myocytes leading to increased cardiac contractility.
Educational Objective:
The class IA antiarrhythmics (quinidine, procainamide and disopyramide) are sodium channel
blocking agents that slow phase 0 of the ventricular myocyte action potential and prolong
repolarization as well as the refractory period of these cells.
USMLE WORLD STEP 1 PHARMACOLOGY

Q NO 171:A 40-year-old male is diagnosed with new-onset schizophrenia. After careful


consideration of various factors you decide to proceed treatment with a long active
fluphenazine preparation because of medication compliance issues. This treatment is most
likely to affect which of the following?

A. Orthostatic blood pressure


B. Bladder emptying
C. Sleep/wakefulness
D. Negative’ symptoms
E. Skeletal muscle tone
Traditional high potency agents — Haloperidol, fluphenazine, pimozide Typical
Traditional low potency antipsychotics — Chiorpromazine, thioridazine Atypical I ) Clozapine,
risperidone, olanzapine, quetiapine
Traditional high potency agents are more likely to cause extrapyramidal symptoms and less
likelyto cause anticholinergic and antihistamine side effects. In contrast, low potency
antipsychotics are more likely to cause anticholinergic and anti histamine side effects.
Pseudoparkinsonism and acute dystonias are treated with anticholinergic drugs such as
benztropine, diphenhydramine, and trihaxyphenidyl.
The nigrostriatal pathway primarily regulates coordination of voluntary movements. In this
pathway, dopamine inhibits the release of acetjlcholine. High potency antipsychotics act
primarily on D2 receptors of the mesolimbicmesocortical pathway but at the same time block
dopamine in the nigrostriatal pathway as well. This causes decreased dopamine action and
subsequent increased acelylcholine, leading to hyperkinetic disorders such as Parkinsonism
and acute dystonias.
(Choice A) Alpha-adrenergic blockade causes orthostatic hypotension. Along with
anticholinergic and antihistamine properties, low potency antipsychotics also block alpha
adrenergic receptors and can cause orthostatic hypotension, especially in elderly patients.
Orthostatic hypotension results in recurrent falls/syncope.
(Choice B) Anticholinergic side effects include dry mouth, blurred vision (problems with
accommodation), constipation leading to adynamic ileus, and urinary hesitancy or obstruction
(decreased bladder emptying).
(Choice C) Sedation and weight gain result from histamine blockade.
(Choice D) Negative symptoms of schizophrenia include a decrease in emotional range,
poverty of speech, loss of interests, and loss of drive. Unlike first-generation (typical)
neuroleptics, second-generation agents (clozapine, risperidone, olanzapine) improve both
positive and negative symptoms of schizophrenia.
Educational Objective:
Traditional high potency agents are more likely to cause extrapyramidal symptoms and less
likelyto cause anticholinergic and antihistamine side effects. In contrast, low potency
antipsychotics more likelyto cause anticholinergic and anti histamine side effects.
USMLE WORLD STEP 1 PHARMACOLOGY

Q NO 172:A 35-year-old Caucasian male presents to the ER complaining of generalized


motion restriction and shaking of his right hand. He admits that he was admitted to the
hospital three weeks ago after voices in his head persuaded him to leave home. Which of
thefollowing isthe besttreatmentforthis patient?

A. Levodopa/carbidopa
B. Bromocriptine
C.Benztropine
D.Diazepam
E. Haloperidol

neff this jäuent Wa seen in me n’Osp’itai trire’ë’weeKs ao, ne w’as most iiRely given an
antipsychotic for his auditory illucinations and has now developed drug-induced Parkinsonism.
In patients with drug-induced Parkinsonism, ntrally-acting antimuscarinic agents are usually
preferred. Levodopa cannot be used to treat drug-induced rkinsonism because it can
precipitate psychosis!! Dopamine agonists cannot be used either as they can :acerbate
underlying psychosis and can reduce the effects of antipsychotic medications. Anticholinergics,
such as hexjphenidyl and benztropine, are most commonly used. These drugs are particularly
effective in alleviating ?mor. These drugs are not typically first-line choices in idiopathic
Parkinson disease because they have numerous le effects, especially in elderly patients.
Instead, the more effective levodopa/carbidopa or dopamine agonists are )ically initiated in
idiopathic Parkinsonism
Pull
Iinc_JndqrI1inn rlrijjjy ciirh1 ,c hcrnrtrnnincm ncj ,nhcn1H,I chnijlri tt!,)atients with benign
perplasia (BPH) and angle closure glaucoma. ‘Urj mouth may cause some patients to develop
acute parotitis.
Levodopa is contraindicated in Parkinsonism induced by antipsychotic medications because it
can underlying psychosis.
I Bromocriptine is a dopamine agonist and can be used in idiopathic Parkinsonism; however, it
may underlying psychosis if given to patients who have drug-induced Parkinsonism.
Furthermore, it reduces )f many antipsychotics. Dopamine agonists such as bromocriptine are
the treatment of choice for
ia. Dopamine agonists normalize prolactin (PRL) levels and prevent amenorrhea-galactorrhea
in the :hese patients.
I Diazepam is a benzodiazepine and is used to treat seizures, anxiety disorders, and muscle
Ithough diazepam prevents tremors associated with delirium tremens and other conditions, it
has no effect )nism. The anxiolytic and muscle relaxation effects of diazepam are mediated
bythe a2 GABAA receptor.
Haloperidol is a first-generation antipsychotic; most of these first generation drugs are
dopamine-receptor iich is why they can cause Parkinsonian symptoms. Second-generation
antipsychotics primarily exert fects at serotonin receptors, although they do possess
dopamine-blocking activity as well. This patient’s ike symptoms most likely represent
extrapyramidal side effects from dopamine-receptor blockers, so any ic, especially a first-
generation one, would only exacerbate symptoms by introducing more of the offending
____ A rcIvirF
-r ‘ UL prostatic in
sup p u rati v
(Choice , exa c e rb at’
(Choice E exacerbat’
the effects prolacti noi majority of
(Choice E spasms.
on Parkim
(Choice E blockers inhibitory E Parkinson
anti psychc agent.
a
Educational Objective:
1. Antimuscarinic agents are usually preferred in patients with medication-induced
Parkinsonism.
USMLE WORLD STEP 1 PHARMACOLOGY

2. Levodopa is contraindicated for drug-induced Parkinsonism because it can precipitate


psychosis.
USMLE WORLD STEP 1 PHARMACOLOGY

Q NO 173:A 53-year-old Caucasian male is recovering from an acute myocardial infarction.


His total serum cholesterol level is 155 mg/dL, with an HDL of 30 mg/dL. Which of the
following agents would be most effective at increasing this patient’s HDL cholesterol?

A. Enzyme inhibitor
B. Transcription factor ligand
C. Absorption inhibitor
D. Cationic exchange resin
E. Vitamin supplement
E. Diet, exercise and weight loss

Explanation:
Subjects with low serum HDL concentration (males <40 mg!dL and females < 50 mg/dL) are
at increased risk for coronary artery disease. Raising one’s HDL cholesterol level is associated
with a reduction in the rate arthrosclerosis progression. Even small increases in HDL are
beneficial. Niacin is the most effective agent currently available for increasing serum HDL
levels (increases HDL cholesterol an average of 25-35%).
HDL is involved in reverse cholesterol transport, helping to remove cholesterol from peripheral
tissues and transport it to the liverfor metabolism. This is accomplished bytwo different
pathways. In the direct pathway, HDL delivers cholesterol esters directly to the liver via a
scavenger receptor (SR BP-1) on the hepatocyte cell membrane. In the indirectpathway,the
cholesterol in HDL istransferredto LDL and VLDL bythe cholesterol estertransferprotein (CETP).
Newer medications that inhibit OTEP increase HDL concentrations by 45-50%. One of the
drugs from this class, torcatrapib, is presently in phase three trials.
(Choice A) Statins competitively inhibit HMG-CoA reductase and significantly decrease serum
LDL concentration. However, statins have only modest effects on HDL concentration (lypically
increase HDL levels by 2-
15%).
(Choice B) Eibrates exert their lipid-lowering effects by activating the peroxisomal proliferator-
activated receptor alpha (PPAR-alpha). Eibric acid derivatives are most useful for decreasing
triglyceride levels. They increase the activilyoflipoprotein lipase, thus causing lipolysis of
lipoprotein triglycerides. Eibrates also increase HDL by 10-25% (i.e. better HDL-increasing
effects than statins, but less HDL benefit than niacin).
(Choice C) Ezetimibe is also used in the treatment of hypercholesterolemia. It decreases LDL
concentration by inhibiting cholesterol absorption from the gastrointestinal tract. Ezetimibe
does not increase HDL concentration. It is mainly used in combination with statins to achieve
greater LDL reduction. This drug is also used extensively as monotherapy in patients who
cannot take statins due to significant hepatotoxicity or myopathy.
(Choice D) Cationic exchange resins such as cholestyramine decrease LDL cholesterol by
interfering with the enterohepatic circulation of bile acids. These drugs have only minimal
effects on HDL concentration. Importantly, bile acid binding resins can significantly increase
triglyceride levels.
(Choice F) Weight loss, smoking cessation, exercise, and moderate use of alcohol are all
associated with increases in HDL levels. However, lifestyle changes exert less of a beneficial
effect on HDL concentrations than niacin. On average, a 5-10% increase in HDL is observed
with stringent lifestyle measures.
(Choice G) Omega-S fatty acids are sometimes used in the treatment of hyperlipidemia. They
can cause slight increases in HDL concentration (an effect less dramatic than that associated
with niacin).
Educational Objective:
1. Patients with familial hypoalphalipoproteinemia (low HDL) are at increased risk of
developing premature coronary artery disease. Niacin (vitamin B3) is the best agent currently
available to increase HDL cholesterol levels.
2. Statins and ezetimibe are mainly used for hypercholesterolemia (high LDL). High LDL is a
riskfactorfor atherogenesi s.
3. Eibric acid derivatives are mainly used for hypertriglyceridemia. Remember that severe
hypertriglyceridemia can cause pancreatitis.
USMLE WORLD STEP 1 PHARMACOLOGY

Q NO : 174 When tazobactam is used togetherwith piperacillin the spectrum of antimicrobial


activity is much greaterthan if either agentwere used alone. The mechanism forthis increased
efficacy is:

A. Increasing piperacillin’s abililyto penetrate bacteria


B. Decreasing destruction of piperacillin
C. Slowing the hepatic metabolism of piperacillin
D. Attacking bacteria via a different mechanism
E. Providing anaerobic coverage

Explanation:
Many Gram positive and Gram negative bacteria have acquired resistance to the penicillin
family of antibiotics. One mechanism of resistance is production of beta-lactamase, an enzjme
that disrupts the beta-lactam ring of penicillins and cephalosporins, effectively inactivating
these medications.
Certain antibiotics have combinations of chemicl arpus around the beta-lactam lana tht
Drevent beta-lactamar ! 3ccess. These antibiotics are considered beta-lactamase resistant. It
is through this mechanism that third generation cephalosporins have better activity against
Gram negative bacteria than first or second generation
cephalosporins. Susceptible beta-lactam antibiotics can also avoid destruction by beta-
lactamases by being given in combination with a beta-lactamase inhibitor, such as clavulanic
acid, sulbactam, ortazobactam.
Choice A) This mechanism describes the synergy between penicillins and aminoglycosides.
Penicillins inhibit Deptidoglycan cell wall synthesis, allowing aminoglycosides to gain access to
the cell interior where they act on the 305 subunit of the bacterial ribosome.
Choice C) This effect may occurwith drugs metabolized bythe hepatic cytochrome P-450
system. Eor example, the Tiacrolide antibiotics erythromycin and clarithromycin inhibit the p-
450 system and thereby increase the serum concentrations of drugs metabolized by this
system, such as ketoconazole.
(Choice D) Additive antimicrobial activity occurs when antibiotics from two different classes
are used in combination, :or example a fluoroquinolone (DNA gyrase inhibitor) with
vancomycin (cell wall synthesis inhibitor). However, :azobactam is a beta-lactamase inhibitor
(lacks independent antimicrobial activity), not an antibiotic, making additive antimicrobial
activity between tazobactam and piperacillin impossible.
Choice E) Tazobactam is not an antimicrobial agent and thus does not alone cover any
organisms. (Regarding anaerobic coverage, a good rule of thumb is that clindamycin is used
for anaerobic coverage above the diaphragm Nhile metronidazole is used for coverage below
the diaphragm.)
Educational Objective:
Olavulanic acid, sulbactam and tazobactam are beta-lactamase inhibitors that extend the
spectrum of penicillin—family antibiotics to include beta-lactamase producing organisms such
as S. eureus, H. influenzee, Bacteroides, and other gram-negative bacteria.
USMLE WORLD STEP 1 PHARMACOLOGY

Q NO 175: In a patient with severe retrosternal chest pain, nitroglycerin infusion results in
pain relief and a decrease in blood pressure. Which of thefollowing isthe
mostlikelyintracellularconsequence of nitrate administration intargettissues?

A. Actin phosphorylation
B. Tyrosine kinase activation
C. Myosin dephosphorjlation
D. Cyclic mononucleotide degradation

Explanation:
The common mechanism of action of all of the nitrate drugs begins with the conversion of the
drug into nitric oxide (NO) at the vascular smooth muscle cell membrane. Within the vascular
smooth muscle cell, NO stimulates guanylate cyclase to convert GTP into cyclic guanosine
monophosphate (cGMP).
This increases the cGMP concentration within the cell with resultant decreased intracellular
calcium concentration, decreased activity of myosin light-chain kinase and finally myosin light
chain dephosphorjlation and smooth muscle relaxation.
(Choice A) Actin phosphorylation has no role in smooth or skeletal muscle relaxation. In
smooth muscle a stimulus to contract leads to increased intracellular calcium which leads to
MYOSIN phosphorjlation. Once myosin is phosphorjlated, it is able to bind directly to actin and
cause contraction.
(Choice B) Tyrosine kinase is a transmembrane protein and is part of the second-messenger
system that leads to the intracellular effects of insulin and insulin-like growth factor. When
stimulated, lyrosine kinase will phosphorylate lyrosine residues on intracellular proteins
leading to a poorly elucidated cascade of intracellular events leading to the end effects of
insulin.
(Choice D) Cyclic mononucleotide degradation is incorrect because nitrates lead to an
INCREASE in cGMP. Increases in cGMP mediate the effects of endothelium-derived relaxing
factor (nitric oxide) and atrial natriuretic peptide.
Educational Objective:
Nitrate drugs mimic the action of endothelial derived relaxing factor (nitric oxide, NO). They
are transformed to NO at the vascular smooth muscle cell membrane which leads to increased
cGMP, decreased intracellular calcium and myosin dephosphorjlation.
USMLE WORLD STEP 1 PHARMACOLOGY

Q NO 176: A 64-year-old male presents to your office with six month history of urination
difficulty, with frequent straining and dribbling. The symptoms have increased over time and
seem to affect his quality of life significantly. You proceed with drug therapy in this patient,
which brings moderate symptomatic relief. However, he notices an increase in his hair growth.
Which of the following best explains the effect of the drug in this patient?

A. Decreased Leydig cell stimulation


B. Decreased Leydig cell androgen synthesis
C. Decreased peripheral androgen aromatization
D. Decreased peripheral androgen reduction
E. Impaired ligand-receptor interaction
E. Impaired second messenger action

Explanation:
Pinasteride is an anti-androgen that inhibits the action of 5-alpha-reductase. This enzyme is
responsible for peripheral conversion of testosterone to its active metabolite
dihydrotestosterone (DHT). DHT has a much higher affinily to the testosterone receptor and
mediates the majorily of testosterone effects, including development of male external genitalia
and prostate enlargement. Pinasteride counteracts these effects of DHT and is used for
treatment of benign prostatic hyperplasia (BPH).
Patients with androgenetic alopecia have higher levels of 5-alpha-reductase and a higher
quantity of androgen receptors. Thus reducing the 5-apha-reductase level with finasteride is
useful for treating androgenetic alopecia.
See the diagram below for the summary of action of major anti-androgens.
(Choice A) Decreased Leydig cell stimulation by LH occurs during treatment by GnRH
(leuprolide, goserelin, nafarelin and histrelin). Einasteride does not affect LH synthesis by
anterior pituitary.
(Choice B) Ketoconazole is aweakanti-androgenthatdecreases synthesis of testosterone in
Leydig cells of the testes. It also inhibits steroid hormone production bythe adrenals.
(Choice C) Decreased peripheral androgen aromatization refers to anastrozole, an aromatase
inhibitorthat blocks estrogen production selectively. Anastrozole is an effective treatment for
postmenopausal women with breast cancer in whom the greatest source of estrogen is the
conversion of endrostenedione, produced in the adrenal glands, to estrone in liver, muscle,
and fat, through aromatization.
(Choice E) Interaction of testosterone and dihydrotestosterone (DHT) with their receptors is
inhibited byflutamide, cyproterone acetate, and spironolactone.
(Choice F) Finasteride does not affect second messenger action.
Educational Objective:
Finasteride is a 5 cc-reductase inhibitorthat suppresses peripheral conversion of testosterone
to DHT. Itis used for treatment of benign prostatic hyperplasia (BPH) and male baldness.
USMLE WORLD STEP 1 PHARMACOLOGY

Q NO 177: A medical student is observing a pharmacology experimentwhere drug A is being


intravenously administered to a pregnant dog. Some parameters that are being recorded
during the experiment include heart rate, blood pressure, pupil size, and uterine contractions.
The following diagrams illustrate the measured parameters and observed changes after
Which of the following best characterizes drug A?

A. Beta-adrenergic agonist
B. Alpha and beta-adrenergic agonist
C. Alpha-adrenergic agonist
D. Beta-adrenergic antagonist
F. Alpha-adrenergic antagonist and beta-adrenergic agonist

Explanation:
Although the most pronounced effects of adrenergic agonists are those on the heart and
vasculature, these drugs also affect many other organs. The graph above illustrates the effects
of adrenergic stimulation on the eye and uterus. The pupillary dilator muscle of the eye
possesses al receptors, and the stimulation of these al receptors results in contraction of the
pupillarj dilator muscle with subsequent dilation of the pupil (mydriasis). Ophthalmic
preparation of phenylephrine (selective a-adrenergic agonist) is used to dilate the pupil before
ophthalmoscopic examination.
Adrenergic receptors of the uterus belong to the j32 group whose stimulation leads to uterine
relaxation (an effect called tocolysis) which is used in obstetrics to deter premature labor.
Adrenergic 132 agonists such as ritodrine and terbutaline are used for this purpose. The drug
in question, therefore, displays both a and j3-adrenergic effects. See the table for the
summary of end-organ effects of adrenergic agonists.

Receptor Target organ Effect of stimulation Example


Epinephrine
Peripheral tSBP
Norepinephrine
vasculature Contraction of external urethral sphincter
Alpha-i
Bladder Mydriasis (contraction of pupillary dilator
Phenylephrine
Eye muscle)
Methoxamine
Epinephrine
Dopamine
Beta-i Heart tHR, contractility and conductance
Dobutami ne
Isoproterenol
Peripheral
vasculature Isoproterenol
Vasodilatation, i.-DBP
Beta-2 (skeletal muscle) Terbutaline
Bronchodilatation Relaxation (tocolysis)
Bronchi Ritodrine
Uterus

(Choice A) Beta-adrenergic agonists stimulate j32-adrenergic receptors of the uterus and


diminish uterine contractions. They do not, however, affect al-receptors of the eye and would
not produce mydriasis.
(Choice C) Alpha-adrenergic agonists would produce mydriasis (al effect), but not uterine
relaxation (j32 effect).
(Choice D) Beta-adrenergic antagonists block j32 receptors of the uterus and enhance uterine
contraction. They do not affect al-adrenoreceptors of the pupillarj dilator of the eye.
(Choice E) Alpha-adrenergic antagonist and beta-adrenergic agonistwould cause miosis
(pupillarj constriction) and uterine relaxation.
Educational Objective:
Inhibition of uterine contractions (tocolysis) is a result of 132 adrenergic receptor stimulation.
Alpha -l receptor stimulation causes contraction of the ocular pupillary dilator muscle, resulting
in mydriasis (pupillarj dilation).
USMLE WORLD STEP 1 PHARMACOLOGY

Q NO 178: A 50-year-old male begins treatment with bupropion for depression. After three
weeks of treatment. he reports that his symptoms have improved somewhat but he still feels
‘really down’ from time to time. You contemplate increasing the dose of his medication. Which
of the following side effects is your biggest concern?

A. Urinary retention
B. Cardiac arrhythmias
C. Seizures
D. Orthostatic hypotension
E.Sedation
F. Sexual dysfunction

Explanation:
Seizures are a feared side effect of bupropion therapy, especially when the drug is given at
high doses. The risk is even greater in patients with a pre-existing seizure disorder, bulimia
and/or anorexia. Bupropion was withdrawn from the market several years ago because of
reports of increased seizure activity in patients with bulimia. The drug has since returned to
the market for treatment of depression, though extreme caution must be exercised when
treating patients with bulimia or anorexia.
(Choice A) Urinary retention may be seen with tn cyclic antidepressants due to anticholinergic
effects on the bladder musculature.
(Choice B) Cardiac arrhythmias are a feared side effect of the tn cyclic antidepressants, which
may produce quinidinelike prolongations of the QRS and QT intervals.
(Choice D) Orthostatic hypotension is a side effect of both the monoamine oxidase inhibitors
(especially phenelzine) and the tn cyclic antidepressants.
(Choice E) Sedation is common with tricyclic antidepressants because of their antihistamine
effects.
(Choice F) Sexual dysfunction is a relatively common side effect of the SSRls, which limits
their use for many patients. Bupropion can be used as an alternative in these patients.
Educational Objective:
Remember — drugs that may cause seizures:
Bupropion (antidepressant)
Isoniazid (anti-tuberculosis drug, if given without pyridoxine)
Imipenem (antibiotic)
USMLE WORLD STEP 1 PHARMACOLOGY

Q NO 179: Rifampin is an antibiotic effective against many Gram-positive and Gram-negative


bacteria. Which of the following would be an indication for rifampin monotherapy?

A. Tuberculosis caused by susceptible bacteria


B. Atypical mycobacterial infection
C. Penicillin-resistant pneumococcal meningitis
D. Staphylococcal endocarditis
E. Meningococcal exposure

Explanation:
Rifampin is well known to induce resistance in many bacterial pathogens when used as
monotherapy. Resistance occurs through spontaneous genetic mutations in the bacterial DNA-
dependent RNA polymerase. Currently, rifampin is used as a component of multiagenttherapy
in the treatment of mycobacterial infections or leprosy and as prophylactic monotherapy in
those exposed to H. influenzae or N. meningitidis.
Rifampin is an inducer of the CYP45O system in the liver (rifAMPin AMPlifies CYP45O), so other
drugs processed through this mechanism will be metabolized more rapidly when taken in
conjunction with rifampin.
(Choice A) When treating active tuberculosis, rifampin is administered with isoniazid and
pyrazinamide, ethambutol, and/or streptomycin.
(Choice B) Alypical mycobacterial infections — such as those caused by the Mycobacterium
aviurn complex (MAC) — are similarto infections with Mycobecterium tuberculosis in that
combination therapy is necessary. The recommended treatment regimen for MAC includes a
macrolide antibiotic (eg, clarithromycin or azithromycin) coupled with rifampin and
ethambutol.
(Choice C) Penicillin-resistant pneumococcal meningitis is effectivelytreated with a combination
of ceftriaxone and vancomyci n.
(Choice D) Staphylococcal endocarditis is treated aggressivelywith intravenous penicillin
antibiotics (eg, nafcillin) and an aminoglycoside (eg, gentamicin) for a synergistic effect. If
MRSA is suspected or prevalent in the communily, then treatmentwith vancomycin and
rifampin oran aminoglycoside is recommended. Rifampin is inappropriate monotherapy in the
treatment of staphylococcal endocarditis.
Educational Objective:
Rifampin is the preferred prophylaxis for persons who have been definitively exposed to N.
meningitidis. Rifampin can eliminate the carrier state as well as prevent active infection in
those who have been exposed to N. meningitidis. Rifampin is also used as a component of
multi-agent therapy for both lypical and atypical mycobacterial pulmonary infections, leprosy,
and staphylococcal endocarditis, When used alone, bacteria rapidly acquire resistance to
rifampin through spontaneous genetic mutations of the bacterial DNA-dependent RNA
polymerase.
USMLE WORLD STEP 1 PHARMACOLOGY

Q NO 180: A 28-year-old male experiences fever and muscle rigidily soon after a major
surgical procedure. His blood pressure is 190/1 00 mmHg and his heart rate is 130/mm.
Physical examination reveals muscle stiffness and cyanotic skin mottling. The appropriate
treatmentforthis patient’s condition includes an agentthat affects which of the following
physiologic processes?

A. Alpha-receptor mediated sympathetic hyperactivily


B. GABA-mediated neuron hyperpolarization
C. Acelylcholine release in neuromuscular juncti on
D. Ligand-receptor interaction in cholinergic synapses
E. Acetylcholine degradation in cholinergic synapses
F. Intracellular calcium release in skeletal muscles

Explanation:
Malignant hyperthermia is a syndrome that occurs due to hypersensitivity of skeletal muscles
to inhalation anesthetics (especially halothane) and muscle relaxant succinylcholine.
Susceptibililyto malignant hyperthermia is inherited as an autosomal dominant trait. In the
majority of cases it is related to the defect on rjanodine receptors of sarcoplasmic reticulum.
Ryanodine receptor is located on the surface of the sarcoplasmic reticulum of skeletal muscles
and is a calcium channel. It releases small amounts of calcium in the cytoplasm of the muscle
fiber during muscle contraction. Abnormal rjanodine receptors release large amounts of Ca
after exposure to anesthetic. Excess of free Ca in the cytoplasm of muscle fibers stimulates its
ATP-dependent reuptake by sarcoplasmic reticulum. Excessive consumption of ATP generates
heat; loss of ATP along with high temperature induces muscle
damage. Rhabdomyolysis leads to release of potassium, myoglobin and creatine kinase into
circulation.
Clinically, malignant hyperthermia presents with fever and muscle rigidity soon after surgery
under general anesthesia. Tachycardi, hypertension1 hyperkalemi a and myoglobi nemi a are
characteristic. Malignant hyperthermi a is a life-threatening condition and should be treated
promptly. Dantrolene is a muscle relaxant effective in malignant hyperthermia. It acts on
ryanodine receptor and prevents further release of Ca into the cytoplasm of muscle fibers.
(Choice A) a-adrenergic antagonists (such as phentolamine and phenoxybenzamine) inhibit a-
receptor mediated sympathetic activity. These medications are used for treatment of
pheochromocytoma.
(Choice B) Barbiturates and benzodiazepines affect GABA receptors and facilitate GABA-
mediated neuron hyperpolarization. Dantrolene does not affect GABA action.
(Choice C) A number of neuromuscular blocking agents interfere with acelylcholine action in
the neuromuscular junction. Nondepolarizing neuromuscular blockers prevent acetylcholine
binding to the receptor, while depolarizing blockers causes constant stimulation of the
receptor.
(Choice D) Cholinergic antagonists (such as atropine, scopolamine and ipratropium) bind to
cholinergic receptors and prevent their interaction with acelylcholine. This mode of action is
not characteristic for dantrolene.
(Choice E) Acelylcholine degradation in cholinergic synapses is affected by acelylcholinesterase
inhibitors (physostigmine, neostigmine, pyridostigmine and edrophonium). Neostigmine and
pyridostigmine are used for treatment of myasthenia gravis, while edrophonium is useful for
its diagnosis.
Educational Objective:
Malignant hyperthermia occurs after administration of inhalation anesthetics and/or
succinylcholine to genetically susceptible individuals. It is treated with dantrolene that blocks
ryanodine receptors and prevents release of Ca into the cytoplasm of skeletal muscle fibers.
USMLE WORLD STEP 1 PHARMACOLOGY

Q NO 181: A 65-year-old Caucasian male presents to your office with increasing muscle
rigidity and gait instability. He was diagnosed with Parkinson disease iwo years ago and has
been treated with selegiline. His past medical hi stow is otherwise insignificant. His father
suffered from Parkinson disease and his mother died of an extensive stroke. You consider
switching to levodopa to control this patient’s symptoms adequately and discuss the adverse
effect profile of levodopa with him. Which of the following adverse effects is most likely to
persist after adding carbidopa to levodopa treatment?

A. Nausea and vomiting


B. Tachyarrhythmias
C. Hotflashes
D. Postural hypotension
E. Anxiety and agitation

Explanation:
Anxiety and agitation are central effects of dopamine and are caused by L-dopa, regardless of
whether carbidopa is added to levodopa treatment. In fact, anxiety and agitation can be
increased because more dopamine is available to the brain. (Adding carbidopa inhibits the
peripheral conversion of levodopa, making more of it available to the brain.) High-dose
combination levodopa/carbidopa can cause various behavioral changes such as anxiety,
agitation, insomnia, confusion, delusions, and hallucinations. Reducing the dose is helpful in
alleviating these adverse effects. If reduced dosage is not possible, atypical antipsychotics,
such as clozapine, may be helpful.
(Choice A) Nausea and vomiting are seen in up to 80% of patients when levodopa is used
alone. Dopamine stimulates the emetic center located in the brainstem (which is outside of the
blood brain barrier). Adding a dopa decarboxjlase inhibitor reduces nausea and vomiting and
significantly increases patient compliance. Most patients develop tolerance to nausea after
several months.
(Choice B) Tachyarrhythmias, such as ventricular extrasystoles and atrial fibrillation, are
attributed to the increased peripheral formation of catecholamines. Adding a peripheral
decarboxjlase inhibitor reduces these arrhythmias.
(Choices C and D) Postural hypotension and hot flashes also result from the peripheral
conversion of levodopa to dopamine, so a peripheral decarboylase inhibitorwould decrease
these problems.
Educational Objective:
Adding carbidopa can reduce most of the peripheral side effects of levodopa. However,
behavioral changes from levodopa can actually worsen with addition of carbidopa because
more dopamine becomes available to the brain.
USMLE WORLD STEP 1 PHARMACOLOGY

Q NO 182: A 67-year-old Caucasian male with metastatic prostate cancer is treated with oral
morphine for chronic back pain. He complains of decreased efficacy of the drug in controlling
the pain despite increasing doses. Which of the following drugs may precipitate opioid
withdrawal symptoms if used in this patient?

A. Methadone
B. Codeine
C. Meperidine
D.Fentanyl
E. Levorphanol
F. Orjmorphone
G.Pentazocine

Explanation:
Thetherapeutic effects of opioid narcotics depend highlyontheirbinding affinityto differenttypes
of opiate receptors. A majority of the clinically available opioids work by selectively binding to
mu receptors and exerting their effects by mimicking the actions of endogenous opioid
peptides.
Pentazocine is an opioid narcotic with partial agonist activity and weak antagonist activity at
mu receptors. Itwas originally designed to provide effective analgesia with little to no abuse
potential.
When used in combination with morphine or other opioids that are selective mu receptor
agonists, pentazocine can competitively inhibit mu receptors and produce antagonistic effects
reducing opioid analgesic effects. In addition to this, when pentazocine is given to patients
dependent on morphine, withdrawal symptoms can also occur secondarjto its antagonistic
effects ay mu receptors.
(Choices A, B, C, D, E, and F) All agents are opioid narcotics that are selective mu receptor
agonists. Use of these agents in equal analgesic doses in morphine dependent patients should
not cause withdrawal.
Educational objectivç;3 A I
3jority of opioici narcOtics aië selective mu receptor agonists that work by binding to mu
receptors and mimicking thE
tiects of endogenous opioid peptides. Pentazocine is an opioid narcotic specifically designed to
produce an
esic effectswith littleto no abuse potential. Pharmacologically, itworksthrough partial
agonistactivi1yand cinnict crIrti\?jji ajJnLIjjfntnrc Poraiico nf tc woLz 2nt2nnnictic offctc it r
c1a nitnthdcpII ents whi
we dependent or tolerant to morphine or other opioids.
To assess bioavailabilily of a drug, investigators administered it intravenously to a volunteer
and measured the plasma concentration of the drug overtime. The investigators subsequently
administered the same dose of the drug orally to the same person and similarly measured the
plasma concentration of the drug overtime. The curves were plotted on the graph below.
USMLE WORLD STEP 1 PHARMACOLOGY

Q NO 183 : Which of the following is the best determinant of oral bioavailability of the drug in
this person?

A. Slope of the IV curve divided by the slope of the oral curve


B. Area under the oral curve
C. Maximal concentration atthe peak of the oral curve
D. Rate of elimination for the IV curve divided by rate of elimination for the oral curve
E. Area underthe oral curve divided by area underthe IV curve

Explanation:
Bioavailability refers to the fraction of administered drug that reaches the systemic circulation
in a chemically unchanged form. In mathematical calculations, bioavailabilityr is represented
by the letter F, which is equal to the percentbioavailabililydivided by 100. Generally1 an
intravenouslyadministered drug has 100% bioavailability(F =
1). For other modes of drug administration, such as oral, intramuscular, inhalational, or
topical, the bioavailability is usually less than 100%, so F is typically less than 1.
The bioavailabilily of a drug after non-intravenous administration can only be determined after
conducting a study of the pharmacokinetics of the drug. This study must plotthe plasma drug
concentration as a function of time fora given dose of drug administered by both the
intravenous route and the other route being studied (i.e. P0, SQ. IM, PR. inhalation,
SL,transdermal). Oral bioavailabilily(F) of the drug in question can then be calculated by
dividing the area under the curve (AUC) of oral administration by the AUC of intravenous
administration, assuming that the dose administered by each route is equal. If the
administered doses are not equivalent, the calculation may be carried out as follows:
F = (area under oral curve x IV dose) I (area under IV curve x oral dose)
In the case of oral administration, bioavailabilily is affected by gastric acidity and motility, the
capability of the drug to be absorbed by the gut, the presence of material such as food or
other drugs in the gut, and first-pass metabolism by the liver.
(ChoiceA) While dividingthe slope of the IV curve bythe slope of the oral curvewould produce
afraction,thatfraction would be greaterthan one, which is not a sensible answerto this question
as the oral bioavailability can never be higher than the IV bioavailabilily.
(Choice B) The area under the oral curve simply represents the total amount of drug that was
absorbed by the body and made available in the systemic circulation from the time of
administration to the time of complete elimination.
(Choice C)The maximal concentration atthe peak of the oral curve represents the peak plasma
level of drug after administration of a given dose. This peak concentration is dose-dependent
and has no bearing on bioavailabilily.
(Choice D)The rate of eliminationforthelV curve orforthe oral curve hasto dowiththe mode of
elimination used by the bodyto eliminatethe drug (zero-orderkinetics,first-orderkinetics). If a
constantfraction of drug is eliminated per unit time based on plasma concentration, then first
order elimination is occurring. If a constant amount of drug is eliminated for every unit time
regardless of plasma concentration, then zero-order kinetics is occurring. However, elimination
is notwhatthe question is asking about, since bioavailabililyis dependent on the process of
absorption.
Educational Objective:
Bioavailabilily refers to the fraction of administered drug that reaches the systemic circulation
in a chemically unchanged form. Bioavailability for a drug administered by a non-intravenous
route is always less than 1. It can be determined by examining a graph of plasma
concentration versus time and then applying the formula:
F = (AUC oral x dose IV) I (AUC IV x dose oral)
USMLE WORLD STEP 1 PHARMACOLOGY

Q NO 184: A 66-year-old male is treated for stable angina and long-standing hypertension.
Physical examination findings are within normal limits. His laboratory values are:
Sodium 140 mEq/L
Potassium 5.6 mEq/L
Chloride 100 mEq/L
Bicarbonate 22 mEq/L
Calcium 9.5 mg/dL
Creatinine 1.0 mg/dL
Which of the following drugs in combination with lisinopril is most likely to account for this
patient’s laboratory abnormality?

A. Verapamil
B. Metoprolol
C. Indapamide
D. Hydrochlorothiazide
E. Eurosemide
E.Amiloride
G. Amlodipine

Explanation:
ACE inhibitors work by blocking the conversion of angiotensin Ito angiotensin Il. This reduces
systemic vasoconstriction and decreases aldosterone secretion, allowing for effective blood
pressure reduction. By decreasing aldosterone secretion, ACE inhibitors increase Na÷
excretion and potassium retention. This effect is responsible for the hyperkalemia often seen
in patients taking ACE inhibitors. Significant hyperkalemia is not common in patients with
normal renal function who are nottaking other drugs that can elevate potassium. However, in
patients with renal insufficiency, in patients taking potassium sparing diuretics (amiloride,
triamterene, spironolactone), or in patients taking potassium supplements, the riskfor
hyperkalemia is much greater. When initiating therapy, it is recommended to check potassium
levels at baseline, after any dose adjustments, and periodicallythereafter. Other side effects of
ACE inhibitors include cough, first-dose hypotension, and angioneurotic edema.
(Choice A) Verapamil is a calcium channel blocker. It works by blocking calcium channels in
the vascular smooth muscle and the myocardium, resulting in relaxation of smooth muscles
and coronary arteries. Common side effects include constipation, peripheral edema, and
bradycardia. Verapamil does not induce any electrolyte abnormalities.
(Choice B) Metoprolol is a selective beta-i blocker. It works by blocking beta-i receptors in the
myocardium resulting in decreased contractility and heart rate. Common side effects include
diiness and bradycardia. More severe and less common side effects include AV block,
insomnia, depression, bronchospasm, and sexual dysfunction. Metoprolol has no effect on
electrolytes. Although beta-2 agonists do stimulate N+-K+ ATPase pumps causing
hypokalemia, beta -i blockade does not cause hyperkalemia.
(Choices C, D, and E) Both indapamide and hydrochlorothiazide are thiazide type diuretics
thatwork by blocking Na+-CI- symporters in the distal tubules resulting in increased sodium
and water excretion. Furosemide is a loop diuretic and works by blocking Na+K+-2C1-
symports in the ascending limb of the loop of Henle. All agents cause hypokalemi a.
(Choice G) Amlodipine is a calcium channel blocker from the dihyrdopyridine class. It works by
blocking calcium channels in the vascular smooth muscle and the myocardium, resulting in
relaxation of smooth muscles and coronary arteries. Because dihydropyridines cause
significant peripheral vasodilation, they cause mild to moderate tachycardia. They can also
cause peripheral edema. They have no effect on electrolytes.
Educational Objective:
Decreased angiotensin II reduces vasoconstriction and decreases aldosterone release. The
decreased aldosterone secretion observed with ACE inhibitors causes increased potassium
retention and can potentially lead to hyperkalemia. Hyperkalemia secondarjto ACE
inhibitortherapyis most common in patients with renal insufficiency1 in patients taking
potassium sparing diuretics (amiloride, triamterene, spironolactone), and in patients taking
potassium supplements.
USMLE WORLD STEP 1 PHARMACOLOGY

Q NO 185: A 79-year-old man develops visual abnormalities, nausea, and anorexia. He has a
history of atrial fibrillation. forwhich he takes a weight-adjusted average adult dose of digoxin.
Age-related changes in which of the following factors most likely accounts for this patient’s
symptoms?

A. Intestinal absorption
B. Liver enzyme activily
C. Renal clearance
D. Plasma protein binding
E. Muscle mass

Explanation:
This patient is experiencing symptoms classically associated with digoxin toxicity, which
include visual changes and gastrointestinal disturbances. Digoxin is a renally cleared agent
with a half-life of approximately 1 .5 days in patients with normal kidneyfunction. As patients
age, renal function decreases progressively. This decrease in renal function is often not
accompanied by a concomitant rise in serum creatinine, since creati nine is derived from
muscle tissue and lean body mass also decreases with age. Thus, all elderly patients (even
those with a normal serum creatinine) should receive reduced doses of renally cleared
medications to prevent toxicity.
(Choice A) Intestinal absorption affects the dose of digoxin that reaches the bloodstream.
Some preparations of digoxin may have a higher bioavailabilitythan others, and some patients
may carry gut bacteria that degrade digoxin before it has the ability to cross into the
bloodstream.
(Choice B) Liver enzyme activity plays little role in the elimination of digoxin, as this agent is
primarily excreted unchanged by the kidney.
(Choice D) Plasma protein binding of digoxin is not as significant as renal clearance when
considering toxicity. Typically less than one4hird of the digoxin that gains access to the blood
stream will be protein-bound; the remainder of the dose circulates unbound.
(Choice E) The patient’s lean muscle mass is importantto considerwhen dosing digoxin based
on weight, as this serves as the primary site where digoxin is stored in the body. Lean body
mass does tend to decrease in an agerelated fashion, but age-related renal insufficiency is
most directly related to toxicity of this agent in the elderly.
Educational Objective:
Digoxin is a cardiac glycoside that is predominantly cleared bythe kidneys. Elderly patients
typically exhibit agerelated renal insufficiency, even in the presence of normal creatinine
levels. The dose of digoxin must be reduced in these patients in order to prevent toxicity.
USMLE WORLD STEP 1 PHARMACOLOGY

Q NO 186: A 64-year-old female is admitted to the hospital with transient numbness and
tingling of the right arm, which resolves completely within 20 minutes. MRI of the brain,
carotid Doppler and echocardiogram of the heart are all within normal limits. Her lipid profile is
also within normal range. She has no other medical problems. She is started on a medication
to prevent similar attacks in the future. If she continues to take this medication long-term,
which of the following side effects is she most likely to eventually experience?

A. Eecal blood loss


B. Nasal polyps
C. Hyperpnoea and vertigo
D. Skin rash
E. Glucose intolerance
F. Gouly arthritis

Explanation:
Low dose aspirin (81-325 mg per day) is commonly used to prevent primary and secondarj
coronary artery events and ischemic strokes, Aspirin works by irreversibly acetjlating and
inhibiting the cycloorjgenase (COX) enz/mes. At low doses, aspirin predominantly inhibits COX
1, whereas high-dose aspirin inhibits both COX 1 and COX 2 enzymes.
Platelets and vascular endothelial cells express COX I and COX 2. Both cell types process
prostaglandin H2 (PGH2), platelets to produce thromboxane A2 and vascular endothelial cells
to produce prostaglandin 12 (PGI2). Thromboxane A2 causes platelet aggregation,
vasoconstriction and proliferation of vascular smooth muscle cells. In contrast, PGI2 inhibits
platelet aggregation, causes vasodilation and inhibits proliferation of smooth muscle cells.
Permanent inactivation of COX 1 by aspirin may lead to bleeding complications. At least two
distinct COX 1- dependent mechanisms contribute to the increased risk of upper
gastrointestinal bleeding associated with aspirin therapy: inhibition of thromboxane A2-
mediated platelet aggregation and impairment of PGE2- and PGI2-dependent protection of the
gastrointestinal mucosa. The irreversible inhibition of platelet aggregation occurs with low as
well as high doses1 whereas impairment of gastric cytoprotection occurs onlywith high doses.
The risk of upper gastrointestinal bleeding is increased 2 to 34o1d even with low-dose aspirin.
The relative risk increases when high- dose aspirin is used because there is loss of
cytoprotection as well as inhibited platelet aggregation. Proton pump inhibitors can help
reduce the risk for upper gastrointestinal bleeding in patients taking aspirin.
(Choice B) Samter’s triad is the triad of asthma, aspirin hypersensitivily (nasal symptoms,
bronchospasm, facial flushing) and nasal polyposis that occurs in 10% of asthmatics treated
with aspirin. These symptoms occur due to the overproduction of leukotrienes that occurs
when aspirin blocks the cyclooxjgenase pathway causing arachidonic acid metabolites to be
diverted into the lipoxjgenase pathway. Other NSAIDs can cause similar symptoms.
(Choice C) Very high-dose aspirin can cause salicylism, a condition characterized by vertigo,
tinnitus, and hearing loss. Atverj high doses, aspirin also stimulates respiratory drive. causing
hyperpnea. Hyperpnea can cause respiratory alkalosis and salicylate accumulation, leading to
metabolic acidosis.
(Choice D) Some patients may experience skin rash with aspirin therapy, butthis complication
is much less common compared to fecal blood loss.
(Choice E) Glucose intolerance has not been reported with aspirin therapy.
(Choice F) In lower doses (up to 2g per day), aspirin may increase uric acid levels. Higher
doses have a uricosuric effect, causing serum uric acid to decrease.
Educational Objective:
Increased fecal blood loss is the most common side effect of aspirin. The relative risk of
gastrointestinal bleeding is increased when high-dose aspirin is used because there is loss of
gastric cytoprotection in addition to impaired platelet aggregation.
USMLE WORLD STEP 1 PHARMACOLOGY

Q NO 187: A 44-year-old female is treated with propranolol for recently diagnosed


hypertension. Several months after initiation of the therapy her blood pressure decreases to
normal values. She is compliant with her medication and experiences no serious side effects.
Which of the following is the most likely combination of changes in response to this patient’s
treatment?
Renin Angiotensin I Angiotensin II Aldosterone Bradykinin

A. Increase Increase Decrease Decrease Increase


B. Increase Increase Increase Decrease Increase
C. Increase Increase Increase Decrease No change
D. Increase Increase Decrease Decrease Decrease
E. Increase Decrease Decrease Decrease Decrease
F. Decrease Decrease Decrease Decrease No change

Explanation:
USMLE WORLD STEP 1 PHARMACOLOGY

Q NO 188: A new drug has been developed that is found to be an effective sulfhydrjl group
donor. The drug would be most useful in the treatment of overdose for which of the following
medications?

A. Iron
B. Aspirin
C. Acetaminophen
D. Amitriptyline
E.Barbiturates
E.Opiates

Explanation:
Acetaminophen is available over-the-counter; therefore, patient overdose and toxicityr are
common. Toxicityr is likely to occur with a single dose of 250-mg/kg bodyweight or a
cumulative 24-hour dose of more than 12 g/day.
therapeutic f’abolized in the liver by sulfation and glucuronide conjugation. - remainder is
eliminated in the livervia oxidation bythe cytochrome- P450 system and by urinary excretion
of unmetabolized drug. The metabolite of the cytochrome -P450 oxidase pathway is called N-
acetyl-p-benzoquinoneirr (NAPQI), a toxic and highly reactive compound. In therapeutic
doses, a small amount of NAPQI is generated, whic metabolized by hepatic glutathione into
non-toxic compounds. With acetaminophen toxicitl, sulfation and glucuror conjugation in the
liver are saturated and excess NAPQI is formed by the cytochrome- P450 enzymes. With deple
of glutathione, NAPQI is free to interactwith the cells causing hepatocellular injury and
centrilobular necrosis.
Administration of N-acelyl cysteine (NAC) is used fortreating acetaminophentoxicitj. NAC acts
as a glutathione substitute and binds to the toxic metabolite. NAC also provides sulfhydryl
groups to enhance the non-toxic sulfatio elimination of acetaminophen.
(Choice A) Deferoxamine is a specific chelator of iron and is the antidote of choice in serious
iron poisoning.
(Choice B) Management of salicylate intoxication includes gastric decontamination, measures
to decrease gastrointestinal absorption, correction of fluid and electrolyte imbalance, and
increasing urinary excretion by alkalinization of urine.
(Choice D) Cardiac arrhythmias are the most common cause of death in patients with tricyclic
antidepressant poisoning. Administration of sodium bicarbonate is very useful in improving
cardiovascular manifestations oftricy anti depressant toxicily.
(Choice E) The characteristic feature of barbiturate poisoning is depression of the central
nervous system. Treatr includes supportive care, gastric decontamination, decreasing
gastrointestinal absorption, and forced alkaline diuresis (combination of diuretic and urinary
alkalinization).
(Choice F) High doses of opioids can be lethal. Clinical features include respiratory depression,
pupillary miosis, bradycardia, hypothermia, and coma. Treatment of opioid intoxication is
administration of an opioid antagonist, e naloxone or naltrexone.
Educational Objective:
Acetaminophen toxiciw can be effectively treated by sulfhydryl group supplementation. N-
acetjl cysteine provides sulfhydryl groups. NAC also acts as a glutathione substitute and binds
to the toxic metabolite.
USMLE WORLD STEP 1 PHARMACOLOGY

Q NO 189: A 10-year-old African American male treated with cromolyn for bronchial asthma
experiences a significant decrease in the frequency of his symptoms. The drug used in this
patient most likely affects which of the following?

A. A
B.B
C.C
D.D
E. E

Explanation:
The mast cell plays a pivotal role in the pathophysiology of bronchial asthma. Mast cells help
to initiate the acute response to allergens and other nonimmune stimuli, such as exercise, cold
exposure, chemical irritants, environmental pollutants, and viral infections. Other cells, such
as macrophages, eosinophils, T-lymphocytes, epithelial cells, fibroblasts and nerve cells also
contribute to the pathophysiology of bronchial asthma.
Acute bronchial constriction begins when an allergen interacts with IgE antibody attached to a
mast cell. This interaction stimulates the release of a number of chemical mediators, which
together cause bronchial constriction, bronchial wall edema, chemoattraction of other
inflammatory cells, and increased mucus production. Even in asymptomatic asthmatics, airway
inflammation occurs. Important chemical mediators liberated from mast cells include
histamine, leukotrienes, prostaglandins, platelet activating factor, interleukin-4, interleukin-5,
tumor necrosis factor alpha, and transforming growth factor beta.
Cromolyn and nedocromil do not influence bronchial constriction directly. Instead, they inhibit
mast cell degranulation, and in this way preventthe release of chemical mediators. These
drugs are typically used forthe prevention of acute attacks, rather than to treat acute
bronchoconstriction itself. The inhibition of mast cell degranulation by cromolyn and
nedocromil occurs independently of any stimuli. These agents are very effective prophylactic
agents for patients with seasonal symptoms, aspirin hypersensitivity, and exercise-induced
asthma, though, in general, they are less efficacious than inhaled glucocorticoids.
(Choice A) Anti-IgE antibody, omalizumab, inhibits IgE binding to mast cells. This antibody
binds mast cells without provoking mast cell degranulation. Clinical trials have shown that
long-term use of omalizumab by asthmatic subjects lowers IgE levels and significantly relieves
allergen-induced bronchial constriction.
(Choices B and C) Many of the chemical mediators involved in bronchial asthma are generated
from arachidonic acid. Cleavage of membrane phospholipid by the enzyme phospholipase A2
leads to the formation of arachidonic acid, which is in turn converted into prostaglandins and
leukotrienes by the cyclooxjgenase and lipoxjgenase pathways, respectively. By inhibiting
phospholipase A2 and downstream chemical mediators, glucocorticoids decrease airway
hyperresponsiveness and inflammation. Zileuton is a selective inhibitor of the liporjgenase
pathway that leads to decreased formation of leukotrienes.
(Choice E) Zafirlukast and montelukast are leukotriene D4 receptor antagonists. Thus,
theywork by blocking the receptors for chemical mediators involved in asthma. These drugs
are typically used for chronic asthma prophylaxis, ratherthan for acute exacerbations. They
have clinical effects similarto those of zileuton.
USMLE WORLD STEP 1 PHARMACOLOGY

Q NO 190: A 56-year-old female begins treatmentfor severe joint pain and swelling. Several
weeks later, she presents to her physician complaining of cough and hemoplysis. Sputum
stains reveal acid-fast bacilli. The medication used to treat this patient’s joint pain inhibits
which of the following processes?

A. Purine synthesis
B. Pyrimidine synthesis
C. Phospholipid degradation
D. Prostaglandin synthesis
F. TNE-alpha action

Explanation:
This patient was most likely being treated with a tumor necrosis factor alpha (TNF-a) inhibitor
for rheumatoid arthritis. TNF-a inhibitors are powerful anti-inflammatory agents with
demonstrated abililyto improve synovitis, reduce joint erosion, and slow disease progression in
patients with rheumatoid arthritis, The TNE-ct inhibitors include infliximab, etanercept, and
adalimumab.
TNE-a is a pro-inflammatory cytokine involved in macrophage functioning. Treatment with
TNE-a blockers predisposes patients to infection. Reactivation of latenttuberculosis has been
reported. Porthis reason, PPD skin tests to screen for latent tuberculosis should be performed
before starting therapy with these agents. The acid-fast bacilli in this patient’s sputum suggest
that she has developed an active tuberculosis infection. While the risk for tuberculosis may
also be increased with some other rheumatoid arthritis therapies (e.g. corticosteroids), the
risk with TNF-a inhibitors is especially high. The riskfor infections byfungi and atjpical
mycobacteria is also increased.
(Choices A and B) Agents inhibiting purine and pyrimidine synthesis have been used
successfully for the treatment of rheumatoid arthritis. These drugs include methotrexate,
leflunomide, and azathioprine. Methotrexate is considered a first-line drug for established
disease or newly diagnosed moderate to severe disease.
(Choice C) Corticosteroids potently suppress inflammation and other immune processes. They
can be used to manage the acute pain and functional limitation in patients with active
rheumatoid arthritis. Corticosteroids work by inhibiting phospholipase A2, which in turn leads
to decreased synthesis of prostaglandin and leukotrienes.
(Choice D) Non-steroidal anti-inflammatory drugs (NSAID5) inhibit the enzyme cyclooxjgenase
to decrease prostaglandin synthesis. They are very useful for the treatment of acute joint pain
and swelling in patients with rheumatoid arthritis. The main adverse effects are
gastrointestinal ulceration and bleeding. NSAIDs can also cause fluid retention and
aggravation of hypertension. There is no increased riskfor infection in patients being treated
with NSAIDs.
Educational Objective:
TNE-a inhibitors (infliximab, etanercept, and adalimumab) decrease macrophage function and
may cause reactivation of latent tuberculosis.
USMLE WORLD STEP 1 PHARMACOLOGY

Q NO 191: A 34-year-old male is brought to the ER with severe headache, blurred vision and
hand tremor soon after dining at a
local Pira restaurant. His past medical historj is significant for severe, atypical depression. His
blood pressure is
210/1 30 mmHg and heart rate is 110/mm. The medication used to treat this patient’s
depression most likely affects
which of the following?

A. Presynaptic selective serotonin uptake


B. Presynaptic selective norepinephrine uptake
C. Presynaptic non-selective monoamine uptake
D. Enzymatic monoamine degradation
E. Postsynaptic monoamine receptor transmission

Explanation:
Monoamine oxidase is a mitochondrial enzyme, responsible for degradation of excess
neurotransmitter amines. It works as a “safely valve”.
Hypertensive crises on food intake are lypical for MAO inhibitors. MAO inhibitors block a major
degradative pathway for the amine neurotransmitters, which permits more amines
(norepinephrine, etc.) to accumulate in presynaptic stores and be released. Foods containing
tjramine, a sympathomimetic, are usually metabolized in the gastrointestinal tract.
However,the MAO inhibitors blockthis degradation and allowittoflowintothe circulation. This
results in severe hypertensive crisis.
Phenelzine is a MAO inhibitor that may lead to severe hypertensive crisis with tyramine-
containing foods [cheese, sausage, and wine]. This phenomenon is the most commonly tested
USMLE Step 1 side effect.
(Choice A) Presynaptic selective serotonin uptake refers to the SSRI anti-depressants such as
fluoxetine, sertraline, and paroxetine that block the re-uptake of serotonin at the pre-synaptic
neuron. This enables more serotonin to accumulate within the synaptic cleft and affect post-
synaptic neuronal transmission, counteracting depressive symptoms.
(Choice B) Presynaptic selective serotonin uptake refers to bupropion. Bupropion’s mechanism
of action involves norepinephrine transmission, whereas a relatively negligible effect is noted
on serotonin, acelylcholine, or histamine metabolism. As a result of its mechanism of action,
bupropion may be associated with agitation, insomnia, and seizures.
(Choice C) Presynaptic non-selective monoamine uptake refers to the tricyclic antidepressants
such as imipramine, amitriplyline, and clomipramine. The tricyclic anti-depressants block the
amine (norepinephrine or serotonin) reuptake pumps which terminate amine
neurotransmission. Such action permits these amines to remains for longer time periods at the
receptor site.
(Choice E) Postsynaptic monoamine receptor transmission is not correct. MAO inhibitors block
a major degradative pathwayforthe amine neurotransmitters, which permits more aminesto
accumulate in presynaptic stores and be released.
Educational Objective:
Watch outforthe depressed patient on the USMLE Step lwho has a hypertensive crisis
afterawine and cheese party! Think MAO inhibitor-lyramine crisis.
USMLE WORLD STEP 1 PHARMACOLOGY

Q NO 192: A 30-year-old Caucasian female presents to your office complaining that she
worries excessively about insignificant details in her daily life. She also says that she feels
tense frequently and fatigues easily. You run a couple of studies including thyroid function
tests, and the resulting values are within normal limits. You decide to proceed with buspirone
therapy. Which of the following is the best statement regarding buspirone therapy?

A. It has significant sedative effects


B. It has muscle relaxant properties
C. It can prevent some lypes of seizures
D. It has minimal hypnotic effects
E. It can cause prominent euphoria

Explanation:
USMLE WORLD STEP 1 PHARMACOLOGY

Q NO 193: A 70-year-old Caucasian female with sleep problems responds poorlyto non-
pharmacologic measures. You consider short4erm pharmacologic treatment in this patient.
She requests a medication that won’t make her “addicted to sleeping pills.” Which of the
following medications is preferred in this patient?

A.Temazepam
B. Zolpidem
C. Estazolam
D.Triazolam
E. Elurazepam

Explanation:
Zolpidem is a short-acting hypnotic agent structurally unrelated to benzodiazepines. Despite
chemical differences, the mechanism of action of zolpidem and benzodiazepines is similar both
bind to the same portion of the GABAA
receptor and enhance the inhibitory action of GABA on the ONS. Zolpidem is used for short-
term treatment of insomnia. It has a rapid onset of action (15 mm after administration) and is
metabolized by liver P450 microsomal oxidases, Its elimination half-life is normally about 3
hours1 but may be prolonged in patients with liver disease.
Zolpidem has the following important properties:
1. Less potential for tolerance and addiction. The incidence of withdrawal symptoms (and,
therefore, physical dependence) is much lower than with benzodiazepines.
2. No anticonvulsant properties in regular doses.
3. No muscle relaxing effects and not used for anesthesia.
(Choices A and C) Temazepam and estazolam are benzodiazepine hypnotics with medium
durations of action. As with all benzodiazepines, these two drugs have a higher risk of
addiction and tolerance than zolpidem.
(Choice D) Triazolam is a short-acting hypnotic (T112 = 2-3 hours) belonging to the
benzodiazepines class. Because the half-life is so short, it causes much more physical
dependence than zolpidem.
(Choice E) Elurazepam is a long-acting benzodiazepine used for the treatment of insomnia. It
is more likely to cause tolerance and dependence than zolpidem.
Educational Objective:
Zolpidem is a short-acting hypnotic medication chemically unrelated to benzodiazepines. It has
the same mechanism of action as benzodiazepines, but a much lower risk of tolerance and
dependence.
USMLE WORLD STEP 1 PHARMACOLOGY

Q NO 194: A 56-year-old male presents to your office with recent-onset fatigue and muscle
pain. His past medical history is significant for an acute myocardial infarction four months ago.
Laboratory testing reveals elevated serum creatine kinase activit’. These findings are most
likely related to the patient’s use of a medication for which of the following conditions?

A. Hypertension
B. Recurrent venous thrombosis
C. Heart failure
D. Lipid abnormalities
E. Osteoarthritis
F. Migraine headaches

Explanation:
Statins are commonly prescribed for patients who have suffered a myocardial infarction
because they have been shown to significantly decrease both the incidence of a second
myocardial infarction and mortality in these patients. These benefits are due to statins’
cholesterol lowering effects, as well as to statins’ direct abilitjto stabilize atheromatous
plaques.
Hepatotoxi city and myopathy are common side effects associated with statin therapy.
Myositis causes elevations of creatinine kinase and usually occurs when higher doses of statins
are used, whereas myalgias are a more common symptom not associated with a rise in serum
creatinine kinase. The risk of statin myopathy is increased when fibrates and/or niacin are
used concomitantly. The pathophysiology of statin myopathy is not completely understood.
There maybe alinkto decreased formation of coenzyme Q1O in skeletal muscle.
Other drugs that can cause myopathy include fibrates, niacin, hydroxjchloroquine,
glucocorticoids, colchicine, interferon alpha and penicillamine.
Educational Objective:
Myopathy is a well-known side effect of some hypolipidemic drugs (especially statins).
USMLE WORLD STEP 1 PHARMACOLOGY

Q NO 195: A 56-year-old male with type 2 diabetes mellitus is treated with a single daily dose
of NPH insulin. After adding daily pioglitazone, the patient’s need for insulin decreases and his
glycemic control improves as verified by his reduced HbA
1 ‘ Which of the following is the most likely cellular target of pioglitazone?

A. Membrane ion channel


B. Surface lyrosine kinase-coupled receptor
C. Surface adenylate cyclase-coupled receptor
D. Intracellular nuclear receptor
E. Surface membrane-bound enzyme
F. Intracellular microsomal enzyme

Explanation:
Thiazolidinediones (TZD5) are new group of antidiabetic medications, which exert their
glucose-lowering effect by improving insulin resistance. TZDs achieve this result by binding to
peroxisome proliferator activated receptor gamma (PPAR-gamma), which is a transcriptional
regulator of the genes involved in glucose and lipid metabolism. PPAR-gamma belongs to the
nuclear receptor superfamily that regulates gene expression after ligand binding. TZD binding
to PPAR-gamma causes conformational change, which allows its binding to another
coactivator. This complex then binds to the transcriptional regulatory sequence of genes that
are responsible for glucose and lipid metabolism, in a process called “transactivation.”
ie most important gene regulated by PPAR-gamma codes for adiponectin, a cytokine secreted
by fat ;sue. Adiponectin levels are low in lype 2 diabetes, and treatment with TZD5 increases
these levels. Increased vels of adiponectin are one of the main mechanisms bywhich TZDs
decrease insulin resistance.
:hoice A) Sulfonylureas bind to the regulatory subunits of the KATP channel on pancreatic beta
cells, which causes sure of these channels. Beta cells then become depolarized and voltage-
dependent calcium channels are opened, ding to calcium influx. High intracellular calcium
levels within beta cells lead to insulin release by
tcytosis. Newer, short-acting non-sulfonylurea medications, such as repaglinide and
metaglinide also act by nding to the regulatory subunit of KATP channels.
hoice B) Insulin acts by binding to a cell-surface receptor on target cell. The receptor for
insulin is a tetrameric ‘ucture consisting of iwo alpha and two beta subunits. The alpha
subunits are located extracellularly and provide nding sites for insulin. Beta subunits are
located intracellularly and contain the tyrosine kinase domains that are livated by insulin’s
attachmentto the extracellularalpha subunits of the insulin receptor.
hoice C) Glucagon like polypeptide I (GLP-1) is an incretin hormone that is secreted by
intestinal L cells in sponse to food intake. GLP-1 decreases blood glucose by inducing satiety,
decreasing gastric emptying, and :reasing insulin release from pancreatic beta cells. GLP-1
acts through cell surface receptors coupled with G otein-adenyl cyclase system. GLP-1 cannot
be used clinically because of its very short half-life, so scientists eated a long-acting GLP-1
analogue called “exenatide” which is approved forthe treatment of type 2 diabetes in 3tents
with suboptimal glucose control, despite adequate doses of metformin and sulfonylureas.
hoices E and F) Enzyme inactivation is an important mechanism of action for metformin and
alpha-glucosidase iibitors. Metformin decreases the enzyme activity responsible for the
gluconeogenesis located in the hepatic icrosomes. Alpha-glucosidase inhibitors decrease the
enzyme activity of the membrane-bound disaccharidases on intestinal brush border.
Educational Objective:
Thiazolidinediones bind to peroxisome proliferator-activated receptor-gamma (PPAR-y), a
receptor that belongs to the steroid and thyroid superfamily of nuclear receptors.
USMLE WORLD STEP 1 PHARMACOLOGY

Q NO 201: Isolates of M. tuberculosis growing in culture lose acid-fastness and


rapidly stop proliferating when exposed to Drug A. This drug behaves in a
similar fashion to:
A. Isoniazid
B. Rifampin
C. Ethambutol
D. Streptomycin
E. Ciprofloxacin

Explanation:
Isoniazid is an antibiotic active only against mycobacteria. The
specificity of this drug is derived from its inhibition of mycolic acid
synthesis. As the long branched-chain fatty acids in the outer portion
of the mycobacterial peptidoglycan cell wall1 mycolic acids are
essential for proper cell wall structure as well as in the synthesis of
mycobacterial virulence factors (eg, sulfa tides, wax D and cord
factor). If the mycobacteria are unable to synthesize mycolic acids
they cannot create proper cell walls or divide to form new cells. The
presence of mycolic acids in the cell wall is what makes mycobacteria
acid-fast when stained with carbolfuchsin dye and treated with an acid-
alcohol decolorizing agent.
(Choice B) Rifampin is an essential component of the multi-drug regimen
used to treat active tuberculosis. Rifampin halts bacterial protein
synthesis by inhibiting bacterial DNA-dependent RNA polymerase, thereby
preventing transcription of DNA into mRNA. Rifampin would inhibit
mycobacterial growth in an in vitro culture, but would not cause the
mycobacteria to lose their acid-fastness.
(Choice C) Ethambutol acts by an unclear mechanism to inhibit
mycobacterial cell wall synthesis but has no effect on the creation of
mycolic acid. Ethambutol is specific for mycobacteria and has no effect
on other bacterial organisms.
(Choice D) Streptomycin is an aminoglycoside antibiotic that functions
by inhibiting the bacterial 308 (small) ribosomal subunit, thereby
halting protein synthesis. Mycobacterial cell growth is restricted and
eventually bacterial cell death results. The acid-fastness of the
organism is not affected, however. Streptomycin is also indicated in
the treatment of plague (caused by Yersinia pestis) and tularemia
(caused by Francisella tularensis).
(Choice E) Ciprofloxacin is a fluoroquinolone antibiotic. The
fluoroquinolones are emerging as new components of the multi-drug
regimen used to treat active infections with M. tuberculosis.
Ciprofloxacin, like all other fluoroquinolones, functions by inhibiting
DNA gyrase. Fractures in the bacterial chromosome result and the
bacteria become unable to code for proteins necessary for survival.
Fluoroquinolones would not have an immediate effect on the acid-
fastness of mycobacteria.

Educational Objective:
Isoniazid is an anti mycobacterial agent that specifically inhibits the
synthesis of mycolic acids. Mycolic acids are essential components of
the unique mycobacterial peptidoglycan cell wall. Without mycolic acids
the mycobacteria lose their acid-fastness and become unable to
synthesize new cell walls or multiply.

16
USMLE WORLD STEP 1 PHARMACOLOGY

Q NO 202: A 34-year-old male with advanced HIV infection is hospitalized with a


vesicular skin rash suggestive of varicella zoster virus (VZV) infection. Viral
strains isolated from this patient lack phosphorylating enzymes. Which of the
following drugs is most likely to be effective in treating this patient’s infection?

A. Acyclovir
B. Ganciclovir
C. Valacyclovir
D. Famciclovir
E. Cidofovir

Explanation:
An absence of phosphorylating enzymes (eg, thymidine kinase [TK]) in
the herpesvirus confers resistance to nucleoside analog antiviral drugs
like acyclovir. This phenomenon occurs because a normally functioning
viral thymidine kinase is necessary to convert a nucleoside analog drug
into its monophosphate form. Once that has taken place, cellular
kinases then convert the drug nucleoside monophosphate into a
nucleoside triphosphate that interferes with herpesvirus replication.
Thymidine kinase-deficient (and therefore acyclovir-resistant)
varicella zoster virus isolates tend to be obtained almost exclusively
from AIDS patients. These immunocompromised patients are best treated
with foscarnet—a pyrophosphate analog viral DNA polymerase inhibitor
that does not require viral kinase activation—or with cidofovir.
Cidofovir is a broad-spectrum antiviral nucleotide analogue of cytosine
monophosphate. Because cidofovir is already a nucleoside monophosphate,
its efficacy does not depend upon the presence of a virally encoded
kinase. Instead, cellular kinases are all that is necessary to convert
the cidofovir into its active nucleoside triphosphate form (which
competitively inhibits viral DNA polymerase).

Educational Objective:
Acyclovir, the acyclovir prodrug valacyclovir, famciclovir, and
ganciclovir are all nucleoside analogues that require conversion to
nucleoside monophosphates by herpesviral kinases. Cidofovir is already
a nucleoside monophosphate and therefore only requires cellular kinases
for conversion to the active nucleoside triphosphate form.

17
USMLE WORLD STEP 1 PHARMACOLOGY

Q NO 203: A 56-year-old Caucasian male is brought to the ER with severe right foot
pain. On physical examination, there is a swollen tender first metatarsophalangeal
joint. His past medical hi stow is significant for hyper lipidemia treated with lifestyle
modifications and several medications. Which of the following drugs is most likely to
have precipitated this patient’s condition?

A. Atorvastatin
B. Niacin
C. Gemfibrozil
D. Cholestyramine
E. Ezetimibe

Explanation:
The patient in the vignette has an acute flare of gouty arthritis
likely a side effect of treatment with niacin. Oftentimes, patients
being treated with nicotinic acid experience increases in serum uric
acid levels, which can precipitate acute gouty arthritis. Other side
effects of nicotinic acid include facial flushing and warmth, which can
be prevented by pre administration of aspirin. Hepatotoxicity can occur
with high doses of nicotinic acid. Patients with diabetes may
experience worsening hyperglycemia.
Niacin improves hyperlipidemia by decreasing hepatic synthesis of
triglycerides and VLDL. Nicotinic acid increases HDL by 25-30% making
it the most effective agent available for raising HDL. Despite niacin’s
well-documented efficacy, large numbers of patients experience side
effects, particularly at the initiation of therapy.
The medications listed in the other answer choices do not alter serum
uric acid levels. Hydrochlorothiazide, cyclosporine, and pyrazinamide
are other medications that can increase serum uric acid levels, putting
patients at increased risk for gout attacks.
(Choice A) The major side effects of statins include hepatitis,
myopathy and myalgias.
(Choice C) Gemfibrozil and fenofibrate are fibric acid derivatives that
are the first-line treatment for hypertriglyceridemia. The main side
effects of fibrates include myopathy and hepatotoxicity (as with
statins). The risk for severe myopathy is increased when fibrates are
combined with statins. Fibric acid derivatives also increase the risk
of cholesterol gallstones.
(Choice D) The main side effects of bile acid-binding resins are
gastrointestinal upset, impaired absorption of nutrients and drugs, and
hypertriglyceridemia. Like fibric acid derivatives, these drugs also
increase the risk of cholesterol gallstone formation, reflecting the
increased synthesis of cholesterol.
(Choice E) Ezetimibe is a new drug that selectively inhibits intestinal
absorption of cholesterol. This medication is primarily used in
conjunction with statin therapy for the treatment of
hypercholesterolemia. The rare side effects of ezetimibe include
elevations in liver enzymes and myopathy.

18
USMLE WORLD STEP 1 PHARMACOLOGY

19
USMLE WORLD STEP 1 PHARMACOLOGY

Q NO 204: A 36-year-oldwoman complains of dyspnea and weakness. Her mother


had similar symptoms and died at42years of age. After an extensive work-up, a
lung biopsy was performed that showed decreased intraluminal diameter of the
small branches of pulmonary artery and medial hypertrophy. The patient was
subsequently scheduled for a lung transplant. Which of the following medications
is indicated for her management during the waiting period?
A. Enalapril
B. Clopidogrel
C. Bosentan
D. Indometacin
E. Etanercept

Explanation:
Pulmonary hypertension (increased pressure in pulmonary artery) causes
specific morphologic changes in the branches of the pulmonary artery.
Increased thickness of the smooth muscle of the arterioles (medial
hypertrophy) and fibrosis of the intima may lead to the narrowing of
the lumina of these vessels. Such morphological findings can be caused
by both secondary (associated with underlying lung, vascular or cardiac
disease) and primary idiopathic pulmonary hypertension.
The patient described in this clinical vignette has characteristic
histologic findings of pulmonary hypertension but no underlying lung or
heart disease. Her family history suggests that she has an inherited
form of primary pulmonary hypertension (PPH). This disease is
associated with decreased apoptosis of endothelial and smooth muscle
cells in pulmonary arterioles. PPH presents in women aged 20-40 years
with dyspnea and exercise intolerance.
Although lung transplant is the ultimate treatment of this disease
vasodilators have been effective for improving symptoms. Bosentan is an
oral endothelin-receptor antagonist recently approved for treatment of
pulmonary hypertension. Endothelin is a potent vasoconstrictor and
stimulant of endothelial proliferation. Bosentan, by blocking
endothelin, has been shown to decrease pulmonary arterial pressure and
lessen the progression of vascular and right ventricular hypertrophy.
(Choice A) Enalapril is an angiotensin converting enzyme inhibitor.
Itis used for treatment of CHE, hypertension and diabetic nephropathy.
(Choice B) Clopidogrel inhibits ADP induced platelet aggregation. It is
used in atherosclerotic ischemic disease and to prevent acute stent
thrombosis following PCI.
(Choice D) Indomethacin is a nonspecific cyclooxygenase (COX 1 & 2)
inhibitor that acts to suppress prostaglandin synthesis. It is used as
in anti-inflammatory agent and pain reliever.
(Choice E) Etanercept is a humanized monoclonal antibody that binds
tumor necrosis factor (TNF) thereby decreasing its concentration in the
serum. Etanercept is an anti-inflammatory agent indicated for treatment
of rheumatoid arthritis as well as psoriasis and psoriatic arthritis.

Educational Objective:
Bosentan is a competitive antagonist of endothelin receptors used for
treatment of primary (idiopathic) pulmonary arterial hypertension.

20
USMLE WORLD STEP 1 PHARMACOLOGY

Q NO 205: A 47-year-old male presents to the emergency department with a


painful erection that has persisted for the past 8 hours. He has never had this
problem before. His past medical history is significant for treatment-resistant
depression and insomnia. Which of the following medications do you suspect
may be responsible?
A. Fluoxetine
B. Phenelzine
C. Bupropion
D. Imipramine
E. Trazodone
F. Amoxapine

Explanation:
Trazodone has sedating effects and can be used with fluoxetine to treat
patients with depression and accompanying insomnia. Trazodone has no
anticholinergic or cardiac effects, but can cause priapism, making it
relatively contraindicated for use in adolescent boys.
(Choice A) Fluoxetine and other SSRIs cause decreased libido,
anorgasmia and increased latency to ejaculation. They can be used to
treat premature ejaculation.
(Choice B) Phenelzine is a monoamine oxidase inhibitor (MAO-I) used in
the treatment of depression. Monoamine oxidase is a mitochondrial
enzyme that deaminates primary and secondary aromatic amines. Priapism
is not associated with the MAO-Is.
(Choice C) Unlike SSRIs, bupropion is not associated with sexual
dysfunction. It is also not associated with priapism.
(Choice D) Imipramine is a tricyclic antidepressant. The tricyclics can
cause orthostatic hypotension, cardiac arrhythmias and other anti
cholinergic side effects.
(Choice F) Amoxapine is a tri cyclic anti depressant that can cause
mild parkinsonian symptoms, but not priapism.

Educational Objective:
Trazodone can cause priapism early in the treatment course. This side
effect is a relative contraindication for use in adolescent boys.
Trazodone is a highly sedating anti-depressant that can be used to
treat insomnia or as an adjunct to the SSRls when there is depression-
associated insomnia.

21
USMLE WORLD STEP 1 PHARMACOLOGY

Q NO 206: A 55-year-old Caucasian female is being treated with tamoxifen. She


experiences hot flashes that come and go in waves and affect her sleep
significantly. Which of the following conditions is the patient also likely to
experience as a result of this therapy?

A. Decrease in bone mass


B. Ductal hyperplasia in her breast
C. Increase in LDL level
D. Decrease in HDL level
E. Endometrial hyperplasia

Explanation:
Tamoxifen and raloxifene are given the name “selective estrogen
receptor modulators (SERM5)” because of their tissue-selective estrogen
agonist and antagonist properties. In other words, the activity of
these drugs varies with tissue type. In breast tissue tamoxifen’s
effects are predominantly anti-estrogenic hence its utility in the
treatment of estrogen receptor-positive breast cancer. When used as an
adjuvant agent in the treatment of estrogen receptor- positive breast
cancer tamoxifen decreases the risk of recurrence and increases
disease-free survival.
In endometrial tissue, however tamoxifen has a stimulatory effect.
Thus, its use is associated with the development of endometrial polyps,
endometrial hyperplasia, and endometrial cancer. Because of the
increased risk of endometrial cancer patients on tamoxifen are advised
to have annual gynecological examinations. Additionally abnormal
vaginal bleeding in these patients warrants prompt investigation.
(Choice A) Tamoxifen acts as a partial estrogen receptor agonist in
bone and is thus typically associated with an increase in bone mineral
density.
(Choice B) In breast tissue tamoxifen’s predominant effect is anti-
estrogenic. Thus, its use decreases cancer recurrences even in the
contralateral breast.
(Choices C and D) Tamoxifen generally has a favorable effect on the
serum lipid profile. Breast cancer patients undergoing chemotherapy
often develop ovarian failure which causes elevations in total
cholesterol and LDL levels. In these patients tamoxifen helps to
normalize total cholesterol and LDL levels without causing significant
changes to the HDL. Serum triglycerides may increase in patients
genetically predisposed to hypertriglyceridemia.

Educational Objective:
Tamoxifen and raloxifene are called “selective estrogen receptor
modulators (SERMs)” because of their tissue selective estrogen agonist
and antagonist properties. Tamoxifen is useful for the treatment of
osteoporosis and breast cancer. However it is associated with an
increased incidence of endometrial cancer and thromboembolic disease.

22
USMLE WORLD STEP 1 PHARMACOLOGY

Q NO 207: A 75-year-old male is hospitalized with an episode of severe hematuria.


He subsequently develops back pain and mild fever. Which of the following anti
inflammatory agents has minimal affect on platelet aggregation and thus can safely
be given to this patient?

A. Aspirin
B. Rofecoxib
C. Diclofenac
D. Fenoprofen
E. Ibuprofen
F. Indomethacin
G. Ketorolac
H. Piroxicam

Explanation:
All of the agents listed are non-steroidal anti-inflammatory drugs
(NSAIDs) medications commonly used to treat pain arthritis and fever.
NSAIDs work by inhibiting the enzyme cyclooxygenase (OCX), thus
blocking prostaglandin synthesis. Most NSAIDs inhibit both the COX 1
and COX 2 isoenzymes. Whereas COX 1 plays a physiologic role in a
number of normally functioning tissues (e.g. platelets, the
gastrointestinal tract) COX 2 is preferentially expressed at sites of
inflammation. Many of the potential NSAID adverse effects, including GI
ulceration and bleeding, are related to inhibition of COX 1. Selective
COX2 inhibitors were developed to avoid these complications. In a
patient with pain, fever, and hematuria, a selective COX 2 inhibitor
such as rofecoxib, would be an appropriate choice.
Both COX 1 and COX 2 are expressed in renal tissues. Both traditional
NSAIDs and selective COX 2 inhibitors can cause fluid retention and
aggravation of hypertension.
The COX 2 enzyme is also expressed in vascular endothelial cells and
vascular smooth muscle cells, and may play a role in the local
production of prostacyclin (PGI2), a substance that promotes
anticoagulation and vascular dilatation. Some selective COX 2
inhibitors have been associated with an increased incidence of
cardiovascular events, which may be related to decreased production of
PGI2.
(Choice A) Aspirin (acetylsalicylic acid) is an NSAID with the unique
property of irreversibly inhibiting platelet aggregation by acetylating
serine residues of the platelet cyclooxygenase enzyme. Aspirin
inhibition of platelet aggregation lasts for the lifetime of the
platelet (3-7 days). A medication with anti-platelet effects would be
inappropriate for a patient with severe hematuria.
(Choices C - F) These drugs are commonly used NSAIDs that reversibly
inhibit both COX 1 and COX 2. The COX 1 effect will cause inhibition of
platelet aggregation, making these medications less than ideal for a
patient with severe hematuria.

Educational Objective:
Selective COX2 inhibitors have potent anti-inflammatory effects without
the side effects of bleeding and gastrointestinal ulceration associated
with non-selective COX inhibitors. Selective CCX 2 inhibitors do not
impair platelet function because platelets predominantly express COX 1.

23
USMLE WORLD STEP 1 PHARMACOLOGY

Q NO 208: A 12-year-old Caucasian male hemorrhages from his mouth after a tooth
extraction. The hemorrhage lasts for more than 12 hours despite local applications
of thrombotic agents. The hemorrhage finally stops after desmopressin (DDAVP)
administration. Which of the following mechanisms most likely explains the effect of
the drug in this patient?

A. Vasoconstriction
B. Increase in liver protein synthesis
C. Increase in endothelial protein release
D. Inhibition of fibrinolytic system
E. Blood cell mobilization from the spleen

Explanation:
The patient described in the vignette is likely to have mild form of
von Willebrand’s disease. Patients with the mild form of von
Willebrand’s disease do not have major spontaneous bleeding; instead,
most patients with this disorder present with increased skin
bruisability, mild mucosal bleeding, and prolonged bleeding during
minor surgical procedures, like a tooth extraction. It is hard to
diagnose Von Willebrand disease on routine blood labs because the
bleeding time is only intermittently and intermediately increased in
these patients.
Von Willebrand factor (vWF) is secreted by endothelial cells and
circulates in multimers that are non-covalently attached to VIII, and
is thought to increase the stability of clotting factor VIII. vWF is
also responsible for augmenting platelet binding to the endothelial
cells at the initial injury which helps information of the initial
platelet plug. Desmopressin (DDAVP) is a synthetic analog of
vasopressin, or anti diuretic hormone (ADH) normally released by the
posterior pituitary. It is well-established that DDAVP increases vWF
release from endothelial cells. After injection of DDAVP, von
Willebrand factor levels increase in 30-60 minutes. This transient
increase in vWF levels for mildly affected patients makes DDAVP very
helpful in controlling bleeding during surgical procedures. In severe
disease when there is almost complete deficiency of vWF, DDAVP is not
beneficial. Because the patient did respond to DDAVP, we can surmise
that he has only a mild form of the disease.
(Choice A) Although DDAVP is very similar to vasopressin, it is not
identical. DDAVP has a minimal effect on V1 vasopressin receptor. As
the V1 receptor primarily causes vasoconstriction, this phenomenon is
not seen with DDAVP use.
(Choice B) Most clotting factors are formed in the liver. However, vWF
is formed in the endothelial cells. DDAVP increases the release (no
increase in the synthesis) of stored vWF by the endothelial cells.
DDAVP does not increase the synthesis of other clotting factors.
(Choices D and E) DDAVP does not have any effect on the fibrinolytic
system or mobilization of cells from the spleen.

Educational Objective:
The desmopressin analog DDAVP is used in patients with von Willebrand
disease because it induces endothelial procoagulatory protein release
(including vWF). Remember, that desmopressin is also used for the
treatment of enuresis.

24
USMLE WORLD STEP 1 PHARMACOLOGY

Q NO 209: A 60-year-old male is treated with losartan for hypertension. His blood
pressure decreases to normal value over several weeks of treatment. He seems to
be compliant with his medication and experiences no significant side effects.
Which of the following is the most likely combination of changes in response to
this patients treatment?

Renin Angiotensin I Angiotensin II Aldosterone Bradykinin


A. Increase Increase Decrease Decrease Increase
B. Increase Increase Increase Decrease Increase
C. Increase Increase Increase Decrease No change
D. Increase Increase Decrease Decrease Decrease
E. Increase Decrease Decrease Decrease Decrease
F. Decrease Decrease Decrease Decrease No change

Explanation:
The renin-angiotensin-aldosterone system (RAAS) is one of the most
important neurohormonal systems that regulate arterial blood pressure
and sodium and fluid content in the body. In response to decreased
sodium, fluid volume, or arterial blood pressure, renin is released
from the kidneys converting angiotensinogen to angiotensin I.
Subsequently, angiotensin converting enzyme (ACE) converts angiotensin
Ito angiotensin II and is also responsible for the breakdown of
bradykinin. Angiotensin II stimulates angiotensin-1 receptors (AT-1)
throughout the body constricting vascular smooth muscle and increasing
aldosterone secretion from the adrenal cortex. AT-1 receptors are also
involved in a negative feedback mechanism regulating the RAAS.
Stimulation of AT-1 receptors on juxtaglomerular cells will decrease
renin release from the kidneys, consequently decreasing angiotensin I
and II levels.
Angiotensin receptor blockers like losartan (ARB5) competitively bind
to AT-i receptors and block the effects of angiotensin II. This will
result in vascular smooth muscle relaxation and decreased aldosterone
secretion. Since ARBs directly block AT-i receptors, they do not affect
ACE activity and should not affect bradykinin degradation or levels.

Educational Objective:
ARBs work by blocking AT-i receptors and inhibiting the effects of
angiotensin II. This results in arterial vasodilation and decreased
aldosterone secretion. Since ARBs work at the level of AT-i receptors
and do not affect ACE activity. they do not interfere with bradykinin
degradation or levels. By blocking AT-i receptors, AREs also interfere
with negative feedback mechanisms resulting in increased renin,
angiotensin I, and angiotensin II levels.

25
USMLE WORLD STEP 1 PHARMACOLOGY

Q NO 210: A 32-year-old male presents to the emergency room reporting right


knee pain and swelling for the past 12 hours. On physical examination, his
knee is swollen and red. Synovial fluid analysis reveals a white blood cell count
of 100000/mm3. The best initial treatment for this patient is:

A. Indomethacin
B. Colchicine
C. Ceftriaxone
D. Allopurinol
E. Observation

Explanation:
Septic arthritis, usually due to gonococcus, is the most common cause
of acute monoarticular arthritis in this age group. Gout and pseudogout
can also cause acute monoarticular arthritis, but the synovial fluid
white blood cell count is usually <20,000/mm3. To make the definitive
diagnosis, Gram stain, culture, and microscopy for crystals should be
performed. While test results are pending ceftriaxone should be given
to cover for gonococcus.
(Choice A) Indomethacin is an NSAID that might be prescribed for non-
infectious inflammatory arthritis (e.g. gout). NSAIDs could be
administered to the patient above for pain relief but would not be the
best initial treatment.
(Choice B) Colchicine reduces the acute inflammation of gouty arthritis
by inhibiting neutrophil migration into the inflamed areas.
(Choice D) Allopurinol is an isomer of hypoxanthine that lowers serum
uric acid levels. It is prescribed to prevent attacks of acute gout
arthritis but is not useful in the treatment of acute flares.

Educational Objective:
A high synovial fluid white blood cell count (100,000/µL) and absent
crystals on microscopic examination strongly suggest bacterial joint
infection. Septic arthritis requires immediate antibiotic treatment to
prevent joint destruction, osteomyelitis, and sepsis.

26
USMLE WORLD STEP 1 PHARMACOLOGY

Q NO 211: A 72-year-old female is diagnosed with atrial fibrillation during routine


check-up. She has no symptoms and her past medical history is insignificant.
Soon after initiation of therapy with warfarin, the patient is hospitalized with
severe skin and subcutaneous fat necrosis. Drug effects on which of the following
processes are most likely responsible for this patient’s condition?

A. Prothrombin conversion
B. Fibrinogen conversion
C. Factor VIIa activity
D. Factor XIa activity
E. Protein C synthesis
F. Factor IX synthesis

Explanation:
Warfarin inhibits vitamin K dependent-carboxylation of glutamic acid
residues of clotting factors II, VIII IX and X (also known as vitamin K
dependent clotting factors). This results in production of
dysfunctional coagulation proteins. Warfarin also decreases
-carboxylation and function of the naturally occurring anticoagulant
proteins C and
S. Naturally occurring negative regulators of blood coagulation such as
antithrombin-III protein C, protein S and tissue pathway factor
inhibitor (TPFI) are normally present in blood to prevent excessive
coagulation. Activated protein C deactivates factor V and VIII by
proteolysis. The actions of protein C are increased in the presence of
protein
S. Deficiency of protein C and S (and other naturally occurring
anticoagulants) typically results in hypercoagulable states.
Inhibition of the anticoagulant activity of protein C can predispose
patients to warfarin-induced skin necrosis. This complication typically
occurs during the first week of warfarin therapy. When warfarin is
started the activity of protein C is reduced to 50% within one day.
Factor VII has a similar half-life but the other procoagulant vitamin K
dependent factors (II, IX, and X) decline at a slower rate. This
results in a transient hypercoagulable state. The risk of warfarin
induced skin necrosis is increased in patients with a preexisting
protein C deficiency, as well as in patients started on a large loading
dose of warfarin.
(Choices A and B) Heparin acts as an anticoagulant by increasing the
effect of the naturally occurring anticoagulant antithrombin-III.
Heparin does not have any effect on protein C, protein S or TPFI.
Complications of heparin therapy include bleeding and heparin-induced
thrombocytopenia.

Educational Objective:
Antithrombin-III protein C and protein S are natural anti coagulants
that are present in the blood. Warfarin inhibits protein C and S
synthesis and thus can pose a risk of paradoxical thrombosis in
patients with congenital deficiency of protein C and S. This is usually
seen in the first week of therapy.

27
USMLE WORLD STEP 1 PHARMACOLOGY

Q NO 212: A 76-year-old Caucasian female is brought to the emergency


Serum
department with nausea vomiting anorexia and confusion. She has been
treated for atrial fibrillation and heart failure for the last several months.
Her laboratory values are:
sodium 140 mEq/l
Serum potassium 5.7 mEq/l
Serum calcium 9.3 mg/dL
Serum creatinine 1.5 mg/dL
EKS shows a heart rate of 50 beats/min. Her blood pressure is 130180
mmHg. Increased blood level of which of the following medications is
most likely responsible for her symptoms?

A. Metoprolol
B. Digoxin
C. Verapamil
D. Furosemide
E. Spironolactone
F. Amiodarone
G. Aspirin
H. Quinidine

Explanation:
Though calcium channel blockers and beta-blockers are the preferred
treatments for patients with atrial fibrillation with rapid ventricular
response digoxin is a very commonly used second-line treatment. It is
particularly used in patients with underlying systolic cardiac
dysfunction. It acts primarily by inducing stimulation of the AV node
by the vagus nerve causing a slowing of conduction through the AV node.
It is a difficult medication to dose properly, and this is why the
clinical presentation of digitalis toxicity is so frequently tested on
exams. Symptoms include fatigue blurry vision, changes in color
perception, nausea and vomiting, diarrhea, abdominal pain, headache
dizziness, confusion, and delirium. Bradycardia from increased AV nodal
block occurs and is followed by junctional escape beats, sustained
junctional escape rhythms and eventually ventricular tachycardia or
ventricular fibrillation. These rhythms cause the mortality associated
with digitalis toxicity.
The elevated plasma potassium is an additional clue that digitalis
toxicity may be the cause of this patient’s symptoms. Digitalis,
through its action on a Na+/K+ ATPase causes an increase in plasma
potassium that is indicative of the amount of digitalis the patient has
taken. The higher the tissue concentration of digitalis the higher the
serum potassium will be.
The treatment of digitalis toxicity includes:
1. Oral activated charcoal as a GI decontaminant, especially in the
case of an attempted suicide by digitalis ingestion.
2. Management of serum potassium levels by the use of insulin
Kayexalate or hemodialysis. Calcium gluconate should be avoided in
digitalis intoxication.
3. Use of digoxin-specific antibody fragments to bind digoxin both in
the tissues and the vascular spaces.

28
USMLE WORLD STEP 1 PHARMACOLOGY
Educational Objective:
Digitalis (digoxin) is a commonly used drug with a well-characterized
side effect profile making it a favorite testing item among question
writers. It leads to AV block and ventricular tachyarrhythmias.
Hyperkalemia is frequently found in digoxin toxicity.

29
USMLE WORLD STEP 1 PHARMACOLOGY

Q NO 213: A 60-year-old male is brought to the ER with confusion, fever and


cutaneous flushing. His blood pressure is 70/40 mmHg and is heart rate is
120/mm. Both pupils are dilated and equally reactive to light. His daughter
who accompanies him says an empty bottle of amitriptyline is found next to
him. The patient dies despite the resuscitation. The patient’s death is most
likely related to which of the following?
A. Synaptic serotonin accumulation
B. Uncontrolled presynaptic dopamine release
C. Synaptic norepinephrine accumulation
D. Chloride channel overactivity
E. Sodium channel inhibition

Explanation:

TCAs inhibit the reuptake of norepinephrine and serotonin, and are used
as a treatment for depression. They also inhibit fast sodium channel
conduction, resulting in arrhythmia, the most common cause of death in
patients with antidepressant intoxication. Tricyclic anti-depressants
are notorious for inhibiting ‘fast myocardial sodium channels. Remember
that phase-zero myocardial depolarization is prolonged when sodium
conductance is inhibited resulting in decreased conduction with a
prolonged ORS complex (>100 msec) and negative inotropic effects.
Impaired excitation-contraction coupling within myocardial cells and
diminished release of calcium from sarcoplasmic calcium stores decrease
contractility, which can lead to cardiac arrhythmia and death.
Refractory hypotension resulting from decreased cardiac contractility
and direct peripheral vasodilatation (from peripheral alpha adrenergic
receptor antagonism) also is a major cause of mortality in these
patients.
(Choices A and C) The tricyclic anti-depressants (imipramine,
amitriptyline, and clomipramine) block the amine reuptake pumps within
the synaptic cleft, which normally terminate amine transmission. This
effect permits a longer interaction between the neurotransmitter and
the receptor, which is thought to account for the anti-depressant
properties of the TCAs. Synaptic norepinephrine accumulation is not
attributed to death from overdose of these antidepressant agents.
(Choice B) Amphetamines have a direct CNS stimulant effect and a
sympathetic nervous system effect by releasing catecholamines from a-
and 3-adrenergic nerve terminals.
(Choice D) Binding of barbiturates to the GABA receptor both enhances
and mimics the action of GABA by increasing chloride conductance
through the ion channel, causing hyperpolarization of the cell
membrane, and thus increasing the threshold of excitability of the
postsynaptic neuron.

30
USMLE WORLD STEP 1 PHARMACOLOGY

Educational Objective:
The most common cause of death in patients with TCAs overdose is
refractory hypotension and cardiac arrhythmias. Inhibition of fast
sodium channels in cardiac myocytes (and His-Purkinje system) thought
to be major underlying cellular event. Thus, hypertonic sodium
bicarbonate administration is crucial in these patients.

31
USMLE WORLD STEP 1 PHARMACOLOGY

Q NO 214: An 8-year-old male is brought to your office by his mother for a routine
check-up. He was diagnosed with absence seizures one year ago and has been
undergoing treatment with ethosuximide. The mother says that the patient is doing
well and his school performance improved significantly. Which of the following is
the most likely mechanism of action of ethosuximide in this patient?

A. Decreases calcium current in thalamic neurons


B. Decreases sodium current in cortical neurons
C. Binds to GABA receptors and increases chloride current
D. Blocks NMDA receptors in hippocampal neurons

Explanation:
Ethosuximide is approved for the treatment of absence seizures.
Ethosuximide blocks T-type Ca2 channels that trigger and sustain
rhythmical burst discharges in thalamic neurons.
(Choice B) Decreases sodium current in cortical neurons refers to
phenytoin. Phenytoin inhibits neuronal high frequency firing by
blocking sodium (Na+) channels and increasing the refractory period of
the neuron.
Carbamazepine and valproic acid mechanisms of actions are similar to
the phenytoin.
(Choice C) GABA is the principal inhibitory neurotransmitter in the
mammalian CNS. Binding of barbiturates to the GABAA receptor both
enhances and mimics the action of GABA by increasing chloride
conductance through the ion channel, causing hyperpolarization of the
cell membrane, and thus increasing the threshold of excitability of the
postsynaptic neuron.
(Choice D) Valproic acid reduces abnormal electric activity in the
brain by blocking NMDA receptors and affecting potassium current. This
drug also affects Na+ channels and GABA receptors.

Educational Objective:
1. Phenytoin, carbamazepine and valproic acid inhibits neuronal high-
frequency firing by reducing the ability of sodium (Na+) channels to
recover from inactivation.
2. Ethosuximide is believed to block T-type calcium channels in
thalamic neurons causing hyperpolarization and is approved for the
treatment of absence seizures.

32
USMLE WORLD STEP 1 PHARMACOLOGY

Q NO 215: A 57-year-old male with suspected bacterial pneumonia is admitted to


the hospital and given ceftriaxone and azithromycin for treatment. Soon after the
first dose of ceftriaxone, he complains of difficulty breathing, abdominal cramps,
and lightheadedness. His current blood pressure is 70/50 mmHg, while his heart
rate is 120/mm. Physical examination reveals a diffuse maculopapular rash.
Which of the following drugs should be administered next to this patient?

A. Corticosteroids
B. Epinephrine
C. Norepinephrine
D. Dobutamine
E. Diphenhydramine

Explanation:
Dyspnea, hypotension, and tachycardia soon after administration of β-
lactam antibiotics are suggestive of anaphylactic shock. Hypotension
occurs in anaphylactic shock secondary to collapse of peripheral
vascular resistance, increases in vascular permeability, and leakage of
capillary fluid. Stimulation of the smooth muscle tone within the
bronchial wall, along with an increase in bronchial secretion, accounts
for the dyspnea seen in anaphylaxis. Skin symptoms (urticaria and
angioedema) may occur secondary to vasodilatation and increased
vascular permeability of skin capillaries. Increases in GI smooth
muscle tone may result in vomiting, abdominal cramps, and diarrhea.
Epinephrine is the drug of choice for the treatment of anaphylactic
shock due to its ability to reverse all of the pathophysiologic
mechanisms of anaphylaxis. Stimulation of α1 receptors counteracts the
vasodilatation of cutaneous and viscera vasculature, thus increasing
blood pressure. Epinephrine-mediated increases in cardiac contractility
(β1 effect) and cardiac output also increase blood pressure and improve
peripheral perfusion. Epinephrine-induced stimulation of β2 receptors
results in bronchodilatation, making it also a popular choice for the
treatment of severe asthmatic reactions.
(Choice A) Steroids inhibit inflammation by reducing capillary
permeability and suppressing neutrophil activity. Steroids also inhibit
phospholipase A2, resulting in decreased formation of prostaglandin
inflammatory mediators. Because steroids anti-inflammatory effects are
not acute, they are not effective in the acute treatment of life-
threatening anaphylaxis. Epinephrine should be given prior to steroids
and antihistamines in the treatment of anaphylaxis.
(Choice C) Norepinephrine has a predominantly alpha-1 adrenergic effect
thus, it can cause intense vasoconstriction, which may limit cardiac
output. Furthermore, it has little effect on the beta-2 adrenoceptor,
so it has little or no bronchodilator action.
(Choice D) Dobutamine is a synthetic drug with primary beta-i
adrenergic action that can cause an increased cardiac output without
the other effects of epinephrine.
(Choice E) Diphenhydramine is a first generation antihistamine drug
that competitively inhibits peripheral Hi receptors in the GI tract,
blood vessels, and respiratory tract. Diphenhydramine may be used for
the treatment of anaphylaxis after the patient is stabilized with
epinephrine.

33
USMLE WORLD STEP 1 PHARMACOLOGY
Educational Objective:
Anaphylactic shock is characterized by vasodilatation, increased
vascular permeability, and bronchoconstriction. Epinephrine counteracts
these physiological mechanisms and is the drug of choice for the
treatment of anaphylaxis.

34
USMLE WORLD STEP 1 PHARMACOLOGY

Q NO 216: A leak of radioactive material occurs at a plant that handles the disposal
of such waste. A number of workers are believed to have been exposed to this
material which contains heavy isotopes. Which of the following should be
immediately administered to prevent tissue damage in these individuals?

A. Propylthiouracil
B. Potassium iodide
C. Propranolol
D. Prednisone
E. L-thyroxine

Explanation:
The first step in the formation of thyroid hormone is energy-dependent
transport of inorganic iodide into the thyroid follicular cell. This
process is called iodide trapping and is accomplished by the sodium
iodide symporter (NIS) located at the basolateral membrane of the
thyroid follicular cell. The thyroid follicular cells also take up
other ions such as perchlorate and pertechnetate and even radioactive
iodine—all through the NIS. Since all three types of chemicals—
inorganic iodide, other ions, and radioactive iodine—use the same
transporter, each competitively inhibits the other. That is, high
levels of any one substance will significantly reduce the uptake of the
others. Physicians can use this principle by administering potassium
iodide to a person exposed to radioactive isotopes of iodine. By
competitive inhibition, less radioactive material will enter the
thyroid, which will significantly reduce the amount of tissue damage.

35
USMLE WORLD STEP 1 PHARMACOLOGY
(Choice A) Propylthiouracil (PTU) is a thionamide medication used in
the treatment of hyperthyroidism. PTU decreases formation of thyroid
hormone by inhibiting the enzyme thyroid peroxidase. PTU therapy has no
direct effect on iodine uptake.
(Choices C and D) Neither propranolol nor prednisones affect the uptake
of iodine in the thyroid follicular cells.
(Choice E) Levothyroxine, a synthetic form of T4, is used for the
treatment of hypothyroidism. High doses of levothyroxine suppress TSH,
just as high amounts of natural T4 would. Since TSH stimulates iodine
trapping high doses of levothyroxine would ultimately reduce iodine
uptake in the thyroid follicular cells. However, this process requires
several days to weeks. By then, the uptake of radioactive iodine would
already have occurred.

Educational Objective:
Administration of potassium iodide may prevent thyroid absorption of
radioactive iodine isotopes by competitive inhibition.

36
USMLE WORLD STEP 1 PHARMACOLOGY

Q NO 217: An 8-year-old male suffers from fever, headache and nausea. Several
hours later, he experiences serial tonic-clonic seizures. When brought to the ER,
his rectal temperature is 42 C. Which of the following is the best emergency
treatment for this patient?

A. Oral aspirin
B. Rectal acetaminophen
C. IV prednisone
D. Cold blankets

Explanation:
Core body temperature is normally maintained within the range of 37-
38°C. The term “fever” refers to a temperature greater than 38.3°C. The
development of fever is triggered by exogenous pyrogens, inflammatory
processes, drugs or malignancy. Exogenous pyrogens such as bacterial
lipopolysaccharide stimulate the synthesis of endogenous pyrogens such
as IL-1, IL-6 and tumor necrosis factor (TNF). These cytokines enter
the brain with arterial blood and stimulate the release of arachidonic
acid from cell membranes by phospholipase A2. Synthesis of
prostaglandin E2 follows and leads to an increase in the
thermoregulatory set point in the anterior hypothalamus.
Body temperature greater than 40°C is called hyperpyrexia. It can occur
with severe infection, malignant hyperthermia, the neuroleptic
malignant syndrome and in heat stroke. Temperatures above 42 °C may
cause permanent neurological sequelae and lead to mental status
changes, convulsions, delirium and coma. Purkinje cells of cerebellum
are shown to be especially sensitive to heat damage. A body temperature
greater than 43°C is considered incompatible with life.
Hyperpyrexia should be treated immediately to prevent brain damage.
Emergent measures should include facilitation of the heat loss by the
body and decrease of the hypothalamic thermoregulatory set point. Body
heat loss is promoted by increasing evaporation with fans, removal of
clothing and moving to a cool environment. Cooling blankets,
application of cool saline bags to the groin and axilla, and cold water
enemas are also used. Cooling measures should be used in association
with oral antipyretics. NSAIDs and acetaminophen inhibit PGE2
production in brain and thereby lower the hypothalamic temperature set
point.
(Choice A) Aspirin should not be used in children less than 12 years
old, as it is associated with development of Reye syndrome.
(Choice B) Acetaminophen is the antipyretic of choice in children. It
acts by inhibition of cyclooxygenase in the CNS with a resultant
decrease in PGE2 synthesis. Diminished PGE2 levels leads to a decreased
hypothalamic set point. Oral and rectal administration of acetaminophen
is equally effective but cooling of the patient should be performed
first in this setting.
(Choices C) IV prednisone is not used in hyperpyrexia.

Educational Objective:
Body temperature greater than 40°C is called hyperpyrexia and may lead
to permanent brain damage if left untreated. Emergent treatment of
hyperpyrexia should consist of increasing body heat loss (cooling) and
decreasing the hypothalamic set point (antipyretics). Facilitating body
heat loss takes precedence because it is effective immediately as
opposed to antipyretics which take time to act.

37
USMLE WORLD STEP 1 PHARMACOLOGY

Q NO 218: A 70-year-old Caucasian male presents to your office complaining of


muscle rigidity that impairs his daily activities. He was diagnosed with Parkinson
disease 5 years ago and takes levodopa/carbidopa. His initial response to this
medication was allowed him to keeping doing all of his usual activities.
Recently, he has experienced severe restriction in his movements, sometimes
“freezing up” altogether. He also admits to episodes when he “flies” with no
rigidity at all but these episodes happen once in a while and lost for only several
hours. He requests relief because he feels that he can still “do so much” with his
life. In discussing the problem with the patient, which of the following should be
emphasized?

A. Drug response is unpredictable


B. Enhanced drug metabolism is the cause
C. Drug holidays are helpful
D. Stopping carbidopa is the best management
E. The problem is temporary and will resolve spontaneously

Explanation:
The “on-off” phenomenon characteristic of advanced Parkinson disease is
usually unpredictable and dose- independent; it consists of a sudden
loss of the anti-Parkinsonian effects of levodopa (L-dopa), resulting
in hypokinesia and rigidity. The loss of efficacy of L-dopa therapy may
lost for several hours and can occur sporadically throughout the day.
This on-off phenomenon usually develops some time after a patient has
been on L-dopa.
L-dopa is usually administered several times per day. If the L-dopa
dosage is high enough to be effective the patient is in an “on” period.
During an on period the patient is mobile and usually feels well.
However during an “off” period a patient’s status may actually be worse
than if the patient had taken no L-dopa at all. It is postulated that
the amount of L-dopa remaining from the first dose falls below a
certain threshold level and can actually inhibit the voluntary motor
system.
It has been found that if the dose is kept constant the on-off effects
are minimized. Also if higher doses of L-dopa are administered there
are no benefits as to increased motor capabilities and the beneficial
effects lasts longer and delay the onset of off periods. However high
doses of L-dopa also carry an increased risk of dyskinesia—nearly all
patients treated with L-dopa for any period of time will experience
dyskinesia, or uncontrollable choreiform movements of the face and
feet.
(Choice B) Enhanced drug metabolism is the cause of neither the “on-
off” phenomenon (see above explanation) nor the “wearing off”
phenomenon. The wearing off phenomenon is due to progressive
destruction of striatonigral dopaminergic neurons over a period of
time. Unlike the on-off phenomenon the wearing off phenomenon is dose
dependant, and more predictable.
(Choice C) Drug holidays are not particularly helpful in preventing the
“on-off” phenomenon. Drug holidays are no longer used in the management
of Parkinson disease for various reasons.
(Choice D) Stopping carbidopa is not the best management. The
peripheral conversion of levodopa to dopamine is largely responsible
for the nausea and vomiting associated with levodopa use. The addition
of dopa-decarboxylase inhibitors such as carbidopa decreases dopamine
formation peripherally, which decreases side effects.
38
USMLE WORLD STEP 1 PHARMACOLOGY
(Choice E) The “on-off” phenomenon is not temporary and will not
usually resolve spontaneously.

Educational Objective:
The “on-off phenomenon is an unpredictable and dose-independent
characteristic of advanced Parkinson disease—there is no clear etiology
this phenomenon. On the other hand the “wearing off” phenomenon of
Parkinson disease is due to progressive destruction of striatonigral
dopaminergic neurons over a period of time.

39
USMLE WORLD STEP 1 PHARMACOLOGY

Q NO 219: A 64-year-old male with stable angina is being treated with atenolol and
aspirin. He reports that over the last week, his symptoms have been worsening.
You add a new medication to his regimen, but several days later he presents to the
emergency room complaining of severe dizziness. On physical examination, his
blood pressure is 100/70 mmHg and his heart rate is 38 beats per minute. The
medication responsible is most likely:

A. Nifedipine
B. Captopril
C. Verapamil
D. Isosorbide dinitrate
E. Prazosin

Explanation:
When used concomitantly non-dihydropyridine-type calcium channel
blockers (e.g. verapamil, diltiazem) and β- adrenergic blocking agents
can have additive negative effects on heart rate AV node conduction,
and myocardial contractility. Significant sinus bradycardia (or even
occasionally sinus arrest with an AV junctional or idioventricular
rhythm) and hypotension may occur. Patients taking combinations of
these medications should be carefully monitored.
(Choice A) Nifedipine is a dihydropyridine calcium channel blocker with
minimal effects on the sinoatrial node and cardiac conduction. It acts
predominantly as a vasodilator and can cause a reflex increase in heart
rate. It would be unlikely to cause synergistic slowing of the SA node
firing rate when combined with a β-blocker.
(Choice B, D and E) ACE inhibitors (e.g. captopril), nitrates and
peripheral al-selective adrenergic blockers (prazosin) predominantly
cause vasodilatation and reflex tachycardia. Bradycardia is not
expected.

Educational Objective:
Combined use of non-dihydropyridine calcium channel blockers (e.g.
verapamil, diltiazem) and β-adrenergic blockers (e.g. atenolol) can
have additive negative chronotropic effects yielding severe bradycardia
and hypotension.

40
USMLE WORLD STEP 1 PHARMACOLOGY

Q NO 220: Glanzmann thrombasthenia, a rare bleeding disorder, is caused by a lack


of functional platelet surface receptors necessary for aggregation. Treatment with
which of the following drugs can most closely mimic this congenital condition?

A. Heparin
B. Argatroban
C. Warfarin
D. Streptokinase
E. Aspirin
F. Ticlopidine
G. Abciximab
H. Cilostazol

Explanation:
Platelets are responsible for the formation of platelet plugs (primary
hemostasis) which stop bleeding from injured vessels. Injury to a
vessel wall exposes subendothelial collagen and matrix, providing a
surface for the attachment of platelets. Platelets adhere to the vessel
collagen by glycoprotein Ia/IIa. The platelet attachment is
strengthened by the binding of glycoprotein Ib/IXb with von Willebrand
factor on the vessel wall. Attachment of platelets to collagen not only
causes a change in their shape but also causes platelet activation.
Activated platelets then release a number of mediators such as ADP and
thromboxane A2 into circulation, which in turn activates other
platelets. Activation of platelets leads to conformational change of
glycoprotein IIb/IIIa structure, which then can bind to fibrinogen (see
diagram). One molecule of fibrinogen binds to two molecules of
glycoprotein IIb/IIIa. Each platelet has thousands of copies of
41
USMLE WORLD STEP 1 PHARMACOLOGY
glycoprotein IIb/IIIa on its surface. Therefore, binding of
glycoprotein IIb/IIIa-fibrinogen-glycoprotein IIb/IIIa forms a giant
platelet plug. Abciximab is one of the glycoprotein IIb/IIIa inhibitors
that inhibit its binding to fibrinogen. Abciximab and other
glycoprotein IIb/IIIa inhibitors are useful drugs for treatment of
unstable angina and acute coronary syndrome, particularly those
undergoing percutaneous coronary intervention.
Glanzmann thrombasthenia is a disorder caused by deficient or defective
glycoprotein IIb/IIIa on platelet surfaces. Patients usually present in
childhood with mucocutaneous bleeding. Peripheral smear of their blood
shows no platelet clumping, which is an important clue for making a
diagnosis of Glanzmann thrombasthenia.
The other drugs listed do not work through platelet glycoprotein
IIb/IIIa.

Educational Objective:
Abciximab is a blocker of GP IIb/IIIa receptor. GP IIb/IIIa is either
deficient or defective in patients with Glanzmann thrombasthenia.

42
USMLE WORLD STEP 1 PHARMACOLOGY

Q NO 221: A 45-year-old male is being treated with atorvastatin following an acute


myocardial infarction. His blood cholesterol level decreases significantly after
starting treatment. Which of the following is likely to increase?

A. VLDL concentration
B. Sterol absorption
C. LDL receptor density
D. Plasma free fatty acids
E. ApoB 100 concentration
F. Biliary excretion of cholesterol

Explanation:
The statins are used in the treatment of hypercholesterolemia.
Treatment generally produces a 20-50% reduction in serum cholesterol
and LDL concentration. Statins work by inhibiting HMG CoA reductase,
the rate-limiting enzyme of hepatic cholesterol synthesis. In response
to these changes hepatocytes increase their surface expression of the
LDL receptor to increase uptake of circulating LDL.
(Choice A) Statins produce modest decreases in VLDL. Fibrates and
nicotinic acid therapy decrease hepatic production of VLDL and are the
mainstay of treatment for primary hypertriglyceridemia (increased
VLDL).
(Choice B) Ezetimibe decreases serum LDL (cholesterol) levels by
decreasing the intestinal absorption of dietary cholesterol. Statins do
not affect the absorption of dietary cholesterol. When used in
combination statins and ezetimibe produce additive effects towards
reducing serum LDL.
(Choice D) Statins have a minimal effect on plasma free fatty acid
concentration.
(Choice E) ApoB 100 is an apoprotein present in VLDL and LDL. Decreases
in circulating LDL and VLDL will cause the apoB 100 concentration to
decrease as well.
(Choice F) The biliary excretion of cholesterol decreases in patients
on statins due to decreased hepatic synthesis.

Educational Objective:
Treatment with statins causes hepatocytes to increase their LDL
receptor density as a means of increasing their uptake of circulating
LDL.

43
USMLE WORLD STEP 1 PHARMACOLOGY

Q NO 222: A 65-year-old male with atrial fibrillation has been taking warfarin and
atenolol for one year. Two weeks after starting a new drug, he experiences
sudden onset hemiplegia. Transesophageal echocardiography reveals a small
thrombus in the left atrium. The recently added drug was most likely:

A. Penicillin
B. Clarithromycin
C. Phenobarbital
D. Cimetidine
E. Nifedipine
F. Fluconazole
G. Ciprofloxacin

Explanation:
This patient has developed complications from chronic atrial
fibrillation — a left atrial mural thrombus and a thromboembolic stroke
— indicating that recently, his anticoagulation with warfarin has been
inadequate. This question thus asks which of the medications listed has
the potential to decrease warfarin’s anticoagulant effects.
Phenobarbital and other barbiturates can induce hepatic microsomal
enzymes, causing increased metabolism and clearance of warfarin.
(Barbiturates may also increase hepatic synthesis of clotting factors.)
Common cytochrome p450 inducers and inhibitors that all students should
know for Step 1 of the USMLE include:

(Choice A) Oral penicillin V and warfarin do not have any significant


specific interactions. Antibiotics in general reduce the intestinal
bacterial load, which reduces vitamin K synthesis and could potentiate
warfarin’s anticoagulant effects.
(Choice B, D, F, and G) Erythromycin, clarithromycin, cimetidine,
ciprofloxacin, and azole antifungals increase warfarin levels and can
cause bleeding complications.

Educational Objective:
Barbiturates induce the hepatic microsomal enzymes, increasing warfarin
metabolism and reducing its anticoagulant activity.

44
USMLE WORLD STEP 1 PHARMACOLOGY

Q NO 223: A 21-year-old Caucasian male has asthma that is not adequately


A.
controlled with occasional (PRN) albuterol. You decide to start him on
aerosol flunisolide therapy. The patient should be instructed to follow which
of the following recommendations?
Low sodium diet
B. Foot care
C. Oral rinsing
D. Fluid restriction
E. Vitamin supplementation

Explanation:
Inhaled glucocorticoids are the most commonly used prophylactic therapy
for patients with persistent bronchial asthma. Treatment goals for
these patients include:
1. Prevention of symptoms
2. Maintenance of near-normal pulmonary function
3. Maintenance of normal activity levels
Low-dose inhaled glucocorticoids are tolerated yew well and rarely
cause systemic side effects. The most common side effect of inhaled
glucocorticoids is oropharyngeal candidiasis. By using a spacer and
rinsing one’s mouth after glucocorticoid inhalation, patients can avoid
this complication. Dysphonia unrelated to oral candidiasis has also
been reported; this may be due to myopathy of laryngeal muscles.
Systemic effects may be seen with higher doses of inhaled
glucocorticoids. These may include: increased intraocular pressure,
cataracts growth retardation in children, bone loss, and suppression of
the hypothalamic-pituita-adrenal axis. Development of florid Gushing
syndrome from inhaled steroids is exceedingly rare.
(Choices A and D) While fluid retention is occasionally seen in
patients receiving systemic glucocorticoid therapy, it is extremely
rare in patients receiving inhaled glucocorticoids, even at high doses.
Thus, low sodium diets and fluid restriction are not among the
recommendations made to patients beginning inhaled steroid therapy.
(Choice B) Vigilant foot care is a recommendation made to patients
diagnosed with diabetes mellitus.
(Choice E) The relationship of inhaled glucocorticoid use with bone
loss is controversial because many patients taking inhaled
glucocorticoids also receive systemic glucocorticoids periodically.
Patients may take calcium and vitamin D supplements to help meet the
recommended daily allowances, but use of these supplements is not
considered essential for patients taking low-dose inhaled
glucocorticoids.

Educational Objective:
Every patient using inhaled corticosteroids should be instructed about
oral rinsing to prevent oropharyngeal candidiasis.

45
USMLE WORLD STEP 1 PHARMACOLOGY

Q NO 224: A new aminoglycoside antibiotic is developed that is believed to be


particularly effective against Pseudomonas. The volume of distribution of the
drug is measured in a group of volunteers and is determined to be 4.5 L. This
new drug is most likely to have which of the following properties?
A. It has low molecular weight
B. It is lipophilic
C. It does not bind to albumin
D. It is highly charged
E. It has high oral bioavailability

Explanation:
The volume of distribution (Vd) refers to a hypothetical volume of
fluid into which the administered amount of drug would need to be
uniformly distributed to produce the observed plasma concentration. The
volume of distribution is determined by administering a given amount of
drug by the intravenous route and subsequently measuring the initial
plasma concentration of the drug. The formula is as follows:
Vd (L) = amount of drug given (mg) / plasma concentration of drug
(mg/L)
The average total body water is approximately4l liters. Of that the
extracellular fluid volume is about 14 L, or 1/3 of total body water.
With in the extracellular fluid the plasma volume is about3 L, and
interstitial fluid makes up the rest. Initially the drug enters the
plasma compartment by the IV route. If a drug has a large molecular
weight, is bound extensively to plasma proteins or is highly charged
(hydrophilic), then the drug generally remains in the plasma
compartment and the volume of distribution is usually low as in the
case above (about 3-5 liters). If the drug has a small molecular weight
but is hydrophilic, it can distribute into the interstitial fluid
compartment outside of the blood vessels as well as in the
intravascular compartment. In these cases the volume of distribution is
limited to a total volume of about 14-16 liters (plasma volume plus
interstitial volume). If the drug has a small molecular weight and is
also uncharged (hydrophobic or lipophilic), then the drug can cross
cell membranes and reach intracellular compartment. These drugs have
the highest volume of distribution (41 liters). Drugs that are avidly
bound in the tissues exhibit the highest volumes of distribution often
much higher than the total body water volume, because these drugs
accumulate readily within cells thereby maintaining low plasma
concentrations.
(Choice A) lf the molecular weight of the drug is low then the drug
will tend to easily pass through endothelial junctions into the
interstitial fluid making the volume of distribution at least 14
liters. The Vd may be higher if the drug is also hydrophobic.
(Choice B) Lipophilic drugs tend to readily cross cell membranes and
distribute widely outside the plasma and interstitial compartments.
This tendency to collect within cells gives these drugs a high volume
of distribution.
(Choice C) A drug with a Vd of 4.5 L which is approximately the same as
the plasma volume is likely to be highly bound to plasma proteins such
as albumin. Binding to plasma proteins tends to retain drug in the
plasma compartment and prevent the diffusion of drug into the
extravascular compartments. If this drug were not bound to albumin it
would be more likely to diffuse into the interstitium and a higher Vd
would be expected.

46
USMLE WORLD STEP 1 PHARMACOLOGY
(Choice E) Bio availability is a measure of absorption and is unrelated
to the distribution of a drug.

Educational Objective:
Characteristics of a drug such as high molecular weight high plasma
protein binding, high charge, and hydrophilicity tend to trap the drug
in the plasma compartment resulting in a low Vd (3-5 L).

47
USMLE WORLD STEP 1 PHARMACOLOGY

Q NO 225: A 23-year-old African-American female with a history of sickle cell


anemia is admitted with severe, diffuse body pains. She has already had five
similar episodes this year. Her physician considers adding hydroxyurea to her
treatment regimen. Which best describes the mechanism of action of
hydroxyurea?
A. Increases hemoglobin A 2 synthesis
B. Increases hemoglobin E synthesis
C. Decreases cellular dehydration
D. Directly inhibits polymerization
E. Stimulates red cell production

Explanation:
Sickle cell anemia is the most common clinically-significant hemoglobin
abnormality seen in the United States. Hemoglobin S (Hb 3) is the
result of a point mutation that substitutes a valine in the sixth
position of the beta chain for glutamic acid. The abnormal beta chain
of Hb S causes it to polymerize when deoxygenated or dehydrated
resulting in the characteristic sickling of red blood cells.
Heterozygotes for Hb S tend to be asymptomatic, but homozygotes have
severe disease.
Recent therapies for sickle cell disease include hydroxyurea.
Hydroxyurea, a drug originally developed to battle neoplasia, increases
fetal hemoglobin (Hb F) synthesis through an incompletely understood
mechanism. An increased percentage of Hb F confers protection against
the polymerization of sickle cells.
(Choice C) Preventing intracellular dehydration is a treatment modality
for sickle cell anemia that is employed not by hydroxyurea, but by
Gardos channel blockers. The calcium-dependent (Gardos) potassium
channel regulates the transport of potassium and water through the red
blood cell membrane—when blocked, potassium and water efflux is
reduced, preventing dehydration of erythrocytes and reducing the
polymerization of Hb S.
(Choice D) Direct chemical inhibition of polymerization is being
studied at this time, but is not the mechanism of action of
hydroxyurea.
(Choice E) Erythropoietin stimulates red cell production.

Educational Objective:
1. Hydroxyurea increases fetal hemoglobin (Hb E) synthesis by an
unknown mechanism. Hydroxyurea is reserved for patients with frequent
pain crises.
2. Gardos channel blockers hinder the efflux of potassium and water
from the cell, preventing dehydration of erythrocytes and reducing the
polymerization of Hb S.

48
USMLE WORLD STEP 1 PHARMACOLOGY

Q NO 226: A 34-year-old male diagnosed with acute myelogenous leukemia


A.
recently underwent successful induction chemotherapy with doxorubicin.
Several weeks later, he presents to your office complaining of progressive
exertional dyspnea and orthopnea. Which of the following is most likely
responsible for this patient’s symptoms?
Right ventricular overload
B. Focal myocardial scarring
C. Restrictive cardiomyopathy
D. Hypertrophic cardiomyopathy
E. Dilated cardiomyopathy
F. Pericardial fibrosis

Explanation:
The anthracyclines (daunorubicin, doxorubicin, epirubicin and
idarubicin) are chemotherapeutic agents associated with severe
cardiotoxicity. The generation of free radicals is implicated in the
unique ability of these agents to cause cardiotoxicity.
Dilated cardiomyopathy is cumulative dose-dependent and may present
many months after discontinuation of the drug. Swelling of the
sarcoplasmic reticulum is the morphologic sign of an early stage of
doxorubicin-associated cardiomyopathy. It is followed by loss of
cardiomyocytes (“myofibrillar dropout”). Its symptoms are those of
biventricular CHF including dyspnea on exertion orthopnea, and
peripheral edema.
The most effective method of preventing doxorubicin cardiomyopathy is
dexrazoxane. Itis an iron-chelating agent that decreases formation of
oxygen free radicals by doxorubicin and other anthracyclines.
(Choice A) Right ventricular overload (cor pulmonale) presents with
fatigue dyspnea on exertion, and peripheral edema. Physical examination
reveals accentuation and splitting of the pulmonary component of 52,
distended neck veins and hepatomegaly with hepatojugular reflux. Right
ventricular failure is commonly a consequence of pulmonary hypertension
or left heart failure.
(Choice C) Restrictive cardiomyopathy is associated with
hemochromatosis, amyloidosis, and sarcoidosis and radiation therapy.
(Choice D) Hypertrophic cardiomyopathy is an autosomal dominant
disorder caused by mutation of the β-myosin heavy chain.
(Choice B) Focal myocardial scarring commonly results following a
myocardial infarction.
(Choice F) Pericardial fibrosis usually follows cardiac surgery,
radiation therapy or viral infections of the pericardium.

Educational Objective:
The anthracycline chemotherapeutic agents (doxorubicin, daunorubicin,
epirubicin and idarubicin) form free radicals in the myocardium. Their
most severe side effect is a cumulative dose-related dilated
cardiomyopathy. It presents with symptoms of left and right ventricular
CHF.

49
USMLE WORLD STEP 1 PHARMACOLOGY

Q NO 227: A 23-year-old male is treated with clozapine for schizophrenia after


unsuccessful trials of other medications. He experiences significant
improvement of symptoms several weeks after initiation of the treatment.
Which of the following should be followed periodically in this patient?
A. Electrocardiogram
B. Complete blood count
C. Liver function tests
D. Creatinine and BUN
E. TSH and T3/T4

Explanation:
Straightforward questions about clozapine causing granulocytopenia are
quite common. Clozapine is more effective in treatment-resistant
schizophrenic patients for both positive psychotic symptoms (delusions
and hallucinations), and negative symptoms (social withdrawal
anhedonia, blunted affect, and poor initiative). Unlike other
traditional antipsychotics, which act on D2 receptors, clozapine acts
on D4 receptors. Clozapine is not likely to cause dopaminergic side
effects such as pseudoparkinsonism, tardive dyskinesia, and
hyperprolactinemia.
Due to the risk of life-threatening agranulocytosis with clozapine, the
FDA requires periodic monitoring of white blood cell count (WBC) for
the duration of treatment. The other important side effect of clozapine
is seizures.
(Choice A) Among all the antipsychotics, ziprasidone is the one that is
notorious for causing prolonged QT. Thus ECGs should be obtained in
these patients.
(Choice C) Liver function tests maybe mildly elevated with use of many
medications such as carbamazepine, which may cause a transient rise in
hepatic enzymes, or isoniazid (INH), which can cause full-blown
hepatitis.
(Choice D) Aminoglycosides and vancomycin are the drugs commonly known
to cause renal failure.
(Choice E) TSH and T3/T4 should be assayed with the use of medications
such as lithium, which can cause hypothyroidism, and amiodarone, the
cardiac medication which may also cause hypothyroidism.

Educational Objective:
Unlike other traditional antipsychotics, which act on D2 receptors,
clozapine acts on D4 receptors. Due to the risk of life-threatening
agranulocytosis with clozapine, the FDA requires periodic monitoring of
the white blood cell count (WBC) for the duration of treatment. The
other important side effect of clozapine is seizures.

50
USMLE WORLD STEP 1 PHARMACOLOGY

Q NO 228: A 34-year-old male is treated with prednisone for relentless bronchial


asthma. The frequency and severity of his attacks decrease as a result of this
treatment. Which of the following tissues is most likely to increase its protein
synthesis in response to this therapy?

A. Skeletal muscles
B. Skin
C. Lymphoid tissue
D. Bones
E. Liver

Explanation:
Cortisol is a potent stimulator of liver gluconeogenesis. Thus, there
is an increase in liver protein synthesis secondary to glucocorticoid
administration due to an elevation in the levels of the enzymes of
gluconeogenesis and glycolysis. Cortisol also antagonizes the actions
of insulin in muscle and adipose tissue, thus favoring catabolism of
these tissues. This proteolysis and lipolysis provides the substrates
for gluconeogenesis.
(Choice A) Cortisol’s effect on skeletal muscle is predominantly
proteolytic. Proximal muscle weakness is a typical feature of cortisol
excess. (The muscle weakness of cortisol excess is also secondary to
the hypokalemia induced by cortisol’s mineralocorticoid effects.)
(Choice B) Cortisol inhibits fibroblast proliferation and collagen
formation in the skin. As a result, there is skin thinning in the
presence of excessive amounts of cortisol, making the skin more
susceptible to damage from minor trauma. Purple striae are one of the
clinical manifestations of this change. The connective tissue that
supports the capillaries is also thinned; thus capillary injury and
easy bruising are also common features of cortisol excess. Wound
healing is impaired as well.
(Choice C) Cortisol is a potent immunosuppressive agent.
Pharmacological doses of glucocorticoids are used as immunosuppressants
in organ transplantation patients. High cortisol levels decrease the
number of circulating T lymphocytes particularly helper T-lymphocytes,
and decrease their ability to migrate to the sites of antigenic
stimulation. Eosinophil counts also decrease with cortisol excess.
Neutrophil counts actually increase acutely upon glucocorticoid
administration, as a result of “demargination” from the blood vessel
wall.
(Choice D) Cortisol decreases bone mass by multiple mechanisms
(decrease gastrointestinal calcium absorption: increase renal calcium
excretion; directly inhibit osteoblast bone-forming activity). Because
of these effects, patients on corticosteroid therapy should receive
calcium and vitamin D supplementation to prevent osteoporosis.

Educational Objective:
Corticosteroids are predominantly catabolic, causing muscle weakness,
skin thinning, impaired would-healing, osteoporosis, and
immunosuppression. However, there is an increase in liver protein
synthesis (enzymes for glycogen synthesis and gluconeogenesis), which
results in hyperglycemia.

51
USMLE WORLD STEP 1 PHARMACOLOGY

Q NO 229: A new class of drugs called vasopeptidase inhibitors is developed. The


drugs in this class inhibit the angiotensin converting enzyme and inhibit
degradation of natriuretic peptide. Which of the following adverse effects should
be expected due to these pharmacologic properties?

A. Cold extremities
B. Throbbing headaches
C. Sodium and fluid retention
D. Angioedema
E. Cardiac arrhythmias

Explanation:
Vasopeptidase inhibitors such as omapatrilat are not yet available in
the United States but their dual action of inhibiting angiotensin
converting enzyme (ACE) and promoting vasodilatation and diuresis by
the natriuretic peptide mechanism makes them attractive as
antihypertensives, especially in heart failure. This class of drugs
inhibits ACE just as the ACE inhibitors do. The ACE inhibitors are
associated with decreased destruction of kinins (which are metabolized
by ACE) and kinins have been implicated as one of the causes of
angioedema. So by inhibiting ACE, the vasopeptidase inhibitors decrease
the destruction of kinins and lead to a high incidence of angioedema
(Choice D)
(Choice A) Cold extremities can be a symptom of Raynaud phenomenon.
Raynaud phenomenon most frequently affects the hands and is caused by
inappropriate constriction of the blood vessels of the extremities in
response to cold ambient temperatures or stressful situations. It can
be primary when it exists in the absence of other disease or it can be
secondary when itis associated with connective tissue diseases.
Frequently the symptoms of Raynaud phenomenon are treated with calcium
channel blockers.
(Choice B) Throbbing headache is the most common side effect of
nitroglycerin a drug used to treat the symptoms of angina.
(Choice C) Sodium and fluid retention is caused by aldosterone which
has the effect of resorbing sodium and water and wasting potassium and
hydrogen ions (acid) in the distal portion of the nephron.
(Choice E) Cardiac arrhythmias can be caused by drugs including digoxin
and many of the other antiarrhythmic medications.

Educational Objective:
Even if you are not familiar with a drug class you can often
rationalize the correct answer if you are provided with the mechanism
of action of that drug class in the question stem. ACE metabolizes
kinins, and increased plasma levels of kinins are associated with
angioedema, which can be a life threatening condition.

52
USMLE WORLD STEP 1 PHARMACOLOGY

Q NO 230: A 62-year-old male presents to ER with severe shortness of breath and


inability to lie down. His past medical history is significant for long-standing
hypertension and myocardial infarction experienced one year ago. After initial
treatment the patient experiences brisk diuresis and significant relief of symptoms.
The drug used to treat this patient’s condition predominantly acts on which of the
following nephron segments?

A. A
B. B
C. C
D. D
E. E
F. F

Explanation:
Loop diuretics work by inhibiting Na-K-2C symporters in the ascending
limb of the loop of Henle. Loop diuretics bind to symporters and
effectively block Na and Cl transport resulting in increased Na, Cl and
fluid excretion. Since the ascending limb of the loop of Henle has
significant reabsorptive capacity, loop diuretics are the most potent
diuretics with excellent efficacy. Since only a small portion of
filtered Na reaches distal tubules, diuretics that work beyond the loop
of Henle are not as efficacious.
Available loop diuretics differ in potency and half-life and include
furosemide, torsemide, bumetanide, and ethacrynic acid. They are
commonly used in treating pulmonary edema, venous and pulmonary
congestion secondary to congestive heart failure and peripheral edema.
Common side effects include hypokalemia, hypomagnesemia, and
hypocalcemia. Less common side effects include volume depletion,
hyponatremia, decreased GEP, hypotension, and ototoxicity.
(Choice A)The proximal tubule has a significant amount of the enzyme
carbonic anhydrase which is responsible for accelerating the reaction
necessary for NaHCO3 reabsorption in the proximal tubule. Carbonic

53
USMLE WORLD STEP 1 PHARMACOLOGY
anhydrase inhibitors block the reabsorption of NaHCQ3 and work in the
proximal tubule.
(Choice B) This area is the straight part of the proximal tubule and
also contains an excess of the carbonic anhydrase enzyme for carbonic
anhydrase inhibitors to effectively work.
(Choice C) The descending limb of the loop of Henle carries fluids from
the proximal tubule to the ascending limb of the loop of Henle in the
medulla. The descending limb is very permeable to water, allowing water
to diffuse into interstitial fluids to produce more concentrated
tubular fluid. No diuretics are known to work in the descending limb of
the loop of Henle.
(Choice E) The distal tubule actively transports Na and Cl and is
impermeable to water. Thiazide diuretics work in the distal tubule.
(Choice F) The collecting duct system includes the connecting tubules
and ducts. Here aldosterone and ADH make final adjustments to
electrolytes and water content. Potassium sparing diuretics and
aldosterone antagonists also work in the collecting duct.

Educational Objective:
Loop diuretics work by inhibiting Na-K-2Cl symporters in the ascending
limb of the loop of Henle and effectively block Na and Cl transport
resulting in increased Na, Cl and fluid excretion. They are the most
potent class of diuretics and are used for treating edema in many
different conditions. Common side effects include hypokalemia,
hypomagnesemia, and hypocalcemia.

54
USMLE WORLD STEP 1 PHARMACOLOGY

Q NO 231: A 60-year-old male is hospitalized with an arrhythmia. The agent used


to treat this patient’s condition is known to significantly prolong the QT interval on
ERG, but despite this the drug is associated with a low incidence of torsade de
pointes. Which of the following agents was most likely used in this patient?

A. Amiodarone
B. Esmolol
C. Lidocaine
D. Procainamide
E. Verapamil
F. Adenosine
G. Digoxin

Explanation:
Amiodarone is a class Ill anti arrhythmic drug that also exhibits
properties of class I, II and IV agents. It is commonly known to
prolong the QT interval via its effect on potassium mediated
repolarization (phase 3) of myocardial cells. QT interval prolongation
for any reason is associated with an increased risk of a form of
ventricular tachycardia called torsade de pointes, but interestingly
the prolongation caused by amiodarone is associated with low risk of
torsade de pointes unlike that caused by class IA drugs or most other
class Ill agents.
(Choice B) Esmolol is a rapidly acting and rapidly cleared beta-
adrenergic blocking agent and is a class II antiarrhythmic. The beta-
blockers have no effect on the OT interval.
(Choice C) Lidocaine is a class IB antiarrhythmic drug. It does not
have much effect on QT interval or PR interval.
(Choice D) Procainamide is a class IA antiarrhythmic drug. Procainamide
can cause a prolongation of the OT interval just as amiodarone does,
but the prolongation of the QT caused by procainamide DOES predispose
to torsade de points, and the prolongation caused by amiodarone does
not.
(Choice E) Verapamil is a cardioselective calcium channel blocker and
is a class IV antiarrhythmic. Verapamil would prolong the PR interval
on the EKG and could predispose to AV nodal block, but it would not
prolong the QT interval.
(Choice F) Because its primary effect is on the AV node, one would
expect that adenosine’s effect on the EKG would lie in prolonging the
PR interval. Because itis so quickly cleared (T1/2 = <10 seconds) any
effect it would have on the EKG would be transient.
(Choice C) Digoxin is a commonly used drug which has the effect of
prolonging AV nodal conduction by stimulation of parasympathetic
signals there as well as increasing cardiac contractility by increasing
intracellular calcium in the ventricular myocardium. It will prolong
the PR interval on the ERG, but it has no effect on the QT interval.

Educational Objective:
Amiodarone along with the class IA and the remainder of the class II
antiarrhythmic agents will cause a prolongation of the OT interval on
the EKG. These drugs have this effect because they all slow phase 3
repolarization in the ventricular myocardium. Amiodarone is unique in
that it does not predispose to torsade de pointes as the remainder of
these agents do.

55
USMLE WORLD STEP 1 PHARMACOLOGY

Q NO 232: In animal experiments the insulin concentration in the pancreatic veins


is measured after the administration of various agents. Epinephrine administration
causes a decrease in insulin. After pretreatment with Drug A, however,
epinephrine causes a paradoxical increase in insulin concentration in the
pancreatic veins. Drug A is most likely which of the following agents?

A. Diphenhydramine
B. Propranolol
C. Phenoxybenzamine
D. Ephedrine
E. Oxybutynin

Explanation:
Besides glucose other factors are also involved in insulin secretion.
Parasympathetic and sympathetic nerves innervate beta cells.
Parasympathetic activation is responsible for insulin secretion and is
induced by the smell and/or sight of food. The sympathetic effect on
beta cells can be either inhibitors or stimulatory, depending on the
type of receptor activation. Stimulation of alpha-adrenergic receptors
inhibits insulin release, whereas stimulation of betaadrenergic
receptor stimulates insulin secretion. Phenoxy benzamine is a long-
acting alpha-blocker commonly used for the treatment of
pheochromocytoma. Since epinephrine can bind to both alpha and beta
receptors, blockage of alpha-receptors allows more epinephrine bind to
the beta-receptors and results in an increase in insulin secretion.
(Choice A) Diphenhydramine is a commonly used antihistaminic agent that
acts by blocking the Hi histamine receptor. Diphenhydramine also has
anticholinergic actions, which can abort insulin secretion induced by
parasympathetic stimulation. However diphenhydramine will not interfere
with the action of epinephrine on beta cells.
(Choice B) Propranolol is a non-cardioselective beta-blocker. Beta-
blockers inhibit insulin secretion by blocking the stimulatory effect
of beta-receptors on insulin producing beta cells. This property of
beta-blockers might be responsible for the increased risk of diabetes
mellitus associated with long-term beta-blocker use.
(Choice D) Ephedrine, like epinephrine, is a sympathomimetic agent.
Pretreatment with ephedrine would have an additive effect to
epinephrine.
(Choice E) Oxybutynin is an anticholinergic agent used for the
treatment for overactive bladder (urge incontinence). Oxybutynin acts
as an antispasmodic agent by providing antimuscarinic action to smooth
muscle cells. It does not block the nicotinic actions of acetylcholine,
however, and will therefore not block the predominantly nicotinic
action of acetylcholine on insulin secretion.

Educational Objective:
Alpha-receptors inhibit insulin secretion and beta-receptors stimulate
insulin secretion. Pretreatment with an alpha blocker would result in
predominance of beta-effects.

56
USMLE WORLD STEP 1 PHARMACOLOGY

Q NO 233: A 75-year-old male is hospitalized after an uncomplicated right carotid


endarterectomy. He experienced myocardial infarction two years ago and
underwent coronary artery bypass surgery for stable angina one year ago. Which
of the following should be considered most if treatment with ramipril is
contemplated in this patient?

A. Aortic regurgitation
B. Post-infarction left ventricular aneurysm
C. Severe iliofemoral occlusive disorder
D. Bilateral renal artery stenosis
E. First degree AV block
F. Polymorphic ventricular extrasystoles

Explanation:
Optimal blood pressure control is one of many important therapeutic
goals in managing patients with cardiovascular and peripheral vascular
disease. Although no specific agent is recommended, ACE inhibitors do
have established benefits in this patient population. The HOPE trial
demonstrated that ACE inhibitors not only reduce blood pressure, but
also decrease the risk for cardiovascular events (MI1 stroke, death) in
high risk patients with cardiovascular and peripheral vascular disease.
Patients with atherosclerotic cardiovascular and peripheral vascular
disease can also develop renovascular disease, manifesting as renal
artery stenosis. Patients with renal artery stenosis are dependent on
ACE mediated efferent arteriole constriction to maintain renal
perfusion and GFP. Use of ACE inhibitors will block this effect and
reduce renal perfusion potentially leading to APE. Because ACE
inhibitors can induce APE in patients with renal artery stenosis, they
are considered contraindicated in patients with bilateral renal artery
stenosis and unilateral stenosis in a solitary functioning kidney. So
although patients with extensive atherosclerosis do benefit from ACE
inhibitors, it is important to consider the presence of bilateral renal
artery stenosis before initiating ACE inhibitor therapy.
(Choice A) For aortic regurgitation, ACE inhibitors have been shown to
decrease left ventricular volume in some studies and are often widely
prescribed.
(Choice B) ACE inhibitors have been shown to reduce infarct expansion
and progressive left ventricular remodeling in post-infarction left
ventricular aneurysm and thus have an established therapeutic role.
(Choice C) In patients with iliofemoral occlusive disorder, ACE
inhibitors may be of significant benefit. Based on the HOPE trial, ACE
inhibitors not only reduce blood pressure, but also decrease the risk
for cardiovascular events (Ml, stroke, death) in high risk patients
with cardiovascular and peripheral vascular disease.
(Choice E) ACE inhibitors have no direct effect on heart conduction and
thus would not be problematic in the setting of first degree AV block.
(Choice F) In general, there is no therapeutic role for ACE inhibitors
in polymorphic ventricular extrasystoles. ACE inhibitors have no direct
effect on heart conduction. So for those patients who may require ACE
inhibitors for a concomitant condition, they can be used safely without
any contraindications or detrimental effects.

57
USMLE WORLD STEP 1 PHARMACOLOGY
Educational Objective:
The HOPE trial demonstrates the potential benefits of ACE inhibitors in
patients with significant cardiovascular and peripheral vascular
disease. Although ACE inhibitor therapy should be considered in most
patients the development of APE may preclude the use of these agents in
patients with extensive atherosclerotic disease and concomitant
bilateral renal artery stenosis.

58
USMLE WORLD STEP 1 PHARMACOLOGY

Q NO 234: Nitrates are useful agents for treatment of the symptoms of ischemic
heart disease. Oral nitrate preparations can be used for chronic treatment of
stable angina. Which of the following nitrate agents has the highest
bioavailability if given orally?

A. Nitroglycerin
B. Isosorbide dinitrate
C. Isosorbide mononitrate
D. Amyl nitrite
E. Sodium nitroprusside

Explanation:
To give a drug orally (PO) is to have a patients wallow the agent and
absorption of that agent then takes place in the gut where it must
undergo first-pass metabolism in the liver before entering the systemic
circulation. Bio availability of a drug refers to the percentage of the
active metabolite of the administered medication that survives first-
pass metabolism to enter the systemic circulation. Intravenous
administration of a drug always has 100% bioavailability. Isosorbide 5-
mononitrate is an active metabolite of isosorbide dinitrate; its
bioavailability is close to
100%.
(Choice A) Nitroglycerin is usually given sublingually (SL) where it is
directly absorbed into the systemic circulation through the oral
mucosa. This provides for the rapid onset of action of nitroglycerin.
If this drug were to be swallowed it would undergo significant first-
pass metabolism in the liver.
(Choice B) Isosorbide dinitrate is the parent compound of isosorbide
mononitrate. It undergoes extensive first-pass metabolism by
glucuronidation in the liver.
(Choice D) Amyl nitrate is a volatile inhalant used historically in the
treatment of angina pectoris but its side effects of extreme cutaneous
flushing and postural hypotension limit its use. It is not possible to
take this medication orally.
(Choice E) Sodium nitroprusside is a drug that is only available as an
intravenous agent; it is used in the treatment of hypertensive
emergency. It is most notable for its ability to cause cyanide
toxicity.

Educational Objective:
Nitroglycerin and Isosorbide dinitrate undergo considerable first-pass
metabolism in the liver when taken by the oral route. Interestingly,
isosorbide mononitrate is nearly 100% bioavailable when taken by the
oral route.

59
USMLE WORLD STEP 1 PHARMACOLOGY

Q NO 235: A 72-year-old female with chronic renal insufficiency is hospitalized for


moderate shortness of breath and peripheral edema. She also complains of fever
burning during urination and frequency. S he is treated with gentamicin for
urinary tract infection. Several days after admission the patient feels much better
but notes that she has difficulty hearing. She turns her TV very loud disturbing her
family members. Which of the following drugs can be responsible for this patient’s
problem?

A. Hydrochlorothiazide
B. Furosemide
C. Spironolactone
D. Triamterene
E. Mannitol
F. Acetazolamide

Explanation:
This patient unfortunately has both urinary tract infection and
pulmonary and peripheral edema due to chronic renal insufficiency.
Patients with chronic renal insufficiency usually require higher dose
of diuretics for good therapeutic response. Loop diuretics work by
inhibiting Na-R-2Cl symporters in the ascending limb of the loop of
Henle. Available loop diuretics differ in potency and half-life and
include furosemide, torsemide, bumetanide, and ethacrynic acid. They
are commonly used in treating pulmonary edema secondary to congestive
heart failure or renal failure.
Common side effects include hypokalemia, hypomagnesemia, and
hypocalcemia. Less common side effects include volume depletion
hyponatremia, decreased SEP. hypotension, and ototoxicity. Ototoxicity
secondary to loop diuretics usually occurs with higher dosages, rapid
intravenous administration or when loops are used in combination with
other ototoxic agents (aminoglycosides, salicylates, cisplatin).
Although the exact mechanism of ototoxicity is unknown itis believed to
due to effects on Na-K-2C1 symporters in the inner ear. It typically
presents as tinnitus, vertigo, hearing impairment, or deafness. Hearing
impairment and deafness are usually reversible but has been reported to
be permanent in some cases. Of the loop diuretics ethacrynic acid
appears to have the greatest risk for ototoxicity.
(Choice A) More common side effects of HCTZ include hypokalemia,
hyponatremia and hypomagnesemia. Less common side effects include
hypotension, volume depletion, and hypercalcemia.
(Choice C) Spironolactone is a mild diuretic that works by antagonizing
the effects of aldosterone in the distal tubule and collecting duct. By
blocking the effects of aldosterone, spironolactone alters Na
conductance, leading to enhanced sodium and fluid excretion.
Hyperkalemia is one of the most common side effects of spironolactone.
Other common side effects include gynecomastia, impotence, and
decreased libido.
(Choice D) Triamterene is a potassium sparing diuretic that works by
blocking Na channels in the distal tubule and collecting duct. This
results in increased Na and fluid excretion and hyperkalemia.
(Choice E) Mannitol is an osmotic diuretic that works by increasing
plasma and tubular osmolality resulting in increased Na and fluid
extraction. Common side effects include nausea, vomiting, headache, and
hyponatremia.

60
USMLE WORLD STEP 1 PHARMACOLOGY
(Choice F) Acetazolamide is a diuretic that works by inhibiting the
enzyme carbonic anhydrase. Common side effects of carbonic anhydrase
inhibitors include somnolence, paresthesias, and urine alkalinization.

Educational Objective:
Ototoxicity secondary to loop diuretics usually occurs with higher
dosages, rapid intravenous administration, or when loops are used in
combination with other ototoxic agents (aminoglycosides, salicylates,
and cisplatin). Hearing impairment is usually reversible but has been
reported to be permanent in some cases.

61
USMLE WORLD STEP 1 PHARMACOLOGY

Q NO 236: A 63-year-old male is hospitalized with unstable angina and is treated


with continuous heparin infusion. Several hours after hospitalization he
experiences bloody vomiting and hypotension. Heparin infusion is stopped. Which
of the following can be used to reverse the anticoagulant effect of heparin?

A. Vitamin K
B. Protamine
C. Aminocaproic acid
D. Fresh frozen plasma
E. Tranexamic acid

Explanation:
Unfractionated heparin is the most commonly used anticoagulant in the
hospital setting. Heparin increases the effect of the naturally
occurring anticoagulant antithrombin-lll. Activated partial
thromboplastin time (aPTT) is measured to monitor the therapeutic
effect of heparin. Bleeding and heparin-induced thrombocytopenia are
important complications of heparin therapy. Bleeding due to heparin
toxicity is treated with protamine sulfate a specific antagonist of
heparin. Protamine sulfate is a peptide that binds to heparin forming a
complex that has no anticoagulant activity. Protamine is not very
effective in treating toxicity caused by low molecular weight heparin.
(Choices A and D) Vitamin K is used for reversal of warfarin overdose.
However, it requires new synthesis of coagulation factors and thus
takes some time. For this reason, in acute life threatening bleeding,
fresh frozen plasma should be given. Fresh frozen plasma contains
clotting factors.
(Choices C and E) Aminocaproic acid and tranexamic acid inhibits
fibrinolysis. They inhibit plasminogen activation.

Educational Objective:
Protamine sulfate binds with heparin causing chemical inactivation.
Vitamin K is used for reversal of warfarin effect. Aminocaproic acid
and tranexamic acid inhibits fibrinolysis.

62
USMLE WORLD STEP 1 PHARMACOLOGY

Q NO 237: A group of investigators are studying pharmacokinetic properties of


thiopental. A dose-time relationship in various tissues after a single bolus of
the drug is presented on the graph below. The blue line on the graph most
likely corresponds to which of the following tissues?

A. Liver
B. Kidney
C. Pancreas
D. Skeletal muscle
E. Lungs

Explanation:
Thiopental is a general anesthetic administered intravenously. It is a
short-acting barbiturate that affects GABA receptors and increases
their inhibitory potential. Thiopental is highly lipid-soluble. It
equilibrates in brain tissue within 1 mm after its administration and
causes loss of consciousness. Accumulation of thiopental in the brain
is followed by its rapid “redistribution” into skeletal muscles and
adipose tissue (within 5-10 mm of administration). In a classic
pharmacologic study, researchers demonstrated that awakening from
thiopental occurred because the plasma level declined rapidly. They
further demonstrated that the cause of the rapid plasma decay of
thiopental was not metabolism of the drug but rather redistribution of
the drug to other tissues throughout the body (lean tissue). This rapid
brain clearance leads to recovery from anesthesia. The blue curve on
the diagram above shows redistribution of thiopental into the skeletal
muscles and fat.
Due to its rapid onset of action and rapid recovery, thiopental is used
for induction of anesthesia. General anesthesia is usually maintained
with an inhaled anesthetic.
(Choices A, B, C and E) Peripheral redistribution of thiopental results
in its presence in many organs including liver, kidney, pancreas and
lungs. Its highest concentrations, however, are found in skeletal
muscles and adipose tissue.

Educational Objective:
Thiopental is a short-acting barbiturate used for induction of
anesthesia. After equilibration with the brain it rapidly redistributes
into skeletal muscles and adipose tissue, which results in rapid
recovery from anesthesia.

63
USMLE WORLD STEP 1 PHARMACOLOGY

Q NO 238: A 56-year-old male with type 2 diabetes mellitus is treated with


A.
pioglitazone. Two weeks after initiation of therapy, there is no meaningful
improvement in glycemic control. You soothe the patient’s concerns that
‘this new medication just isn’t working” by explaining that the mechanism
of action of the drug involves which of the following?
Hormone secretion
B. Decrease glucose absorption
C. Receptor down-regulation
D. Enzyme inactivation
E. Transcription modulation
Explanation:
Thiazolidinediones (TZD5) are new group of antidiabetic medications
that exert their glucose-lowering effect by improving insulin
resistance. TZDs decrease insulin resistance by binding to peroxisome
proliferator activated receptor gamma (PPAR-gamma), which is a
transcriptional regulator of the genes involved in glucose and lipid
metabolism. PPAR-gamma belongs to the nuclear receptor superfamily that
also includes thyroid receptors, and all of which regulate gene
expression after ligand binding. When TZD binds to PPAR-gamma, a
conformational change ensues, which allows another coactivator to
attach. This complex then binds to the transcriptional regulatory
sequence of genes responsible for glucose and lipid metabolism in a
process is called “transactivation.”
The most important gene regulated by PPAR-gamma is adiponectin, a
cytokine secreted by fat tissue (generically termed “adipocytokine”).
Adiponectin levels are low in type 2 diabetes—treatment with TZDs
increases adiponectin levels. Increased levels of adiponectin are one
of the main mechanisms by which TZDs decrease insulin resistance.
As the glucose-lowering effect of TZDs requires alteration in gene
transcription and protein production it takes few days to weeks after
commencing TZD therapy to observe significant reduction in glucose
levels, which is the case with the patient in this scenario.
(Choice A) The glucose-lowering effect of sulfonylureas is achieved by
increasing insulin release from the pancreatic beta cells.
(Choice C) Receptor downregulation typically leads to insulin
resistance and hyperglycemia. High-dose insulin therapy sometimes
causes receptor downregulation through accelerated degradation of the
insulin receptor.
(Choices B and D) Enzyme inactivation is an important mechanism of
action for metformin and alpha-glucosidase inhibitors. Metformin
decreases the enzyme activity responsible for gluconeogenesis and
causes a significant reduction in hepatic glucose production.
Alpha-glucosidase inhibitors (acarbose and miglitol) decrease the
enzyme activity of disaccharidases found in the plasma membranes of the
intestinal brush border. Because carbohydrates are absorbed as
monosaccharides the prevention of disaccharide breakdown by alpha-
glucosidase inhibitors, delays the absorption of carbohydrates.
Educational Objective:
1. TZD5 decrease insulin resistance by binding to peroxisome
proliferator activated receptor gamma (PPAR-gamma), which is a
transcriptional regulator of genes involved in glucose and lipid
metabolism.
2. Adiponectin levels are low in type 2, diabetes and treatment with
TZDs increases the levels of adiponectin.

64
USMLE WORLD STEP 1 PHARMACOLOGY

Q NO 239: An 18-year-old presents to her physician because of severe cutaneous


erythema, blistering, and desquamation on the face and other sun-exposed areas.
She was recently seen in the clinic for acne. The most likely cause of her current
condition is:

A. Benzoyl peroxide
B. Erythromycin
C. Tetracycline
D. Minocycline
E. Clindamycin
F. Metronidazole

Explanation:
This patient is likely suffering a tetracycline-associated
photosensitivity reaction. Doxycycline can also cause such a reaction.
Minocycline and the other medications listed are also used to treat
acne but are not associated with severe photosensitivity reactions.

Educational Objective:
Medications commonly associated with cutaneous photosensitivity include
sulfonamides, amiodarone, and tetracyclines.

65
USMLE WORLD STEP 1 PHARMACOLOGY

Q NO 240: A 67-year-old man admitted for right lower lobe pneumonia


A.
subsequently develops hypotension and lactic acidosis. He is started on
a norepinephrine intravenous drip. A few hours later, the antecubital
vein being used for the infusion blanches and the tissues surrounding
the IV site become cold, hard, and pale. Local injection of the affected
tissues with which of the following agents is most likely to be of
greatest benefit:
Calcium gluconate
B. Phentolamine
C. Heparin
D. Isoproterenol
E. Lidocaine

Explanation:
Blanching of a vein into which norepinephrine (NE) is being infused
together with induration and pallor of the tissues surrounding the IV
site is signs of norepinephrine (NE) extravasation. The norepinephrine
(NE) leak causes intense a receptor mediated vasoconstriction which can
lead to local tissue necrosis. Such necrosis can be prevented by
infiltration (using a syringe with a fine hypodermic needle) throughout
the affected area with 10—15cc of sodium chloride solution containing 5
—10mg of phentolamine mesylate, an a receptor blocker. This antidote
must be given within 12 hours of extravasation to be effective.
Remember that phentolamine mesylate and other a receptor blockers lead
to vasodilatation, thus counteracting the al receptor mediated
vasoconstriction of NE.
(Choices A and C) Calcium gluconate is administered in severe
hypocalcemia and would not lead to the desired effect of
vasodilatation. Heparin is indicated if there is an arterial thrombosis
but does not cause significant vasodilatation.
(Choice D) Isoproterenol can mediate vasodilation via j32 receptor
activation particularly in striated muscle, renal, and mesenteric
vascular beds, leading to decreased peripheral vascular resistance with
increased cardiac output. Subcutaneous tissue blood vessels have a
relatively low j32 receptor density compared to al receptors, thus, an
alpha blocker would likely be more effective in reversing cc1 receptor
mediated vasoconstriction.
(Choice E) Lidocaine can relax vascular smooth muscle via blockade of
Na current mediated action potentials but tends to be less effective in
ischemic, acidotic tissues. The most direct and effective treatment for
this patient remains an alpha blocker, such as phentolamine.

Educational Objective:
Blanching of a vein into which norepinephrine (NE) is being infused
together with induration and pallor of the tissues surrounding the IV
site is signs of NE extravasation and resulting vasoconstriction.
Tissue necrosis is best prevented by local injection of an alpha 1
blocking drug, such as phentolamine.

66
USMLE WORLD STEP 1 PHARMACOLOGY

Q NO 241: A 24-year-old female with a history of Munchausen syndrome presents


to the emergency room with epistaxis, bleeding gums, bruises and petechiae. She
has an episode of bloody emesis and has guaiac positive stool. After much
questioning, she admits to having ingested rat poison. Immediate treatment of
this patient should include:

A. Syrup of ipecac
B. Fresh frozen plasma
C. Cryoprecipitate
D. Platelet infusion
E. Protamine sulfate

Explanation:
Most available rodenticides contain brodifacoum, a long acting 4-
hydroxycoumarin derivative. Thus, this patient has depleted herself of
vitamin K dependent clotting factors, causing an acquired coagulopathy.
To prevent life- threatening GI bleeding, this patient requires
restoration of clotting factors II, VII, IX, and X to adequate levels.
The fastest way to replenish these factors is through IV administration
of fresh frozen plasma (IFP). Vitamin K1 should also be given.
(Choice A) Syrup of ipecac is an emetic treatment that would have been
of value immediately following the ingestion. The fact that this
patient has already started bleeding indicates that several hours or
days have passed since her ingestion.
(Choice C) Cryoprecipitate contains factor VIII, factor XIII, von
Willebrand’s factor, and fibrinogen, and thus would not be of
significant benefit where the vitamin K dependent clotting factors (II,
VII, IX, X) require replenishment.
(Choice D) Rat poison does not affect platelet count.
(Choice E) Protamine is used for heparin overdose.

Educational Objective:
Most available rodenticides contain brodifacoum, a long acting 4-
hydroxvcoumarin derivative. If a patient has ingested a quantity of
rodenticide sufficient to cause coagulopathy and abnormal bleeding,
immediate treatment with fresh frozen plasma and vitamin K1 is
required.

67
USMLE WORLD STEP 1 PHARMACOLOGY

Q NO 242: A 32-year-old HIV-positive male is diagnosed with pulmonary


A. Up
tuberculosis and started on four-drug combination therapy. Several days
later, he presents complaining of red urine and red staining of his contact
lenses. His sputum isolates of M. tuberculosis are resistant to several
chemotherapeutic drugs. Which of the following best explains the
bacterial resistance to the drug that is responsible for this patient’s
complaints?
regulation of beta-lactamase synthesis
B. Decreased activity of bacterial catalase-peroxidase
C. Altered structure of bacterial ribosomal proteins
D. Altered structure of enzymes involved in DNA winding-unwinding
E. Altered structure of enzymes involved in RNA synthesis

Explanation:
This HIV-positive patient has active pulmonary tuberculosis, which is
not an uncommon scenario. The rise in HIV infections in the United
States has been paralleled by a rise in M. tuberculosis infections
typically from reactivation of latent infection after the dramatic
onset of HIV-induced immunosuppression. Rifampin is one of the drugs
used in the treatment of active M. tuberculosis infection. It is given
with isoniazid and pyrazinamide, ethambutol, and/or streptomycin.
Rifampin inhibits DNA-dependent RNA-synthesis, thereby preventing
transcription. The subsequent lack of mRNA leads to a deficiency of
those proteins necessary for mycobacterial survival. An alteration of
the structure of this DNA-dependent RNA polymerase causes bacterial
resistance to rifampin. Unlike isoniazid, rifampin is not specific to
the mycobacteria alone; it can also be used in the treatment of other
bacterial infections.
(Choice A) Up regulation of beta-lactamase synthesis would cause
bacterial resistance to beta-lactam drugs such as penicillins,
aminopenicillins, cephalosporins and monobactams. Infections caused by
organisms expressing beta lactamase require the addition of a beta-
lactamase inhibitor (eg, clavulanic acid sulbactam or tazobactam) or
the use of a carbapenem.
(Choice B) Decreasing the activity of bacterial catalase-peroxidase is
a mycobacterial response to isoniazid. Isoniazid requires conversion to
its active compound by the mycobacterial enzyme catalase-peroxidase
before it can inhibit mycolic acid synthesis.
(Choice C) Antibiotics that act on the ribosome can be divided into two
groups: those that act on the 50S subunit and those that act on the 30S
subunit. Chloramphenicol, clindamycin, linezolid and macrolides (eg,
erythromycin, azithromycin, and clarithromycin) act on the large 50S
subunit while tetracycline doxycycline and the aminoglycosides (eg,
gentamicin, streptomycin and others) act on the smaller 30S ribosomal
subunit.
(Choice D) Structural alteration of enzymes involved in DNA winding-
unwinding would endow bacteria with resistance to the fluoroquinolone
class of antibiotics. The fluoroquinolones bind to and inhibit DNA
gyrase in bacterial cells, causing the DNA chain to fracture.
Educational Objective: Rifampin is an antimycobacterial drug that
blocks the action of the bacterial DNA-dependent RNA polymerase,
thereby inhibiting transcription. The subsequent absence of mRNA leads
to a deficiency of proteins necessary for bacterial survival. Rifampin
resistance is acquired by modification of the rifampin binding site on
the bacterial DNA- dependent RNA polymerase.

68
USMLE WORLD STEP 1 PHARMACOLOGY

Q NO 243: A 30-year-old Caucasian male presents to your office with fatigue,


muscle weakness and occasional headaches. His blood pressure is 180/110
mmHg and his heart rate is 80/mm. Laboratory evaluation reveals severely
depressed plasma renin activity, and a CT scan demonstrates a right-sided
adrenal mass. After treatment for several weeks, the patient’s symptoms
resolve, his blood pressure is decreased to 130/10 mmHg and his heart rate
is 75/mm. Which of the following drugs was most likely used in this patient?
A. Clonidine
B. Propranolol
C. Captopril
D. Hydrochlorothiazide
E. Eplerenone
F. Verapamil
G. Amlodipine
H. Isosorbide dinitrate

Explanation:
This patient is suffering from an aldosterone secreting tumor (adenoma)
leading to primary hyperaldosteronism (Conn’s Syndrome). Presenting
signs of hyperaldosteronism most commonly include hypertension
hypokalemia, metabolic alkalosis and decreased plasma renin activity.
Aldosterone causes resorption of sodium and water and wasting of
potassium and hydrogen ions (acid) at the distal portion of the
nephron, leading to hypokalemia and alkalosis. Additionally,
inappropriately high aldosterone will suppress renin activity as part
of a feedback inhibition loop. The treatment for a unilateral adenoma
secreting aldosterone, as is found in this patient, can be either by
surgical resection or by medical therapy with aldosterone antagonists.
Spironolactone is the most frequently used first- line drug, and
eplerenone is a new aldosterone antagonist that has fewer side effects
than spironolactone and is often used in those that can not tolerate
spironolactone.
The most frequently mentioned side effect of these medications is their
ability to cause gynecomastia (approximately 1% with eplerenone, 9%
with spironolactone).
Other drugs mentioned in the other choices are not used in Conn’s
syndrome.

Educational Objective:
Aldosterone excess will cause hypertension, hypokalemia, metabolic
alkalosis and depressed renin. Alternatively, hypoaldosteronism is the
cause of type IV renal tubular ACIDOSIS. Aldosterone antagonists such
as spironolactone or eplerenone can be used as medical therapy for
Conn’s syndrome.

69
USMLE WORLD STEP 1 PHARMACOLOGY

Q NO 244: A 54-year-old man is hospitalized after a planned abdominal surgery.


One of his physicians administers a new drug whose mechanism of action you do
not know. Shortly after administration of the drug the patient develops flushing,
diaphoresis and nausea. His blood pressure is 100/70 mmHg and heart rate is
55/mm. His pupils are constricted but reactive to light. This medication is most
likely given for which of the following conditions?

A. Urinary tract infection


B. Urinary obstruction
C. Atonic bladder
D. Fluid overload
E. Hypovolemic shock

Explanation:
The patient described in this vignette displays symptoms associated
with cholinergic agonists. These medications increase GI tract smooth
muscle tone to produce nausea, vomiting, abdominal cramps and diarrhea.
Cholinergic agonists decrease heart rate, cardiac conduction and
contractility, and may result in bradycardia, partial or complete heart
block, and hypotension. Cholinergic-associated peripheral
vasodilatation decreases blood pressure due to the stimulation of
endothelial nitric oxide synthesis. Cholinergic agonists also increase
secretion, leading to excessive sweating, salivation and lacrimation.
These drugs can cause dyspnea via bronchoconstriction (increased smooth
muscle tone in the tracheobronchial tree) and increased bronchial
secretion.
There are several cholinergic agonists used in medicine. Bethanechol is
a cholinergic agonist used to stimulate peristalsis in postoperative
ileus. This drug is also used to treat non-obstructive urinary
retention (atonic bladder). Carbachol and pilocarpine are used to lower
intraocular pressure in glaucoma. Cholinergic-associated miosis causes
the iris to move further from the cornea. This widens the anterior
chamber angle and allows for better outflow of the aqueous humor.
(Choices A and B) Cholinergic agonists are not used to treat un flaw
tract infections or urinary obstruction.
(Choice D) Clinically evident volume overload is treated with loop
diuretics. Cholinergic agents will not help this condition.
(Choice E) Hypovolemic shock is treated with lV fluids. Administering a
cholinergic agent would likely worsen hypovolemic shock.

Educational Objective:
Cholinomimetics are indicated in non-obstructive urinary retention
paralytic ileus, and glaucoma. Their side effects include nausea,
vomiting, abdominal cramps, diarrhea dyspnea and increased secretions
(sweating lacrimation and salivation).

70
USMLE WORLD STEP 1 PHARMACOLOGY

Q NO 245: Which of the following medications used to treat pulmonary tuberculosis


requires an acidic environment to 1< Mycobacterium tuberculosis, and is thus most
useful against intracellular organisms?

A. Isoniazid
B. Ethambutol
C. Pyrazinamide
D. Rifampin
E. Streptomycin

Explanation:
Pyrazinamide (PZA) works best at relatively acidic pHs, as within
phagolysosomes. It is therefore most bactericidal to M. tuberculosis
(MTB) organisms engulfed by macrophages. The other first-line agents
for MTBI namely isoniazid, rifampin, and ethambutol, have better
activity against extracellular MTB (including organisms in necrotic
foci and/or tissue cavities) than PZA.
(Choice A) Isoniazid inhibits synthesis of mycolic acid, an important
constituent of mycobacterial cell walls.
(Choice B) Ethambutol inhibits mycobacterial cell wall synthesis by
blocking arabinosyl transferase.
(Choice D) Rifampin inhibits a mycobacterial DNA-dependent RNA
polymerase.
(Choice E)The aminoglycoside streptomycin is a second-line agent for
MTB. It has poor cellular penetration and is inactive at acidic pHs.

Educational Objective:
Of the first-line agents for Mycobacterium tuberculosis, only
pyrazinamide requires an acidic environment (as is present within
macrophage phagolysosomes) to exert antimicrobial effects. Isoniazid,
rifampin, and ethambutol are more active than PZA against extracellular
mycobacteria.

71
USMLE WORLD STEP 1 PHARMACOLOGY

Q NO 246: Enterococci isolated from a patient with a severe urinary tract infection
are shown to produce an enzyme located at the surface of the cytoplasmic
membrane that is involved in acetyl group transfer to exogenous substances. The
bacteria are most likely resistant to which of the following antibiotics?

A. Ampicillin
B. Gentamicin
C. Vancomycin
D. Minocycline
E. Ciprofloxacin

Explanation:
The enterococci are a concerning organism due to the fact that this
genus has acquired resistance to multiple antibiotics including but not
limited to penicillin, penicillinase-resistant penicillins,
aminoglycosides, trimethoprim sulfamethoxazole, and most famously
vancomycin. Each of these resistances occurs by different mechanisms
specific for the drug class. The enterococci have achieved
aminoglycoside resistance by producing aminoglycoside modifying enzymes
that transfer different chemical groups (acetyl groups adenyl groups,
and phosphate groups) to the aminoglycoside antibiotic molecule outside
of the bacterium, thereby decreasing the ability of these drugs to bind
to ribosomes and exert their antimicrobial effects. This resistance is
not chromosomally mediated; itis an acquired resistance mediated by
plasmids or transposons.
(Choice A) Enterococci accomplish penicillin resistance by two
mechanisms: 1) production of beta-lactamase and 2) production of a low-
affinity penicillin binding protein.
(Choice C) Vancomycin is frequently used synergistically with
aminoglycosides in the treatment of enterococcal infections that are
resistant to penicillins. Vancomycin-resistance in enterococci (VRE) is
an acquired resistance mediated by plasmids or transposons. The
resistance occurs when proteins are synthesized by the resistant
enterococci that act as Ii gases that alter the D-alanyl D-alanine
cell-wall precursors which are the targets of vancomycin. The most
famous of these is the V anA ligase which substitutes D-alanine D-
lactate for D-alanyl D alanine thus preventing binding of vancomycin.
(Choice D)The mechanism of resistance to tetracyclines occurs by two
mechanisms: 1) synthesis of a protein that allows the ribosome to do
translation even in the presence of drug, and 2) decreasing
intracellular concentrations of the drug by increased efflux or
decreased influx.
(Choice E) Ciprofloxacin is a fluoroquinolone antibiotic.
Fluoroquinolone resistance is usually due to mutations in the DNA
gyrase or topoisomerase genes.

Educational Objective:
Enterococci produce aminoglycoside-modifying enzymes that transfer
different chemical groups (acetyl, adenyl or phosphate) to the
aminoglycoside molecule and therefore impair antibiotic binding to
ribosomal subunits.

72
USMLE WORLD STEP 1 PHARMACOLOGY

Q NO 247: A 75-year-old female is brought to your office by her son because of


worsening forgetfulness over the past few years and several recent episodes of
getting lost while performing errands around the house. She exhibits no signs or
symptoms of depression. On physical examination, the patient has a blood
pressure of 139/88 mmHg. She is alert and cooperative but disoriented to time
and can only recall of 3 items on a short term memory test. An extensive work-
up, including appropriate laboratory tests and a brain MRI scan, reveals diffuse
cortical atrophy with no other abnormalities. This patient would likely benefit
from treatment with which of the following:

A. Lorazepam
B. Desipramine
C. Haloperidol
D. Donepezil
E. Folic acid
F. Vitamin A
G. Sertraline
H. Melatonin

Explanation:
This is a patient with chronically progressive dementia. The MRI
findings of cortical atrophy and the absence of other reversible causes
of dementia point to a clinical diagnosis of Alzheimer’s disease (AD),
a diagnosis of exclusion. Currently available AD-specific therapies
offer either: 1.) Enhanced cholinergic neurotransmission, 2.) Neuro
protection via antioxidants, or 3.) NMDA receptor antagonism. Of the
answer choices provided, only donepezil, a cholinesterase inhibitor, is
an AD medication. The cholinesterase inhibitors appear to improve
cognition, behavior, and functioning in activities of daily living in
AD patients. Although they do not appear to prevent the ultimate
progression of cerebral neuro degeneration, the cholinesterase
inhibitors may delay institutionalization and mortality.
Regarding the other classes of AD drugs vitamin E (a-tocopherol) is a
neuroprotective agent that may slow functional losses in AD. Memantine
is a NMDA receptor antagonist. It is thought that CNS NMDA-receptor
overstimulation by glutamate may contribute to AD symptoms.
(Choice A) In general, benzodiazepines should be avoided in AD given
their depressive effects on cognition (e.g. lethargy, confusion). In
some circumstances, however, they may be prescribed to AD patients to
treat associated anxiety and insomnia.
The other agents listed do not have applications in AD.

Educational Objective:
A chronically progressive pre-senile dementia with cortical atrophy but
no other radiological or laboratory abnormalities permits a clinical
diagnosis of Alzheimer’s disease (AD), which is a diagnosis of
exclusion. Current AD specific therapies include cholinesterase
inhibitors (Donepezil), antioxidants (vitamin E), and NMDA receptor
antagonists (e.g. memantine).

73
USMLE WORLD STEP 1 PHARMACOLOGY

Q NO 248: A 62-year-old African American woman is brought to ER with acute chest


pain. She also complains of perspiration nausea and lightheadedness. Her blood
pressure is 90/60 mmHg and her heart rate is 42 beats/min. An ECG reveals ST
segment elevation in the inferior leads. After initial treatment her blood pressure is
120/70 mmHg and heart rate is 76/mm. She now complains of severe right-sided
eye pain. Which of the following is the most likely eye problem in this patient?

A. Iridocyclitis
B. Retinal artery thrombosis
C. Acute conjunctivitis
D. Glaucoma
E. Cataract
F. Retinal vein thrombosis
G. Retinal detachment
H. Vitreous hemorrhage

Explanation:
ST segment elevation in the inferior leads is diagnostic of an inferior
myocardial infarction (MI). Inferior MIs are often due to blockage of
the right coronary artery the artery responsible for sinoatrial (SA)
node perfusion. Thus, inferior MIs are often associated with
bradycardia. Atropine blocks vagal influence on the SA node and is
effective in increasing heart rate in such patients. Atropine’s side
effects are due to muscarinic receptors blockade in other organs. In
the eye, atropine causes mydriasis, resulting in a narrowing of the
anterior chamber and diminished outflow of aqueous humor. This can
precipitate glaucoma in patients with increased intraocular pressure.
Acute closed angle glaucoma presents with unilateral severe eye pain
and is described in this clinical scenario. African Americans are at
increased risk of glaucoma.
The clinical indications for anticholinergic medications are summarized
in the table below:

(Choice A) Anticholinergic medications (cycloplegics) reduce pain and


prevent the formation of adhesions in iridocyclitis (uveitis). Atropine
does not precipitate iridocyclitis.
Anticholinergic drugs do not affect the other conditions mentioned in
this question.

74
USMLE WORLD STEP 1 PHARMACOLOGY
Educational Objective:
Atropine is indicated for the treatment of bradycardia since it
decreases vagal influence on the SA node. A common side effect of
atropine is increased intraocular pressure. It may precipitate closed
angle glaucoma in susceptible individuals.

75
USMLE WORLD STEP 1 PHARMACOLOGY

Q NO 249: A 56-year-old male with severe joint pain and swelling is treated with
colchicine. He experiences significant relief of symptoms 12 hours after the
treatment was initiated. The drug used in this patient most likely affects which of
the following steps shown on the diagram?

A. A
B. B
C. C
D. D
E. E
F. F

Explanation:
The preferred first line treatment of acute gout arthritis is the use
of nonsteroidal antiinflammatory drugs. However, colchicine is
sometimes used to treat acute gouty arthritis. Treatment with
colchicine results in resolution of symptoms in more than 85% of
patients with acute gouty arthritis. Colchicine primarily works by
inhibition of microtubular polymerization. Colchicine binds to the
protein subunit of microtubules and prevents their aggregation. This in
turn disrupts membrane-dependent functions such as chemotaxis and
phagocytosis (choice F). Colchicine also reduces the formation of
leukotriene B 4. Important adverse effects of colchicine are nausea,
abdominal pain, and diarrhea.
Colchicine should not be used in patients who are elderly or have renal
dysfunction. Chronic low-dose colchicine has been utilized as
prophylactic treatment for the prevention of acute gout arthritis.
(Choices A, E and F) Glucocorticoids produce their anti-inflammatory
effects by inhibition of the enzyme phospholipase A2, which is
responsible for conversion of membrane phospholipids to arachidonic
acid. This in turn decreases the formation of prostaglandins and
leukotrienes. Intra-articular glucocorticoid therapy is a preferred
treatment for acute gouty arthritis in elderly patients or those with
renal failure.
(Choices B and D) The leukotriene pathways can be blocked by inhibition
of the 5-lipoxygenase pathway or by blocking the leukotriene receptors.

76
USMLE WORLD STEP 1 PHARMACOLOGY
The drug Zileuton is a specific inhibitor of the enzyme 5-lipoqgenase,
and thus inhibits leukotriene formation (LTB, LTC, LTD and LTE 4).
Montelukast is a drug which inhibits LTD 4 receptors. Leukotriene
modifiers are used for preventing the recurrence of asthma attacks and
allergic rhinitis, and they have no role in the treatment of acute or
chronic gout.
(Choice C) Non-steroidal anti inflammatory agents are inhibitors of the
cyclooxygenase enzymes. These agents have analgesic, anti-inflammatory,
and antipyretic effects. NSAIDs are the most commonly used drugs for
treatment of acute gouty arthritis.

Educational Objective:
Colchicine is used for both acute gout arthritis and prophylaxis of
recurrent gouty arthritis. Colchicine inhibits leukocyte migration and
phagocytosis by blocking tubulin polymerization. Colchicine does not
have any effect on the metabolism or urinary excretion of uric acid.
The most troublesome side effects of colchicine are nausea and
diarrhea.

77
USMLE WORLD STEP 1 PHARMACOLOGY

Q NO 250: A 65-year-old immigrant from Eastern Europe presents to your office


with a four week history of low-grade fever, multiple joint pains, and a well-
demarcated erythematous rash on his face and trunk. His serum tests positive
for the presence of anti-nuclear antibody (ANA). He reports a past medical
history of coronary artery disease, congestive heart failure, and ‘cardiac rhythm
abnormalities. The patient should be specifically questioned about the intake of
which of the following medications?

A. Amiodarone
B. Propranolol
C. Lidocaine
D. Procainamide
E. Verapamil
F. Adenosine
G. Quinidine

Explanation:
This vignette describes a patient with signs and symptoms of lupus.
Procainamide is a class 1A antiarrhythmic drug that can cause serious
side effects including a drug-induced lupus syndrome, ventricular
arrhythmias, and psychiatric disturbances like psychosis and
depression.
Procainamide is metabolized via hepatic acetylation. Individuals who
are “slow acetylators” are at greatest risk for the drug-induced lupus
syndrome. Individuals who metabolize the drug more rapidly are less
likely to experience this adverse effect. Hydralazine is another
medication commonly associated with drug-induced lupus.
(Choice A) The most notable side effects of amiodarone include thyroid
disease, lung fibrosis and liver toxicity, as well as a blue or grey
discoloration of the skin.
(Choice B) The most frequently encountered adverse effects of
nonselective beta-blockers are negative inotropy and
bronchoconstriction in patients with asthma or COPD.
(Choice C) The adverse effects of lidocaine are characteristically non
local neurologic signs like tremor, drowsiness, and change in mental
status, though lidocaine can also lead to generalized seizures.
(Choice E) Verapamil is a calcium channel blocker and class 4
antiarrhythmic. The most frequently noted adverse reactions with
verapamil are negative inotropy, constipation and gingival hyperplasia.
(Choice F) Adenosine is used as a vasodilator in chemical cardiac
stress tests and is also useful as a fast acting anti arrhythmic for
stopping acute supraventricular tachycardias. Adenosine has an
extremely short half-life, and its immediate adverse effects include
chest burning, flushing and transient hypotension.
(Choice G) Quinidine’s characteristic side effects include weakness GI
effects, and “cinchonism,” a syndrome characterized by tinnitus,
dizziness, and occasional fainting.

Educational Objective:
Procainamide, hydralazine and isoniazid have all been shown to cause a
drug-induced lupus syndrome in some patients.

78
USMLE WORLD STEP 1 PHARMACOLOGY

Q NO 251: A 72-year-old Caucasian male was diagnosed with Parkinson disease five
years ago and is treated with levodopa/carbidopa. He presents to your office
complaining of stiffness that is only temporarily corrected by his medication. He asks
if the dose should be increased. You suggest instead that the patient add a second
drug called tolcapone. The effect of tolcapone can be best described by which of the
following?

A. Enhances the effect of endogenous dopamine


B. Stimulates dopamine receptors
C. Decreases central dopamine degradation
D. Increases dopamine availability to the brain
E. Inhibits central muscarinic receptors

Explanation:
Levodopa is the immediate precursor of dopamine, a neurotransmitter
that is absent in the Nigro striatum of patients with Parkinson Disease
(PD). Dopamine itself cannot be administered directly because it is
unable to cross the blood-brain barrier. Levodopa, the precursor of
dopamine, however, can cross this barrier. Unfortunately levodopa is
rapidly metabolized in the peripheral tissues and only small percentage
reaches the brain. Thus levodopa has traditionally been administered
with carbidopa, a peripheral dopa decarboxylase inhibitor. Even with
carbidopa, however, only 5-10% of levodopa reaches the brain. One of
the main enzymes responsible for this peripheral catabolism of levodopa
is Catechol-O-methyl-transferase (COMT).

79
USMLE WORLD STEP 1 PHARMACOLOGY
Tolcapone is a COMT inhibitor that primarily serves to increase the
bioavailability of levodopa.
Catechol-O-methyl-transferase (COMT) inhibitors, such as entacapone and
tolcapone, are a newer class of medications that prolong the “on”
response to levodopa, reduce motor fluctuations effectively, and allow
a reduction in daily levodopa dosage. Remember that catechol-O-
methyltransferase (COMT) inhibitors should only be used in combination
with levodopa. Tolcapone can cause severe drug-induced hepatitis, so
frequent liver function testing must be performed.
(Choice A) Amantadine, designed as an anti-influenzal antiviral,
happens to also enhance the effect of endogenous dopamine. Amantadine
is believed to to do so by increasing the synthesis, release, and re-
uptake of dopamine.
(Choice B) Dopamine agonists, such bromocriptine, pergolide,
pramipexole, and ropinerole, stimulate dopamine receptors
(preferentially, D2 receptors).
(Choice C) Selegiline, an inhibitor of the brain’s monoamine oxidase-B
(MAO-B) enzyme, decreases central dopamine degradation. Selegiline is
believed to have a neuroprotective effect and may retard the
progression of PD, although studies have shown conflicting results.
(Choice E) Anticholinergics, such as trihexyphenidyl and benztropine,
inhibit central muscarinic receptors. These drugs are mainly used for
drug-induced PD. Patients with tremor as the predominant symptom also
respond well.

Educational Objective:
Both DOPA decarboxylase and COMT blockers increases the dopamine
availability to brain. Entacapone and tolcapone are the currently
available COMT inhibitors.

80
USMLE WORLD STEP 1 PHARMACOLOGY

Q NO 252: A 6-year-old Caucasian female has recurrent lower lobe pulmonary


infiltrates, longstanding steatorrhea and malabsorption, and an abnormally
high concentration of chloride in her sweat. She receives intermittent
inhalational therapy with an N-acetylcysteine-containing aerosol. N-
acetylcysteine may benefit this patient by:
A. Releasing sensory neuropeptides
B. Increasing epithelial chloride transport
C. Reducing intermolecular di sulfide bridges
D. Stimulating secretion of IgA
E. Decreasing epithelial sodium conductance
F. Inhibiting the epidermal growth factor receptor

Explanation:
This clinical vignette describes a patient with cystic fibrosis (CE).
Patients with CE have decreased chloride secretion and increased sodium
reabsorption in the mucous glands of the pulmonary airways. The net
result is isotonic dehydration of the inner surfaces of the
tracheobronchial tree and abnormal thickening (hyperviscosity) of
secreted mucus. In small airways, this hyperviscosity causes impaired
mucociliary clearance and mucus plugging with resultant obstruction and
infection. N-acetylcysteine is a mucolytic agent that loosens the thick
sputum by cleaving disulfide bonds within mucus glycoproteins.

Educational Objective:
N-acetylcysteine is a mucolytic agent used in the treatment of CF. N-
acetylcysteine works by cleaving the disulfide bonds within mucus
glycoproteins, thus loosening thick sputum.

81
USMLE WORLD STEP 1 PHARMACOLOGY

Q NO 253: A new antiviral agent is shown to selectively bind to the HIV gp41
protein. This antiviral agent is most likely to inhibit which of the following?
A. Viral attachment to target cells
B. Viral penetration into target cells
C. Viral DNA synthesis from RNA template
D. Viral polyprotein cleavage
E. Viral DNA integration into the host genome

Explanation:
Drugs that selectively bind to HIV’s transmembrane envelope protein
gp41 are known as “fusion inhibitors.” One of the more commonly used
fusion inhibitors is enfuvirtide. Enfuvirtide binds to the heptad
repeat (HR1) of gp41, thereby preventing gp41 from undergoing the
conformational changes necessary for the viral membrane to fuse with
the target cellular membrane. As a result, the HIV genome entry is
denied into uninfected CD4+ T-cells.
(Choice A) HIV attachment to target cells is mediated by the binding of
viral envelope protein gpl2Otothe CD4 membrane protein of T helper
cells. An agent that selectively binds to gp41 instead of gp120 would
not be expected to prevent viral adherence.
(Choice C) HIV DNA synthesis from the RNA template is impaired by
reverse transcriptase (RT) inhibitors.
(Choice D) HIV polyprotein cleavage is catalyzed by HIV viral protease.
This reaction can be prevented by protease inhibitors (P1).
(Choice E) HIV DNA integration into the host genome is catalyzed by HIV
viral integrase.

Educational Objective:
Antiretroviral agents that selectively bind to the HIV envelope
transmembrane glycoprotein gp41 prevent the conformational changes
necessary for the viral membrane to fuse with the target cellular
membrane. These agents are therefore known as “fusion inhibitors.”

82
USMLE WORLD STEP 1 PHARMACOLOGY

Q NO 254: Isolates of M. tuberculosis obtained from a 23-year-old HI V-positive


male show significantly decreased activity of intracellular catalase-peroxidase.
The isolates would most likely grow in the presence of:

A. Isoniazid
B. Rifampin
C. Ethambutol
D. Streptomycin
E. Pyrazinamide

Explanation:
The mechanism of action of isoniazid is the inhibition of mycolic acid
synthesis by M. tuberculosis. Isoniazid accomplishes this through an
incompletely understood method. However, it is known that isoniazid
must be processed by mycobacterial catalase-peroxidase for the drug to
be activated within the bacteria, and that isoniazid requires a
specific protein sequence in its enzyme target (an enzyme necessary for
mycolic acid synthesis). For this reason, mycobacterial resistance to
isoniazid could be accomplished through non-expression of the catalase-
peroxidase enzyme or through genetic modification of the isoniazid
binding site on the mycolic acid synthesis enzyme.
(Choice B) Resistance to rifampin in M. tuberculosis is primarily due
to a mutation in the gene that codes for a DNA-dependent RNA polymerase
necessary for transcription and RNA prolongation.
(Choice C) Ethambutol appears to inhibit the synthesis of the
mycobacterial cell wall. Resistance to this drug develops when the
mycobacteria increase production of arabinosyl transferase (an enzyme
that polymerizes arabinose into Arabian and then arabin galactan, one
of the key constituents of the mycobacterial cell wall).
(Choice D) Streptomycin is an aminoglycoside antibiotic that inhibits
mycobacterial protein synthesis by disabling the bacterial ribosomal
303 subunit (which disrupts translation). Resistance to this drug is
primarily achieved by a modification of the 303 ribosomal protein
structure such that streptomycin is unable to bind to the ribosomal
subunit, rendering it ineffective.
(Choice E) Pyrazinamide is converted to pyrazinoic acid, which lowers
the environmental pH in susceptible strains of M. tuberculosis. Because
pyrazinamide must be converted to an active form by the mycobacterial
enzyme pyrazinamidase, drug resistance can occur in those organisms
that have modified their pyrazinamidase to be structurally ineffective.

Educational Objective:
Mycobacterial resistance to isoniazid can be accomplished through non-
expression of the catalase-peroxidase enzyme or through genetic
modification of the isoniazid binding site on the mycolic acid
synthesis enzyme.

83
USMLE WORLD STEP 1 PHARMACOLOGY

Q NO 255: A 23-year-old male is brought to your office by his mother who states
that her son lost interest in life. He was diagnosed with schizophrenia six months
ago and treatment was initiated. Although the patient appears not to have
hallucinations any more he shows little interest in his friends or surrounding
environment. His face is emotionless and his speech is scant. Which of the
following drugs would most likely affect this patient’s current symptoms?

A. Chlorpromazine
B. Haloperidol
C. Fluphenazine
D. Thiothixene
E. Thioridazine
F. Quetiapine

Explanation:

Unlike first-generation (traditional) neuroleptics, second-generation


agents (clozapine, risperidone, olanzapine) improve both positive and
negative symptoms of schizophrenia. Positive symptoms of schizophrenia
include psychotic symptoms, such as hallucinations, usually auditors’;
delusions; and disorganized speech and behavior. Negative symptoms
include a decrease in emotional range, poverty of speech, loss of
interests, and loss of drive.
Traditional high potency agents are more likely to cause extrapyramidal
symptoms and less likely to cause anticholinergic and antihistamine
side effects, while low potency antipsychotics are more likely to cause
anticholinergic and anti histamine side effects.

Educational Objective:
The symptoms of schizophrenia include the following:
1. Positive symptoms: hallucinations (usually auditors’), delusions,
and disorganized speech and behavior.
2. Negative symptoms: a decrease in emotional range, poverty of speech,
and loss of interest in living.
Unlike first-generation neuroleptics, second-generation (atypical)
agents (clozapine, risperidone, olanzapine, quetiapine) improve both
positive and negative symptoms of schizophrenia.

84
USMLE WORLD STEP 1 PHARMACOLOGY

Q NO 256: A 34-year-old female with a prior history of bipolar disorder is admitted


to the hospital after a motor vehicle collision. She develops generalized tonic-clonic
seizures. Which of the following would be most helpful in treating both of her
conditions in long run?

A. Lorazepam
B. Valproic acid
C. Sertraline
D. Olanzapine
E. Lithium
F. Ethosuximide

Explanation:
Valproic acid is used as both an anticonvulsant and a mood stabilizer.
It decreases the propagation of abnormal action potentials by enhancing
the inhibitory action of GABA in the CNS. Valproate (valproic acid)
augments GABA by increasing its synthesis and decreasing its breakdown.
As an anticonvulsant, valproic acid is effective for the treatment of
absence myoclonic, and generalized tonic-clonic seizures. Although
valproic acid is the drug of choice for myoclonic seizures, itis a
second-line agent for absence seizures (ethosuximide is preferred). As
a mood stabilizer valproate is used to prevent mania in rapid-cycling
bipolar disorder (>4 episodes/year), in mixed manic-depressive
episodes, and for the management of acute mania.
(Choice A) Lorazepam is an intermediate-acting benzodiazepine. It
enhances the affinity of GABA to its receptor, thus increasing its
inhibitory potential. Lorazepam is generally the first-line medication
for the treatment of active seizures anxiety disorder, and alcohol
withdrawal.
(Choice C) Sertraline is a selective serotonin reuptake inhibitor
(SSRI). It is indicated in the treatment of major depression, obsessive
compulsive disorder, and panic disorder.
(Choice D) Olanzapine is an “atypical” antipsychotic drug effective in
the treatment of schizophrenia. It is also indicated for acute mania.
Side effects of olanzapine include extrapyramidal symptoms and weight
gain. Olanzapine is not indicated for seizures.
(Choice E) Lithium is effective in preventing both manic and depressive
episodes of bipolar disorder. It acts by inhibiting the cellular
concentration of inositol triphosphate (1P3). It is not an
anticonvulsant.
(Choice F) Ethosuximide is the anticonvulsant of choice for absence
seizures. It reduces Ca2 currents in T-type Ca2 channels in the
neurons, thus inhibiting the propagation of action potentials.

Educational Objective:
Lithium, valproic acid, and carbamazepine are mood stabilizing agents.
Valproic acid and carbamazepine also work as anticonvulsants. Valproic
acid increases the inhibitory effect of GABA in the ONS.

85
USMLE WORLD STEP 1 PHARMACOLOGY

Q NO 257: A 78-year-old nursing home resident develops fever and myalgias and is
treated with oseltamivir. Which of the following is most likely impaired in the
patient’s infected cells as a result of the treatment?

A. Viral un coating
B. Foreign protein synthesis
C. Foreign protein splitting
D. Viral RNA replication
E. Virion particle release

Explanation:
Oseltamivir is a neuraminidase inhibitor useful in the treatment of
both influenza A and B virus infections. Neuraminidase is required for
the release of virus from infected cells and for the spread of virus
within the respirator tract. Neuraminidase inhibitors cause the newly
synthesized virions to adhere to the host cell surface and form viral
aggregates, thereby reducing the spread of virus to other host cells.
Oseltamivir (brand name Tamiflu) can shorten the course and
complications of influenza A and B infections if taken within 48 hours
of the onset of symptoms. Additionally, oseltamivir works to slow viral
penetration of the mucous secretions that protect the respiratory

86
USMLE WORLD STEP 1 PHARMACOLOGY
epithelium. Oseltamivir can therefore be used to treat or prevent
influenza infection.
(Choice A) Amantadine impairs un coating or disassembly of the
influenza A virion after host cell endocytosis. Oseltamivir does not
operate via the same mechanism of action.
(Choice B) Foreign (viral) protein synthesis can be inhibited by a- and
3-interferons, which induce the synthesis of proteins that have an
antiviral effect. The antiviral proteins interfere with production of
viral RNA and promote degradation of viral mRNA. Host mRNA is not
affected.
(Choice C) The influenza viral genome consists of segmented RNAI which
is predominantly translated into individual proteins rather than a
large polyprotein product. An example of an agent that impairs foreign
(viral) polyprotein splitting would be a protease inhibitor used to
counteract the HIV polyprotein cleavage mediated by viral protease.
(Choice D) Impairment of viral RNA replication is accomplished by the
reverse transcriptase inhibitors used to combat HIV infection.

Educational Objective:
Oseltamivir is a neuraminidase inhibitor useful in the treatment and
prevention of both influenza A and B virus infections. This medication
impairs the release of newly formed virions from infected host cells
and impairs viral penetration of mucous secretions that overlie the
respiratory epithelium.

87
USMLE WORLD STEP 1 PHARMACOLOGY

Q NO 258: A 45-year-old male with atrial fibrillation receives dofetilide as a means


of pharmacologic cardioversion and the arrhythmia resolves. This medication
exerts its main effect on which portion of the action potential curve?

A. A
B. B
C. C
D. D
E. E

Explanation:
The action potential pictured above is that of a cardiac myocyte with
phases 0(B), 1(C), 2(D), 3(F) and 4 (A). Remember that this differs
from the action potential of cardiac pacemaker cells which have only
phases 4, 0 and 3. Dofetilide is a class 3 (K blocking) antiarrhythmic
agent that slows potassium efflux from the cardiac myocyte. Thus, it
mainly affects phase 3 (Choice E).
(Choice A) Phase 4 of the myocyte action potential is the resting
portion of the curve where ingoing and outgoing currents are balanced.
(Choice B) During phase 0 of the cardiac myocyte action potential,
there is a large influx of sodium ions into the cell that causes the
cell to depolarize. The class I antiarrhythmics alter phase 0 of the
action potential by modulating the sodium permeability of the cell
membrane.
(Choice C) Phase of the action potential is a brief period of cell
membrane repolarization mediated by a sharp decrease in sodium
conductance and an increase in potassium permeability. It is not
modulated to an appreciable extent by anti arrhythmic medications.
(Choice D) Phase 2 of the cardiac myocyte action potential is the
“plateau” of the action potential where movement of ions into (Ca2) and
out of (K+ the cell is relatively equal. K-blocking agents (class 3
antiarrhythmics) do modulate this portion of the action potential but
exert their greatest effect during phase 3, which is completely
dependent on potassium efflux. The antiarrhythmic effect of the calcium
channel blockers applies only to calcium sensitive tissues of the heart
such as the AV node.
Educational Objective: Class 3 antiarrhythmic agents block potassium
efflux from cardiac myocytes and prolong phase 3 of the myocyte action
potential.

88
USMLE WORLD STEP 1 PHARMACOLOGY

Q NO 259: A 44-year-old Caucasian male presents to your office with unbearable


cutaneous flushing and pruritus. He says that the symptoms appear shortly after
he takes his new cholesterol-lowering medication. The drug being used in this
patient acts on which of the following steps of cholesterol metabolism?

A. A
B. B
C. C
D. D
E. E

Explanation:
Nicotinic acid (niacin, vitamin B3) has been utilized for the treatment
of dyslipidemia for almost four decades. Niacin improves hyperlipidemia
by decreasing the synthesis of hepatic triglycerides and VLDL (step C),
an effect due, in part to its suppression of free fatty acid release
from peripheral tissues. Nicotinic acid also decreases VLDL conversion
to LDL, thus decreasing LDL. Nicotinic acid also increases HDL by 25-
30%, more than and other agent currently available.
Although niacin is a very effective agent, side effects are quite
common, especially at the initiation of therapy. Flushing is a
prominent side effect of nicotinic acid: this can be prevented by pre-
treatment with aspirin. Nicotinic acid is better tolerated in slow-

89
USMLE WORLD STEP 1 PHARMACOLOGY
release preparations. Hepatotoxicity can occur with high-dose nicotinic
acid. Additionally, diabetics may experience worsening hyperglycemia.
(Choice A and B) Statins decrease cholesterol synthesis by inhibiting
the enzyme HMG-CoA reductase (step B), which causes up-regulation of
hepatocyte LDL receptors, leading to increased uptake of LDL
cholesterol from the circulation (step A). The side effects experienced
by the patient in the vignette are not associated with statin use.
(Choice D) Bile acid-binding resins, such as cholestyramine and
colestipol, work by binding to bile acid in the gastrointestinal tract
and interfering with its enterohepatic circulation. LDL is reduced as a
result because hepatic cholesterol is used in the re-synthesis of bile
acids, thus causing increased uptake of LDL from the circulation.
The main side effects of bile acid-binding resins include:
gastrointestinal upset (constipation and bloating), impaired absorption
of nutrients and drugs, and most importantly, hypertriglyceridemia.
These medications should not be used in patients with
hypercholesterolemia who have concomitant hypertriglyceridemia. Bile
acid-binding resins are now primarily used in combination with statins.
Because this pairing tends to lead to decreased statin absorption,
these agents should be administered at least four hours apart.
Additionally, bile acid-binding resins should be avoided in patients
with diverticulosis, as the constipation side effect can worsen the
underlying diverticulosis. Malabsorption of vitamin K can occur as
well.
(Choice E) Ezetimibe is a new drug that inhibits the intestinal
absorption of cholesterol. This drug is effective even in the absence
of dietary cholesterol because it also inhibits the enterohepatic
circulation of bile acid (like bile acid resins). This medication is
primarily used in conjunction with statin therapy. This inhibition of
intestinal cholesterol absorption (both dietary and from bile acid)
reduces serum LDL by 20-30%.

Educational Objective:
1. Niacin is believed to inhibit hepatic VLDL production. It is mainly
used to increase HDL levels.
2. Statins inhibit cholesterol synthesis and thereby up-regulate the
LDL receptors.
Bile acid-binding resins can cause constipation and abdominal bloating
(potentially worsening diverticulosis), hypertriglyceridemia,
cholesterol gallstones, and vitamin K malabsorption.

90
USMLE WORLD STEP 1 PHARMACOLOGY

Q NO 260: A new drug is developed that has strong anti-inflammatory properties. It


works by inhibiting both COX 1 and COX2 enzymes by irreversible acetylation. This
new drug most resembles which of the following anti-inflammatory agents?

A. Aspirin
B. Rofecoxib
C. Diclofenac
D. Fenoprofen
E. Ibuprofen
F. Indomethacin
G. Ketorolac
H. Piroxicam

Explanation:
Aspirin acetylates the cyclooxygenase (COX) enzymes, COX 1 and COX 2,
causing irreversible inhibition. At low doses, aspirin inhibits COX 1
alone, whereas at high doses, it inhibits both isoenzymes. In contrast,
the other NSAIDs produce reversible CCX inhibition.
(Choice B) Rofecoxib is a selective CCX 2 inhibitor. CCX 2 is
preferentially expressed at sites of inflammation, whereas COX 1 is
constitutively expressed in platelets and in the gastrointestinal tract
where itis important to normal physiological functioning. Selective
inhibition of COX 2 provides anti-inflammatory benefits without
interfering with the physiological functions of COX 1. In contrast to
the non-selective NSAIDs, selective 00X2 inhibitors have no significant
effect on platelet function and are associated with a lower incidence
of gastrointestinal bleeding. However, recent clinical data suggests
that selective COX 2 inhibitors particularly rofecoxib, are associated
with an increased incidence of thrombotic events.

Educational Objective:
Unlike the majority of non-selective NSAIDs, aspirin has irreversible
effects on COX.

91
USMLE WORLD STEP 1 PHARMACOLOGY

Q NO 261: A 55-year-old male presents to the ER with unbearable right foot pain
that started suddenly in the middle of the night. He has never had such
symptoms before. Physical examination reveals swelling and tenderness of the
first metatarsophalangeal joint. Synovial fluid analysis demonstrates needle-
shaped, negatively birefringent crystals and an excess of neutrophils. The best
initial treatment of this patient would be:

A. COX inhibitor
B. Antibiotics
C. Uricosuric agents
D. Xanthine oxydase inhibitor
E. Lipoxygenase antagonist
F. Glucocorticoid

Explanation:
NSAIDs are the first line treatment for most patients with acute gouty
arthritis. Nonsteroidal anti inflammatory agents exert their anti
inflammatory reaction by inhibiting cyclooxygenase enzymes, thus
decreasing prostaglandin synthesis. NSAIDs are typically initiated in
full dosage at the first sign of acute gouty arthritis. NSAIDs provide
relief of acute symptoms in most patients. NSAIDs should be avoided in
patients with renal and hepatic dysfunction. There is risk for
gastrointestinal bleeding with the use of NSAIDS. NSAIDs can also cause
fluid retention, aggravation of hypertension, and precipitation of
congestive heart failure. NSAIDs are generally avoided in elderly
patients.
(Choice B) Antibiotics are used in patients when acute infectious
arthritis is considered.
(Choice C and D) Both uricosuric drugs and xanthine oxidase inhibitors
(allopurinol) are used as prophylactic therapy for the prevention of
acute gouty arthritis attacks. Prophylactic therapy is used in patients
with more than 3 episodes of acute gouty arthritis and those with
macroscopic tophi. These drugs should not be initiated during an acute
gouty arthritis attack because they can exacerbate acute arthritis.
(Choice E) Lipoxygenase inhibitors such as Zileuton are useful in
patients who have allergic rhinitis and bronchial asthma. Lipoxygenase
antagonists are not useful in the treatment of acute or chronic gout.
(Choice F) Glucocorticoid therapy either systemically or
intraarticular, can also be utilized as a first line treatment for
episodes of acute gout. Intraarticular glucocorticoids are typically
used in elderly patients with renal dysfunction, as these patients
cannot be treated with colchicine or nonsteroidal anti inflammatory
drugs.

Educational Objective:
NSAIDs are the first-line therapy for treatment of acute gouty
arthritis. Colchicine is considered as second-line therapy due to its
side effects of nausea and diarrhea. Glucocorticoids are indicated in
patients with a contraindication to both NSAIDs and colchicine, such as
patients with renal failure. Uricosuric agents and allopurinol are
contraindicated during acute attacks; however, they are used as
prophylactic treatment in certain patients.

92
USMLE WORLD STEP 1 PHARMACOLOGY

Q NO 262: A 64-year-old African American woman suffers from progressive bilateral


vision loss. She takes timolol ophthalmic drops. Her past medical history is
insignificant and she uses no other medications. Which of the following structures is
timolol’s most likely target?

A. Trabecular meshwork
B. Ciliary muscle
C. Pupil sphincter
D. Ciliary epithelium
E. Lens

Explanation:
This patient has chronic glaucoma being treated with timolol. Aqueous
humor is produced by the epithelial cells of the ciliary body. It is
excreted into the posterior eye chamber and transferred through the
pupil into the anterior eye chamber. The anterior chamber angle
(iridocorneal angle) contains a trabecular mesh work though which the
aqueous humor diffuses into Schlemm’s canal (scleral venous sinus).
Schlemm’s canal drains into episcleral and conjunctival veins. The
pathway of aqueous humor is outlined by the red arrow on the diagram
below.

Glaucoma is a chronic eye disease characterized by increased


intraocular pressure. It develops due to diminished outflow or
increased secretion of the aqueous humor. Narrow-angle glaucoma occurs
when the anterior chamber narrows, obstructing the trabecular meshwork.
This type of glaucoma is often precipitated by anticholinergic
medications and manifests with an acute attack of headache and eve
pain. Open-angle glaucoma is typically more chronic and insidious. Both
types of glaucoma can ultimately lead to blindness.
The drugs used to treat glaucoma either decrease the production of
aqueous humor or increase its outflow. Timolol and other non-selective
β-blockers work by diminishing the secretion of aqueous humor by the
ciliary epithelium. Acetazolamide, a carbonic anhydrase inhibitor, also
decreases aqueous humor secretion by the ciliari epithelium.
Prostaglandin F2a (latanoprost, unoprostone, travoprost) and
cholinomimetics (pilocarpine, carbachol) decrease intraocular pressure
by increasing the outflow of the aqueous humor.
(Choices B and E) Cholinomimetics promotes ciliary muscle contraction,
causing the lens to become more convex.
(Choices A and C) Cholinergic agonists (pilocarpine and carbachol)
cause miosis by promoting contraction of the sphincter of iris. This

93
USMLE WORLD STEP 1 PHARMACOLOGY
causes the anterior chamber angle to become wider; this makes the
trabecular meshwork more accessible to outflow of aqueous humor.

Educational Objective:
Glaucoma, a disease due to increased intraocular pressure, develops due
to decreased outflow or increased production of aqueous humor. Timolol
and other beta-blockers decrease aqueous humor production by the
ciliary epithelium.

94
USMLE WORLD STEP 1 PHARMACOLOGY

Q NO 263: An elderly immigrant from the Middle East presents to your office
complaining of numbness and tingling of his hands and feet. He was
diagnosed with pulmonary tuberculosis several months ago and is currently
receiving isoniazid and rifampin. Neurologic testing demonstrates sensory
ataxia and decreased pain sensation in the distal extremities. This patient’s
condition is most likely caused by:
A. Immune-mediated demyelinization
B. Bacterial toxin-mediated axonal damage
C. Drug-mediated toxic neuron loss
D. Vitamin deficiency
E. Diabetic neuropathy

Explanation:
Isoniazid is chemically similar to pyridoxine (vitamin B6). Because of
this similarity isoniazid can compete with vitamin B6 in the synthesis
of multiple neurotransmitters (including GABA) resulting in defective
end products. Isoniazid also increases the urinary excretion of
pyridoxine, and a frank deficiency of this vitamin can eventually
develop. Vitamin B6 deficiency most commonly occurs in patients treated
with isoniazid who are elderly, alcoholics, or who have other
comorbidities (eg, kidney or liver dysfunction). When isoniazid
neuropathy occurs it usually clinically manifests as peripheral
neuropathy. To prevent these adverse effects, supplemental vitamin B6
is typically prescribed in conjunction with isoniazid.
(Choice A) Immune-mediated demyelinization occurs in multiple sclerosis
and the Guillain-Barre syndrome. These conditions are not associated
with antibiotic therapy of any kind.
(Choice B) Bacterial toxin-mediated axonal damage occurs in infections
secondary to Corynebacterium diphtheriae, a microorganism that
classically causes pseudomembranous pharyngitis. C. diphtheriae
synthesizes a potent exotoxin associated with cardiac and neural
toxicity.
(Choice C) Pyridoxine deficiency, not drug-mediated toxic neuron loss,
is responsible for this patient’s neuropathy. Drugs known to be
directly toxic to nerves include the chemotherapeutic agents that
inhibit mi CRO tubule formation (eg, vincristine and paclitaxel).
(Choice E) There is no evidence or clinical history suggestive of
diabetic neuropathy in this patient. The primary cause of diabetic
neuropathy is glucose-induced neural toxicity. High concentrations of
glucose result in the formation of glycosylation products (eg,
hemoglobin A1c) and the accumulation of sorbitol within the nerves, a
phenomenon described as metabolic nerve damage.

Educational Objective:
Isoniazid is structurally similar to pyridoxine (vitamin B6). As a
result this antibiotic increases the urinary excretion of pyridoxine —
often causing a frank deficiency of vitamin B6-- and competes for
vitamin B6-binding sites, leading to the defective synthesis of
neurotransmitters like GABA. Isoniazid-induced neuropathy can usually
be prevented with pyridoxine supplementation.

95
USMLE WORLD STEP 1 PHARMACOLOGY

Q NO 264: A 28-year-old Caucasian female is treated with high-dose prednisone for


severe lupus nephritis. Several hours after initiating therapy, she experiences fever
chills and severe fatigue. Her blood pressure is 120/70 mmHg and her heart rate is
110 beats per minute. Serum and urine are collected for CBCI blood culture,
urinalysis and urine culture, to more fully evaluate the patient’s condition. An
increase in which of the following blood cell counts might be attributable to this
patient’s therapy?

A. Neutrophils
B. Eosinophils
C. Basophils
D. Lymphocytes
E. Monocytes

Explanation:
Neutrophil counts increase following glucocorticoid administration, as
a result of “demargination” of leukocytes previously attached to the
vessel wall.
(Choice B) Eosinophil counts decrease significantly with glucocorticoid
administration. In fact this effect has been used historically as a
bioassay for glucocorticoids. Remember that glucocorticoids are used in
allergic conditions because they reduce the eosinophil count and
therefore also the release of mediators from the eosinophils.
(Choice C) Glucocorticoids also reduce basophil count. This decreases
local inflammatory responses by preventing histamine release.
(Choice D) Glucocorticoids have profound effects on the immune system,
by suppressing immunologic responses. This property has been utilized
to treat a variety of autoimmune and inflammatory conditions. The
immunosuppressive effects are mediated at many levels. In the
peripheral blood, glucocorticoids reduce lymphocyte counts within
minutes. T-lymphocyte counts are reduced to a greater degree than B-
lymphocyte counts. The acute response occurs as a result of the
redistribution of lymphocytes from the intravascular compartment to the
spleen, lymph nodes and bone marrow. Additionally, there are direct
effects of glucocorticoids on both T- and B-lymphocytes, including
inhibition of immunoglobulin synthesis and stimulation of lymphocyte
apoptosis.
(Choice E) Glucocorticoids also inhibit monocyte differentiation into
macrophages, thus decreasing the rate of antigen presentation by
macrophages to T-lymphocytes.

Educational Objective:
The acute effects of corticosteroids on the CBC include: increased
neutrophil count, and decreased lymphocyte, monocyte, basophil, and
eosinophil counts. The increase in neutrophil count results from
“demargination” of neutrophils previously attached to the vessel wall.

96
USMLE WORLD STEP 1 PHARMACOLOGY

Q NO 265: A 34-year-old Caucasian male is being treated with simvastatin and


cholestyramine for hyperlipidemia. His past medical history is significant for
myocardial infarction six months ago. Which of the following best describes the
effects of simvastatin and cholestyramine, respectively, on hepatic cholesterol
synthesis?

A. Increase, Increase
B. Increase, Decrease
C. Decrease, Increase
D. Decrease, Decrease
E. Increase, No change

Explanation:
Statins are first-line therapy for most patients with
hypercholesterolemia. These drugs competitively inhibit HMG-CoA
reductase, the enzyme responsible for the conversion of HMG-CoA to
cholesterol, the last step in hepatic cholesterol synthesis. As a
result, there is an up-regulation of LDL receptors, causing increased
uptake of LDL from the circulation. Statins are very effective agents
for reducing LDL cholesterol. Statins effects on triglyceride and HDL
levels are comparatively modest.
Bile acid-binding resins work by binding bile acids in the
gastrointestinal tract, thereby interfering with the enterohepatic
circulation of bile acids. As a result, there is hepatic synthesis of
new bile acids, a process which consumes liver cholesterol stores.
Hepatic uptake of LDL from the circulation is increased for continued
bile acid synthesis. Bile acid production and secretion are increased
by 10-fold due to this interruption of the enterohepatic circulation of
bile acids.
In summary, bile acid-binding agents increase hepatic cholesterol
synthesis, an effect opposite that of statins. Fibrates also increase
hepatic cholesterol synthesis and excretion. Agents that increase
hepatic cholesterol synthesis and excretion (e.g. bile acid-binding
agents, fibrates) put patients at increased risk for gallstone
formation. Other hyperlipidemia medications, such as ezetimibe and
niacin, do not alter hepatic cholesterol production.

Educational objective:
Simvastatin decreases and cholestyramine increases hepatic cholesterol
production. Agents that increase hepatic cholesterol synthesis (e.g.
fibrates, bile acid-binding agents) are associated with increased risk
for gallbladder stones.

97
USMLE WORLD STEP 1 PHARMACOLOGY

Q NO 266: A 50-year-old male has had increasingly severe epigastric pain that
failed to resolve with over-the-counter antacids. Upper 01 endoscopy shows
multiple ulcerations in the stomach, duodenum, and jejunum. Fasting serum
gastrin levels are elevated. Imaging studies show a pancreatic tumor with
diffuse metastasis. Which of the following is the most effective acid suppressive
therapy for this patient?
A. Proton pump inhibitor
B. High dose H2 receptor blockers
C. Combination of antacids and H2 receptor blockers
D. Sucralfate and H2 receptor blocker therapy
E. Octreotide

Explanation:
The patient described has upper gastrointestinal ulceration with
increased gastrin levels, which is very suggestive of a gastrinoma. The
typical location of gastrinoma is within the pancreas and sometimes in
peripancreatic soft tissues around head of the pancreas. The majority
of patients with gastrinomas present with upper gastrointestinal
ulceration and diarrhea.
Gastrin increases stomach-acid production by increasing the activity of
H+/ K+/ ATPase pump as well as increasing the production of histamine
from enterochromaffin cells. Suppression of gastric acid production is
the first line of therapy in most patients with gastrinomas. Proton
pump inhibitors are the drugs of choice for this purpose. Proton pump
inhibitors bind covalently at H+/K+ ATPase pump causing irreversible
inhibition of this pump. Although PPIs have a short half-life, the
irreversible inhibition of H7KATPase pump means that their actual
activity lasts up to 24 hours, until the pump regenerates.
(Choice B) Although, H2-receptor-blockage decreases gastric acid
production, it does not do so as effectively as proton pump inhibitors.
H2 receptor blockers are not first line therapy of gastrinomas.
(Choice C) Antacids are weak bases that neutralize stomach acids. The
duration of most antacids is extremely short. Even in combination with
H2 receptor blockers, antacids do not provide the excellent acid
control seen with proton pump inhibitors.
(Choice D) Sucralfate is the complex salt of sucrose with aluminium
hydroxide. Sucralfate binds to the sides of gastrointestinal erosions
and ulcerations, acting as a mucosal protectant. Sucralfate is not
absorbed into systemically and is very safe for most patients. However,
it impairing the absorption of other medications, as can most salts,
essentially by chelation. With the advent of newer medications, such as
proton pump inhibitors, the clinical utility of sucralfate is limited.
(Choice E) Somatostatin normally decreases gastric-acid production by
inhibiting the secretion of gastrin and histamine. Octreotide is the
long-acting somatostatin analog that can reduce gastric production in
patients with gastrinomas that express somatostatin receptors on the
tumor! Octreotide is not the first choice, however, because some
gastrinomas do not express somatostatin receptors. Octreotide is
expensive and can only be used parenterally.

Educational Objective:
Proton pump inhibitors decrease gastric acid production by irreversibly
inhibiting the H7RATPase pump and are first line therapy for most
patients with a gastrinoma.

98
USMLE WORLD STEP 1 PHARMACOLOGY

Q NO 267: A 30-year-old Caucasian male with adult polycystic kidney disease


presents to the emergency department with sudden onset of severe headache
and confusion. Neurological examination reveals no meningeal signs or focal
neurological deficits. Emergency CT shows blood in subarachnoid space. On
the fifth day after admission, the patient begins to complain of weakness in
his right arm and leg. Which of the following drugs could most likely have
prevented this patient’s neurological sequelae?
A. Beta-adrenergic blocker
B. Central sympatholytic
C. ACE inhibitor
D. Thiazide diuretic
E. Osmotic diuretic
F. Calcium channel blocker
G. Nitrate

Explanation:
Vasospasm leading to clinical symptoms occurs in approximately 20 to 30
percent of patients with subarachnoid hemorrhage (SAH) caused by a
ruptured intracerebral aneurysm such as a berry aneurysm. The question
stem hints at the presence of a berry aneurysm by mentioning that the
patient has adult polycystic kidney disease. Symptoms of cerebral
vasospasm include an altered mental status as well as focal
neurological deficits. Substances generated by the degradation of
subarachnoid blood clots are believed to be the etiologic agents.
Symptoms manifest no earlier than three days after the hemorrhage, and
most frequently occur seven to eight days following SAH.
Calcium channel blocker is the correct answer because Nimodipine has
been shown in studies and reviews to decrease morbidity and mortality
due to cerebral vasospasm when used as a preventative agent following
SAH. It is identified as a cerebral selective drug, though it is
unclear whether the mechanism of its cerebral protection in this
clinical setting is due to vasodilation or neural protection.
(Choice A) Beta-adrenergic blockers such as metoprolol would serve to
decrease the heart rate and blood pressure as well as cause
bronchoconstriction (a concern in asthmatics). They do not have a use
in the treatment of SAH or intracerebral vasospasm.
(Choice B) Central sympatholytics such as methyldopa and clonidine
stimulate alpha-2A receptors centrally which causes a decrease in
generalized sympathetic outflow. This would cause a decrease in blood
pressure in addition to other effects of decreasing both alpha and beta
sympathetic stimulation. They do not have a role in therapy of
vasospasm following SAH though they could possibly be used in treatment
of other intracranial hemorrhages such as those caused by hypertension.
(Choice C) ACE inhibitors such as captopril and enalapril decrease
blood pressure and cardiac hypertrophy by inhibiting the conversion of
Angiotensin Ito Angiotensin II. They are useful in hypertension,
prevention of diabetic nephropathy, and in heart failure with systolic
dysfunction. Their major side effects include cough and angioedema.
(Choice D) Thiazide diuretics are presently the first line treatment
for essential hypertension in the outpatient setting; they have no
indication for vasospasm prevention following SAH.
(Choice E) Osmotic diuretics such as mannitol are part of the treatment
for increased intracranial pressure.

99
USMLE WORLD STEP 1 PHARMACOLOGY
(Choice G) Nitrates such as nitroglycerin and isosorbide mononitrate
are used as vasodilators primarily for the treatment of angina
pectoris.

Educational Objective:
Calcium channel blockers, specifically Nimodipine, can be used to
assist in the prevention of cerebral vascular spasm following SAH. This
is an alternative use of calcium channel blockers.

100
USMLE WORLD STEP 1 PHARMACOLOGY

Q NO 268: A new adrenergic agonist is infused to a healthy volunteer and


hemodynamic parameters are recorded. The following curves are
obtained. Effects of the agent tested in this experiment would most likely
include which of the following?

A. Increased renal perfusion


B. Decreased heart afterload
C. Decreased AV conduction
D. Decreased venous tone
E. Decreased pulmonary capillary wedge pressure

Explanation:
The diagram above shows increase in both systolic and diastolic blood
pressure associated with decrease in the heart rate. These effects
occur after the administration of a-adrenergic agonists (such as
phenylephrine or methoxamine). Stimulation of α1-adrenoreceptors in the
blood vessels of the skin and viscera causes vasoconstriction. Increase
in systemic blood pressure stimulates baroreceptors of carotid sinus
and aortic arch, with subsequent increase of vagal influence on heart.
This effect results in decrease in heart rate, contractility and
conductance. Stimulation of α2-adrenoreceptors inhibits norepinephrine
release, which may also contribute to bradycardia.
The other effects of a-adrenergic agonists include mydriasis
(stimulation of pupillary dilator muscle) and increase in tone of
external urethral sphincter with delayed urination. Constriction of
blood vessels of nasal mucosa associated with these drugs makes them
useful as decongestants.
(Choice A) a-adrenergic agonists cause vasoconstriction of renal and
splanchnic arteries. They decrease perfusion to the kidneys and other
viscera.
(Choice B) Increased tone of the vascular walls associated with a-
adrenergic agonist raises afterload. Stronger contraction of the
myocardium is required to expel the blood against the increased
resistance.
(Choice D) a-adrenergic agonists increase both arterial and venous
tone.
(Choice E) Pulmon capillary wedge pressure equals the pressure in the
left atrium. Itis increased after administration of a-adrenergic

101
USMLE WORLD STEP 1 PHARMACOLOGY
agonists due to increase in peripheral vascular resistance and higher
venous return to the heart.

Educational Objective:
α-adrenergic agonists increase both systolic and diastolic blood
pressure via stimulation of al-adrenoreceptors in the vascular walls.
Elevated systemic blood pressure is associated with reflex increase in
vagal influence on heart. It results in decrease in heart rate,
contractility and conductance.

102
USMLE WORLD STEP 1 PHARMACOLOGY

Q NO 269: A 57-year-old female presents to the emergency department after


vomiting a substance that resembled “coffee grounds.” She reports that she
now feels lightheaded as well. Her past medical history is significant for deep
venous thromboses, for which she takes warfarin, and occasional joint pain,
for which she takes aspirin. Her blood pressure is 90/60mm Hg and her pulse
is 110 beats per minute. Which of the following substances would provide the
fastest reversal of warfarin’s effects?
A. Vitamin K
B. Protamine
C. Aminocaproic acid
D. Fresh frozen plasma
E. Cryoprecipitate
F. Desmopressin

Explanation:
Warfarin is the most commonly used agent for long-term anticoagulation
in the setting of venous thromboses and pulmonary thromboembolism.
Warfarin inhibits the vitamin K dependent g-carboxylation of the
glutamic acid residues of clotting factors II, VII, IX and X, causing
production of dysfunctional coagulation proteins.
The therapeutic effect of warfarin is monitored by measuring the
prothrombin time (PT). The international normalized ratio (INR) is a
standardized PT. The target INR for therapeutic warfarin
anticoagulation is 2-3. Bleeding is a common complication of warfarin
therapy and the risk is increased with INRs above 3.0. Treatment of
life-threatening bleeding requires rapid reversal using fresh frozen
plasma. Vitamin K can help to reverse warfarin action but the effects
take time.
(Choice B) Protamine is used for heparin reversal. It acts by binding
and chemically inactivating heparin.
(Choice C) Aminocaproic acid is an antifibrinolytic agent that inhibits
plasminogen activators and, to a lesser degree antiplasmin activity.
Aminocaproic acid helps achieve hemostasis when fibrinolysis is the
cause of bleeding.
(Choice E) Whereas fresh frozen plasma contains all of the coagulation
factors cryoprecipitate contains only cold- soluble proteins (Factor
VIII, fibrinogen, von Willebrand factor and vitronectin).
(Choice F) Desmopressin is a synthetic analogue of anti-diuretic
hormone (ADH) used in the treatment of diabetes insipidus. At high
doses desmopressin increases factor VIII activity in patients with
hemophilia A and von Willebrand disease. High-dose desmopressin can
help control bleeding associated with these disorders.

Educational Objective:
Both vitamin K and fresh frozen plasma are used for reversing warfarin-
induced anticoagulation. Fresh frozen plasma rapidly reverses
warfarin’s effects whereas vitamin K requires time for clotting factor
re-synthesis.

103
USMLE WORLD STEP 1 PHARMACOLOGY

Q NO 270: A 40-year-old female with a history of schizophrenia is treated with


risperidone. She was brought to the hospital by her group home because she
has not had a menstrual period in 6 months. Her BMI is 35 kg/m2. Which of
the following is the most likely cause of her amenorrhea?
A. Schizophrenia
B. Risperidone
C. Premature menopause
D. Pregnancy
E. Polycystic ovarian disease

Explanation:
Risperidone is a very effective anti-psychotic drug often used in the
management of schizophrenia. One side effect of risperidone is
hyperprolactinemia, a condition that can cause amenorrhea.
The secretion of prolactin is unique in that it is regulated by the
inhibitory effect of hypothalamic dopamine. Dopamine acts on the D2
receptors of lactotrophs, causing the release and synthesis of
prolactin to decrease. As risperidone has anti-dopaminergic action, it
decreases dopaminergic D2 receptor activity, causing a loss of feedback
inhibition on prolactin-producing lactotrophs. Hyperprolactinemia then
causes hypogonadism because prolactin inhibits the hypothalamus from
releasing gonadotrophin releasing hormone (GnRH).
(Choice A) It would be yew unusual indeed for schizophrenia itself to
lead to amenorrhea and/or hyperprolactinemia.
(Choice C) Premature menopause is defined as loss of ovarian function
in female less than 35 years of age. This patient is 40 years old.
(Choice D) Pregnancy should be ruled out in any patient with
amenorrhea. However, pregnancy is not the most likely answer as the
chances of this patient becoming pregnant at forty are less than her
chance of having risperidone induced hyperprolactinemia. Furthermore, a
pregnant woman at six months gestation would be clinically obvious.
(Choice E) Polycystic ovarian disease is a condition characterized by
infertility, oligomenorrhea, and hyperandrogenism. Typically,
polycystic ovarian disease begins sometime in the teenage years.
Occurrence of polycystic ovarian disease for the first time in a 40-
year-old female is unlikely. Most patients with polycystic ovarian
disease are obese, although the disease can be seen in leaner subjects.
Insulin resistance is thought to play a central role in the
pathogenesis of polycystic ovarian disease.

Educational Objective:
The secretion of prolactin is controlled by the inhibitory effect of
hypothalamic dopamine. Hyperprolactinemia causes hypogonadism by
inhibiting the release of gonadotrophin releasing hormone from the
hypothalamus. Risperidone and other anti psychotics cause
hyperprolactinemia by their anti dopaminergic action.

104
USMLE WORLD STEP 1 PHARMACOLOGY

Q NO 271: A group of investigators examined the effects of different muscarinic


agonists and antagonists on the bladder musculature. The curves presented
below were constructed after application of different agents. Curve A represents
the effect of bethanechol, a pure cholinergic agonist. Which of the curves most
likely represents the effect of bethanechol after application of a reversible
competitive antagonist?

A. Curve A
B. Curve B
C. Curve C
D. Curve D
E. Curve E

Explanation:
Antagonists are agents that bind to but do not activate receptors.
Antagonists can be either competitive or noncompetitive. Competitive
antagonists bind to the exact same receptor binding sites as endogenous
or exogenous agonists, thereby preventing agonist binding and
activation of the receptor. Competitive antagonism can be reversible
(ionic bond) or irreversible (covalent bond). The effect of a
reversible competitive antagonist can be overcome by high
concentrations of agonist, which cause displacement of the antagonist
from the receptors by mass action.
Noncompetitive antagonists on the other hand bind to receptors at a
site other than the primary agonist binding site. There by causing a
conformational (allosteric) change in the receptor protein that
modifies the agonist binding site and prevents agonist binding. By
binding to a different site than the agonist noncompetitive antagonists
are able to remain bound to the receptor even when high concentrations
of agonist are present because the agonist is unable to displace
noncompetitive antagonists from the receptor?
The question has asked how the curve for bethanechol (Curve A) would be
affected by the introduction of a reversible competitive antagonist. A
reversible competitive antagonist is expected to require higher doses
of the full agonist (bethanechol)to be present to achieve the same
effect at each point along the curve. If enough of the original
substrate is added the reaction can still reach the same maximum
effect. Thus the log dose-response curve for a full agonist combined
with a reversible competitive antagonist will exhibit a parallel shift
to the right in the log-dose response curve, with an increase in the
ED50 and no change in the maximum effect (Emax).
(Choice A) Curve A is the dose-response curve for the original agonist
drug in the absence of modifiers such as antagonists.
(Choices B and C) Both Curves B and C represent the effect of adding a
noncompetitive agonist to the agonist. Note that the ED50 is unchanged
(i.e. the ED50 for Curves A, B and C occur at the exact same log drug

105
USMLE WORLD STEP 1 PHARMACOLOGY
concentration). This is a distinguishing characteristic of
noncompetitive antagonism. The maximum effect (Emax) of Curves B and C
are reduced because the noncompetitive agonist has effectively reduced
the number of receptors available for binding. The decrease in
Emax<will depend on the dose of noncompetitive antagonist present and
is illustrated by the difference between Curves B and C. where a higher
concentration of antagonist is present in Curve C than in Curve B.
(Choice D) Curve D illustrates a rightward shift in the dose-response
relationship, but the shift is not a parallel shift because the shape
of the curve differs from that of Curve A, with changes in both Emax
and ED50. This curve is not characteristic of any single antagonist
effect and may be the result of adding multiple types of antagonists.

Educational Objective:
The changes in the log dose-response curve expected for the effect of a
reversible competitive antagonist added to a full agonist are: 1) a
parallel shift to the right in the log-dose response curve,
illustrating an increase in the ED50, and 2) no change in the maximum
effect (Emax).
Competitive=change ED50=shift right, noncompetitive = change Emax =
Shift down.

106
USMLE WORLD STEP 1 PHARMACOLOGY

Q NO 272: A 40-year-old female with a history of depression and hypertension is


brought to the ER after being found obtunded in her apartment. She is
hypotensive and bradycardic on physical examination. Intravenous glucagon is
administered, and her condition improves. Which of the following intracellular
changes is most likely responsible for the improvement in her condition?
A. Increased synaptic release of glutamate
B. Decreased cAMP in vascular smooth muscle
C. Decreased DAG in vascular smooth muscle
D. Increased cAMP in cardiac myocytes
E. Increased IP3 in cardiac myocytes

Explanation:
This patient has most likely overdosed on beta blocker medications.
Beta blocker overdose causes diffuse non- selective blockade of
peripheral beta adrenergic receptors, causing depression of myocardial
contractility, bradycardia, and varying degrees of AV block. The result
is a low cardiac output state.
Glucagon is the drug of choice for beta blocker overdose. Glucagon acts
on G protein-coupled receptors, increasing intracellular cAMP and thus
increasing the release of intracellular calcium during muscle
contraction. This increases heart rate and cardiac contractility.
Improvements in heart rate and blood pressure may be observed within
minutes.

Educational Objective:
Patients who have overdosed on beta blockers should be treated with
glucagon, which increases heart rate and contractility independent of
adrenergic receptors. Glucagon activates G-protein-coupled receptors on
cardiac myocytes, causing activation of adenylate cyclase and raising
intracellular cAMP. The result is calcium release from intracellular
stores and increased sinoatrial node firing.

107
USMLE WORLD STEP 1 PHARMACOLOGY

Q NO 273: A new drug is shown to block ergosterol synthesis in susceptible cells by


interfering with lanosterol demethylation. This agent would most likely have which
of the following activities?

A. Bactericidal
B. Fungicidal
C. Bacteriostatic
D. Fungi static
E. Antiviral

Explanation:
Triazoles are the class of antifungals that inhibit the synthesis of
ergosterol (a component of the fungal cell membrane). This inhibition
involves blockade of the demethylation of lanosterol into ergosterol.
In other words the triazoles inhibit ergosterol synthesis. Triazoles
are fungistatic.
The triazoles include ketoconazole, fluconazole, itraconazole and
voriconazole. Most fungi that cause systemic mycoses are sensitive to
the triazoles. Ketoconazole, the first triazole, has the narrowest
antifungal spectrum and is rarely used today. All triazoles inhibit
cytochrome P450 interfering with the liver metabolism of many other
medications.
(Choices A and C) Ergosterol is a component of the fungal cell
membrane, and is not present in bacteria. The drug in question
therefore is not bactericidal or bacteriostatic.
(Choice B)The most intensive synthesis of cell components occurs in
growing cells. For this reason triazoles are mostly fungistatic. Some
fungicidal action may be seen at increased doses.
(Choice E) Ergosterol is not a component of viruses. Thus, inhibition
of ergosterol synthesis does not have an antiviral effect.

Educational Objective:
Azole antifungals include ketoconazole, fluconazole, itraconazole and
voriconazole. These drugs inhibit the demethylation of lanosterol into
ergosterol, thus suppressing the synthesis of the fungal cell membrane.
These drugs are fungistatic.

108
USMLE WORLD STEP 1 PHARMACOLOGY

Q NO 274: In vitro studies demonstrate that tumor cell lines can become resistant
after exposure to various anticancer agents. These cells express a specific cell
surface glycoprotein that has which of the following functions?

A. Tyrosine kinase-coupled receptor


B. Adenylate cyclase-coupled receptor
C. Cell adhesion molecule
D. Transmembrane ion channel
E. ATP-dependent transporter

Explanation:
Human tumor cells have developed the ability to resist chemotherapy in
much the same way that many bacteria have developed resistance to
antibiotics. One well-studied mechanism by which human tumor cells
resist these agents is via the human multidrug resistance (MDR1) gene.
The prototype product of this gene is P-glycoprotein, a transmembrane
protein that functions as an ATP-dependent efflux pump. P-glycoprotein
is normally expressed in intestinal and renal tubular epithelial cells
and functions to eliminate foreign compounds from the body. This
protein is also present in the capillary endothelium of the vessels
that form the blood-brain barrier. In this location P-glycoprotein
prevents the penetration of foreign compounds into the CNS. In tumor
cells, this ATP-powered transmembrane pump protein actively removes
chemotherapeutic agents particularly hydrophobic agents like the
anthracyclines.
Drugs such as verapamil, diltiazem, and ketoconazole, among others,
have been shown to reduce the action of this multidrug resistance
protein in experimental models: the development of agents specifically
intended to abolish this mechanism of resistance is currently underway.
(Choice A) Tyrosine kinase receptors generally mediate the effects of
hormones that promote anabolism and cell growth such as insulin,
insulin-like growth factor epidermal growth factor, platelet-derived
growth factor, and others.
(Choice B) cAMP is the second messenger in the G-protein / adenylate
cyclase second messenger system. This system is utilized by numerous
hormone receptors including beta-adrenergic receptors, V2 ADH
receptors, and calcitonin, glucagon, TSHI ACTHI and HCG receptors,
among many others.
(Choice C) Cell adhesion molecules (CAMs) are proteins located on the
cell surface that mediate binding with other cells or with the
extracellular matrix. Examples include selectins, integrins, and
cadherins. Cell adhesion molecules are generally downregulated in
malignant tumors, which allows these cells to spread from their site of
origin.
(Choice D) Transmembrane ion channels selectively allow certain ions to
pass into or out of the cell. These channels facilitate the formation
of a voltage gradient across the cell membrane. Examples of ion
channels, transporters, and exchangers are ubiquitous. These proteins
participate in the formation of urine, nerve and myocyte
depolarization, muscle contraction, sweat formation, exocrine secretion
formation, and many other processes.

109
USMLE WORLD STEP 1 PHARMACOLOGY
Educational Objective:
The human multidrug resistance (MDR1) gene codes for P-glycoprotein, a
transmembrane ATP-dependent efflux pump protein that has a broad
specific for hydrophobic compounds. This protein can both reduce the
influx of drugs into the cytosol and can increase efflux from the
cytosol, thereby preventing the action of chemotherapeutic agents.

110
USMLE WORLD STEP 1 PHARMACOLOGY

Q NO 275: A 40-year-old female presents to your office because of diarrhea that is


accompanied by occasional abdominal cramps. After basic evaluation you proceed
with diphenoxylate therapy in this patient. Which of the following is the primary
target for the drug you prescribed to this patient?

A. Absorption
B. Secretion
C. Digestion
D. Inflammation
E. Motility

Explanation:
Diphenoxylate is an opiate anti-diarrheal structurally related to
meperidine. It binds to mu opiate receptors in the gastrointestinal
tract and slows motility. Low therapeutic doses allow for potent anti-
diarrheal effects without other significant opiate effects (euphoria
and physical dependence). However, higher doses can produce morphine
like euphoria and physical dependence. Thus therapeutic doses of the
drug are combined with small doses of atropine and marketed under the
brand name Lomotil. As higher doses of diphenoxylate are used, higher
doses of atropine will produce adverse symptoms (dry mouth blurry
vision nausea) and discourage abuse. Common side effects of
diphenoxylate include bloating and mild sedation.
(Choice A) One primary cause of diarrhea involves decreased absorption
of water or electrolytes. Substances that are poorly absorbed can
retain fluid and cause diarrhea. No drugs used in treating diarrhea
target enhanced absorption of fluids or electrolytes by the gut.
Rather, adsorbents, like kaolin-pectin and attapulgite, adsorb fluids
toxins, nutrients, and digestive juices to make stools less watery.
They allow for effective symptomatic relief of diarrhea.
(Choice B) Excess secretions of fluid and electrolytes in the gut can
also lead to diarrhea. Secretory diarrhea can be due to many different
substances (vasoactive intestinal peptide, bacterial toxins, etc.) that
increase secretions. Drugs that target secretory types of diarrhea
include bismuth subsalicylate, probiotics, and octreotide.
(Choice C) Diarrhea secondary to malabsorption and poor digestion is
primarily due to inability to digest carbohydrates and fats. For
carbohydrates, lactase deficiency is the most common problem seen and
causes inability to breakdown lactose leading to diarrhea. Lactase
enzyme supplements are available to assist patients with lactose
digestion. Fat malabsorption is mainly due to pancreatic disease. In
pancreatic disease pancreatic enzyme deficiency decreases fat breakdown
leading to diarrhea. Pancreatic enzymes are also available for
supplementation in patients with deficiencies.
(Choice 0) Ulcerative colitis and Crohn’s disease are inflammatory
bowel diseases that commonly present with diarrhea. Inflammation of the
gut causes mucus protein, and bloody discharge leading to diarrhea.
Treatment involves anti-inflammatory agents corticosteroids, and other
immunosuppressive agents.

111
USMLE WORLD STEP 1 PHARMACOLOGY
Educational Objective:
1. Diphenoxylate is an opiate anti-diarrheal structurally related to
meperidine. It binds to mu opiate receptors in the gastrointestinal
tract and slows motility. Low therapeutic doses allow for potent anti-
diarrheal effects without euphoric effects. Since higher doses can lead
to euphoria and physical dependence the drug is combined with atropine
at therapeutic doses to discourage abuse.
2. Octreotide is helpful for secretary diarrhea.

112
USMLE WORLD STEP 1 PHARMACOLOGY

Q NO 276: A 72-year-old male is treated for congestive heart failure, hypertension


and diabetes mellitus. He experienced myocardial infarction two years ago. He
presents to your office saying that he has ‘cancer because his right breast got
enlarged. Which of the following drugs can be responsible for this patient’s
problem?

A. Hydrochlorothiazide
B. Torsemide
C. Spironolactone
D. Triamterene
E. Acetazolamide

Explanation:
Spironolactone is an aldosterone antagonist with very mild diuretic
effects. Aldosterone is a steroid hormone that binds to
mineralocorticoid receptors to produce Na and fluid retention.
Additionally, elevated aldosterone levels are known to cause cardiac
remodeling and worsen left ventricular dysfunction in heart failure
patients. By competitively inhibiting aldosterone, spironolactone
effectively blocks aldosterone’s detrimental cardiac effects. Based on
results from the PALES trial, addition of low dose spironolactone to
standard therapy significantly reduces morbidity and mortality in class
Ill and IV heart failure patients.
Spironolactone has a potassium sparing effect and can cause significant
hyperkalemia. Since it is structurally similar to steroids,
spironolactone can cause endocrine effects including gynecomastia,
decreased libido, hirsutism, and impotence. Gynecomastia occurs in
approximately 10 % of patients. Although it is generally bilateral, it
can present unilaterally. The exact mechanism is veri complex and may
involve increased estradiol and androstenedione levels and other
various anti-androgen effects. Eplerenone is a newer and more selective
aldosterone antagonist that may produce less endocrine effects.
(Choice A) More common side effects of thiazides include hypokalemia,
and hypomagnesemia. Less common side effects include hypotension,
volume depletion, hypercalcemia, and hyponatremia.
(Choice B) Common side effects of torsemide include hypokalemia,
hypomagnesemia, and hypocalcemia. Less common side effects include
volume depletion, hyponatremia, decreased GEP, hypotension, and
ototoxicity.
(Choice D) Triamterene is a potassium sparing diuretic that works by
blocking Na channels in the distal tubule and collecting duct. This
results in increased Na and fluid excretion. The most common side
effects of triamterene are hyperkalemia, nausea, vomiting, dizziness,
and leg cramps.
(Choice E) Common side effects of carbonic anhydrase inhibitors include
somnolence, paresthesias, and urine alkalinization. More rare side
effects include metabolic acidosis, dehydration, hypokalemia, and
hyponatremia.
Educational Objective: Spironolactone is an aldosterone antagonist
commonly used in treating class Ill and IV heart failure patients.
Since it is structurally similar to steroids, spironolactone can cause
endocrine effects including gynecomastia, decreased libido, hirsutism,
and impotence. Gynecomastia occurs in approximately 10 % of patients.
Eplerenone is a newer and more selective aldosterone antagonist that
may produce less endocrine effects.

113
USMLE WORLD STEP 1 PHARMACOLOGY

Q NO 277: A 65-year-old male is hospitalized in the ICU with septic shock. His
blood pressure is 60/40 mmHg despite aggressive intravenous hydration.
Norepinephrine is administered in response to the patient’s hypotension.
Which of the following cellular process is seen with norepinephrine therapy?
A. cAMP increase in vascular smooth muscle cells
B. DAG decrease in vascular smooth muscle cells
C. cAMP increase in cardiac muscle cells
D. cAMP decrease in bronchial smooth muscle cells
E. IP3 increase in cardiac muscle cells

Explanation:
Norepinephrine is an adrenergic agonist that affects predominantly α
and β1 receptors with little influence on β2 receptors. Its most
clinically significant effects include:
1. Stimulation of al adrenoreceptors leading to vasoconstriction in the
skin and viscera, resulting in increases in systolic and diastolic
blood pressure with decreased renal and hepatic blood flow. Increased
peripheral vascular resistance causes reflex bradycardia that
counteracts β1-mediated positive chronotropic effect (see later).
2. Stimulation of β adrenoreceptors of the heart increases cardiac
contractility and conduction. Beta-i stimulation also leads to
increased heart rate (positive chronotropic effect), but this effect is
counteracted by the baroreceptor-mediated reflex bradycardia seen with
increased peripheral resistance. As a result of these effects after
administration of epinephrine, the heart rate remains largely
unchanged.
Each type of adrenoreceptors linked to a specific signal transduction
system. Stimulation of these receptors leads to a change in
concentration of the second messenger that mediates the effects of
norepinephrine (“the first messenger”) on the target organ. Seethe
chart below for the summary of norepinephrine effects on second
messenger synthesis.

* Remember that norepinephrine does not affect β2-adrenoreceptors.


Their effect is given here for your reference. Norepinephrine effects
are highlighted in the blue background.
(Choice A) cAMP increases in vascular smooth muscle cells occur after
stimulation of β2 adrenoreceptors. Norepinephrine stimulates only α and
β but not β2. adrenoreceptors.
(Choice B) Norepinephrine-mediated stimulation of al adrenoreceptors
produces increases in IP3 and DAG synthesis (increased calcium) in the
vascular smooth muscle cells.
(Choice D) Stimulation of β2 adrenoreceptors in the bronchial smooth
muscle cells leads to increased cAMP synthesis and subsequent
bronchodilatation.

114
USMLE WORLD STEP 1 PHARMACOLOGY
(Choice E) Cholinergic muscarinic receptors are linked to the 1P3
signal transduction system. Stimulation of these receptors in the heart
leads to increases in IPS concentration in cardiac myocytes.
Norepinephrine’s cardiac effects are mediated by β1 adrenoreceptors
that are linked to the cAMP signal transduction pathway.

Educational Objective:
Norepinephrine stimulates cardiac β adrenoreceptors, which are linked
to the cAMP signal transduction pathway. Stimulation of these receptors
by norepinephrine causes increases in cAMP concentration within cardiac
myocytes.

115
USMLE WORLD STEP 1 PHARMACOLOGY

Q NO 278: A new inhaled anesthetic has been developed and is tested in a series of
experiments. It turns out that the anesthetic has a very high arteriovenous
concentration gradient soon after beginning inhalation. Which of the following best
describes the properties of the new anesthetic?

A. Low tissue uptake


B. Slow onset of action
C. Fast equilibration with the brain
D. Low potency
E. Low tissue solubility

Explanation:
Before inhalation anesthetics reach the target organ (brain) they move
through the number of compartments (inhaled air — lungs — blood—.
brain). The onset of anesthesia occurs when a sufficient concentration
of anesthetic is transferred to the brain. The following factors
influence the rate of transfer of anesthetic through the compartments
and determine the onset of action:

Arteriovenous concentration gradient (the difference between the


concentration of gas in arterial and venous blood) reflects the
solubility of the gas in tissues. If the solubility is high, a large
amount of anesthetic is taken up by target tissues from the arterial
blood and its venous concentration is low. Highly soluble anesthetic is
characterized by high arteriovenous gradient (high tissue uptake) and
low onset of action (more gas is needed to saturate the tissues).
Anesthetic gas with low solubility, to the contrary, has a low
arteriovenous gradient and rapid onset of action.
Low tissue uptake (choice A) is characteristic for gases with low
solubility in tissues (choice E). These anesthetics have low
arteriovenous concentration gradient, and fast equilibration with
tissues (choice C).

116
USMLE WORLD STEP 1 PHARMACOLOGY
(Choice D) Potency of the anesthetic is determined by MAC (minimal
alveolar concentration). The potent anesthetics have low MAC.
Arteriovenous concentration gradient reflects the rate of induction but
not the potency.

Educational Objective:
Arteriovenous concentration gradient reflects the solubility of
anesthetic in tissues. Highly soluble anesthetics are characterized by
high arteriovenous concentration gradients and slow onset of action.

117
USMLE WORLD STEP 1 PHARMACOLOGY

Q NO 279: A 60-year-old male was diagnosed with diabetes mellitus six years ago.
His current treatment regimen includes a long-acting insulin injection once a day.
In reviewing this patient’s glycemic control, you notice that postprandial glucose
levels are too high. Which of the following insulin preparations has the most rapid
action peak and is ideal for postprandial hyperglycemia?

A. Regular insulin
B. NPH insulin
C. Insulin aspart
D. Insulin glargine
E. Lente insulin

Explanation:
Over the past decade, insulin therapy has tremendously improved.
Although human rapid-acting regular insulin manufactured by recombinant
DNA technology is pure, it is problematic in its delayed peak time. Its
action starts within 30-60 minutes and peaks at two to three hours.
Unfortunately the effects of regular insulin peak after postprandial
peak glucose concentration. This mismatch between insulin and glucose
peak generally causes inadequate control of glucose following meals.
The delayed onset in the action of regular insulin is due to dimer and
hexamer formation. Dimers and hexamers take time to disassociate after
injection which causes some absorption delay. To overcome this problem
ultra rapid-acting insulins were produced. These monomeric insulins
such as lispro and aspart, have significantly improved insulin therapy.
Because dissociation from hexamers is not required these monomeric
insulins absorb rapidly and their onset of action is under 15 minutes
peaking between 30-90 minutes, a pattern that closely mimics the
endogenous postprandial insulin response of normal individuals. A third
rapidly acting insulin now available is called Glulysine.
(Choices B and E) Insulin can be modified to suspension, which prolongs
action by delaying absorption from the subcutaneous injection site.
This modification is accomplished by the addition of protamine to NPH
(neutral protamine Hagdedom) insulin and by zinc to Lente insulin. NPH
and Lente insulin were very commonly used in the past but are being
replaced by a ultra-long-acting insulin called glargine insulin. The
118
USMLE WORLD STEP 1 PHARMACOLOGY
actions of NPH and Lente insulin begin between one to three hours and
peak between 4-12 hours with a total duration up to 18-24 hours.
(Choices D) Insulin glargine is a very long-acting insulin analog.
After injection glargine insulin precipitates in the subcutaneous
tissue which allows very slow absorption. This insulin does not have a
peak and its action lasts up to 24 hours.

Educational Objective:
Diabetic patients need two types of insulin: both a basal long-acting
insulin and a short-acting insulin. The latter covers post-meal
hyperglycemia. The best basal, long-acting insulin is glargine insulin
(Lantus), which is supplemented by at least 3 shots of short-acting
insulin at mealtimes. Before the advent of glargine, NPH was the best
long-acting basal insulin. Since NPH is only good for 12-18 hours,
patients need two shots per day, plus the short-acting mealtime shots.
Glargine allows only four shots per day, while the use of NPH requires
five shots.
The best short-acting insulins are aspart and lispro. Before the advent
of these two drugs, regular insulin was the best option for post meal
hyperglycemia. Insulin aspart and insulin lispro have a very rapid
onset of action.

119
USMLE WORLD STEP 1 PHARMACOLOGY

Q NO 280: A thin 46-year-old Caucasian female presents to clinic for routine


A.
examination. Her last menstrual period was one year ago. Her diet consists
mainly of vegetables and fruit and she takes a daily multivitamin. She enjoys
jogging but has little time for outdoor activities. Her mother died of breast
cancer at the age of 52 years. The patient is concerned that her family
history puts her at risk for breast cancer. X-ray absorptiometry studies
demonstrate abnormally decreased bone density in the lumbar vertebrae.
Which of the following drugs can decrease the risk of both bone fractures
and breast cancer in this patient?
Ethinyl estradiol
B. Medroxyprogesterone
C. Alendronate
D. Calcium and vitamin D
E. Raloxifene
F. Leuprolide
G. Finasteride

Explanation:
As a selective estrogen receptor modulator (SERM)I raloxifene binds to
estrogen receptors and exhibits tissue specific behavior that can
imitate or antagonize the effects of natural estrogen. Raloxifene has
estrogen agonist activity on bone, the cardiovascular system, and blood
lipoproteins. Net bone resorption is decreased, thereby slowing
postmenopausal osteoporosis. In the breast and uterus, however,
raloxifene behaves as an estrogen antagonist. One large research trial
concluded that raloxifene administration for a median of 40 months
resulted in an 84% relative reduction in the incidence of estrogen
receptor-positive breast cancer.
(Choice A) Estradiol can lower the risk of postmenopausal osteoporosis
but would not be expected to lower the risk of estrogen receptor-
positive breast cancer.
(Choice B) Oral medroxyprogesterone acetate reduces the incidence of
endometrial hyperplasia and the risk of endometrial carcinoma in
postmenopausal women on estrogen replacement therapy.
Medroxyprogesterone acetate is also used parenterally as a long-acting
contraceptive. Prolonged use of parenteral medroxyprogesterone is
associated with decreased bone mineral density in women of all ages.
(Choice C) Alendronate is a synthetic bisphosphonate analog of
pyrophosphate. As a bone resorption inhibitor, it prevents and treats
osteoporosis in postmenopausal women and in corticosteroid-treated
individuals. Alendronate is not known to protect against the
development of breast cancer.
(Choice D) Vitamin D and calcium supplementation can counteract
postmenopausal and senile osteoporosis. However this vitamin and
mineral combination is not known to affect the risk of breast cancer.
(Choice F) Leuprolide is a GnRH analog with agonist properties when
administered in a pulsatile fashion and antagonist properties when
administered continuously. If given as an agonist to a woman, her
estrogen levels would likely increase protecting her against
osteoporosis but not breast cancer. If given as an antagonist to a
woman her estrogen levels would likely decrease protecting her against
estrogen receptor-positive breast cancer but not osteoporosis.
(Choice C) Finasteride is an inhibitor of the 5-a reductase utilized by
tissues to convert testosterone to dihydrotestosterone. As an anti-
androgen, finasteride would not be expected to have a marked effect on
120
USMLE WORLD STEP 1 PHARMACOLOGY
the incidence of estrogen level-dependent osteoporosis or estrogen
receptor-positive breast cancer.

Educational Objective:
As a selective estrogen receptor modulator (SERM) raloxifene binds to
estrogen receptors and exhibits tissue specific behavior that either
imitates or antagonizes the effects of natural estrogen. In bone the
estrogen agonist effects of raloxifene predominate and osteoporosis is
inhibited. In mammary tissue, the estrogen antagonist effects of
raloxifene predominate and protection is provided against estrogen
receptor-positive breast cancer.

121
USMLE WORLD STEP 1 PHARMACOLOGY

Q NO 281: A 52-year-old male with advanced HIV infection is treated with high-
dose ganciclovir for hemorrhagic colitis. Antiretroviral therapy is also begun.
Giving zidovudine to this patient is most likely to precipitate which of the
following conditions?

A. Pancreatitis
B. Hepatitis
C. Neutropenia
D. Renal failure
E. Hypocalcemia
F. Toxic epidermal necrolysis

Explanation:
Ganciclovir triphosphate interferes with human host cell DNA synthesis
to a greater extent than does acyclovir triphosphate. Major adverse
effects of ganciclovir include neutropenia, anemia thrombocytopenia,
and impaired renal function. As ganciclovir is used to treat
cytomegalovirus infections, it is commonly administered to patients
with advanced HIV.
Zidovudine triphosphate can bind to and inhibit some mammalian cellular
and mitochondrial DNA polymerases, particularly the β- and y-
polymerases. The major adverse effect of zidovudine is bone marrow
suppression (eg, anemia granulocytopenia).
Because ganciclovir and zidovudine can similarly impair hematopoietic
cell DNA synthesis, there is an increased incidence of neutropenia and
anemia in patients using this drug combination?
(Choice A) Pancreatitis is an important side effect of didanosine.
(Choice D) Impaired renal function is associated with the use of
ganciclovir, not zidovudine.
(Choice E) Foscarnet is associated with numerous electrol4e
abnormalities, including hypocalcemia, hypokalemia, hypomagnesemia, and
hypophosphatemia.
(Choice F) Cutaneous drug hypersensitivity reactions such as Stevens-
Johnson syndrome and toxic epidermal necrolysis are associated with
nevirapine and efavirenz, two antiretroviral medications.

Educational Objective:
Neutropenia is a significant adverse effect of ganciclovir therapy, and
its incidence is increased with co-administration of zidovudine. Both
drugs can affect DNA synthesis in hematopoietic stem cell lines
resulting in bone marrow suppress i on.

122
USMLE WORLD STEP 1 PHARMACOLOGY

Q NO 282: A 62-year-old male who was treated for acute episodes of gout arthritis
experiences some relief of symptoms but complains of diarrhea and mild nausea.
The drug used in this patient most likely affects which of the following cell
structures?

A. Nucleus
B. Rough endoplasmic reticulum
C. Smooth endoplasmic reticulum
D. Golgi apparatus
E. Microsomes
F. Cytoskeleton
G. Peroxisomes

Explanation:
Treatment options for acute gouty arthritis are nonsteroidal
antiinflammatory drugs colchicine, and glucocorticoids. The preferred
first line treatment of acute gouty arthritis is the use of
nonsteroidal antiinflammatory drugs. However colchicine is sometimes
used to treat acute gouty arthritis. Colchicine binds to the
intracellular protein tubuli n and inhibits its polymerization into
microtubules. This, in turn disrupts membrane-dependent functions such
as chemotaxis and phagocytosis.
For treatment of acute gouty arthritis, colchicine is administered 0.6
mg every 1-2 hours for a maximum 6mg cumulative dose (24 hours);
gastrointestinal toxicity (diarrhea) develops at higher doses.
Important adverse effects of colchicine are nausea, abdominal pain, and
diarrhea. Colchicine should be avoided in patients who are elderly or
have renal dysfunction.

Educational Objective:
Colchicine affects tubulin polymerization into microtubules. Important
side effects of colchicines are nausea, abdominal pain, and diarrhea.

123
USMLE WORLD STEP 1 PHARMACOLOGY

Q NO 283: A 55-year-old Caucasian female is treated with ramipril for


hypertension. Her blood pressure decreases to normal value over
several weeks of treatment. She seems to be compliant with her
medication and experiences no significant side effects. Which of the
following is the most likely combination of changes in response to this
patient’s treatment?
Renin Angiotensin I Angiotensin II Aldosterone Bradykinin
A. Increase Increase Decrease Decrease Increase
B. Increase Increase Increase Decrease Increase
C. Increase Increase Increase Decrease No change
D. Increase Increase Decrease Decrease Decrease
E. Increase Decrease Decrease Decrease Decrease
F. Decrease Decrease Decrease Decrease No change

Explanation:
The renin-angiotensin-aldosterone system (RAAS) is one of the most
important neurohormonal systems that regulate arterial blood pressure
and sodium and fluid content in the body. In response to decreased
sodium, fluid volume, or arterial blood pressure, renin is released
from the kidneys converting angiotensinogen to angiotensin I.
Subsequently, angiotensin converting enzyme (ACE) converts angiotensin
Ito angiotensin II. Angiotensin II then increases aldosterone secretion
from the adrenal cortex leading to increased sodium and fluid retention
in the distal tubules of the kidneys. Angiotensin II is also a potent
vaso constrictor that ultimately increases systemic vascular resistance
and arterial pressure.
Angiotensin II itself is involved in two negative feed back mechanisms
that help regulate the RAAS. In short loop negative feedback, elevated
angiotensin II stimulates the angiotensin receptors on juxtaglomerular
cells to inhibit renin release. In long loop negative feedback elevated
blood pressure and sodium levels secondary to angiotensin II eventually
decrease renin release via intrarenal baroreceptor and macula densa
pathways, respectively.
ACE inhibitors, like ramipril, block ACE and decrease the conversion of
angiotensin I to angiotensin II, effectively blocking arteriolar
vasoconstriction and aldosterone secretion. In addition to decreased
blood pressure and sodium levels, decreased angiotensin II levels will
also interfere with negative feedback mechanisms ultimately activating
the RAAS to promote renin release. Since ACE is also responsible for
the breakdown of bradykinin, ACE inhibitors can cause an increase
bradykinin levels too.

Educational Objective:
ACE inhibitors block the effect of ACE, decreasing angiotensin II and
aldosterone levels. By decreasing angiotensin II levels, ACE inhibitors
directly interrupt negative feedback loops, thus increasing renin and
angiotensin I levels. Without effective ACE, bradykinin metabolism
decreases and bradykinin levels will increase.

124
USMLE WORLD STEP 1 PHARMACOLOGY

Q NO 284: A 21-year-old female is taking medication for a recently diagnosed


medical problem. While at a college party, she develops facial flushing,
headache, nausea, vomiting, and abdominal cramps immediately after having
an alcoholic drink. This patient is most likely being treated for which of the
following conditions?
A. Chlamydial urethritis
B. Candida vaginitis
C. Hypothyroidism
D. Pharyngitis
E. Menstrual cramps
F. Trichomonas vaginitis

Explanation:
Patients receiving metronidazole treatment commonly develop symptoms
like those described above shortly after ethanol consumption.
Metronidazole is commonly used to treat trichomonas vaginitis and
bacterial vaginosis. Metronidazole’s interaction with alcohol is
thought to result from its inhibition of alcohol oxidizing enzymes,
which causes acetaldehyde to accumulate and thus the unpleasant
effects. Disulfiram is an aldehyde dehydrogenase inhibitor that also
causes acetaldehyde accumulation and a similar adverse reaction with
ethanol consumption. Disulfiram is used in recovering alcoholics to
prevent them from relapsing to alcohol use.
(Choices A - E) Chlamydial urethritis is treated with doxycycline or
azithromycin. Candida vaginitis is treated with fluconazole.
Hypothyroidism is treated with thyroid hormone replacement. Bacterial
pharyngitis is commonly treated with penicillin or amoxicillin.
Menstrual cramps are treated with nonsteroidal anti inflammatory drugs.
None of these drugs is known to cause a disulfiram-like reaction when
mixed with ethanol.

Educational Objective:
Oral metronidazole can cause disulfiram-like effects when combined with
alcohol. It is used to treat giardiasis, trichomonas vaginitis and
bacterial vaginosis.

125
USMLE WORLD STEP 1 PHARMACOLOGY

Q NO 285: A 62-year-old male is treated for congestive heart failure. He


experienced acute myocardial infarction two years ago and has a long
history of hypertension. After physical examination and laboratory
testing you adjust his medications but his serum calcium level increases
alter this adjustment. The diuretic used in this patient predominantly
acts on which of the following nephron segments?

A. A
B. B
C. C
D. D
E. E
F. F

Explanation:
Thiazide diuretics work by blocking Na-Cl symporters in distal
convoluted tubules. By blocking Na-Cl symporters, thiazides cause
enhanced Na, Cl and water excretion. Since only a small amount of
filtered Na reaches distal tubules, thiazides are not as efficacious
diuretics as loop diuretics. Thiazides also cause hypokalemia. Unlike
loop diuretics thiazides cause hypercalcemia. The exact mechanism of
hypercalcemia is unknown, but most likely involves increased
reabsorption in proximal and distal tubules.
There are a number of thiazide and thiazide-like diuretics available.
Examples include hydrochlorothiazide, chlorothiazide, indapamide, and
metolazone. They all differ in potency bioavailability, and half-life.
Thiazide diuretics are used in treating edema secondary to heart
failure, renal disease, and liver disease. Thiazides are also very
efficacious and commonly used agents in treating hypertension. More
common side effects include hypokalemia, hyponatremia and
hypomagnesemia. Less common side effects include hypotension, volume
depletion, and hypercalcemia.
(Choice A) Carbonic anhydrase inhibitors block the reabsorption of
NaHCO3 and work in the proximal tubule.
(Choice B) This area is the straight part of the proximal tubule and
also contains an excess of the carbonic anhydrase enzyme for carbonic
anhydrase inhibitors to effectively work.
(Choice C) No diuretics are known to work in the descending limb of the
loop of Henle.
(Choice D) Loop diuretics work in the ascending limb and are the most
potent diuretics.
(Choice F) The collecting duct system includes the connecting tubules
and ducts. Here, aldosterone and ADH make final adjustments to

126
USMLE WORLD STEP 1 PHARMACOLOGY
electrolytes and water content. Potassium sparing diuretics and
aldosterone antagonists also work in the collecting duct.

Educational Objective:
Thiazide diuretics work by blocking Na-Cl symporters in distal
convoluted tubules resulting in enhanced Na, Cl, and water excretion.
Since only a small amount of filtered Na reaches distal tubules
thiazides are not as efficacious diuretics as loop diuretics. Unlike
loop diuretics thiazides cause hypercalcemia.

127
USMLE WORLD STEP 1 PHARMACOLOGY

Q NO 286: A 28-year-old female presents to the ER complaining of eye irritation


and double vision. She also complains of recent weight loss mood swings, and
heart palpitations. Her blood pressure is 140/70 mmHg, and heart rate is
110/mm. Physical examination reveals bilateral eye redness and severe
proptosis. She is prescribed some medication and sent home. The patient
returns two weeks later with resolved eye symptoms; her proptosis has
decreased significantly, and no redness is noted. The drug prescribed most likely
affected which of the following?

A. Iodine uptake by the thyroid


B. T3/T4 release by the thyroid
C. Iodine organification
D. Inflammatory infiltration
E. Sympathetic hyperactivity

Explanation:
The patient described has hyperthyroidism due to Graves’ disease and
was most likely given high-dose glucocorticoids. Patients with Graves’
disease, in addition to hyperthyroidism, can have extrathyroidal
manifestations, such as infiltrative ophthalmopathy and pretibial
myxedema. Infiltrative ophthalmopathy is characterized by edema and
infiltration of lymphocytes and macrophages into the extraocular
muscles and connective tissue—clinically, the patient’s eyeballs look
like they are “popping out” and the entire globes will be tender and
red. Pathophysiologically, retro orbital fibroblasts are stimulated to
produce excessive glycosaminoglycans. The volume of retro orbital
tissue is increased which pushes from behind the eyeballs. Generally,
the eye disease worsens for 12-18 months and then stabilizes.
Eventually, the inflammation becomes fibrosis, which restricts
extraocular movements, causing diplopia.
Severe ophthalmopathies characterized by worsening diplopia,
extraocular muscle involvement, and exposure keratitis. High-dose
glucocorticoids are helpful in decreasing the severity of inflammation
and decreasing extraocular volume. Glucocorticoids are generally used
for the treatment of severe infiltrative ophthalmopathy and to prevent
worsening of ophthalmopathy induced by radioactive iodine treatment
glucocorticoids also decrease peripheral conversion of T4 to T3.
Antithyroid drugs do not have a direct effect on ophthalmopathy.
Radioactive iodine in some cases (particularly in smokers) leads to
worsening of ophthalmopathy.
(Choices A and B) Iodine uptake by the thyroid is an energy-dependent
process, which utilizes the sodium iodine symporter. This uptake of
iodine occurs against the concentration gradient. Glucocorticoids do
not have any effect on iodine uptake by the thyroid gland, nor do they
affect release of thyroid hormones into the circulation.
(Choice C) Iodine organification is the process in which dietary,
inorganic iodine is oxidized by the enzyme thyroid peroxidase to
organic iodine. Organic iodine is then carried to the thyroglobulin
where it combines with tyrosine residues to form mono- and
diiodotyrosine, triiodothyronine, and thyroxin. Glucocorticoids have no
effect on the organification process or on the formation of thyroid
hormones.
(Choice E) Sympathetic activation with hyperthyroidism is responsible
for a number of symptoms, including tachycardia and tremors. Beta-
blockers are useful agents for providing symptomatic relief in these

128
USMLE WORLD STEP 1 PHARMACOLOGY
cases. Glucocorticoids do not have a significant anti-sympathetic
effect.

Educational Objective: High-dose glucocorticoids, like prednisone, are


used to control severe Graves ophthalmopathy. They are helpful in
decreasing the severity of inflammation and decreasing extraocular
volume. Remember that conventional antithyroid drugs do not improve
ophthalmopathy.

129
USMLE WORLD STEP 1 PHARMACOLOGY

Q NO 287: A new inhaled anesthetic is found to have an acceptable safety profile


when compared to fluorinated anesthetics. It also demonstrates relatively high
potency. The value of which of the following best reflects this last statement
about the drug?

A. Blood gas partition coefficient


B. Blood lipid partition coefficient
C. Steepness of arterial tension curve
D. Minimal alveolar concentration
E. Arteriovenous concentration gradient

Explanation:
A state of general anesthesia is achieved when a certain threshold
amount of anesthetic is accumulated in the brain. The potency of an
anesthetic is determined by the minimal dose needed to depress the CNS.
When the brain tissue is saturated with inhalation anesthetic the
partial pressure of the gas in the brain equals its partial pressure in
the other compartments (inhaled air, lungs, and arterial blood). Due to
this equilibration of pressures throughout all compartments, it is
possible to use alveolar concentration of the inhalation agent as a
measure of its potency.
Minimal alveolar concentration (MAC) refers to the concentration of
inhalation anesthetic in the gas mixture that renders 50% of the
patients unresponsive to painful stimuli. On the dose-effect curve MAC
corresponds to ED5O (minimal dose that results in standardized effect
in 50% of the patients). Potency is inversely proportional to the value
of MAC (the lower the MAC the more potent the anesthetic).
MAC is an intrinsic property of the anesthetic. It does not depend on
the type of surgery, duration of anesthesia sex height and weight of
the patient. It does however depend on the ambient temperature and the
age of the patient. Usually MAC is given for4O year old patients, and
it decreases with increasing age.
(Choice A) Blood/gas partition coefficient corresponds to the
solubility of anesthetic in the blood. Highly soluble gases have a high
blood/gas partition coefficient and slow onset of action.
(Choices B and E) Blood/lipid partition coefficient is a measure of the
solubility of anesthetic in tissues. It is directly proportional to the
arteriovenous concentration gradient. Highly lipid-soluble anesthetics
have high blood/lipid partition coefficient, high arteriovenous
concentration gradient and slow onset of action.
(Choice C) Steepness of the arterial tension curve depends on
solubility of anesthetic in the blood. In less-soluble gases the
partial pressure in the blood rises rapidly and the curve is steep.

Educational Objective:
Minimal alveolar concentration is the best measure of potency of an
inhaled anesthetic (actually ED50). The less MAC is required for
anesthesia. the more potent the inhalation anesthetic is.

130
USMLE WORLD STEP 1 PHARMACOLOGY

Q NO 288: A 69-year-old Caucasian male with Parkinsonism and a gait disturbance


presents to your office complaining of insomnia. He started having trouble sleeping
two months ago, and the lack of sleep seems to affect his quality of life
significantly. You consider short-term pharmacologic therapy along with non-
pharmacologic measures in this patient. Which of the following medications would
present the greatest fall risk for this patient?

A. Alprazolam
B. Zolpidem
C. Oxazepam
D. Triazolam
E. Flurazepam

Explanation:
Benzodiazepines bind to GABAA receptors in the CNS and enhance the
inhibitory effect of the neurotransmitter GABA. Benzodiazepines are
indicated as hypnotics (to treat insomnia), anxiolytics (for panic
attacks and anxiety disorders) and anticonvulsants (especially for
seizures associated with alcohol withdrawal). Benzodiazepines are also
used as general anesthetics because they cause amnesia and muscle
relaxation.
The main side effect of benzodiazepines is daytime drowsiness. This
“hangover” effect is more than just inconvenient. as it can be
associated with impaired judgment and concentration—it is also
associated with ataxia, which obviously increases the risk of falling.
Certain classes of benzodiazepines have a shorter duration of action
which decreases the risk of daytime drowsiness: however, these shorter-
acting medications are also more addictive (see below).
Tolerance readily develops with the long-term use of benzodiazepines.
Physical dependence may also occur and manifests with withdrawal
symptoms such as anxiety and insomnia upon discontinuation of the drug.
Generally the risk of dependence is directly proportional to the rate
of clearance of a particular drug. To phrase it another way the shorter
the duration of action of a drug the more likely withdrawal symptoms
are to occur. According to their pharmacokinetic properties,
benzodiazepines are divided into the following groups:

131
USMLE WORLD STEP 1 PHARMACOLOGY
(Choices A, C and D) Alprazolam, triazolam, and oxazepam are short-
acting benzodiazepines. They are used to treat insomnia, and they
produce less daytime drowsiness than long-acting benzodiazepines.
(Choice B) Zolpidem is a non-benzodiazepine hypnotic. It has a rapid
onset of action and a short half-life. Zolpidem is distinguished by its
low incidence of side effects and low potential for tolerance and
dependence.

Educational Objective:
Benzodiazepines can cause daytime drowsiness (a “hangover’) and can
increase the risk of falls in elderly. The severity of this side effect
depends on the half-life of the drug. Long-acting benzodiazepines cause
more severe drowsiness than short-acting ones. On the other hand long-
acting benzodiazepines are far less likely to cause dependence (i.e.
less addicting).

132
USMLE WORLD STEP 1 PHARMACOLOGY

Q NO 289: A 46-year-old female is treated with glyburide for type 2 diabetes. After
adding acarbose, the patient’s glycemic control improves significantly, as reflected
by a decreased HbA1C. Which of the following is the most likely cellular target of
acarbose?

A. Membrane ion channels


B. Surface tyrosine kinase-coupled receptors
C. Surface adenylate cyclase-coupled receptors
D. Intracellular nuclear receptors
E. Surface membrane-bound enzymes
F. Intracellular microsomal enzymes

Explanation:
Alpha-glucosidase inhibitors decrease the activity of disaccharidases
on the intestinal brush border (remember, disaccharidases are membrane-
bound enzymes). Since carbohydrates are absorbed as monosaccharides,
the inhibition of disaccharide breakdown by alpha-glucosidase
inhibitors delays carbohydrate absorption. In the United States,
acarbose and miglitol are the two alpha glucosidase inhibitors
available for clinical use. Both acarbose and miglitol are taken with
meals for maximal effect. The major side effects of alpha-glucosidase
inhibitors are flatulence, gastrointestinal bloating, abdominal pain,
and rash. These drugs should therefore not be used in patients with
inflammatory bowel disease malabsorption, intestinal obstruction, or
colonic ulceration.
(Choice A) This is the mechanism of action of sulfonylureas.
(Choice B) This is the mechanism of action of insulin.
(Choice C) Glucagon like polypeptide-1 (GLP-1) is an incretin hormone
secreted by intestinal L cells in response to food intake. GLP-1
decreases glucose by inducing satiety, decreasing gastric emptying, and
increasing insulin release. GLP-1 acts through cell surface receptors
that are coupled with a G protein-adenyl cyclase system. Exenatide, a
long-acting GLP-1 analogue is approved for the treatment of type 2
diabetes mellitus in patients with suboptimal glucose control, despite
adequate doses of metformin and sulfonylureas.
(Choice D) TZDs decrease insulin resistance by binding to peroxisome
proliferator activated receptor gamma (PPAR-gamma), which is a
transcriptional regulator of the genes involved in glucose and lipid
metabolism. PPAR gamma belongs to the nuclear receptor super family
that regulates gene expression after ligand binding.
(Choice F) Alteration of intracellular enzymes is the primary mechanism
of action of metformin (biguanide). Metformin reduces glucose levels by
enhancing glycolysis and inhibiting gluconeogenesis.

Educational Objective:
Alpha-glucosidase inhibitors decrease the activity of the membrane-
bound disaccharidases on the intestinal brush border. Carbohydrates are
absorbed as monosaccharides; therefore, the action of alpha-glucosidase
inhibitors in preventing disaccharide breakdown allows the delay in
carbohydrate absorption. In the United States, acarbose and miglitol
are the two alpha glucosidase inhibitors available for clinical use.

133
USMLE WORLD STEP 1 PHARMACOLOGY

Q NO 290: A 70-year-old Caucasian male is brought to emergency department after


a syncopal episode. He also complains of recent onset of severe constipation. He
was recently admitted to the hospital with atrial fibrillation and rapid ventricular
response. His past medical history is also significant for severe chronic obstructive
pulmonary disease. An ERG done in the emergency department demonstrates
second degree AV block. Which of the following drugs was most likely used to treat
this patient’s atrial fibrillation?

A. Propranolol
B. Verapamil
C. Amlodipine
D. Digoxin
E. Lidocaine

Explanation:
Verapamil is a non-dihydropyridine calcium-channel blocker and is often
used as a therapy for rate control in atrial fibrillation with rapid
ventricular response due to its ability to slow conduction through the
atrioventricular (AV) node. It is a potent negative inotrope and is
also approved for use as an anti-anginal medication and an
antihypertensive.
AV nodal conduction abnormalities are a characteristic side effect of
verapamil which slows conduction through the AV node by its action on
open, depolarized calcium channels on cardiac pacemaker cells. This can
lead to bradycardia as well as first, second and third degree AV
blocks, though these side effects are only seen in approximately 1-2%
of patients. Constipation is seen in over 10% of patients treated. A
contraindication to the use of verapamil would be in the setting of CHE
due to verapamil potent negative inotropic effect.
(Choice A) Severe CQPD and asthma are contraindications to the use of
non-selective beta-blockers, so propranolol is a less likely cause of
this patient’s complaints. Cardioselective beta-blockers such as
metoprolol and atenolol can be used in patients with mild to moderate
asthma.
(Choice C) Amlodipine is a dihydropyridine calcium channel blocker and
differs from verapamil in that it is selective for the vascular smooth
muscle and does not affect the heart as verapamil does. The most
commonly noted side effect of amlodipine is peripheral edema. These
drugs can cause reflex tachycardia and should not be used in patients
with acute coronary syndrome, as the oxygen consumption of the
myocardium is increased in states of reflex tachycardia.
(Choice D) Digoxin is a very commonly used treatment for impaired left
ventricular function. Digoxin is also a second- line treatment for
atrial fibrillation with rapid ventricular response through its ability
to slow conduction through the AV node. Digoxin usually causes nausea,
vomiting and diarrhea and does not cause constipation.
(Choice E) Lidocaine is not used in atrial arrhythmias. It is mainly
used for ventricular arrhythmias, especially those arising after an
acute myocardial infarction.
Educational Objective:
It is important to know the difference in the cardio selectivity and
vascular selectivity of the 3 classes of calcium channel blockers.
Verapimil has the most effect on the heart while nifedipine is most
selective for the peripheral vasculature with diltiazem having an
effect somewhere in between. The most frequent adverse reactions noted

134
USMLE WORLD STEP 1 PHARMACOLOGY
with verapamil are constipation and gingival hyperplasia, though they
also are known to cause bradycardia as well as first, second or third
degree AV nodal block in 1-2% of patients treated.

135
USMLE WORLD STEP 1 PHARMACOLOGY

Q NO 291: A 59-year-old man being treated for hypercholesterolemia, diabetes and


hypertension presents to his primary care provider three weeks after starting a new
medication. Follow-up laboratories show an interim increase in potassium from 4.8
to 5.2 and a creatinine elevation from 1.1 to 1.7 Which of the following drugs is
most likely responsible for this clinical picture?

A. Lisinopril
B. Metoprolol
C. Atorvastatin
D. Hydrochlorothiazide
E. Furosemide
F. Metformin
G. Prazosin

Explanation:
Angiotensin-converting enzyme (ACE) inhibitors (typically named
“April”) are one of the most important agents in treating hypertension
heart failure, and renal failure with or without, proteinuria. They
work by preventing the conversion of angiotensin I to angiotensin II.
This prevents constriction of the efferent arteriole greater than the
afferent arteriole thus decreasing the glomerular pressure and GER.
Itis expected for the GER to decrease in all patients initially. Most
clinicians do not worry unless the creatinine increases by greater than
3O% because the long-term benefits of ACE inhibitors is well studied.
Other common side-effects of ACE inhibitors are hyperkalemia and cough.
(Choice B) Metoprolol is a beta-blocker; typical side-effects include
bradycardia and erectile dysfunction. It does not decrease GER.
(Choice C) Atorvastatin is an HMG-CoA reductase inhibitor; worrisome
side-effects include muscle pain and hepatic dysfunction. It does not
decrease GER.
(Choices D and E) Hydrochlorothiazide is a potassium-wasting thiazide
diuretic that may decrease GER if it results in volume depletion and
pre-renal azotemia; however it is a potassium-wasting diuretic.
Furosemide is also potassium wasting loop diuretic.

Educational Objective:
Common side-effects of ACE-inhibitors are decreased GFRI hyperkalemia,
and cough.

136
USMLE WORLD STEP 1 PHARMACOLOGY

Q NO 292: A 62-year-old male is brought to ER unresponsive after sustaining


multiple injuries in a major motor vehicle accident. After initial treatment
severe tachypnea and decreased oxygenation are observed and the patient
dies of pulmonary edema. Which of the following drugs could contribute to
this patient’s condition?
A. Chlorthalidone
B. Bumetanide
C. Spironolactone
D. Triamterene
E. Mannitol

Explanation:
Mannitol is an osmotic diuretic that works by increasing plasma or
tubular fluid osmolality. Increased plasma and fluid osmolality causes
extraction of water from the interstitial space into the vascular space
or tubular lumen. In the kidneys, osmotic diuretics primarily work in
the proximal tubule and the loop of Henle to produce diuresis. In the
brain water extraction from tissues into the plasma helps to reduce
edema and intracranial pressures. Thus osmotic diuretics are used
therapeutically to treat cerebral edema. Common side effects of osmotic
diuretics include headache, nausea, and vomiting. One of the more
severe toxicity of osmotic diuretics involves pulmonary edema. Osmotic
diuretics increase extracellular osmolality causing increase water
extraction into the extracellular space. In certain patients, increased
volume expansion can lead to pulmonary edema and thus osmotic diuretics
should be used cautiously in high risk patients (patients with heart
failure or preexisting pulmonary congestion). Overaggressive treatment
with osmotic diuretics can also lead to excessive volume depletion and
hypervolemia.
(Choice A) Thiazide diuretics are used in treating edema secondary to
heart failure, renal disease, and liver disease. More common side
effects include hypokalemia, and hypomagnesemia. Less common side
effects include hypotension, volume depletion, hypercalcemia, and
hyponatremia. (Choice B) Bumetanide is a loop diuretic that works by
inhibiting Na-K-2Cl symporters in the ascending limb of the loop of
Henle, effectively blocking Na and Cl transport resulting in increased
Na, Cl, and fluid excretion. They are commonly used in treating
pulmonary edema venous and pulmonary congestion secondary to congestive
heart failure, and peripheral edema. Common side effects include
hypokalemia, hypomagnesemia, and hypocalcemia.
(Choice C) Spironolactone is an aldosterone antagonist with mild
diuretic effects. Spironolactone has a potassium sparing effect and can
cause significant hyperkalemia. Since itis structurally similar to
steroids, spironolactone can cause endocrine effects including
gynecomastia, decreased libido, hirsutism, and impotence. (Choice D)
Triamterene is a potassium sparing diuretic that works by blocking Na
channels in the distal tubule and collecting duct. This results in
increased Na and fluid excretion.

137
USMLE WORLD STEP 1 PHARMACOLOGY
Educational Objective:
Mannitol is an osmotic diuretic that works by increasing plasma or
tubular fluid osmolality. Increased plasma and fluid osmolality causes
extraction of water from the interstitial space into the vascular space
or tubular lumen with subsequent diuresis. In the brain, water
extraction from tissues into the plasma helps to reduce edema and
intracranial pressures in the setting of cerebral edema. One of the
more severe toxicities of osmotic diuretics involves pulmonary edema.

138
USMLE WORLD STEP 1 PHARMACOLOGY

Q NO 293: An experimental medication produces the hemodynamic response curve


shown below. This new drug is most similar to which of the following substances?

A. Norepinephrine
B. Isoproterenol
C. Phentolamine
D. Labetalol
E. Phenylephrine

Explanation:
This experimental drug causes a dose-dependent increase in cardiac
contractility and a dose-dependent decrease in systemic vascular
resistance. These effects are most similar to those elicited by
isoproterenol. Isoproterenol increases cardiac contractility by acting
on myocardial β-adrenergic receptors. In lower doses it selectively
binds to β2-receptors, causing relaxation of vascular smooth muscle.
Isoproterenol has negligible effects on a-receptors.
(Choice A) Norepinephrine (NE) acts on al-receptors, causing an
increase in systemic vascular resistance. NE can also act as a weak
agonist at β1-receptors, potentially increasing myocardial
contractility.
(Choice C) Phentolamine predominantly acts as a peripheral a-adrenergic
blocker, causing peripheral vasodilation (mainly arterial).
(Choice D) Labetalol is a non-selective β-blocker that also has al-
receptor blocking effects. Both myocardial contractility and systemic
vascular resistance would decrease with this medication.
(Choice E) Phenylephrine is a sympathomimetic drug with α1 > α2 agonist
effects. Infusion of this drug would increase systemic vascular
resistance.

Educational Objective:
Isoproterenol is an agonist at both β1 and β2-adrenergic receptors and
has little or no a-adrenergic agonist effects. It causes a dose-
dependent increase in myocardial contractility and a dose-dependent
decrease in systemic vascular resistance.

139
USMLE WORLD STEP 1 PHARMACOLOGY

Q NO 294: A 26-year-old male with a history of childhood dermatitis, asthma, and


hay fever has routinely been applying a corticosteroid cream to the flexor areas of
his skin for many years. The treated areas have remained free of erythema,
papules vesicles, crusts, or scales. A punch biopsy from one of the areas subjected
to this topical therapy would most likely show which of the following?

A. intraepidermal vesicles
B. Epidermal hyperkeratosis
C. Epidermal hyperplasia
D. Dermal atrophy
E. Dermal perivascular lymphocytosis
F. Adipose tissue hyperplasia

Explanation:
This patient has been successfully treated for atopic eczematous
dermatitis. Hence, marked histological manifestations of active eczema
such as intraepidermal vesicles, superficial epidermal hyperkeratosis
producing scales epidermal hyperplasia (acanthosis), and/or a chronic
inflammatory infiltrate within the dermis would not be expected. This
dermatitis cure comes with a price, however; along with anti-
inflammatory actions, corticosteroids decrease the production of
extracellular matrix collagen and glycosaminoglycans. Consequences
include atrophy of the dermis with loss of dermal collagen, drying,
cracking and/or tightening of the skin. Telangiectasias, ecchymoses
from mild trauma, and atrophic striae may also be found.
(Choice F) Via an uncertain mechanism, chronic application of topical
corticosteroids may also cause atrophy, not hyperplasia, of
subcutaneous tissues, including adipocytes.

Educational Objective:
Local cutaneous adverse effects of chronic topical corticosteroid
administration include atrophy/thinning of the dermis that is
associated with loss of dermal collagen, drying, cracking, and/or
tightening of the skin telangiectasias, and ecchymoses.

140
USMLE WORLD STEP 1 PHARMACOLOGY

Q NO 295: Three alpha-agonist drugs are tested as potential vasoconstrictors. The


degree of vasoconstriction is determined by measuring the cross-sectional area of
an isolated vessel after application of the drug. The following curves are obtained:
Which of the following is the best statement concerning the effects of these drugs?

A. Drug 1 has lower potency than Drug 2


B. Drug 2 has higher affinity for alpha-receptors than Drug 3
C. Drug 1 demonstrates the highest efficacy
D. Drug 2 and Drug 3 bind to different loci of alpha-receptors
E. Drug 2 and Drug 3 demonstrate a similar potency

Explanation:
Efficacy and potency are terms that are commonly used and often
confused. In pharmacology, efficacy refers to the intrinsic ability of
a drug to elicit an effect, such as receptor activation or dilation of
a vessel as described in the question stem. Itis a measure of the
maximum ceiling of activity [Emax] of a drug with respect to a
particular pharmacodynamic end point. For example, loop diuretics such
as furosemide or bumetanide will cause greater diuresis or natriuresis
than any thiazide diuretic, irrespective of dose.
Potency, on the other hand, refers to the dose of drug that is required
to produce a given effect. The potency of a drug is primarily affected
by the affinity of the drug for its receptor and the amount of drug
that is able to reach the target tissues Therefore, drugs that bind
their receptors with a higher affinity or are better able to gain
access to their target tissues will have greater potency. The potency
of different agents can be compared by determining the dose of drug
that is required to produce one-half (500k) of the maximum biological
response. This dose is known as the ED50. The lower the ED50 of a drug,
the more potent it is. For example, among the thiazide diuretics, 25mg
of hydrochlorothiazide and 5mg chlorthalidone produce similar drops in
blood pressure (similar efficacy), but chlorthalidone requires a lower
dose, or lower ED50, than HCTZ to cause a similar effect (higher
potency).
With respect to the question, drug 2 and drug 3 have similar maximum
effects (efficacy). A much lower dose of drug 2 is required to produce
the same effect as that seen with drug 3; therefore drug 2 is more

141
USMLE WORLD STEP 1 PHARMACOLOGY
potent than drug 3 and has a lower ED50. To achieve this higher potency
the binding of drug 2 to its receptors (affinity) must be higher than
that of drug 3 (Choice B) or drug 2 must be better able to reach its
target (penetration) than drug 3.
(Choice A) Drug 1 has higher potency than Drug 2 because a flow
biological activity it can cause the same effect with much lower
concentrations of drug. However it does not reach the same maximum
effect as Drug 2.
(Choice C) Drug 1 demonstrates the lowest efficacy (ceiling of
biological effect) of the three drugs graphed. Lower efficacy may be a
result of partial agonism and other factors.
(Choice D) Drugs 2 and 3 exhibit a parallel shift in their relative
biological effects exhibiting similar efficacy but different potency
(The shapes of their graphs are identical but at different dose
ranges). Drug 3 may contain the same basic pharmacologic agent as Drug
2 but with a competitive antagonist mixed in; thus the Emax is the same
but the ED50 is higher. A competitive antagonist would bind to the same
location as the original substance.
(Choice E) Drug 2 has a lower ED50 (greater potency) than drug 3. Both
have equal efficacy.

Educational Objective:
Efficacy is a measure of the maximum pharmacodynamic effect achievable
with a drug. Potency refers to the dose of drug that is required to
produce a given effect. Drugs that bind their receptors with a higher
affinity or are better able to gain access to their target tissues will
have greater potency (lower ED5Q).

142
USMLE WORLD STEP 1 PHARMACOLOGY

Q NO 296: A 65-year-old healthy Caucasian male presents to your office for a


routine check-up. He was diagnosed with diabetes mellitus seven years ago
and to flows a strict diet to control his blood sugar. His recent HbA1C value
was 7.4% (normal is less than 7.0%). His blood pressure is 139/88 mmHg
and heart rate is 10/mm. Physical examination reveals decreased lower
extremity sensation when examined with a 10g monofilament. Laboratory
studies reveal an increased urinary albumin excretion, but a conventional
urinalysis is within normal limits. Which of the following drugs is most
appropriate for this patient?
A. Prazosin
B. Atenolol
C. Lisinopril
D. Hydrochlorothiazide
E. Spironolactone
F. Nifedipine
G. Isosorbide dinitrate

Explanation:
ACE inhibitors (ACE-I) as well as angiotensin receptor blockers (ARBs)
have been clearly shown to slow the progression of diabetic nephropathy
in Type I and Type II diabetics both prior to the onset of proteinuria
as well as after proteinuria has been documented. Therapy with these
drugs has also been shown to decrease the chances of progressing to
end-stage renal disease and need for dialysis. An additional benefit of
ACE-I or ARB therapy in diabetics is their antihypertensive effect
because the normal blood pressure limits are lower in diabetics. Loss
of albumin in the urine of 20 mg/day or less is considered normal.
Microalbuminuria is defined as urine albumin loss between 30 and 300
mg/day and is indicative of diabetic nephropathy in diabetic patients.
A urinalysis can only detect protein concentrations of greater than
300mg/day. Microalbuminuria can not be detected by a standard dipstick
urinalysis: only albumin excretion greater than 300 mg/day can be
detected by dipstick urinalysis and is labeled microalbuminuria.
(Choice A) Alpha 1-blockers such as Doxazosin, Prazosin and Terazosin
are useful for the treatment of both hypertension and benign prostatic
hyperplasia. While tight blood pressure control is important in
patients with diabetes to reduce cardiac risk, this is not the best
choice for this patient.
(Choice B) Cardioselective beta-blockers such as metoprolol, atenolol
etc. are the drugs of choice in patients with coronary artery disease
and congestive heart failure along with hypertension.
(Choice D) Hydrochlorothiazide is presently the first-line medication
for the treatment of essential hypertension in the general population.
It is also the first line medication for patients with osteoporosis
(since it increases the serum calcium) and isolated systolic
hypertension.
(Choice E) Spironolactone is an aldosterone antagonist and functions as
a potassium-sparing diuretic. It is a weak diuretic and is frequently
used in conjunction with other diuretics for their potassium-sparing
effect or for hyper aldosterone states such as with an aldosterone-
secreting tumor.
(Choice F) Nifedipine is a dihydropteridine calcium channel blocker and
differs from verapamil in that nifedipine is selective for the vascular
smooth muscle and does not affect the heart as verapamil does. While
tight blood pressure control is important in patients with diabetes to
143
USMLE WORLD STEP 1 PHARMACOLOGY
reduce cardiac risk this is not the best choice for this patient with
microalbuminuria.
(Choice G) Isosorbide dinitrate is an intermediate acting nitrate used
in the treatment of stable angina pectoris. It will lead to a decrease
in cardiac output by producing venodilitation causing a decrease in
preload and a decrease in cardiac work. This is why nitrates are
effective as anti-anginal.

Educational Objective:
This is a very common clinical issue given the prevalence of diabetes
in the US population, and the ACE-I class of drugs together with the
ARB class of drugs have been shown to decrease the progression of
diabetic nephropathy.

144
USMLE WORLD STEP 1 PHARMACOLOGY

Q NO 297: A 33-year-old homeless HI V-positive male is hospitalized with recent


weight loss odynophagia and progressive cough. His most recent CD4 count is
45pL. His condition improves on a newly instituted treatment, but four weeks
later at his outpatient followup he is found to have a significant elevation of his
fasting blood glucose level. Which of the following medications is most likely
responsible for his hyperglycemia?

A. TMP-SMX
B. Zidovudine
C. Indinavir
D. Acyclovir
E. Azithromycin
F. Foscarnet

Explanation:
Indinavir is a member of the protease inhibitor class of antiretroviral
drugs. These drugs reversibly inhibit viral protease, an enzyme
responsible for the final step of replication of HIV virus within host
cells. Protease inhibitors are always used as a component of
combination antiretroviral therapy as in HAART (highly active
antiretroviral therapy). They are never used as monotherapy because
viral resistance to the entire class of drugs quickly forms. All
protease inhibitors have the following important adverse effects:
1. Lipodystrophy leads to increased deposition of fat on the back and
abdomen, and decreased adipose tissue on the extremities. This gives
patients a “buffalo hump” appearance with central obesity and
peripheral wasting.
2. Hyperglycemia is a side effect associated with all protease
inhibitors that results from increased insulin resistance and may lead
to frank diabetes.
3. Inhibition of P-450 also occurs with some protease inhibitors and
may cause interactions with other drugs. Rifampin should not be
administered with protease inhibitors because rifampin increases the
activity of P-450 and will therefore decrease the serum levels of
protease inhibitor; rifabutin should be used instead for Mycobacterial
infections in patients on protease inhibitors.
Apart from the side effects mentioned above, each protease inhibitor
has its specific adverse effects. Indinavir can cause nephrotoxicity
and nephrolithiasis.
(Choice A) TMP-SMX (Trimethoprim-Sulfamethoxazole, Bactrim) is used in
HIV patients for prevention and treatment of pneumonia caused by
Pneumocystis jiroveci (formerly P. carinii). The most important side
effects associated with this drug are megaloblastic anemia, Steven-
Johnson syndrome and toxic epidermal necrolysis.
(Choice B) Zidovudine is a nucleoside reverse transcriptase inhibitor
(NFI). Its acts by suppression of DNA synthesis from viral RNA and is
the most widely used anti-HIV drug. Its most common side effect is bone
marrow toxicity resulting in anemia in up to 40% of patients.
(Choice 0) Acyclovir is an antiviral drug active against HSV-1, HSV-2I
and varicella-zoster viruses. Renal toxicity is the most important side
effect of this drug.
(Choice E) Azithromycin is a macrolide antibiotic used in HR patients
for treatment and prophylaxis of Mycobacterium avium intracellulare
(MAC) infections.

145
USMLE WORLD STEP 1 PHARMACOLOGY
(Choice F) Azithromycin is used for treatment of CMV infections in HR
patients. It may cause nephrotoxicity, electrolyte disturbances such as
hypocalcemia, hypomagnesemia, and hypokalemia.

Educational Objective:
Protease inhibitors are anti-HIV medications that inhibit cleavage of
the polypeptide precursor into mature viral proteins. Their side
effects as a class include hyperglycemia, lipodystrophy, and drug-drug
interactions due to inhibition of cytochrome p-450.

146
USMLE WORLD STEP 1 PHARMACOLOGY

Q NO 298: A 25-year-old female presents to your office complaining of a lump on


her neck. She discovered it accidentally while bathing. She denies any weight
loss. Anorexia joint pain chest pain and difficulty breathing but admits feeling hot
from time to time. Her past medical history is significant for epilepsy that is
controlled with medications. Physical examination reveals generalized
lymphadenopathy. If her condition is due to a medication side effect which of the
following is most likely responsible?

A. Carbamazepine
B. Valproic acid
C. Lamotrigine
D. Phenytoin
E. Phenobarbital

Explanation:
Phenytoin can cause generalized lymphadenopathy without a serum
sickness-like syndrome. Phenytoin inhibits neuronal high-frequency
firing by reducing the ability of sodium (Na+ channels to recover from
inactivation, increasing the refractory period. Phenytoin undesirable
cosmetic effects (hirsutism, coarsening of facial features, acneiform
skin rash, and gingival hypertrophy), limit its use. Phenytoin is
useful in treating tonic-clonic and psychomotor seizures as well as for
seizure prophylaxis after head trauma and before neurosurgery.
(Choice A) Carbamazepine is used in the treatment of generalized tonic-
clonic and partial seizures, pain relief in trigeminal neuralgia and
diabetic neuropathy, and bipolar disorders. Agranulocytosis is the most
feared complication.
(Choice B) Valproic acid has been associated with severe
hepatotoxicity. Thus liver function tests should be monitored at the
beginning of medication.
(Choice C) Lamotrigine has been associated with a hypersensitivity skin
rash (Steven Johnson syndrome).
(Choice E) Phenobarbital can precipitate acute intermittent porphyria
and, in toxic doses, can cause sedation or cardiovascular and
respiratory depression.

Educational Objective:
Phenytoin’s undesirable cosmetic effects (hirsutism, coarsening of
facial features, acneiform skin rash, and gingival hypertrophy), limit
its use. It has also been associated with generalized lymphadenopathy
(pseudolymphoma).

147
USMLE WORLD STEP 1 PHARMACOLOGY

Q NO 299: A 57-year-old male presets to ER with severe right-sided eye pain,


decreased vision and nausea. After initial treatment the severity of pain
decreases. He experiences increased diuresis with highly alkaline urine. The
drug used to treat this patient’s condition predominantly acts on which of the
following nephron segments?

A. A
B. B
C. C
D. D
E. E
F. F

Explanation:
Acetazolamide is a diuretic that works by inhibiting the enzyme
carbonic anhydrase. Carbonic anhydrase is found in high concentrations
in the proximal tubule and is responsible for catalyzing reactions
necessariforNaHCO3 reabsorption. By inhibiting carbonic anhydrase,
acetazolamide and other carbonic anhydrase inhibitors effectively block
NaHCO3 and water reabsorption in the proximal tubules. This results in
enhanced HCO3- and water excretion and increased urinary pH and
potential metabolic acidosis.
Carbonic anhydrase is also present in the eyes, pancreas,
gastrointestinal tract CNS, and red blood cells. In eye tissues,
carbonic anhydrase modulates HCO3- formation in the aqueous humor.
Inhibition of carbonic anhydrase will decrease HCO3- and aqueous humor
formation. Thus a number of carbonic anhydrase inhibitors are used to
relieve intraocular pressures in open-angle and secondary glaucoma.
Common side effects of carbonic anhydrase inhibitors include somnolence
paresthesias, and urine alkalinization. More rare side effects include
metabolic acidosis, dehydration hypokalemia, and hyponatremia.
(Choice C)The descending limb of the loop of Henle carries fluids from
the proximal tubule to the ascending limb of the loop of Henle in the
medulla. The descending limb is ver permeable to water, allowing water
to diffuse into interstitial fluids to produce more concentrated
tubular fluid. No diuretics are known to work in the descending limb of
the loop of Henle.
(Choice D) Loop diuretics work in the ascending limb and are the most
potent diuretics.
(Choice E) The distal tubule actively transports Na and Cl and is
impermeable to water. Thiazide diuretics work in the distal tubule.
(Choices B and F) The collecting duct system includes the connecting
tubules and ducts. Here, aldosterone and ADH make final adjustments to

148
USMLE WORLD STEP 1 PHARMACOLOGY
electrolytes and water content. Potassium sparing diuretics and
aldosterone antagonists also work in the collecting duct.

Educational Objective:
Carbonic anhydrase is found in high concentrations in the proximal
tubule and is responsible for catalyzing reactions necessaryforNaHCO3
reabsorption. Acetazolamide is a diuretic that works by inhibiting
carbonic anhydrase, effectively blocking NaHCO3 and water reabsorption
in the proximal tubules. Carbonic anhydrase inhibitors are also used to
relieve intraocular pressure in open-angle and secondary glaucoma.

149
USMLE WORLD STEP 1 PHARMACOLOGY

Q NO 300: A 50-year-old male begins treatment with an antiarrhythmic medication


for recurrent atrial fibrillation. The arrhythmia resolves, and there is now mild
bradycardia and significant QT interval prolongation on his ERG recording. Which of
the following drugs was most likely used in this patient?

A. Metoprolol
B. Propranolol
C. Sotalol
D. Carvedilol
E. Verapamil
F. Diltiazem

Explanation:
The Gil-interval represents the period of ventricular repolarization as
the ventricles prepare for the next beat. Repolarization is mediated by
(efflux from the cardiac myocyte. Sotalol is a beta-blocker with class
3 (K channel blocking) antiarrhythmic properties. Other class 3
antiarrhythmics include amiodarone, ibutilide, and dofetilide. Sotalol
is the only class 3 antiarrhythmic with beta-adrenergic blocking
abilities (causing the patient’s mild bradycardia) as well as class 3
effects (causing the OT interval prolongation).
(Choices A, B, and D) Metoprolol, propranolol and carvedilol are beta-
blockers like sotalol, but these medications lack sotalol’s class 3 K+
blocking effects. Beta-blockers decrease heart rate and cardiac
contractility by slowing AV nodal conduction and the phase 4
depolarization of cardiac pacemaker cells. They are useful as rate
controlling agents, as myocardium protective agents following
myocardial infarction or in CHF resulting from systolic dysfunction,
and as antihypertensives. They do not prolong the OT interval.
(Choices E and F) Verapamil and diltiazem are calcium channel blockers
that affect cardiac tissue and are therefore class 4 antiarrhythmic
agents. Verapamil is the most cardioselective, but both will act on the
peripheral vasculature as well, causing a decrease in blood pressure.
As calcium channel blockers these medications slow the phase 0
depolarization of cardiac pacemaker cells and phase 2 of the myocyte
action potential, neither of which will change the QT interval. Their
adverse effects include negative inotropy and the potential for AV
block. Verapamil is additionally associated with gingival hyperplasia
and constipation.

Educational Objective:
Sotalol has both beta-adrenergic blocking properties and class 3
antiarrhythmic (R channel blocking) properties. It prolongs both the PR
interval and the OT interval.

150
USMLE WORLD STEP 1 PHARMACOLOGY

Q NO 301: A 35-year-old Caucasian male develops a deep vein thrombosis of his


right lower extremity, and you decide to treat him with warfarin. His past medical
history is significant for epilepsy. After initiation of therapy you discover that the
patient requires high doses of warfarin to achieve his target INR values. Which of
the following is the most likely explanation for your discovery?

A. Low intestinal absorption


B. High gastric degradation
C. Increased liver metabolism
D. Increased renal excretion
E. Inappropriate drug formulation

Warfarin is an oral anticoagulant drug that is tightly bound to plasma


proteins after absorption by the gut. Warfarin is metabolized in the
liver by multiple hepatic microsomal mixed function oxidases,
especially cytochrome P450 2C9. Because warfarin is metabolized by this
enzyme system, other drugs or foods that alter the activity of the
hepatic microsomal enzyme will also alter the plasma concentration of
warfarin.
Many drugs can induce the hepatic CYP45O system and thereby accelerate
the metabolism of warfarin leading to reduced anticoagulation and
enhanced thrombotic risk. These inducers of the CYP45O system include
antibiotics such as rifampin and griseofulvin as well as
anticonvulsants such as barbiturates carbamazepine, and phenytoin. It
is likely in this case that the patient is taking anti-epileptics or
possibly other herbs, foods, or medications that may be elicited with a
careful history.
Warfarin metabolism may also be inhibited by many drugs, leading to a
high serum level and an increased bleeding risk. These drugs include
antibiotics such as trimethoprim, isoniazid, and azole antifungals, as
well as other well-known drugs such as cimetidine, SSIRIs, and
amiodarone.
(Choice A) Typically, warfarin administered orally has nearly 100%
bioavailability.
(Choice B) Peptic ulcer disease and increased gastric acidity are not
associated with increased degradation of oral warfarin. If used to
treat peptic ulcer disease or heartburn cimetidine would serve to
inhibit CYP45O thereby increasing the effect of warfarin.
(Choice D) Warfarin is metabolized mostly by the liver: thus, the drug
should not be broken down excessively in the event of increased renal
activity, and similarly, dose reduction is not required in renal
failure. (Choice E) The formulation of warfarin is a standardized
pharmaceutical procedure, with high quality control. Although drugs may
be substandard when counterfeited, this is highly unlikely in a well
developed clinical setting.
Educational Objective:
1. Many drugs can accelerate (induce) the hepatic metabolism of
warfarin leading to reduced anticoagulation and enhanced thrombotic
risk. These include antibiotics such as rifampin and griseofulvin as
well as antiepileptic drugs such as barbiturates, carbamazepine, and
phenytoin.
2. Warfarin metabolism may be inhibited by antibiotics such as
trimethoprim, isoniazid, and fluconazole, as well as other drugs such
as cimetidine. Inhibition of warfarin metabolism can increase the risk
of bleeding.

151
USMLE WORLD STEP 1 PHARMACOLOGY

Q NO 302: A 57-year-old Caucasian male is being evaluated for hyperlipidemia. He


has a history of unstable angina and long-standing hypertension. As you prescribe
the appropriate medications to this patient, you explain that he will likely
experience skin flushing and warmth after taking his pills. Which of the following
mediates the side effect you describe?

A. Histamine
B. Serotonin
C. Prostaglandin
D. Substance P
E. Platelet-activating factor (PAF)

Nicotinic acid, or niacin, has been used in the treatment of


hyperlipidemia for almost four decades. Although it is yen effective,
side effects are common. Niacin’s main side effects are cutaneous
(flushing, warmth, itching). These symptoms typically occur when
treatment is initiated. Niacin is better-tolerated in slow-release
preparations.
This side effect may be mediated by prostaglandins, as evidenced by the
fact that 325mg of aspirin taken 30-60 minutes before nicotinic acid
administration significantly reduces these cutaneous symptoms.
Administration of nicotinic acid with meals also improves flushing. Hot
liquids exacerbate this symptom. Over time many patients develop a
tachyphylactic response, and pre-treatment with aspirin becomes
unnecessary.
The molecules listed in the other answer choices do not play a role in
niacin-induced flushing. The physiology of these substances is briefly
described below.
(Choice A) The body synthesizes histamine from the amino acid
histidine. Histamine is released from mast cells, and is responsible
for allergic responses via its interaction with the Hi receptor.
Histamine is also released from the enterochromaffin cells of the
stomach, and stimulates parietal cell acid production via its
interaction with H2 receptors.
(Choice B) The body synthesizes serotonin from the amino acid
tryptophan. In humans, 90% of serotonin resides in the gastrointestinal
tract; the remainder is in the brain or associated with platelets.
Serotonin is responsible for mood-regulation. Selective serotonin
reuptake inhibitors, or SSRls, are powerful antidepressant medications.
(Choice D) Substance P is an li-amino acid polypeptide, which acts as a
pain neurotransmitter in both the peripheral and central nervous
systems. In the CNSI substance P is also thought to regulate mood,
anxiety, and stress behavior. Capsaicin reduces pain by decreasing the
level of substance P in the peripheral nervous system.
(Choice E) Platelet-activating factor (PAF) is a powerful phospholipid
activator produced by a number of cells, including neutrophils,
basophils, platelets, and endothelial cells. It is a very important
mediator of platelet aggregation.
Educational Objective:
1. The cutaneous flushing associated with niacin is mediated by
prostaglandins and can be prevented with aspirin pre-treatment.
2. Capsaicin reduces pain by decreasing the level of substance P in the
peripheral nervous system.

152
USMLE WORLD STEP 1 PHARMACOLOGY

Q NO 303: A 54-year-old male is hospitalized with atrial fibrillation and rapid


ventricular response. He had a similar episode three months ago. After
treatment, his arrhythmia is corrected and he becomes asymptomatic. You
consider long-term amiodarone therapy to prevent future arrhythmic
episodes. Which of the following should be tested before initiation of the
therapy?
A. Serum prolactin
B. Serum TSH
C. 24-hour urinary cortisol excretion
D. 24-hour urinary VIVA excretion
E. Oral glucose tolerance test
F. Serum calcitonin

Amiodarone is a class III anti-arrhythmic agent used to suppress life-


threatening cardiac conduction abnormalities. It has multiple side-
effects, and most patients on chronic amiodarone therapy can expect at
least one of the following: thyroid dysfunction corneal micro-deposits,
blue-gray skin discoloration, drug-related hepatitis, or pulmonary
fibrosis (rare, but life-threatening). Amiodarone causes thyroid
dysfunction because it is 40% iodine by weight. Amiodarone induced
hypothyroidism is due to excessive iodine and occurs in 5-20% of the
amiodarone-treated patients in iodine- sufficient regions. Patients
with preexisting autoimmune thyroid disease are at a greater risk for
hypothyroidism, which is why thyroid functions are routinely measured
before and during treatment with amiodarone. Amiodarone induced
hypothyroidism is treated with levothyroxine and amiodarone is
typically continued.
Amiodarone can also induce thyrotoxicosis due to excessive production
of thyroid hormone and primarily seen with patients living in iodine-
deficient areas.
(Choice A) Although serum prolactin can be increased in amiodarone-
induced hypothyroidism, it is by deficient thyroid feedback and is not
directly related to amiodarone.
(Choices C and D) Patients who are on amiodarone therapy do not have
changes in the hypothalamic-pituitary adrenocortical axis. Amiodarone
does not have any direct effect on the adrenal medullary gland, either.
(Choice E) The oral glucose tolerance test screens for diabetes and
“glucose intolerance,” diabetes precursor. Although some initial
studies suggested that amiodarone increases patients risk for glucose
intolerance, larger studies have not shown any clear association.
(Choice F) Calcitonin is secreted by the parafollicular C-cells.
Amiodarone does have a significant effect on thyroid follicular cells,
but has no effect on thyroid parafollicular cells.

Educational Objective:
Thyroid function tests should be monitored in patients receiving
amiodarone therapy. Amiodarone is associated with many side effects:
thyroid dysfunction, corneal micro-deposits, blue-gray skin
discoloration, drug-related hepatitis, and pulmonary fibrosis.

153
USMLE WORLD STEP 1 PHARMACOLOGY

Q NO 304: A 56-year-old female is hospitalized for elective hip replacement


A.
surgery. Her blood pressure is 130/80 mmHg and her heart rate is 90
beats/min. Her BMI is 31.5 kg/m2. The drug used to prevent perioperative
venous thrombosis in this patient has which mechanism of action?
Binds tightly to antithrombin
B. Binds to the thrombin active site
C. Blocks ADP receptors
D. Combines with proactivator plasminogen
E. Inhibits arachidonate product formation

Non-ambulatory hospitalized patients are at increased risk for


development of lower extremity deep venous thrombosis (DVT) which can
lead to pulmonary artery embolism. In patients undergoing elective
surgery without prophylaxis against DVl the risk for fatal pulmonary
embolism is about 0.1 to 0.8%. The risk is even higher in certain
orthopedic procedures such as hip and knee replacement.
Unfractionated and low molecular weight heparins are most commonly used
for prevention of deep venous thrombosis in inpatient settings. Heparin
increases the effect of the naturally occurring anti coagulant
antithrombin III Unfractionated heparin bind to antithrombin-lll by a
penta saccharide in the heparin chain. This binding causes a
conformational change of antithrombin-lll, which in turn increases
antithrombin binding and neutralization of thrombin.
(Choice B) Direct thrombin inhibitors do not require antithrombin-lll
for their action. These medications directly inhibit thrombin-mediated
fibrin formation. Lepirudin and argatroban are primarily used for
management of heparin-induced thrombocytopenia.
(Choice C) Ticlopidine and clopidogrel inhibit ADP mediated platelet
aggregation. They are useful following percutaneous coronary
intervention (PCI), and for treatment of unstable angina and non-Q wave
myocardial infarction.
(Choice D) Drugs that convert inactive plasminogen to plasmin result in
fibrinolysis and clot lysis. These medications are used for clot lysis
in acute myocardial infarction, pulmonary embolism and arterial
thrombosis.
(Choice E) Aspirin irreversibly acetylates platelet cyclooxygenase-l
leading to decreased formation of thromboxane A2. Aspirin is used as a
primary and secondary prevention of myocardial infarction and strokes.
Aspirin is not as effective as heparin in preventing perioperative
thromboembolism.

Educational Objective:
Heparin is the drug of choice for prevention of venous thrombosis in
non-ambulatory patients or patients undergoing elective surgery,
especially hip and knee surgery. Heparin increases the effect of the
naturally occurring anticoagulant antithrombin-III.

154
USMLE WORLD STEP 1 PHARMACOLOGY

Q NO 305: A new medication ‘Drug A1 has the following effects on HCI output by
the stomach mucosa:

Drug A is most similar to which of the following substances?

A. Atropine
B. Cimetidine
C. Lansoprazole
D. Sucralfate
E. Bethanechol

This drug inhibits gastric acid secretion in response to the three


major gastric acid secretagogues, acetylcholine (vagal stimulation),
histamine, and gastrin. It therefore likely acts at the parietal cell
apical membrane H+/K+ -ATPase proton pump the final common pathway for
HCl secretion. Drug A is therefore most similar to lansoprazole and
other proton pump inhibitors.
(Choice A) Atropine blocks the gastric parietal cell M3 acetylcholine
receptor.
(Choice B) Cimetidine blocks the gastric parietal cell histamine H2
receptor.
(Choice D) Sucralfate does not inhibit gastric acid secretion, but
rather binds to the base of mucosal ulcers, providing physical
protection against acid.
(Choice E) Bethanechol is cholinomimetic muscarinic agonist used to
treat ileus and urinary retention. Although it is somewhat selective
for the M2 muscarinic receptor, it can also increase gastric acid
secretion.

Educational Objective:
Proton pump inhibitors block the final common pathway of gastric acid
secretion from parietal cells, which is simulated by acetylcholine,
histamine, and gastrin.

155
USMLE WORLD STEP 1 PHARMACOLOGY

Q NO 306: A 68-year-old Caucasian male presents to your office with muscle pain
and fatigue. His past medical history is significant for hypertension and a
myocardial infarction two years ago. His serum creatine kinase activity is
elevated. Which of the following drug combinations is most likely to be responsible
for this patient’s condition?

A. Atorvastatin and ezetimibe


B. Atorvastatin and cholestyramine
C. Atorvastatin and gemfibrozil
D. Gemfibrozil and ezetimibe
E. Niacin and ezetimibe
F. Niacin and gemfibrozil

Statins are the first-line therapy for most patients with


hypercholesterolemia. Drugs of the statin class inhibit
hydroxymethylglutaryl coenzyme A (HMG-CoA) reductase, the enzyme
responsible for the conversion of HMG-CoA to cholesterol. Statins are
very effective at reducing LDL cholesterol.
Serious side effects of statins include myopathy and hepatitis.
Myopathy is a rare complication of statin use, defined clinically as
muscle pain with serum creatine kinase more than 10 times the upper
limit of normal. The risk of myopathy is increased when statins are
used in combination with fibrates, like gemfibrozil and fenofibrate.
Gemfibrozil increases the concentration of most statins, accounting for
the increased risk of myopathy. While fenofibrate does not
significantly alter statin pharmacokinetics, it causes myopathy itself,
thus compounding the risk for myopathy associated with statins alone.
Simvastatin is the statin with the highest associated risk of myopathy.
Its dose should not exceed 10mg when used concurrently with fibrates.
(Choice A) Concurrent administration of statins with ezetimibe has been
reported to cause increased risk of myopathy, though, in general, this
combination of drugs is much safer than the combination of statin plus
fibrate.
(Choice B) Concurrent use of cholestyramine (a bile acid sequestrant)
plus statin does not increase the risk of myopathy beyond that which is
expected with statins alone. However, bile acid sequestrants reduce the
gastrointestinal absorption of statins. To prevent this interaction,
itis recommended that the drugs be dosed at least four hours apart.
(Choice D) Safety data regarding the use of ezetimibe in combination
with fibrates is lacking. Studies in healthy subjects have shown that
co-administration of these two drug types increases plasma ezetimibe
levels by 1 Y2 to 2 times. Despite this fact the risk of clinical
myopathy with concurrent use of fibrates and ezetimibe has not been
studied extensively.
(Choice E and F) There are no significant interactions with the
concurrent use of niacin and ezetimibe or niacin and gemfibrozil.

Educational Objective:
1. The combination of statins plus fibrates increases the risk of
myopathy in patients being treated for hyperlipidemia. Statin use is
also associated with hepatotoxicity.
2. The combination of fibric acid derivates and bile acid-binding
resins increases the risk for cholesterol gallstones. This reflects an
increased cholesterol concentration in bile.

156
USMLE WORLD STEP 1 PHARMACOLOGY

Q NO 307: A 60-year-old farmer is brought to the emergency department due to


confusion, muscle cramps and difficulty breathing. His past medical history is
insignificant. He currently takes no medications. Physical examination reveals
excessive sweating, wheezing and bradycardia. His pupils are constricted,
symmetric and reactive to light. Intravenous atropine is administered and he
gradually improves. Which of the following is still a risk for this patient?

A. Bradycardia
B. Bronchospasm
C. Intestinal obstruction
D. Muscle paralysis
E. Urinary incontinence

Organophosphate poisoning most commonly occurs in farmers and


agricultural workers exposed to excessive amounts of pesticides. These
substances irreversibly inhibit cholinesterase in both muscarinic and
nicotinic cholinergic synapses. They decrease degradation of
acetylcholine and increase acetylcholine concentration in the synaptic
cleft, leading to overstimulation of the corresponding receptors.
The clinical presentation of organophosphate poisoning is that of
excessive cholinergic stimulation and can be memorized with mnemonic
“DUMBELS:”
D: diarrhea (stimulation of smooth muscle of GI tract) diaphoresis
(excessive sweating due to stimulation of secretory glands)
U: urination (stimulation of smooth muscle of detrusor)
M: miosis (constriction of pupillary sphincter)
B: bronchospasm (increased bronchial smooth muscle tone) bronchorrhea
(increased bronchial secretion), bradycardia (decreased conductance)
E: emesis (due to GI stimulation)
L: lacrimation (along with stimulation with other secretor glands)
S: salivation
All the effects listed above are due to muscarinic activation.
Increased acetylcholine concentration in the nicotinic synapses of
neuromuscular junction leads to muscle fasciculations followed by
paralysis. Organophosphates also penetrate the blood-brain barrier and
cause seizures and CNS depression.
Atropine reverses the muscarinic symptoms of organophosphate poisoning
but does not have any effect on nicotinic receptors. Patients treated
with atropine are still at risk of muscle paralysis. Pralidoxime is the
only medication that reverses both muscarinic and nicotinic effects of
organophosphates by “restoring” cholinesterase from its bond with these
substances. Pralidoxime is effective only if given early after the
exposure.

Educational Objective:
Organophosphates stimulate both muscarinic and nicotinic cholinergic
receptors. Atropine reverses muscarinic effects but does not prevent
the development of nicotinic effects such as muscle paralysis.
Pralidoxime is the only medication that reverses both muscarinic and
nicotinic effects of organophosphates by “restoring” cholinesterase
from its bond with these substances.

157
USMLE WORLD STEP 1 PHARMACOLOGY

Q NO 308: A 23-year-old male hospitalized for confusion and seizures is treated


with intravenous high-dose acyclovir. On the third day of hospitalization, his
serum creatinine level increases to 34 mg/dL from a baseline of 0.9 mg/dL at
admission. The observed finding could have been potentially prevented by which
of the following?

A. Careful allergy history taking


B. Monitoring the blood drug levels
C. Pre-treatment with allopurinol
D. Pre-treatment with prednisone
E. Aggressive intravenous hydration

The acute elevation of this patient’s serum creatinine is most likely


secondary to acyclovir nephrotoxicity, a phenomenon that occurs in 5-
10% of patients who receive the drug intravenously. Acyclovir is
excreted principally in the urine via glomerular filtration and tubular
secretion. When the acyclovir concentration in the collecting duct
exceeds its solubility, crystallization, crystalluria, and renal
tubular damage may result. In most cases, this toxic complication is
transient and can be prevented (as well as treated) with adequate
hydration and dosage adjustment, which includes slowing the rate of
intravenous infusion.
(Choice A) Allergic reaction is not the most common cause of acyclovir-
induced renal failure.
(Choice B) Monitoring of acyclovir blood levels is not always effective
in preventing nephrotoxicity. During states of low intravascular volume
(eg, dehydration), nephrotoxicity can develop secondary to drug
crystallization within the collecting duct even when acyclovir blood
levels are not within the established toxic range.
(Choice C) Allopurinol pre-treatment may be used in lymphoma and
leukemia to prevent tumor lysis-associated urate crystal nephropathy.
(Choice D) Pre-treatment with prednisone maybe used to minimize
radiocontrast-induced allergic reactions.

Educational Objective:
Acyclovir can cause crystalline nephropathy if adequate hydration is
not also provided.

158
USMLE WORLD STEP 1 PHARMACOLOGY

Q NO 309: A 5-month-old male is brought to clinic with low-grade fever and runny
nose. Five days later, the child presents to the emergency department with
progressive cough, tachypnea and agitation. Pulse oximetry confirms hypoxia.
Physical examination reveals diffuse bilateral wheezing, prolonged expiration, and
scattered rales. Which of the following drugs has the most potential to be useful in
treating this patient’s condition?

A. Oseltamivir
B. Ganciclovir
C. Ribavirin
D. Lamivudine
E. Amantadine
F. Interferon-a

This patient has developed the signs and symptoms (eg, rhonchi,
wheezing) of turbulent air flow secondary to airway obstruction, a
condition typically associated with asthma or bronchiolitis. Because
atopic asthma is rare in infants and the only medications available to
choose from here are antiviral agents, it can be inferred that this
patient is suffering from a viral bronchiolitis.
Viral bronchiolitis occurs in up to 15% of children under the age of
two years and is most commonly attributed to infection with respiratory
syncytial virus (RSV). In healthy children, RSV bronchiolitis is
usually a self-limited disease with most cases managed supportively.
Children with more severe RSV bronchiolitis can be given warmed,
humidified oxygen and intravenous fluids. Controversy surrounds the
administration of bronchodilators, corticosteroids, or aerosolized
ribavirin in most patients. Ribavirin is a nucleoside analog that
inhibits the synthesis of guanine nucleotides: this medication is
active against RSV and hepatitis C virus, and is most often given to
infants with confirmed RSV infection who are at risk for disease
progression.
(Choice A) Oseltamivir is a sialic acid analogue inhibitor of influenza
A and B viral neuraminidases. Although influenza can cause
bronchiolitis in infants or toddlers, RSV is the more common culprit.
(Choice B) Ganciclovir is an anti-herpesviral guanine nucleoside
analogue that is structurally similar to acyclovir. Ganciclovir is
distinguished from acyclovir in that it expresses greater activity
against CMV DNA polymerase. Herpesviruses are not known to cause
bronchiolitis in infants or toddlers.
(Choice D) Lamivudine is an anti-retroviral nucleoside reverse
transcriptase inhibitor.
(Choice E) Amantadine impairs un coating of the influenza A virion
after host cell endocytosis.
(Choice F) Naturally released by eukaryotic cells in response to viral
infection, interferon-a is pharmacologically used to combat infection
with the hepatitis B and C viruses.

Educational Objective:
Acute obstruction of the small airways in infants is most commonly
secondary to respiratory syncytial virus (RSV) bronchiolitis. Ribavirin
is an antiviral drug that may be of benefit in RSV bronchiolitis.

159
USMLE WORLD STEP 1 PHARMACOLOGY

Q NO 310: A 47-year-old Caucasian female is diagnosed with metastatic breast


cancer, and begins treatment with anastrozole. Soon after initiating therapy,
she experiences relief of her bone pain, and her primary tumor substantially
decreases in size. Which of the following best explains the effect of the therapy
in this patient?

A. Decreased follicular cell stimulation


B. Decreased androgen synthesis
C. Decreased androgen aromatization
D. Impaired ligand-receptor interaction
E. Impaired second messenger action

Estrogen is the main hormone responsible for the growth and development
of malignant breast tumors. The aromatase inhibitors are a group of
drugs that inhibit the synthesis of estrogen from its androgenic
substrate, suppressing estrogen to postmenopausal levels. In the
treatment of metastatic breast cancer, the aromatase inhibitors are
equivalent or superior to tamoxifen.
The aromatase enzyme belongs to the p450 superfamily, and is highly
expressed in ovarian tissue. (Ovarian expression of aromatase is
gonadotropin-dependent.) Aromatase is also present in subcutaneous fat,
and in normal as well as cancerous breast tissue. In the postmenopausal
state, this extra-ovarian aromatase maintains a low level of estrogen
in the peripheral circulation.
Aminoglutethimide and fadrozole were the initial aromatase inhibitors.
However, these drugs lacked potency and specificity. Newer third-
generation aromatase inhibitors are more highly selective and potent.
The three third- generation aromatase inhibitors currently available
are anastrozole, letrozole, and exemestane.
(Choice A) The production of LH and ESH by the pituitary is dependent
upon cyclic stimulation by GnRH from the hypothalamus. Paradoxically,
sustained stimulation of the GnIRH receptors inhibits LH and ESH
release. GnRH analogs with sustained stimulatory activity have been
developed to effectively suppress LH and ESH release.
(Choice B) Although inhibition of androgen synthesis could
theoretically decrease estrogen synthesis, in practice, most inhibitors
of androgen synthesis are nonspecific and suppress other pathways of
steroidogenesis such as cortisol and mineralocorticoid synthesis.
Ketoconazole is an antifungal agent that decreases androgen synthesis
by inhibiting multiple enzyme pathways.
(Choice D) Tamoxifen acts as either an antagonist or partial agonist of
the estrogen receptor, depending on the tissue type. In breast tissue
tamoxifen antagonizes the estrogen receptor, hence its utility as an
adjuvant therapy for estrogen receptor-positive breast cancer. However,
it has partial agonist effects in bone, endometrium, and cardiovascular
tissues, and has pro-coagulation effects as well. Thus, it is
associated with side effects such as deep venous thrombosis,
thromboembolism, and endometrial cancer.
(Choice E) Many patients with breast cancer express HER/neu-2
receptors. Epidermal growth factor (EGE) binds to these receptors,
activating tyrosine kinase, and causing cancer cells to proliferate.
Inhibition of EGF-HEFRJneu-2 pathways can lead to apoptosis of breast
cancer cells. Monoclonal antibodies against HER/neu-2 receptors have
been successfully used in the treatment of breast cancer. Preliminary

160
USMLE WORLD STEP 1 PHARMACOLOGY
reports of one of the monoclonal antibodies against HERfneu-2,
Trastuzumab, have been very encouraging.

Educational Objective:
1. Anastrozole is a selective inhibitor of aromatase, the enzyme
responsible for the conversion of androgens to estrogens.
2. Ketoconazole is an antifungal agent that decreases androgen
synthesis by inhibiting multiple enzyme pathways involved in the
synthesis of androgens.
3. Inhibition of the EGF-HER/neu-2 pathways by trastuzumab leads to
apoptosis of breast cancer cells. HEIR/neu-2 receptor activation leads
to activation of tyrosine kinase.

161
USMLE WORLD STEP 1 PHARMACOLOGY

Q NO 311: A 56-year-old male is diagnosed with type 2 diabetes mellitus. Despite


the introduction of an appropriate diet and physical activity, the patient’s glycemic
control does not improve significantly. You proceed with pioglitazone monotherapy
in this patient. This patient is most likely to experience which of the following side
effects?

A. Hypoglycemia
B. Lactic acidosis
C. Weight gain and edema
D. Agranulocytosis
E. Pulmonary fibrosis
F. Orthostatic hypotension

Thiazolidinediones (TEDs) exert their glucose-lowering effect by


improving insulin resistance, an effect achieved by binding to
peroxisome proliferator activated receptor gamma (PPAR-gamma). As the
glucose-lowering effect of TZDs requires an alteration in gene
transcription and in protein synthesis, a meaningful reduction in
glucose levels takes days to weeks after commencing TZD therapy. The
main side effect of TZDs is fluid retention weight gain (from fluid),
and the precipitation of congestive heart failure. Typically fluid
retention is worse when TZDs are used concurrently with insulin.
(Choice A) Hypoglycemia can occur with the use of sulfonylureas and
insulin unlike with metformin and TZDs. The risk of hypoglycemia does
increase however when insulin therapy occurs in combination with TZDs
or metformin.
(Choice B) Lactic acidosis is a side effect of metformin therapy.
Lactic acidosis is an uncommon complication of metformin when it used
in young patients with normal renal, hepatic and cardiac functions.
(Choice D) Rarely agranulocytosis occurs with sulfonylurea therapy.
(Choice E) None of the antidiabetic medications cause lung fibrosis.
Drugs that are associated with pulmonary fibrosis include amiodarone
bleomycin, mitomycin C, busulphan, and methysergide.
(Choice F) Postural hypotension or orthostatic hypotension is reported
with insulin therapy. The mechanism of insulin-induced postural
hypotension is not clear. Other medications that are commonly
associated with postural hypotension include alpha-adrenergic blockers,
diuretics and tricyclic antidepressants.

Educational Objective:
Fluid retention with resultant weight gain and edema is a common side
effect of thiazolidinedione therapy. This excess fluid can exacerbate
underlying congestive heart failure.

162
USMLE WORLD STEP 1 PHARMACOLOGY

Q NO 312: An infant born to a 28-year-old female expresses shrill crying, tremor,


rhinorrhea, sneezing and diarrhea. The condition is accompanied by myoclonic
jerks and rapidly progresses to seizures. You find out that the patient’s mother
lives on the street and has had little prenatal care. Which of the following would
most likely control this patient’s symptoms?

A. Naloxone
B. Flumazenil
C. Tincture of opium
D. Polio acid
E. Vitamin K
F. Sodium bicarbonate

Because the patient’s mother is homeless and lacked prenatal care, the
likelihood for polydrug abuse is yew high. Drug abuse can involve a
number of different drugs, but the presenting symptoms are more
consistent with neonatal opioid withdrawal.
Acute opioid withdrawal can present with pupillary dilatation,
rhinorrhea, sneezing, nasal stuffiness, diarrhea, nausea, and vomiting.
Chills, tremors, and jittery movements occur commonly as well, and can
rarely lead to seizures. Symptoms usually begin within 24-48 hours
afterbirth, but really depend on the time of the last dose taken by
mother.
The treatment of choice for treating acute opioid withdrawal in
neonates is opium solution. Opium contains morphine and is available in
oral solution as the drug tincture of opium or paregoric. Paregoric was
one of the first agents used in treating neonatal withdrawal, but
tincture of opium is now the preferred agent because paregoric contains
potentially toxic ingredients like alcohol, camphor, anise oil, and
benzoic acid.
(Choice A) Naloxone is a pure opioid receptor antagonist. It
antagonizes effects at all opioid receptors (mu, kappa. delta, etc.)
and is used in the setting of acute opioid intoxication or overdose and
for diagnosing opioid dependence. Administering naloxone to a patient
with opioid withdrawal can result in more severe withdrawal symptoms
and potentially cause seizures.
(Choice B) Flumazenil is a benzodiazepine receptor antagonist used for
benzodiazepine reversal.
(Choice D) Folic acid or folate is a water-soluble vitamin necessary
for red blood cell production and other nonspecific cellular division
and growth processes. It also plays a role in lowering homocysteine
levels and preventing neural tube defects during pregnancy.
(Choice E) Vitamin K is a fat-soluble vitamin essential for maintaining
blood clotting and synthesizing certain clotting factors in the liver.
Therapeutically, itis primarily used as a reversal agent for the
anticoagulant warfarin.
(Choice F) Sodium bicarbonate is an electrolyte combination available
for intravenous or oral use. Intravenously, sodium bicarbonate is used
in treating acidosis and hyperkalemia. Orally, it used as an
alkalinizing agent or an antacid.

163
USMLE WORLD STEP 1 PHARMACOLOGY
Educational Objective:
Acute neonatal narcotic withdrawal presents with symptoms of pupillary
dilatation, rhinorrhea, sneezing, nasal stuffiness, diarrhea, nausea,
and vomiting. Chills, tremors, and jittery movements occur commonly as
well, and can rarely lead to seizures. Opium solution given as diluted
tincture of opium is the treatment of choice and can be given orally.
The dose is initially titrated to the patient’s symptoms and eventually
tapered off.

164
USMLE WORLD STEP 1 PHARMACOLOGY

Q NO 313: A 55-year-old female is diagnosed with metastatic breast cancer. She is


treated with an opioid analgesic for bone pain. Although very effective in controlling
the pain during the first week of therapy the analgesic dose does not make her pain
go away after a while. You explain to the patient that tolerance to the effect of the
drug has developed and a dose increase is necessary. Which of the following opioid
effects is most resistant to tolerance development?

A. Euphoria
B. Sedation
C. Nausea and vomiting
D. Respiratory depression
E. Constipation
F. Cough suppression
G. Urinary retention

The use of opioids can lead to the development of tolerance or a


decrease in opioid effectiveness and physiological response with
continued use. Tolerance to opioid effectiveness is different from
addiction and abuse and can be seen in all patients in varying
magnitudes. In particular, cancer pain usually requires extremely high
doses of opioids for prolonged time periods. The development of
tolerance should be recognized and managed appropriately. Patients who
develop tolerance will require higher dosages to achieve effective and
adequate pain control.
Tolerance to the different side effects of opioids is also expected to
occur. However, tolerance to constipation and miosis does not readily
occur. Opioids bind and stimulate mu receptors in the gastrointestinal
tract causing decreased secretions and gastric motility. Thus,
constipation can occur with continued administration of opioids and
normal bowel function rarely resumes. For patients who will require
opioid therapy, it is recommended to prophylactically treat
constipation with adequate fluid intake and laxatives. Opioids also
cause activation of parasympathetic effects on the pupil resulting in
miosis. Very little tolerance develops to these effects and pupils will
remain constricted in most patients.
(Choice A) Opioids can produce significant euphoria, mood alterations,
and rewarding properties. The exact mechanism of euphoria and rewarding
properties are unknown, buy may involve dopaminergic pathways in the
nucleus accumbens. Tolerance to these effects develops rapidly and can
lead to drug addiction and abuse.
(Choice B) Sedation typically occurs during initiation of therapy and
usually disappears after several days. It is not unusual for patients
to be drowsy and sleep more during first days of therapy.
(Choice C) Nausea with or without vomiting is the most common and
unpleasant side effect of opioid therapy. Opioids stimulate the
chemoreceptor trigger zone leading to emesis. Tolerance will usually
develop after a few days of therapy.
(Choice D) Respiratory depression is the most severe, yet rare side
effect of opioid therapy. Opioids depress respiration by reducing
responsiveness of respiratory centers in the brain stem to carbon
dioxide. Tolerance to respiratory depression develops very rapidly and
is unlikely to occur in patients with active pain or in patients who
have been taking opioids regularly.
(Choice F) Opioids directly affect receptors in the cough center of the
medulla and thus effectively depress cough reflexes. Cough suppression

165
USMLE WORLD STEP 1 PHARMACOLOGY
can be achieved with lower dosages than required for analgesia.
Tolerance to cough suppressant effects of opiates can easily develop.
(Choice G) Opioids block urinary voiding reflexes and also increase
sphincter tone and bladder volume. This results in an antidiuretic
effect and urinary retention. Tolerance develops rapidly to these
effects and symptoms are transient.

Educational Objective:
The use of opioids can lead to the development of tolerance ora
decrease in opioid effectiveness and physiological response with
continued use. Tolerance to opioid induced constipation and miosis does
not readily occur. To prevent bowel complications, itis recommended to
treat patients prophylactically with adequate fluid intake and daily
laxatives.

166
USMLE WORLD STEP 1 PHARMACOLOGY

Q NO 314: A 24-year-old. HI V-positive, African-American male has a CD 4 count of


200. He is started on dapsone in lieu of trimethoprim-sulfamethoxazole (TMP-SMX)
because of a previous adverse reaction. He is also given pneumococcal and
influenza vaccines. A few days later, he comes back to the office complaining of
fatigue jaundice, and dark urine. Laboratory studies show:

Complete blood count


Hemoglobin 9.0 g/L
MCV 85fl
Reticulocytes 7.1%
Platelets 234 000/mm3
Leukocyte count 5500/mm3
Coagulation studies are within normal limits. Peripheral blood smear
shows red cell fragments microspherocytes, and “bite” cells. Which of
the following is most likely responsible for his symptoms?

A. Disseminated intravascular coagulation


B. Vaccine-induced hemolysis
C. Enzyme-deficiency anemia
D. Abnormal sickling due to medication
E. Red blood cell cytoskeleton abnormality

All HI V-positive patients with a CD4 count of c 200 should be


prophylactically treated for pneumocystis carinii pneumonia (PCP). TMP-
SMX is the drug of choice for this purpose, but dapsone is an
alternative agent in cases of allergy, etc. Possible side effects of
dapsone include fever, rash, and methemoglobinemia. Dapsone also puts
oxidative stress on the body, so G6PD levels should be checked before
administering dapsone in order to prevent hemol4ic anemia in patients
with this enzyme deficiency.
Common precipitating factors of glucose-6-phosphate dehydrogenase
(G6PD) deficiency anemia include:
1. Infections
2. Drugs — dapsone, antimalarials, sulfonamide antibiotics (TMP-SMX)
3. Diabetic ketoacidosis
4. Favism (ingestion of fresh fava beans)
Suspicions of dapsone-induced hemolytic anemia in the context of G6PD
deficiency are confirmed by this patient’s anemia, reticulocytosis,
jaundice, and dark urine. Furthermore, his blood smear shows red cell
fragments, microspherocytes, and “bite cells.” Bite cells are typical
of oxidant-induced damage, such as in G6PD deficiency. Heinz body
preparation is also very helpful in the diagnosis—G6PD deficiency will
be demonstrated on crystal violet stain as small, irregular, dark
purple granules in the red blood cells which are called “Heinz bodies.”
Bite cells are often the result of the phagocytic removal of Heinz
bodies by the splenic monocyte-macrophage system.
(Choice A) Disseminated intravascular coagulation is characterized by
thrombocytopenia and a prolonged PT and aPTT.
(Choice B) Hemolytic anemia is not a known complication of either
pneumococcal or influenza vaccines. (Choice D) Abnormal sickling is the
hallmark of sickle cell anemia. This form of anemia is genetic.
Exacerbations of sickle cell anemia may be caused by deoxygenation and
dehydration, but not by dapsone.
(Choice E) Diseases with red blood cell cytoskeleton abnormalities
include hereditary spherocytosis, hereditary elliptocytosis, and

167
USMLE WORLD STEP 1 PHARMACOLOGY
hereditary stomatocytosis. They would have presented early. This is
perhaps a plausible explanation for this patient’s symptoms, but not
the best choice.

Educational Objective:
• Hemolytic anemia is a possible side effect of dapsone and is most
significant in patients deficient for glucose- 6-phosphate
dehydrogenase (G6PD).
• G6PD deficiency anemia is characterized by episodes of hemolytic
anemia precipitated by oxidative stress (drugs, infections). Peripheral
smear typically shows bite cells and Heinz bodies (requires special
preparation).

168
USMLE WORLD STEP 1 PHARMACOLOGY

Q NO 315: A 68-year-old male presents to your office with exertional thigh and leg
pain and decreased sexual performance. His past medical history is significant for
hypertension, diabetes mellitus and myocardial infarction experienced two years
ago. He smokes two packs per day and consumes alcohol occasionally. His dorsalis
pedis pulses are weak in both feet. Evaluation shows moderate peripheral arterial
disease of both lower extremities. This patient experiences symptomatic
improvement from a drug that dilates arteries and inhibits platelet aggregation.
Which of the following drugs is most likely described in this scenario?

A. Heparin
B. Argatroban
C. Warfarin
D. tPA
E. Aspirin
F. Ticlopidine
G. Abciximab
H. Cilostazol

Several molecules, including thrombin, ADP, and thromboxane A2,


activate platelets by acting on cell surface receptors. Interference
with post receptor signaling can alter platelet functions. Agents that
increase platelet cyclic AMP decrease platelet aggregation by
preventing platelet shape change and granule release.
Dipyridamole and cilostazol work by decreasing the activity of platelet
phosphodiesterase, the enzyme responsible for breakdown of cyclic AMP.
Cilostazol, in addition to inhibition of platelet aggregation, also is
a direct arterial vasodilator. Cilostazol is approved by the FDA for
the treatment of intermittent claudication (peripheral arterial
disease).

Educational Objective:
Cilostazol is a PDE inhibitor used in patients with intermittent
claudication. In addition to inhibition of platelet aggregation,
cilostazol is also a direct arterial vasodilator.

169
USMLE WORLD STEP 1 PHARMACOLOGY

Q NO 316: A 14-year-old Caucasian male with seizures that are refractory to


medication is started on lamotrigine. The patient should be instructed to see
his physician immediately if which of the following symptoms occurs?
A. Abdominal pain
B. Diarrhea
C. Cough and fever
D. Urine discoloration
E. Skin rash
F. Heartburn

Lamotrigine is one of the newer anti convulsants and is used mainly for
the treatment of refractory partial seizures. It is also effective for
the management of generalized tonic-clonic seizures, and in the
treatment of bipolar disorder.
Lamotrigine has fewer side effects than other anti convulsants:
however, it is associated with a potentially life- threatening
hypersensitivity reaction that manifests as a skin rash. This is
particularly important in children and requires discontinuation of the
drug immediately.
The other relatively new anti convulsants that are used predominantly
for refractory partial seizures are:
Tiagabine: inhibitor of GABA uptake.
Topiramate: blocks Na channels and enhances the effect of GABA.
Vigabatrin: inhibits GABA-transaminase and increases GABA
concentration.
Gabapentin: increases brain GABA concentration.

Educational Objective:
Lamotrigine is a newer anticonvulsant used for the treatment of
refractory partial seizures. It may cause skin rash. Because this rash
is life threatening in children, the drug should be discontinued
immediately at the first sign of rash.

170
USMLE WORLD STEP 1 PHARMACOLOGY

Q NO 317: A 23-year-oldfemale presents to the ER complaining of restlessness in


her legs and inability to lie or sit still. She was diagnosed with schizophrenia
recently, but she says that she has not been hearing ‘voices any more since the
treatment was initiated. Her blood pressure is 140/90 mmHg and her heart rate is
90/mm. She is alert, awake and oriented to time, place and person. Which of the
following is the most likely diagnosis in this patient?

A. Acute dystonia
B. Drug-induced Parkinsonism
C. Akathisia
D. Tardive dyskinesia
E. Neuroleptic malignant syndrome
F. Drug-induced delirium

The passage describes a classic presentation of akathisia a well-known


complication of neuroleptic therapy. Akathisia can range from a mild
subjective feeling of anxiety or disquiet to intense physical
restlessness. It is typically presents days to weeks after initiation
of treatment and patients complain of feeling “on edge” or tense are
unable to sit or stand in one position, and often manifest repetitive,
purposeless movements such as “restless legs” syndrome. Treatment rests
on decreasing the dose of the neuroleptic or adding antiparkinsonian
agents.
(Choice A) Acute dystonia is the least common, but most disabling, of
the hyperkinetic movement disorders associated with anti-psychotic use.
It is defined by twisting involuntary movements and abnormal postures.
Usually, dystonia occurs within 5 days of anti-psychotic use especially
with high-potency neuroleptics.
(Choice B) Neuroleptic-induced Parkinsonism can also occur, developing
between four days and four months after receiving an antipsychotic
medication. It presents with cogwheel rigidity, masked facies,
bradykinesis, pill-rolling finger tremors, and shuffling gait.
Treatment involves anticholinergics such as benztropine.
(Choice D) Tardive dyskinesia is characterized by involuntary perioral
movements such as biting, chewing, grimacing, and tongue protrusions.
Involuntary choreoathetoid movements of the head, limbs, and trunk may
also be observed. The condition usually arises between four months and
four years of treatment and may be irreversible.
(Choice E) Neuroleptic malignant syndrome (NMS) is an acute potentially
fatal idiosyncratic reaction to neuroleptic medications. The principal
manifestations are due to disorders of thermoregulation and skeletal
muscle metabolism mediated via central mechanisms. The usual
presentation consists of four primary features: (1) hyperthermia, (2)
extreme generalized rigidity (3) autonomic instability, and (4) altered
mental status.
(Choice F) The hallmark of delirium is confusion or clouding of the
sensorium. Patients may appear somewhat dazed and unclear about their
surroundings. They have difficulty perceiving correctly what goes on
around them and one may have difficulty capturing and holding their
attention. Disorientation to time and place are common accompanying
features.

171
USMLE WORLD STEP 1 PHARMACOLOGY
Educational Objective:
Akathisia is a movement disorder characterized by inner restlessness
and an inability to sit or stand in one position. Onset of akathisia
can be a complication of anti-psychotic therapy. A diagnosis of
akathisia is often missed because the movements and restlessness are
misinterpreted as worsening psychotic behavior and agitation and
instead of decreasing the patient’s dose the patient’s neuroleptic
dosing is often increased, exacerbating the akathisia.

172
USMLE WORLD STEP 1 PHARMACOLOGY

Q NO 318: A 75-year-old male is diagnosed with hypertension. On careful


questioning he describes chest pain on exertion. Physical examination is
significant for decreased pulse amplitude over right femoral artery and a
systolic bruit over left carotid arteri. Treatment with ramipril is initiated
that results in a significant increase in serum creatinine level. An effect of
the drug on which of the following renal vessels is most likely responsible
for the abnormality observed in this patient?

A. A
B. B
C. C
D. D
E. E

ACE inhibitors can acutely reduce GFR causing a rise in serum


creatinine by altering the autoregulatory hemodynamics of the kidneys.
Normally, the kidney maintains GFR by dilating the afferent arteriole
and constricting the efferent arteriole. ACE inhibitors reduce
angiotensin II synthesis and activity, causing decreased efferent
arterial tone and subsequent dilation. This effect reduces outflow
resistance from the glomerulus and decreases hydrostatic pressure in
the glomerular capillaries ultimately decreasing renal perfusion
pressure.
It is important to distinguish between acute renal failure and a normal
predictable rise in serum creatinine secondary to ACE inhibitor
therapy. Based on their pathophysiologic effects, an increase in serum
creatinine up to 30 % within 2to
5 days can be expected in most patients started on ACE inhibitors. This
increase usually stabilizes in 2 to 3 weeks and is reversible upon drug
discontinuation.

173
USMLE WORLD STEP 1 PHARMACOLOGY
(Choice A) Although ACE inhibitors do cause constriction in the
afferent arteriole, prostaglandins are also released to produce a net
dilation in the afferent arteriole. This aims to improve blood flow
into the glomerulus and improve renal perfusion.
(Choices C and D) ACE inhibitors have no effect on the descending or
ascending loop of Henle.
(Choice E) ACE inhibitors block the release of aldosterone resulting in
decreased Na reabsorption and increased potassium retention in the
distal and collecting tubules. Although this effect is responsible for
the hyperkalemia often seen with ACE inhibitor therapy, it has no role
in ACE inhibitor induced renal failure.

Educational Objective:
ACE inhibitors cause renal failure by altering renal hemodynamics. ACE
inhibition results efferent arteriole constriction, thereby decreasing
glomerular pressure and renal perfusion. For patients dependent on
efferent arteriole constriction to maintain renal perfusion (those with
renal artery stenosis), ACE inhibitors can be detrimental causing acute
renal failure or complicating existing renal disease.

174
USMLE WORLD STEP 1 PHARMACOLOGY

Q NO 319: A 40-year-old female is brought to the ER suffering involuntary


A.
movements ataxia and tremor. Her past medical history is significant for
bipolar disorder treated with a stable dose of lithium over many years. You
also find out that she was diagnosed with hypertension recently. Which of
the following agents was most likely used to treat this patient’s
hypertension?
Amlodipine
B. Prazosin
C. Hydrochlorothiazide
D. Metoprolol
E. Reserpine
F. Clonidine

This patient most likely has developed lithium toxicity. Lithium is


almost exclusively excreted by kidneys. Most of the filtered lithium is
reabsorbed in the proximal tubules. Lithium reabsorption follows sodium
reabsorption. Thus, anything that increases proximal tubular absorption
of sodium also increases lithium levels and elevates the risk of
lithium toxicity. Thiazide diuretics cause volume depletion; because of
this kidneys two to retain sodium and water. NSAIDs also cause lithium
toxicity. NSAIDs cause relative ischemia (prostaglandin inhibition),
leading kidneys to sense there is not enough intravascular volume and
start retaining sodium and water. This is the same reason that NSAIDs
are relatively contraindicated in heart failure.
Lithium has a very narrow therapeutic index and toxicity typically
occurs at blood levels > 2.0 mEq/L. Important clinical manifestations
include neuromuscular excitability, irregular coarse tremors fascicular
twitching, agitation ataxia, and delirium. Hemodialysis is the most
effective way of acutely reducing the blood lithium level.
The other drugs mentioned in the choices do not cause lithium toxicity.

Educational Objective:
Thiazide diuretics and NSAIDs can cause lithium toxicity. Lithium is
almost exclusively excreted by kidneys. Most of the filtered lithium is
reabsorbed in the proximal tubules. Lithium reabsorption follows sodium
reabsorption. Thus anything that increases proximal tubular absorption
of sodium also increases lithium levels and elevates the risk of
lithium toxicity. Hemodialysis is the most effective way of acutely
reducing the blood lithium level.

175
USMLE WORLD STEP 1 PHARMACOLOGY

Q NO 320: A 60-year-old male with mild to moderate chronic obstructive pulmonary


disease secondary to cigarette smoking is admitted to the hospital because of
sudden onset chest pain. Evaluation shows elevated cardiac troponin and ST
elevation in the inferior leads. You decided to administer a beta blocker. Which of
the following is the most appropriate for this patient?

A. Propranolol
B. Nadolol
C. Metoprolol
D. Timolol
E. Isoproterenol

The patient described in this clinical vignette has a myocardial


infarct, which is an important indication for a β- adrenergic
antagonist (i.e. beta blocker). Beta-blockers are divided into non-
selective (act on both β1 and β2 receptors) and selective (affect
predominantly β receptors). The group of non-selective beta-blockers
includes propranolol, timolol, and nadolol.
The effects of both types of β-adrenoreceptors are summarized in the
table below.

Bronchoconstriction is a potentially serious side effect of non-


selective β-blockers; therefore, these medications are contraindicated
in patients with asthma or COPD. Alternatively, selective β-blockers
affect β1-receptors so they are less likely to cause
bronchoconstriction. Selective β-blockers can be safely used to treat
myocardial infarction in both asthmatic patients and those with COPD.
(Choice E) Isoproterenol is not a β-blocker; it is an adrenergic
agonist that stimulates β1 and β2 adrenergic receptors. Isoproterenol
causes bronchodilatation, increases heart rate and cardiac output and
decreases peripheral arterial resistance. Even though this drug helps
patients with asthma and COPD, it should never be used during an acute
Ml.
Educational Objective:
Non-selective β3-adrenergic blockers cause bronchoconstriction,
peripheral vasospasm, and can predispose diabetic patients to
hypoglycemia. Selective β-blockers (metoprolol, atenolol, acebutolol,
and esmolol) are preferred in patients with COPD and/or asthma.

176
USMLE WORLD STEP 1 PHARMACOLOGY

Q NO 321: A 48-year-old female presents to your office complaining of change in


her body habitus. She says her legs are “like sticks” while her “belly is getting
bigger.” Examination demonstrates loss of adipose tissue from the extremities
and the gluteal region with a simultaneous increase in abdominal girth. If this
patient’s condition is medication-induced, which of the following disorders was
she most likely to have been treated for?

A. HIV infection
B. Thyroid nodule
C. Stable angina
D. Hypercholesterolemia
E. Peptic ulcer disease
F. Hypertension
G. Osteoarthritis

This patient is described as having redistribution of fat from the


extremities to the trunk. Medication-induced body fat redistribution
often accompanies the administration of glucocorticoids or highly
active antiretroviral therapy (HAART). Since none of the conditions
listed above is treated with glucocorticoids we can conclude that this
patient is on HAARTI and probably HIV-1 protease inhibitors in
particular.
Medication-induced body fat redistribution is characterized by an
excess accumulation of fat in the abdominal viscera breasts, posterior
neck (“buffalo hump”), and the supraclavicular area. A dysmetabolism
profile consisting of hyperglycemia hyperlipidemia and hyperinsulinemia
is frequently observed. HIV-1 protease inhibitors are thought to cause
this lipo dystrophic condition by impairing hepatic chylomicron uptake
and triglyceride clearance.
(Choice B) Glucocorticoids are used to treat proptosis in patients with
Graves disease (diffuse thyroid enlargement) but are not used in the
treatment of thyroid nodules.
(Choices C, D, E, F, and G) Stable angina hypercholesterolemia, peptic
ulcer disease hypertension, and osteoarthritis are not typically
treated with drugs known to cause medication-induced body fat
redistribution.

Educational Objective:
HIV-1 protease inhibitors and glucocorticoids are strongly associated
with medication-induced body fat redistribution syndrome which is
characterized by the redistribution of body fat from the extremities to
the abdominal viscera and the subcutaneous adipose tissues of the
thorax, posterior neck and supraclavicular region.

177
USMLE WORLD STEP 1 PHARMACOLOGY

Q NO 322: A 35-year old female presents to the emergency room with a “racing
heart” and shortness of breath. She also admits to insomnia. Cardiomegaly is
noted on chest x-ray. An EKG shows sinus tachycardia at 120 bpm. Serum TSH
is decreased and free T4 is increased. Intravenous propranolol is given to this
patient. In addition to its β receptor blocking actions, propranolol is likely to
benefit this patient by decreasing which of the following?

A. Peripheral conversion of T4 to T3
B. Release of T4 by the thyroid gland
C. New thyroid hormone synthesis
D. Tissue oxygen consumption
F. Serum levels of TSH receptor antibodies

This patient presents with symptoms and signs of a hypermetabolic,


hyper adrenergic state, as well as mild congestive heart failure. Her
laboratory findings indicate thyrotoxicosis. The “triangle of
treatment” for thyrotoxicosis is aimed at minimizing the following:
1. thyroid hormone synthesis and release,
2. peripheral conversion of T4 to the more active T3, and
3. sympathetic outflow and/or its actions on target organs.
β receptor blockers contribute to the latter to therapeutic objectives.
Hyper adrenergic manifestations in thyroid storm reflect a generalized
increased sensitivity to catecholamines via thyroid hormone-mediated up
regulation of adrenergic receptor expression. Thus, propranolol will
reduce this patient’s heart rate and subjective feelings of
anxiety/agitation. Via uncertain mechanisms, propranolol decreases the
peripheral conversion of T4 to T3, thereby decreasing net thyroid
hormone action on target tissues.

178
USMLE WORLD STEP 1 PHARMACOLOGY
(Choice B) β-receptor blockers (i.e. beta blockers) are not known to
affect thyroid hormone release. In thyrotoxicosis, such release can be
reduced by administration of pharmacologic doses of iodine.
(Choice C) Beta blockers are not known to after thyroid hormone
synthesis. Propylthiouracil or methimazole are administered to decrease
the synthesis of thyroid hormone.
(Choice D) In thyrotoxicosis and other hyperthyroid states, beta
blockers improve some manifestations of excessive thyroid hormone
action, but not all. Hyperthyroid symptoms/signs that are not improved
by β blockers include: increased tissue oxygen consumption, circulating
thyroxine levels, weight loss, and—in the case of Grave’s disease—
exophthalmos.
(Choice E) Beta blockers do not affect serum levels of TSH receptor
antibodies.

Educational Objective:
The mechanism of action of receptor blockers in thyrotoxicosis is dual:
there is a decrease in the effect of sympathetic adrenergic impulses
reaching target organs and a decrease in the rate of peripheral
conversion of T4 to T3.

179
USMLE WORLD STEP 1 PHARMACOLOGY

Q NO 323: A 38-year-old Caucasian female presents to your office for a routine


check-up, and requests a simple and reliable method of contraception. She has
no significant past medical history, and does not take any medications other
than a daily multivitamin. Which of the following factors would most affect your
decision to prescribe oral contraceptives to this patient?

A. Diet
B. Physical activity level
C. Smoking status
D. Parity
E. Glucose intolerance
F. Serum HDL level

Oral contraceptive pills are the most reliable reversible form of


female contraception. They typically contain a combination of estrogen
and progestin, and are taken in cycles interrupted by menstrual
bleeding. The estrogen in oral contraceptives prevents pregnancy by
suppressing the midcycle gonadotropin surge, thereby inhibiting
ovulation. Progesterone is added to counteract the increased risk of
endometrial cancer associated with unopposed effect of estrogen.
Additionally, progesterone may enhance the contraceptive efficacy by
making the endometrium less suitable for implantation and by decreasing
the permeability of the cervical mucus to sperm.
Side effects of oral contraceptives may include: breakthrough menstrual
bleeding breast tenderness, and weight gain. Additionally, there is a
risk of more serious, though rare, events, such as deep vein thrombosis
pulmonary embolism ischemic stroke, and myocardial infarction. The risk
of cardiovascular events due to oral contraceptive pills is increased
in smokers and patients over the age of 35. In particular, individuals
who smoke more than 15 cigarettes per day have a much higher incidence
of cardiovascular events.
(Choices A and B) Use of oral contraceptive pills may cause weight
gain. However there is no relation of weight gain to the more serious
potential health risks associated with oral contraceptives.
(Choice D) The risks of oral contraceptive pills are similar in females
regardless of past pregnancy or childbearing status.
(Choice E) Oral contraceptive pills can induce insulin resistance and
cause glucose intolerance. However the risk of glucose intolerance was
most significant with the first generation of high-dose estrogen oral
contraceptives. With the reduced quantities of estrogen in the newer
and 3rd generation oral contraceptive pills glucose intolerance has
become much less common. Adequately controlled diabetes mellitus, in
the absence of other cardiovascular risk factors is not a
contraindication to the use of oral contraception.
(Choice F) Although low HDL is a known cardiovascular risk factor,
there is no data to suggest that patients with low HDL have especially
heightened cardiovascular risks while using oral contraceptives. The
guideline for oral contraceptive use in patients with
hypercholesterolemia is dependent instead upon the LDL level. Oral
contraceptives are generally safe to use in patients with an LDL below
160 mg/dL without any other cardiovascular risk factors.

180
USMLE WORLD STEP 1 PHARMACOLOGY
Educational Objective:
The absolute contraindications to the use of OCPs are:
1. Prior histor of thromboembolic event or stroke
2. History of an estrogen-dependent tumor
3. Women over age 35 years who smoke heavily
4. Hypertriglyceridemia
5. Decompensated or active liver disease (would impair steroid
metabolism)
6. Pregnancy

181
USMLE WORLD STEP 1 PHARMACOLOGY

Q NO 324: A 66-year-old Caucasian male presents to your office with a several


month history of straining during urination hesitancy and intermittent urinary
flow. Upon reviewing his medical record, you also note that his blood pressure
was high (160-170/95-100 mmHg) during his last several check-ups. He is not
taking any medications. Which of the following drugs would address both of
this patient’s current problems?

A. Doxazosin
B. Metoprolol
C. Ramipril
D. Hydrochlorothiazide
E. Eplerenone
F. Amlodipine

In medical practice, patients are often prescribed so many medications


that they have trouble keeping track of them. In order to give them an
opportunity for best compliance with your medical treatment it is
desirable to choose medications which can address multiple issues at
once by taking advantage of the multiple effects of certain drugs on
the body.
Alpha 1-blockers such as Doxazosin Prazosin and Terazosin are useful
for the treatment of both benign prostatic hyperplasia and
hypertension. They act by blocking the alpha-1 adrenergic receptor
leading to a relaxation of smooth muscle in arterial and venous walls
leading to a decrease in peripheral vascular resistance. They do not
have an effect on the chronotropy or inotropy of the heart.
Additionally via their blockade of the alpha-i adrenergic receptor
these drugs induce relaxation of smooth muscle in the bladder neck and
prostate leading to a decrease in urinary obstruction caused by benign
prostatic hyperplasia (BPH). Thus they are useful both as a treatment
for hypertension and as a medical treatment for BPH. Decreased
peripheral vascular resistance can lead to orthostatic hypotension and
vertigo which are common side effects of this class of drugs. The most
notable issue with these medications is their tendency to cause
hypotension when treatment is first started; this is known as a first-
dose effect and can be ameliorated by starting with a very small first
dose.
(Choice B) Beta blockers are indicated in hypertensive patients who
also have evidence of coronary artery disease and congestive heart
failure.
(Choice C) Ramipril is an angiotensin converting enzyme inhibitor (ACE-
I). ACE-Is are the drugs of choice for patients with diabetes and
hypertension. ACE-Is have no effect on BPH.
(Choice D) Hydrochlorothiazide is presently the first-line medication
for the treatment of essential hypertension in the general population.
It is a diuretic that functions at the distal convoluted tubule of the
nephron and prevents the reabsorption of sodium chloride and water by
blocking a Na/Cl cotransporter (Remember: wherever sodium goes water
follows). Thiazides can cause elevation of glucose uric acid and
calcium and thus are not recommended for diabetic patients, patients
with gout or patients with hypercalcemia. They can also cause
hyponatremia.
(Choice E) Eplerenone is a newer aldosterone antagonist and functions
similarly to spironolactone as a potassium sparing diuretic. These
drugs are weak diuretics and are frequently used in conjunction with

182
USMLE WORLD STEP 1 PHARMACOLOGY
other diuretics for their potassium-sparing effect or for hyper
aldosterone states such as with an aldosterone-secreting tumor. The
most frequently mentioned side effect of these medications is their
ability to cause gynecomastia (approximately i% with Eplerenone, 9%
with spironolactone).
(Choice F) Amlodipine is a dihydroptyridine calcium channel blocker and
differs from verapamil in that amlodipine is selective for the vascular
smooth muscle and does not affect the heart as verapamil does. It is
often used as an antihypertensive agent but is second line for most
patients. Calcium channel blockers are particularly useful for patients
with evidence of vasospasm such as Raynaud phenomenon, Prinzmetal
angina etc.

Educational Objective:
1. Alpha 1-blockers such as Doxazosin, Prazosin and Terazosin are
useful for the treatment of both benign prostatic hyperplasia and
hypertension.
2. Patients with coronary artery disease and heart failure along with
hypertension will benefit from cardioselective beta-blockers.
3. Hydrochlorothiazide is presently the first-line medication for the
treatment of essential hypertension in the general population.

183
USMLE WORLD STEP 1 PHARMACOLOGY

Q NO 325: A 61-year-old male is found to have hyperuricemia after an episode of


arthritis. His past medical history is significant for recurrent renal colic. Which of
the following is the best agent for long-term management of this patient?

A. COX inhibitor
B. Antibiotics
C. Uricosuric agents
D. Xanthine oxidase inhibitor
E. Lipoxygenase antagonist
F. Glucocorticoid

Gout is not always a progressive disease; therefore, uric acid lowering


therapy is not required in all patients who present with acute gouty
arthritis. Dietary changes (decrease in purine intake), weight loss
abstinence from alcohol, or change in drug therapy (e.g.
discontinuation of thiazides) can result in substantial reduction in
serum uric acid levels. The major indications for long-term uric acid
therapy are 1) macroscopic tophaceous deposits, 2) more than 3 episodes
of acute gout arthritis per year, 3) uric acid stones or 4) gross
elevation of serum uric acid levels.
Uricosuric drugs (Probenecid or sulfinpyrazone) or xanthine oxidase
inhibitors (allopurinol) are used as long-term serum uric acid lowering
therapy. In hyperuricemic patients, the goal is to reduce the uric acid
level below 6 mg/dL. This can be achieved with either uricosuric drugs
or allopurinol. In patients who excrete large amounts of uric acid,
uricosuric drugs should be avoided to prevent uric acid
nephrolithiasis; allopurinol would be the best option in these
patients. Nonetheless, allopurinol is most commonly used irrespective
of the level of urinary excretion in idiopathic gout.
Chronic therapy with allopurinol leads to a reduction of recurrent
acute gout arthritis and shrinkage of tophi. Precipitation of an acute
gouty arthritis attack can occur in patients during the initiation of
chronic hypouricemic therapy with either xanthine oxidase inhibitors or
uricosuric agents. Febuxostat is a new xanthine oxidase inhibitor that
is thought to be safer in patients with renal dysfunction and has fewer
drug interactions compared to allopurinol.
(Choice A) NSAIDs exert their antiinflammatory effect by COX
inhibition. NSAIDs are the first line therapy for most patients with
acute gout arthritis. They should be used with extreme caution in
patients with increased risk of gastrointestinal bleeding, congestive
heart failure, renal failure, and the elderly. Chronic NSAID therapy is
sometimes used to prevent precipitation of acute gouty arthritis when
chronic hypouricemic therapy is initiated.
(Choice B) Antibiotics are used in patients where acute infectious
arthritis is considered.
(Choice C) Probenecid and sulfinpyrazone are uricosuric agents.
Uricosuric agents are only effective in patients with good renal
function (GFR more than 50-60 mL/min) and those who can maintain
adequate renal flow of more than 1500 mL per day. These agents are
avoided in patients who are excreting excessive amounts of uric acid in
their urine (> 700 mg/24 hr) or in those who have kidney stones.
(Choice E) Lipoxygenase inhibitors such as Zileuton are useful in
patients who have allergic rhinitis and bronchial asthma.

184
USMLE WORLD STEP 1 PHARMACOLOGY
(Choice F) Intraarticular glucocorticoids are typically used in elderly
patients with renal dysfunction as these patients cannot be treated
with colchicine and nonsteroidal anti inflammatory drugs. They should
not be used for chronic gout because of the numerous side effects.

Educational Objective:
Allopurinol is the best long-term treatment choice for chronic
tophaceous gout regardless of the urinary excretion of uric acid. In
patients who excrete large amounts of uric acid, uricosuric drugs
should be avoided to prevent uric acid nephrolithiasis.

185
USMLE WORLD STEP 1 PHARMACOLOGY

Q NO 326: A 9-year-old girl is brought to the ER by her mother after the child was
witnessed to have jerking movements of the right arm. The mother explains that
this is the second episode of such an “attack.” The girl remembers neither episode,
but her mother tells you that the child also made bizarre chewing and sucking
movements. The patient is lethargic after the episodes. She is started on a
medication to treat her condition. Three months later, she is found to have
following blood counts:

Hemoglobin 9.0 g/L


Reticulocytes 0.1%
Platelets 80000/mm3
Leukocyte count 2500/mm3
Which of the following is the most likely mechanism of the action of
the drug used in this patient?

A. Decreases calcium current in thalamic neurons


B. Decreases sodium current in cortical neurons
C. Increases chloride current on multiple levels
D. Blocks NMDA receptors in hippocampal neurons

Seizures that involve only one area of the body are called “partial.” A
simple seizure means that there is no associated impairment of
consciousness during or after the event. Complex seizures to the
contrary, are characterized by loss of memory during the event and a
postictal state. This clinical vignette describes a typical
presentation of a complex partial seizure.
Both simple and complex partial seizures can be effectively treated
with carbamazepine. Carbamazepine is also used for the management of
generalized tonic-clonic seizures as a mood stabilizer in bipolar
disorder. and to treat trigeminal neuralgia. Carbamazepine blocks
voltage-gated Na channels in cortical neurons. It stabilizes these
channels in an inactivated state; fewer Na channels therefore are
available for the propagation of an abnormal action potential.
Carbamazepine may cause the following important adverse effects:
1. Bone marrow suppression may lead to anemia agranulocytosis, and
thrombocytopenia. Low hemoglobin, leukopenia, and thrombocytopenia—as
in the above patient—can result from carbamazepine therapy. Complete
blood counts should be monitored periodically.
2. Carbamazepine is also hepatotoxic. LETs should be monitored
regularly.
3. A carbamazepine-associated increase in ADH secretion may cause
SIADH.
(Choice A) Ethosuximide blocks T-type Ca2 channels and decreases
calcium current in thalamic neurons. This anti convulsant is the agent
of choice for absence seizures. It is not associated with bone marrow
suppression. (Choices C and D) Carbamazepine does not affect chloride
channels or NMDA receptors.

Educational Objective:
Carbamazepine is used for simple partial, complex partial, and
generalized tonic-clonic seizures. It acts by blocking voltage-gated
sodium channels in neuronal membranes. Carbamazepine can cause bone
marrow suppression so blood counts should be reviewed often.

186
USMLE WORLD STEP 1 PHARMACOLOGY

Q NO 327: An 18-year girl is brought to the ER after an attempted suicide. Her


parents report that she consumed two bottles of an arsenic-containing
insecticide. The patient complains of nausea, vomiting, abdominal pain and
bloody diarrhea. Her blood pressure is 90160mm Hg and pulse is 120
beats/min. She has a garlic odor on her breath. Which of the following
medications should be administered immediately?
A. Penicillamine
B. CaNa2 EDTA
C. Dimercaprol
D. Amyl nitrite
E. Deferoxamine
F. Methylene blue

Acute arsenic poisoning most commonly occurs due to ingestion of


arsenic-containing insecticides. It presents with GI symptoms such as
nausea, vomiting, abdominal pain, and diarrhea, a decreased level of
consciousness, hypotension, tachycardia, and a garlic odor on the
patient’s breath. Acute arsenic poisoning is treated with chelating
agents. The drug of choice is dimercaprol, which displaces arsenic ions
from the sulfhydryl groups of enzymes. Dimercaprol is itself toxic and
has a narrow therapeutic index. The most serious side effects of
dimercaprol are nephrotoxicity and hypertension.
(Choice A) D-Penicillamine is used for chelation of copper from the
body and is used in treatment of Wilson disease. It can additionally be
used as a treatment for rheumatoid arthritis.
(Choice B) CaNa2 EDTA is the chelating agent of choice for treatment of
acute lead and mercury poisoning. It acts by forming complexes with
mono-, d1- and trivalent ions.
(Choice D) Amyl nitrite is effective in the treatment of cyanide
poisoning. It forms methemoglobin that binds cyanide ions forming the
non-toxic compound cyano methemoglobin thereby preventing cyanide from
binding to mitochondrial enzymes in the tissues.
(Choice E) Deferoxamine is a chelating agent used to treat iron
poisoning that occurs in patients receiving multiple blood transfusions
for conditions such as thalassemia and in patients who consume large
amounts of supplemental iron. It binds iron in the bloodstream and
facilitates its urinary excretion.
(Choice F) Methylene blue is indicated for treatment of
methemoglobinemia. It is a reducing agent that converts iron in heme
from the Fe3to the Fe2 state.

Educational Objective:
Dimercaprol is a chelating agent used in the treatment of arsenic
poisoning. It displaces arsenic ions from sulfhydryl groups of enzymes
and facilitates their excretion. Arsenic causes poisoning by
inactivating numerous enzymes involved in cellular metabolism. Symptoms
of poisoning include stomach pains, vomiting and delirium in addition
to a garlic odor on the breath.

187
USMLE WORLD STEP 1 PHARMACOLOGY

Q NO 328: A 65-year-old male presents to the ER complaining of suprapubic


A.
discomfort and anuria. His past medical history is significant for benign
prostatic hypertrophy diagnosed one year ago and treated effectively with
finasteride. He also has type-2 diabetes, hyperlipidemia and hypertension.
He was recently seen in the office for painful peripheral neuropathy. Foley
catheter placed in this patient revealed 1000 ml of urine. Which of the
following medications most likely have contributed to this patient’s current
condition?
Finasteride
B. Amitriptyline
C. Duloxetine
D. Gabapentin
E. Capsaicin
F. Carbamazepine
G. Atorvastatin
H. Metformin
I. Tamsulosin

This patient most likely has developed acute urinary retention due to
the anti cholinergic side effect of amitriptyline. TCAs are commonly
used for the treatment of painful diabetic neuropathy.

(Choices A and I) Finasteride and tamsulosin are useful for BPH and
should not cause urinary retention.
(Choice C) Duloxetine is a dual serotonin and norepinephrine reuptake
inhibitor and is promising for the treatment of painful diabetic
polyneuropathy. However, this would not cause anticholinergic side
effects.
(Choices D, E and F) Gabapentin, capsaicin cream and carbamazepine are
all useful for the painful neuropathy but none of them cause
anticholinergic side effects.
(Choices G and H) Metformin causes lactic acidosis and atorvastatin
causes hepatitis and myositis. They do not have anticholinergic side
effects.

Educational Objective:
Tricyclic antidepressants (TCAs) such as imipramine, doxepin,
amitriptyline, and clomipramine have stronger anticholinergic
properties than heterocyclic or SSRIs and should be used with caution
in patients with benign prostatic hypertrophy (BPH), as they may cause
urinary retention.

188

You might also like